Longman preparation course for the toefl test -the paper test-ITP

629 Pages • 292,664 Words • PDF • 86.6 MB
Uploaded at 2021-09-24 18:16

This document was submitted by our user and they confirm that they have the consent to share it. Assuming that you are writer or own the copyright of this document, report to us by using this DMCA report button.


CONTENTS IN T R O D U C T IO N

19

S E C T IO N O N E : L IS T E N IN G C O M P R E H E N S IO N D IA G N O S T IC P R E -T E S T

35

L IS T E N I N G C O M P R E H E N S IO N

42

T h e L is t e n in g P art A Q u e s tio n s

43

S tr a te g ie s SKILL 1: Focus on the second line SK ILL 2: C hoose answers with synonym s SK ILL 3: Avoid similar sounds T O E F L EXERCISE (Skills 1-3)

44 46 48 50

W h o , W h at, W h e r e SK ILL 4: Draw conclusions about who,what, where SK ILL 5: Listen for who and what in passives SK ILL 6: Listen for who and what with m ultiple nou ns T O E FL EXERCISE (Skills 4-6) T O E FL REVIEW EXERCISE (Skills 1-6)

51 54 56 58 59

N e g a tiv e s SKILL 7: Listen for negative expressions SKILL 8: Listen for dou ble negative expressions SK ILL 9: Listen for “alm ost negative” expressions SKILL 10: Listen for negatives with com paratives T O E FL EXERCISE (Skills 7-10) T O EFL REVIEW EXERCISE (Skills 1-10)

60 62 64 66 68 69

F u n c tio n s SKILL 11: Listen for expressions o f agreem en t SK ILL 12: Listen for expressions o f uncertainty and su ggestion SK ILL 13: Listen for em phatic expressions o f surprise T O E FL EXERCISE (Skills 11-13) T O E FL REVIEW EXERCISE (Skills 1-13) C o n tra r y M e a n in g s SK ILL 14: Listen for wishes SKILL 15: Listen for untrue conditions T O EFL EXERCISE (Skills 14-15) T O E FL REVIEW EXERCISE (Skills 1-15)

70 72 75 77 78 7g gj 83 H4

I d io m a tic L a n g u a g e SKILL 16: Listen for two- and three-part verbs SK ILL 17: Listen for idiom s T O E F L EXERCISE (Skills 16-17) T O E FL REVIEW EXERCISE (Skills 1-17)

g5 gy 89 89

12 CO NTEN TS

T h e L is te n in g P art B Q u e s tio n s

91

B e fo r e L is te n in g SK ILL 18: Anticipate the topics SKILL 19: Anticipate the questions

93 95

W h ile L is te n in g SKILL 20: D eterm ine the topic SKILL 21: Draw conclusions about who, what, when, where SKILL 22: Listen for answers in order T O EFL REVIEW EXERCISE (Skills 18-22) T h e L is t e n in g P art C Q u e s tio n s B e fo r e L is te n in g SKILL 23: Anticipate the topics SKILL 24: Anticipate the questions W h ile L is te n in g SKILL 25: D eterm ine the topic SK ILL 26: Draw conclusions about who, what, when, where SK ILL 27: Listen for answers in order T O E F L REVIEW EXERCISE (Skills 23-27) T O E F L P O S T -T E S T

97 98 99 101

102

104 106 108 109 110 112

113

S E C T IO N T W O : S T R U C T U R E A N D W R IT T E N E X P R E S S IO N D IA G N O S T IC P R E -T E S T S T R U C T U R E A N D W R IT T E N E X P R E S S IO N T h e S tr u c tu r e Q u e s tio n s S e n te n c e s SK ILL SK ILL SK ILL SKILL SKILL

w ith O n e C la u se 1: Be sure the sentence has a subject and a verb 2: Be careful o f objects o f prepositions 3: Be careful o f appositives 4: Be careful o f present participles 5: Be careful o f past participles

EXERCISE (Skills 1-5) T O E FL EXERCISE (Skills 1-5) S e n te n c e s w ith M u ltip le C la u s e s SKILL 6: U se coordinate connectors correctly SK ILL 7: U se adverb time and cause connectors correctly SKILL 8: U se other adverb connectors correctly EXERCISE (Skills 6-8) T O E F L EXERCISE (Skills 6-8) T O E F L REVIEW EXERCISE (Skills 1-8)

121

128 129 130 131 133 134 136 138 138

139 141 142 144 144 145

CO N TEN TS

13

M o re S e n te n c e s w ith M u ltip le C la u s e s U se n ou n clause connectors correctly SK ILL 9: SK ILL 10 : U se n ou n clause connector/subjects correctly SK ILL 11 : U se adjective clause connectors correctly SK ILL 12 : U se adjective clause connector/subjects correctly EXERCISE (Skills 9-12) T O E F L EXERCISE (Skills 9-12) T O E F L REVIEW EXERCISE (Skills 1-12)

146 148 149 151 153 153 154

S e n te n c e s w ith R e d u c e d C la u s e s SK ILL 13: U se red uced adjective clauses correctly SK ILL 14: U se reduced adverb clauses correctly EXERCISE (Skills 13-14) T O E FL EXERCISE (Skills 13-14) T O E F L REVIEW EXERCISE (Skills 1-14)

155 158 160 160 161

S e n te n c e s w ith I n v e r te d S u b je cts a n d V erb s SK ILL 15 Invert the subject and verb with SK ILL 16 Invert the subject and verb with SK ILL 17 Invert the subject and verb with SK ILL 18 Invert the subject and verb with SK ILL 19 Invert the subject and verb with EXERCISE (Skills 15-19) T O E F L EXERCISE (Skills 15-19) T O E FL REVIEW EXERCISE (Skills 1-19)

163 164 166 168 170 172 172 173

question words place expressions negatives conditionals com parisons

T h e W r itte n E x p r e s s io n Q u e s tio n s

175

P r o b le m s w ith S u b je ct/V e rb A g r e e m e n t SK ILL 20 Make verbs agree after prepositional phrases SK ILL 21 M ake verbs agree after expressions o f quantity SK ILL 22 Make inverted verbs agree SKILL 23 M ake verbs agree after certain words EXERCISE (Skills 20-23) T O E FL EXERCISE (Skills 20-23) T O E F L REVIEW EXERCISE (Skills 1-23)

176 177 178 180 181 181 182

P r o b le m s w ith P a r a lle l S tr u c tu r e SK ILL 24 U se parallel structure with coordinate conjunctions SKILL 25: U se parallel structure with paired conjunctions SK ILL 26: U se parallel structure with com parisons EXERCISE (Skills 24-26) T O E F L EXERCISE (Skills 24-26) T O E FL REVIEW EXERCISE (Skills 1-26)

184 185 186 188 188 189

P r o b le m s w ith C o m p a r a tiv e s a n d S u p e r la tiv e s SK ILL 27: Form com paratives and superlatives correctly SK ILL 28: U se com paratives and superlatives correcdy SK ILL 29: U se the irregular -er, -er structure correcdy EXERCISE (Skills 27-29) T O E F L EXERCISE (Skills 27-29) T O E F L REVIEW EXERCISE (Skills 1-29)

190 191 192 194 194 195

14 CO NTEN TS

P r o b le m s w ith th e F o rm o f th e V erb SKILL 30: After have, use the past participle SKILL 31: After be, use the present participle or the past participle SKILL 32: After will, would, or other m odals, use the base form o f the verb EXERCISE (Skills 30-32) T O EFL EXERCISE (Skills 30-32) T O E FL REVIEW EXERCISE (Skills 1-32) P r o b le m s w ith th e U s e o f th e V erb SKILL 33: Know w hen to use the past with the present SKILL 34: SKILL 35: SKILL 36:

Use have and had correctly U se the correct tense with tim e expressions U se the correct tense with will and would

EXERCISE (Skills 33-36) TO EFL EXERCISE (Skills 33-36) T O EFL REVIEW EXERCISE (Skills 1-36) P r o b le m s w ith P a ssiv e V erb s SKILL 37: U se the correct form o f the passive SKILL 38: Recognize active and'passive m eanings EXERCISE (Skills 37-38) TO E FL EXERCISE (Skills 37-38) TO E FL REVIEW EXERCISE (Skills 1-38) P r o b le m s SKILL SKILL SKILL SKILL

w ith 39: 40: 41: 42:

N ouns U s e the correct singular or plural noun D istinguish countable and uncountable nouns R ecognize irregular plurals o f nouns Distinguish the person from the thing

EXERCISE (Skills 39-42) T O E FL EXERCISE (Skills 39-42) T O E FL REVIEW EXERCISE (Skills 1-42)

196 '97 198 >99 199 200 201 202 204 205 207 207 208 2 10 211 212 2 >2 213 2142 15 217 218 2 19 220 221

P r o b le m s w ith P r o n o u n s SKILL 43: Distinguish subject and object pronouns SKILL 44: D istinguish possessive adjectives and pronou ns SKILL 45: Check pronou n reference for agreem ent EXERCISE (Skills 43-45) TO E FL EXERCISE (Skills 43-45) TO E FL REVIEW EXERCISE (Skills 1-45)

222 223 225 226 226 227

P r o b le m s w ith A d je c tiv e s a n d A d v erb s SKILL 46: U se basic adjectives and adverbs correctly SKILL 47: U se adjectives after linking verbs SKILL 48: Position adjectives and adverbs correctly EXERCISE (Skills 46-48) TO EFL EXERCISE (Skills 46-48) TO EFL REVIEW EXERCISE (Skills 1-48)

228 230 232 233 234 235

CO NTEN TS

M o re P r o b le m s w ith A d je c tiv e s SK ILL 49: R ecognize -ly adjectives SK ILL 50 U se predicate adjectives correctly SK ILL 51 U se -ed and -ing adjectives correctly EXERCISE (Skills 49-51) T O E FL EXERCISE (Skills 49-51) T O E FL REVIEW EXERCISE (Skills 1-51)

15

236 237 238 239 240 241

P r o b le m s w ith A r tic le s SK ILL SKILL SKILL SKILL

52: 53 54 55

Use articles with singular nouns D istinguish a and an M ake articles agree with nouns Distinguish specific and general ideas EXERCISE (Skills 52-55) T O E F L EXERCISE (Skills 52-55) TO EFL REVIEW EXERCISE (Skills 1-55) P r o b le m s w ith P r e p o s itio n s SK ILL 56: R ecognize incorrect prepositions SK ILL 57: R ecognize when prepositions have been om itted EXERCISE (Skills 56-57) T O E F L EXERCISE (Skills 56-57) T O EFL REVIEW EXERCISE (Skills 1-57) P r o b le m s w ith U s a g e SKILL 58: Distinguish make and do SKILL 59: Distinguish like, alike, unlike, and dislike SKILL 60: Distinguish other, another, and others EXERCISE (Skills 58-60) T O E FL EXERCISE (Skills 58-60) T O E FL REVIEW EXERCISE (Skills 1-60) T O E F L P O S T -T E S T

242 243 244 245 246 247 248 249 250 251 251 252

253 254 255 256 257 258 259

S E C T IO N T H R E E : R E A D IN G C O M P R E H E N S IO N D IA G N O S T IC P R E -T E S T R E A D IN G C O M P R E H E N S IO N T h e R e a d in g C o m p r e h e n s io n Q u e s tio n s

269 281 282

Q u e s tio n s a b o u t th e I d e a s o f th e P a ssa g e 3 J T .I . 1 • Ancm or m n in U n n •.!___ Answer m ain idea questions correctly SKILL 2: R ecognize the organization o f ideas T O E FL EXERCISE (Skills 1-2)

D ir e c tly A n sw e r e d Q u e s tio n s SK ILL 3 Answer stated detail questions correctly SK ILL 4 Find “u nstated” details SK ILL 5: Find pronou n referents T O E FL EXERCISE (Skills 3-5) T O E FL REVIEW EXERCISE (Skills 1-5)

283 287 290 291 296 299 301 303

16 CO NTEN TS

I n d ir e c tly A n sw e r e d Q u e s tio n s SKILL 6: Answer im plied detail questions correctly SKILL 7: Answer transition questions correctly T O E FL EXERCISE (Skills 6-7) T O E FL REVIEW EXERCISE (Skills 1-7) V o c a b u la r y Q u e s tio n s SKILL 8: Find definitions from structural clues SKILL 9: D eterm ine m eanings from word parts SK ILL 10: U se context to determ ine m eanings o f difficult words SK ILL 11: U se context to determ ine m eanings o f sim ple words T O E FL EXERCISE (Skills 8-11) T O E FL REVIEW EXERCISE (Skills 1-11) O v e r a ll R e v ie w Q u e s tio n s SKILL 12: D eterm ine w here specific inform ation is found SK ILL 13: D eterm ine the tone, purpose, or course T O E FL EXERCISE (Skills 12-13) T O E FL REVIEW EXERCISE (Skills 1-13)

307 311 314 316

320 324 326 328 330 333 338 341 346 349 355

T O E F L P O S T -T E S T S E C T I O N F O U R : T e s t o f W r itte n E n g lis h (T W E )

370

T h e Score

370

S a m p le E ssa y s B e fo r e W r itin g SK ILL 1: D ecod e the topic SK ILL 2: D evelop su pp orting ideas

376 379

W h ile W r itin g SKILL 3: SKILL 4: SKILL 5: SKILL 6:

382 385 388 391

W rite the introductory paragraph Write unified su pp orting paragraphs Write the conclu ding paragraph C onnect the su pp orting paragraphs in the essay

A fte r W r itin g SK ILL 7: Edit 7A 7B 7C SK ILL 8:

Edit written expression 8A Inversions and agreem en t 8B Parallel, com parative, and superlative structures 8C 8D 8E 8F

P r a c tic e T e s ts

sentence structure Sim ple sentence structure C om p oun d sentence structure C om p lex sentence structure

Verbs N ou n s and p ronou ns Adjectives and adverbs Prepositions and usage

394 394 396 398 401 401 402 403 404 405 406 407

CO N TEN TS

409

CO M PLETE TESTS CO M PLETE CO M PLETE CO M PLETE CO M PLETE CO M PLETE

TEST TEST TEST TEST TEST

ONE TW O T H R EE F O U R .. FIVE

411 437 463 489 515 541

A P P E N D IX E S A P P E N D IX A P P E N D IX A P P E N D IX A P P E N D IX A P P E N D IX A P P E N D IX A P P E N D IX A P P E N D IX A P P E N D IX

17

A: B: C: D: E: F: G: H: I:

Similar Sounds T w o- and T hree-P art Verbs Idiom s Prepositions W ord Endings Irregular Verb Forms Form ation o f the Passive Irregular Plurals W ord Parts

543 544 548 552 554 560 562 564 565

SCOR ES A N D C H A R TS

579

Scoring Inform ation D iagnostic Charts Progress Chart T O E F L A nswer Sheets

581 583 591 593

R E C O R D IN G S C R IP T A N SW E R KEY

'609 651

« m a n

aãỉ

18

■SECTION ONE.

LISTENING COMPREHENSION

33

34

l

D

l

D

l

D

l

D

l

ü

l

ü

l

n

l

DIAGNOSTIC PRE-TEST S E C T IO N 1 L IS T E N IN G C O M P R E H E N S IO N T im e — a p p r o x im a t e ly 3 5 m in u te s (in c lu d in g t h e r e a d in g o f t h e d ir e c t io n s fo r e a c h p a r t) In this section of the test, you will have an opportunity to demonstrate your ability to understand conversations and talks in English. There are three parts to this section, with special directions for each part. Answer all the questions on the basis of what is stated or im plied by the speakers you hear. Do not take notes or write in your test book at any time. Do not turn the pages until you are told to do so.

Part A Directions: In Part A you will hear short conversations between two people. After each conversation, you will hear a question about the conversation. The conversations and questions will not be repeated. After you hear a question, read the Four possible answers in your test book and choose the best answer. Then, on your answer sheet, find the number of the question and fill in the space that corresponds to the letter of the answer you have chosen. Listen to an exam ple.

Sam ple Answer

On the recording, you w ill hear: (m an) (w om an) (narrator)

That exam was just awful. Oh, it could have been worse. What does the w om an m ean?

In your test book, you w ill read:

(A) (B) (C) (D)

The exam was really awful. It was the worst exam she had ever seen. It couldn't have been more difficult. It wasn't that hard.

You learn from the conversation that the man thought the exam was very difficult and that the woman disagreed with the man. The best answer to the question, “What does the woman mean?" is (D), "It wasn’t that hard.” Therefore, the correct choice is (D).

TOEFL* test directions and form at are reprinted by perm ission of ETS, the copyright owner. However, all exam ples and test questions are provided by P earson Education. Inc.

LISTENING COMPREHENSION PRE-TEST

35

l

D

l

D

l

D

l

D

1. (A) The coffee is much better this morning. (B) He’s feeling bitter this morning. (C) The coffee isn’t very good. (D) He cannot taste the butter. 2. (A) The two classes meet in an hour and a half. (B) The class meets three hours per week. (C) Each half of the class is an hour long. (D) Two times a week the class meets for an hour. 3. (A) A few minutes ago, the flight departed. (B) The fight will start in a while. (C) They are frightened about the departure. (D) The plane is going to take off soon. 4. (A) He hasn’t yet begun his project. (B) He’s supposed to do his science project next week. (C) He needs to start working on changing the due date. (D) He’s been working steadily on his science project. 5. (A) (B) (C) (D)

At the post office. In a florist shop. In a restaurant. In a hospital delivery room.

6. (A) The professor drowned the cells in a lab. (B) The topic was presented in a boring way. (C) The professor divided the lecture into parts. (D) The biologist tried to sell the results o f the experiment. 7. (A) She needs to get a driver's license. (B) It is impossible to cash a check without two pieces of identification. (C) The man should check to see if he needs credit. (D) A credit card can be used to get a driver’s license.

l

D

l

D

l

D

l

8. (A) Housing within his budget is hard to locate. (B) It’s hard to find his house in New York. (C) He can’t afford to move his house to New York. (D) Housing in New York is unavailable. 9. (A) The boss was working on the reports. (B) He would have to finish the reports before the end of next month. (C) He was directed to stay late and finish som e work. (D) He could finish the reports at home. 10. (A) The boisterous students made the teacher mad. (B) The teacher angered the students with the exam results. (C) The students were angry that the teacher was around. (D) The angered students complained to the teacher. 11. (A) The prices are reasonable. (B) The store is too far out of town. (C) He would like the woman to repeat what she said. (D) He agrees with the woman. 12. (A) (B) (C) (D)

It has rarely rained this much. It hardly rained this year. It is barely raining this year. It seldom rains so little.

13. (A) He needs to do a better job writing questions. (B) His writing must certainly be improved. (C) Without the questions, he cannot write the answers. (D) He needs to understand the written questions better.

GO ON TO THE NEXT PAGE

36

LISTENING COMPREHENSION PRE-TEST

l

D

l

D

l

D

l

D

14. (A) The agent was standing in line with his passport. (B) The line to get new passports is very long. (C) The woman must wait her turn to get her passport checked. (D) He can check her passport instead of the agent. 15. (A) He couldn’t finish closing the library book. (B) He hadn’t finished the library assignment, but he was close. (C) He was working on the assignment when the library closed. (D) His homework was incomplete because the library wasn’t open. 16. (A) He wishes the hard work had had a better result. (B) He thinks the lawyer hardly prepared. (C) He wishes the lawyer had prepared. (D) He thinks the lawyer worked for free. 17. (A) The history class begins next week. (B) He thinks the papers should be turned in next week. (C) He has already done the paper for next week. (D) The papers are not due next week. 18. (A) He's not really happy. (B) The contractor’s work was satisfactory. (C) He would rather work with the contractor himself. (D) He was already contacted about the work. 19. (A) The man should try another type of paper. (B) The man should locate a typist tomorrow morning. (C) The man should make a tape in the morning. (D) The man should complete the paper without help.

l

D

l

D

l

D

l

20. (A) She’d like som e pie. (B) It’s easy to buy it. (C) The task the man’s working on isn’t difficult. (D) It’s easier to prepare pie than do what the man is doing. He reported that the tim e for the budget meeting had been set. (B) He is always late in submitting his accounting figures. (C) He never manages to budget his time well. (D) He is never too late in turning in his reports.

21. (A)

22. (A) The repairs would require an extension. (B) The car is going to need a lot of repairs. (C) Buying a new car would be quite expensive. (D) The mechanic extended the repair warranty. 23. (A) Betty wrote the letter as directed. (B) The directions were given to Betty in a letter. (C) Betty will follow the instructions later. (D) Betty worked exactly as instructed. 24. (A) Walter's had a lack of success with his business. (B) Walter’s failed in business. (C) Walter's new company is doing rather well. (D) Walter hoped to succeed in business. 25. (A) He should put the organ in the closet. (B) The closet has already been organized. (C) He needs to rearrange the closet. (D) He wishes the closet were closer. 26. (A) She didn’t do the work. (B) She gave the assignment her best effort. (C) She finished the assignm ent even though it was difficult. (D) She gave the man a signal.

LISTENING COMPREHENSION PRE-TEST

37

l D l D l D l D l D l D l D l 27. (A) (B) (C) (D)

She said some terrible things. She didn’t say anything nice. She didn't have any nice things. She said really wonderful things.

28. (A) New employees are rarely initiated into the company. (B) New workers don’t generally undertake actions on their own. (C) New employees are initially rated. (D) It’s rare for employees to make new suggestions.

38

LISTENING COMPREHENSION PRE-TEST

29. (A) The woman is more than a week late. (B) The children would have wrecked the house later. (C) The woman was so late that she was a wreck. (D) He’s glad that she was not any later. 30. (A) He had not gone to the store. (B) He was still at the market. (C) He was going to take care of the shopping. (D) He always went to the market.

l n l D l D l n l ü l n l D l Part B Directions: In this part of the test, you will hear longer conversations. After each conversation, you will hear several questions. The conversations and questions will not be repeated. After you hear a question, read the four possible answers in your test book and choose the best answer. Then, on your answer sheet, find the number of the question and fill in the space that corresponds to the letter of the answer you have chosen. Remember, you are not allowed to take notes or write in your test book. 31. (A) (B) (C) (D)

She's a senior. She’s a junior. She’s a transfer student. She’s a graduate student.

32. (A) How to transfer to a junior college. (B) How to find his way around campus. (C) The course requirements for a literature major. (D) Who won the campus election. 33. (A) (B) (C) (D)

Three. Five. Eight. Ten.

34. (A) (B) (C) (D)

American literature. World literature. Literary analysis. Surveying.

35. (A) (B) (C) (D)

In a book. From a television program. During a trip that she took. From a lecture.

36. (A) (B) (C) (D)

To communicate with other dolphins. To recognize objects in the water. To learn human language. To express fear.

37. (A) (B) (C) (D)

Five. Fifteen. Fifty. Five hundred.

38. (A) (B) (C) (D)

It is limited. It is greater than human intelligence. It is less than previously thought. We are beginning to learn how m uch they have.

iu im TOEFL* test directions and form at are reprinted by perm ission of ETS, the copyright owner. However, all exam ples and test questions are provided by P earson E ducation, Inc.

4 n u u |

LISTENING COMPREHENSION PRE-TEST

39

l D l D l n l n l D l n l a l Part C D irections: In this part of the test, you will hear several talks. After each talk, you will hear some questions. The talks and questions will not be repeated. After you hear a question, you will read the four possible answers in your test book and choose the best answer. Then, on your answer sheet, find the number of the question and fill in the space that corresponds to the letter of the answer you have chosen. Here is an exam ple. On the recording, you w ill hear: (narrator) (m an)

Listen to an instructor talk to his class about painting. Artist Grant Wood was a guiding force in the school o f painting know n as American regionalist, a style reflecting the distinctive characteristics o f art from rural areas o f the United States. Wood began drawing anim als on the family farm at the age o f three, and when he was thirty-eight one o f his paintings received a remarkable am ount o f public notice and acclaim. This painting, called “American Gothic," is a starkly simple depiction o f a serious couple staring directly o u t at the viewer.

Now listen to a sample question. (narrator)

Sam ple Answer

What style o f painting is known as American regionalist?

In your test book, you w ill read:

(A) (B) (C) (D)

Art from Art from Art from Art from

(A) ® © 9

Americas inner cities. the central region of the U.S. various urban areas in the U.S. rural sections of America.

The best answer to the question, "What style of painting is known as American regionalist?” is (D), "Art from rural sections of America." Therefore, the correct choice is (D). Now listen to another sample question. (narrator)

Sam ple Answer

W hat is the name o f Wood's m ost successful painting?

In your test book, you w ill read:

(A)' (B) (C) (D)

(A )® ® ®

"American Regionalist.” "The Family Farm in Iowa." "American Gothic.” "A Serious Couple.”

The best answer to the question, "What is the name of Wood's most successful painting ?” is (C). “American Gothic.” Therefore, the correct choice is (C). Remember, you are n ot allowed to take notes or write in your test book.

40

LISTENING COMPREHENSION PRE-TEST

TOEFL* test directions and form at arc reprinted by perm ission o f ETS, the copyright owner. However, all examples and test q uestions are provided by Pearson Education, Inc.

l D l D l D l D l D l D l D l 39. (A) (B) (C) (D)

To protect its members. To save the natural environment. To honor the memory o f John Muir. To improve San Francisco's natural beauty.

40. (A) (B) (C) (D)

For less than a year. Only for a decade. For more than a century. For at least two centuries.

41. (A) (B) (C) (D)

San Francisco. All fifty states. The Sierra Nevadas. The eastern U.S.

42. (A) (B) (C) (D)

All over the world. In the entire United States. Only in California. Only in the Sierra Nevadas.

To the To the To the To the

46. (A) (B) (C) (D)

Go to the Art Center. Sign up for sports classes. Visit the exercise room. Watch a football game.

47. (A) (B) (C) (D)

Science. Art. Literature. Music.

tennis courts. arena. gymnasium. Athletic Department office.

48. (A) They are completely different. (B) They are somewhat similar but have an essential difference. (C) They are exactly the sam e in all respects. (D) They are unrelated.

43. (A) Students signing up for athletic teams. (B) Students going on a tour of a university campus. (C) Students playing various sports. (D) Students attending a university dedication ceremony. 44. (A) (B) (C) (D)

45. (A) (B) (C) (D)

Membership on an athletic team. Enrollment in an exercise class. A valid student identification card. Permission from a faculty member.

49.

(A) (B) (C) (D)

Objective. Idealistic. Philosophical. Environmental.

50. (A) (B) (C) (D)

Heredity. Environment. Idealism. Natural laws.

This is the end of the Listening Comprehension Pre-Test. Turn off the recording.

(st o p)

(st o p)

(s t o p )

^ ST Q p j

(s t o p )

(s t o p)

(s t o p )

W h e n y o u fin ish th e test, y o u m ay d o th e fo llo w in g : • T u rn to th e D ia g n o s tic C h art o n p a g e s 585$- '>84, a n d c ir cle th e n u m b e r s o f th e q u e s tio n s th at y o u m isse d . • T u rn to S c o r in g I n fo r m a tio n o n p a g e s 5 8 1 - 5 8 2 , a n d d e te r m in e y o u r T O E F L sc o r e. • T u rn to th e P r o g r e s s C h art o n p a g e 5 9 1 , a n d a d d you r sc o r e to th e ch art.

LISTENING COMPREHENSION PRE-TEST

4I

LISTENING COMPREHENSION T h e first se c tio n o f th e T O E F L test is th e L iste n in g C o m p r e h e n s io n se c tio n . T h is se c tio n c o n sists o f fifty q u e s tio n s (so m e tests m ay b e lo n g e r ). You w ill listen to r e c o r d e d m aterials a n d r e s p o n d to q u e s tio n s a b o u t th e m aterial. You m u st lis te n carefully, b e c a u se y o u will h e a r th e r e c o r d in g p rogram o n e tim e only, a n d th e m aterial o n th e r e c o r d in g is n o t writ­ te n in y o u r test b o o k . T h e r e are th r e e parts in th e L iste n in g C o m p r e h e n s io n se c tio n o f th e T O E F L test: 1.

P a rt A c o n sists o f thirty sh o r t co n v e rsa tio n s, e a c h fo llo w e d by a q u e s tio n . You m u st c h o o s e th e b e st answ er to e a c h q u e s tio n fr o m th e fo u r c h o ic e s in y o u r te st b ook . 2. P a rt B c o n sists o f tw o lo n g c o n v e rsa tio n s, e a c h fo llo w e d by a n u m b e r o f q u estio n s. Y ou m u st c h o o s e th e b e st answ er to e a c h q u e s tio n from th e fo u r c h o ic e s in your 3.

te st b o o k . P art C c o n sists o f th r e e talks, e a c h fo llo w e d by a n u m b e r o f q u estio n s. You m u st c h o o s e th e b e st an sw er to e a c h q u e s tio n fro m th e fo u r c h o ic e s in y o u r test b o o k .

GENERAL STRATEGIES 1. Be fam iliar w ith th e d irection s. The directions on every TOEFL test are the same, so it is not necessary to listen carefully to them each time. You should be completely familiar with the directions before the day of the test. 2. Listen carefully to th e con versation s and talk s. You should concentrate fully on what the speakers are saying in the recording program, because you will hear it one time only. 3. Know w h er e th e e a sier and m ore difficult q u estio n s are generally found. Within each part of the Listening Comprehension section, the questions generally progress from easy to difficult. 4. N e v er leave any answ ers blank on your an sw er sh e e t. Even if you are unsure of the correct response, you should answer each question. There is no penalty for guessing. 5. U s e any rem aining tim e to look ahead at th e an sw ers to th e q u estio n s th a t follow . When you finish with one question, you may have time to look ahead at the answers to the next question.

42

LISTENING PARTA

T HE L IS T E N IN G PA R TA Q U E S T IO N S For e a c h o f th e thirty q u e s tio n s in P art A o f th e L iste n in g C o m p r e h e n s io n s e c tio n o f th e T O E F L test, y o u w ill h e a r a sh o r t c o n v e r s a tio n b e tw e e n two sp ea k er s fo llo w e d by a q u e s ­ tio n . A fter y o u liste n to th e c o n v e r s a tio n a n d q u e s tio n , y o u m u st c h o o s e th e b e s t an sw er to th e q u e s tio n from y o u r test b o o k . Exam ple On the recording, you hear: (man) (woman) (narrator)

I've always wanted to visit Hawaii w ith you. Why not next m onth? What does the w om an mean?

In your test book, you read: (A) (B) (C) (D)

Next month isn't a good time for the trip. She doesn’t want to go to Hawaii. She suggests taking the trip next month. She’s curious about why he doesn't want to go.

A n sw er (C ) is th e b e st answ er to th e q u e s tio n . W hy not n e xt m o n th ? is a s u g g e s tio n th a t th e y take th e trip n e x t m o n th .

STRATEGIES FOR THE LISTENING PARTA QUESTIONS 1. A s you listen t o each sh o r t c o n v e rsa tio n , focu s on th e se c o n d line o f th e con v ersa tio n . The answer to the question is generally found in the second line. 2.

K eep in m ind th a t th e c o r r e c t a n sw er is probably a r e s ta te m e n t o f a key w ord or Idea in th e se c o n d line o f th e co n v e rsa tio n . Think of possible restatements.

3.

K eep in m ind th a t c er ta in str u c tu r es and e x p r essio n s are te s te d regularly in L istening P art A . Listen for these structures and expressions: • structures (passives, negatives, wishes, conditions) • functional expressions (agreement, uncertainty, suggestion, surprise) • idiomatic expressions ( two-part verbs, three-part verbs, idioms)

4.

K eep in m ind th a t th e s e q u e stio n s gen erally progress from easy to difficult. This means that questions I through S will be the easiest, and questions 26 through 30 will be the hardest.

5.

Read th e an sw ers and c h o o se th e b e s t an sw er to each q u estio n . Remember to answer each question even if you are not sure of the correct response. Never leave any answers blank.

6.

Even if you do n o t u n derstand t h e c o m p le te con v ersa tio n , you can find th e c o r r e c t answer. • If you only understood a few words or ideas in the second line, choose the answer that contains a restatement of those words or ideas. • If you did not understand anything at all in the second line of the conversation, choose the answer that sounds the most different from what you heard. • Never choose an answer because it sounds like what you heard in the conversation.

LISTENING COMPREHENSION

T h e fo llo w in g skills will h e lp you to im p le m e n t th e se str a teg ie s in th e L iste n in g Part A s e c tio n o f th e T O E FL test. 5

STRATEGIES S k ill

I:

FO C U S O N T H E SEC O N D LINE

In L iste n in g Part A you w ill h ea r a sh o rt c o n v e r sa tio n in v o lv in g two p e o p le ; th is con versa­ tio n is fo llo w e d by a q u estio n . It is im p o r ta n t to u n d e r sta n d th at th e answ er to this type o f q u e s tio n is m o st o fte n (b u t n o t always!) fo u n d in th e s e c o n d lin e o f th e c o n v e rsa tio n . Exam ple On the recording, you hear: (man) (woman) (narrator)

Billy really made a big m istake this time. Yes, he forgot to turn in his research paper. What does the w om an say about Billy?

In your test book, you read: (A) (B) (C) (D)

It was the first time he made a mistake. He forgot to write his paper. He turned in the paper in the wrong place. He didn’t remember to submit his assignment.

T h e s e c o n d lin e o f th is co n v e rsa tio n in d ic a te s th a t B illy forgot to tu r n in his paper, a n d this m e a n s th a t h e d id not remember to subm it it. T h e b e st an sw er is th e r e fo r e answ er ( D ) . T h e fo llo w in g ch art o u tlin e s th e m o st im p o r ta n t strategy for L iste n in g P art A: STRATEGY#!: FOCUS ONTHE SECOND LINE I.

The second line of the conversation probably contains the answer to the question.

2. Listen to the first line of the conversation. If you understand it, that’s good. If you don’t understand it, don’t worry because it probably does not contain the'answer.

LISTENING PARTA

E X E R C ISE 1: In th is e x e r c ise , y o u s h o u ld fo c u s o n th e s e c o n d lin e o f th e c o n v e r sa tio n , r e a d th e q u e s tio n , a n d th e n c h o o s e th e b e st an sw er to th a t q u e s tio n . R e m e m b e r th a t y o u c a n p rob ab ly an sw er th e q u e s tio n ea sily w ith o n ly th e s e c o n d lin e. (man)

(woman) (narrator)

Can you tell me i f today's matinee is a comedy, romance, or western ? I have no idea. What does the woman mean ?

(A) (B)

She has strong ideas about movies. She prefers com ed ies over westerns and romances. (C) She d o esn ’t like today’s m atinee. (D) She does not know.

2.(wom an)

Was anyone at home at Barb’s house when you went there to deliver the package? (m an) / rang the bell, but no one answered. (narrator) What does the man imply?

(A) Barb answered the bell. (B) T h e house was probably empty. (C) T he bell wasn’t in the house. (D) T he house d o esn ’t have a bell.

3.(woman)

(A)

You ju st got back from the interview fo r the internship. How do you think it went ? (m an) I think i t ’s highly unlikely that I got the job. (narrator) What does the m an suggest ?

It’s unlikely that h e ’ll go to the interview. (B) H e thinks h e ’ll be recom m en ded for a high-level job . (C) T he interview was apparently quite unsuccessful. (D) H e had an excellen t interview.

T O E F L E X E R C ISE 1: In th is e x e r c is e , lis te n ca re fu lly to th e sh o r t c o n v e r s a tio n a n d q u e s­ tio n in th e r e c o r d in g p ro g ra m , a n d t h e n c h o o s e th e b e st answ er to th e q u e s tio n . You s h o u ld fo c u s ca refu lly o n th e s e c o n d lin e . N ow

BEGIN THE RECORDING PROGRAM AT TOEFL EXERCISE

1.

(A) (B) (C) (D)

H e is leaving now. H e has to go out o f his way. H e will n ot be leaving soon. H e will d o it his own way.

2.

(A) (B)

H e locked die door. H e tried unsuccessfully to g e t into the house. H e was able to op en the door. H e left the house without locking the door.

(C) (D) 3.

(A) (B) (C) (D)

4.

(A) (B) (C) (D)

She d oesn ’t like to listen to turkeys. She thinks the d in ner sounds special. She especially likes the roast turkey. Sh e’d prefer a different dinner. H e ’ll be busy with her hom ework tonight. H e can’t help her tonight. H e ’s sorry he can’t ever h elp her. H e ’ll help her with her physics.

I.

5.

(A) H er eyes hurt. (B) She thought the lecture was great. (C) T he class was boring. (D) She did n’t want to watch Professor Martin.

6.

(A) (B) (C) (D)

N ot all the bills have b een paid. They d o n ’t have en ou gh credit to pay the bills. What she said on the p h on e was not credible. H e used a credit card to pay som e o f the bills.

7.

(A) S h e’ll call back quickly. (B) S h e’ll definitely b e back by 4:00. (C) S h e’ll give it back by 4:00. (D) S h e’ll try to return fast.

8.

(A) She hasn’t seen Tim. (B) Tim was there only for a m om ent. (C) Tim was around a short tim e ago. (D) Tim will return in a m inute.

LISTENING COMPREHENSION

(A) (B) (C) (D)

She doesn’t like the place he chose. She doesn’t want to get into the car. S h e’s glad the spot is reserved. They can’t park the car there.

S k i l l 2:

10.

(A) T here’s plenty to eat. (B) T he refrigerator’s broken. (C) The food isn’t in the refrigerator. (D) H e ’s n ot sure if there’s enough.

C H O O S E ANSW ERS W IT H S Y N O N Y M S

O fte n th e c o r r e c t an sw er in L iste n in g Part A is an an sw er th a t c o n ta in s syn on ym s (w ords w ith sim ilar m e a n in g s b u t d iffe r e n t so u n d s) fo r k e y w o r d s in th e c o n v e rsa tio n .

Exam ple On the recording, you hear: (woman) (man) (narrator)

Why is Barbara feeling so happy? She just started working in a real estate agency. What does the man say about Barbara?

In your test book, you read: (A) She always liked her work in real estate. (B) She began a new job. (C) She just bought some real estate. (D) She bought a real estate agency.

In th is c o n v e rsa tio n , th e key w ord started m e a n s began, a n d th e key w ord w orking refers to job. T h e b e st answ er to th is q u e s tio n is th e r e fo r e an sw er ( B ) . T h e fo llo w in g ch a rt o u tlin e s a very im p o r ta n t strategy for L iste n in g Part A: STRATEGY #2: CHOOSE ANSWERS WITH SYNONYMS 1. As you listen to the second line of the conversation,focus on key words In that line. 2.

If you see any synonyms for key words in a particular answer, then you have probably found the correct answer.

EX E R C ISE 2: In th is e x e r c ise , u n d e r lin e key w ord s in th e se c o n d lin e o f e a c h sh o rt c o n ­ v e rsa tio n . T h e n u n d e r lin e syn on ym s for th e se key w ord s in th e answ ers, a n d c h o o s e th e b e st answ er to e a ch q u e s tio n . R e m em b er th a t th e b e s t answ er is p rob ab ly th e answ er th a t c o n ta in s syn on ym s fo r th e key w ord s in th e s e c o n d lin e o f th e co n v ersa tio n . 1.

(wom an) (m an) (narrator)

Did you see the manager about the job in the bookstore? Yes, and I also had to Jill out an application. What does the m an mean?

(A) H e got a job as bookstore manager. (B) The bookstore was not accepting applications. (C) H e saw a book about how to apply for jobs. (D) It was necessary to com plete a form.

LISTENING PART A

2.

(man) (woman) (narrator)

3.

(man) (woman) (narrator)

We 're planning to leave fo r the trip at about 2:00. Couldn’t we leave before noon ? What does the woman ask ?

(A) (B) (C) (D)

If they could leave at n oon . If it is possible to go by 12:00. Why they can’t leave at noon . If they could leave the room.

Was the concert well received ? The audience applauded fo r a long time after the performance. What does the woman say about the concert?

(A)

The perform ance went on for a long time. There was applause throughout the perform ance. The p eople clapped o n and on after the concert. The aud ien ce waited for a long tim e for the concert to begin.

(B) (C) (D)

T O E F L E X E R C ISE 2: In th is e x e r c ise , liste n carefu lly to th e sh o r t c o n v e r s a tio n a n d q u e s ­ tio n in th e r e c o r d in g p r o g r a m , a n d th e n c h o o s e th e b est answ er to th e q u e s tio n . You s h o u ld lo o k fo r syn on ym s fo r key w o r d s in th e s e c o n d lin e. N O W BEGIN THE RECORDING PROGRAM AT TOEFL EXERCISE 2.

(A)

T h e final exam was harder than the others. (B) T here were two exam s rather than one. (C) H e thought the exam would be easier. (D) T h e exam was not very difficult.

6.

T he firefighters saved the hom es for last. (B) A firefighter saved the hillside last night. (C) The hom es on the hillside were burned. (D) The houses w eren’t destroyed.

(A) (B) (C)

7.

(A) (B) (C) (D)

T here’s en ou gh soup. T he spices are adequate. She thinks the sou p’s too salty. The man should add m ore salt and pepper.

8.

(A)

He was lucky to receive a grant for his studies. He used his fortune to pay his fees. He is a scholar at a college with low fees. He paid to get a scholarship.

(D)

H e ’s not feelin g very well. H e ’s rather sick o f working. H e ’s feelin g better today than yesterday. H e ’d really rather n ot answer the question.

(A)

(A)

T he com pany was fou n d ed about a year ago. (B) It was just established that he could go into business. (C) T he family is well established. (D) T he business only lasted a year.

(B) (C) (D) 9.

(A)

H e did not look at the right schedule. (B) T h e plane landed in the right place. (C) T h e plane arrived on time. (D) H e had to wait for the plane to land. (A) (B)

Sh e’d rather go running. She d oesn ’t want to g o into the pool. (C) Sh e’ll change cloth es quickly and go swimming. (D) She needs a sweatsuit to go running.

(A) (B) (C) (D)

10.

(A)

It profited from previous mistakes. It earned a lot o f money. This was the last year that it would make a profit. It was not so successful.

Chuck’s bank account has too m uch m oney in it. (B) H e thinks Chuck has the wrong kind o f bank account. (C) H e thinks that Chuck is on his way h om e from the bank. (D) There isn’t enough m oney in Chuck’s account.

LISTENING COMPREHENSION

S k i l l 3:

A V O ID SIM ILAR SO U N D S

O fte n th e in c o r r e c t answ ers in L iste n in g P art A are answ ers th a t c o n ta in w ord s w ith sim i­ lar s o u n d s b u t very d iffe r e n t m e a n in g s fro m w h a t y o u h e a r in th e r e c o r d in g p rogram . You sh o u ld d e fin ite ly a void th e se answers.

Exam ple On the recording, you hear: (man) (woman) (narrator)

Why couldn't Mark com e w ith us? He was searching for a new apartment. What does the w om an say about Mark?

In your test book, you read: (A) (B) (C) (D)

He was in the department office. He was looking for a place to live. He was working on his research project. He had an appointment at church.

T h e k ey w o rd s in th e s e c o n d lin e o f th e c o n v e r sa tio n are searching a n d apartm ent. In a n ­ sw ers (C ) a n d (D ) th e w ord s research a n d church s o u n d lik e search, so th e se answ ers are in ­ c o r r e c t. In answ ers (A ) a n d (D ), th e w ord s departm ent a n d appointm ent so u n d like apartm ent, so th e se answ ers are in c o r r e c t. T h e b e st an sw er is th e r e fo r e answ er (B ). T h e fo llo w in g ch a rt o u tlin e s a very im p o r ta n t strategy fo r L iste n in g Part A: STRATEGY #3: AVOID SIMILAR SOUNDS 1. Identify key words in the second line of the conversation. 2.

Identify words in the answers that contain similar sounds, and do not choose these answers.

NOTE: InAppendix A there are drills to practice distinguishing similar sounds. You may want to complete these practice drills before trying the following exercises.

E X E R C ISE 3: In this e x e r c ise , u n d e r lin e key w ord s in th e s e c o n d lin e o f e a c h sh o rt c o n ­ v e rsa tio n . T h e n u n d e r lin e w ord s w ith so u n d s sim ila r to th e se key w ord s in th e answ ers, an d c h o o s e th e b e st an sw er to e a c h q u e s tio n . R e m e m b e r th a t th e b e st an sw er is probab ly th e an sw er th a t d o e s n o t c o n ta in w ord s w ith so u n d s th a t are sim ilar to th e so u n d s o f th e key w o rd s in th e s e c o n d lin e o f th e c o n v e rsa tio n . 1.

(wom an)

I heard that Sally jiust moved into a new, big house near the beach. (m an) But Sally doesn '1 have a cent! (narrator) What does the man mean?

(A) (B) (C) (D)

Sally has no sense o f responsibility. Sally sent her friend to the house. Sally has n o money. Sally is on the set with her.

LISTENING PARTA

(woman) (man) (narrator)

(man) (woman) (narrator)

Did they get the new car they wanted ? No, they lacked the money. What does the man mean ?

They locked the map in a car. They looked many times in the car. It cost a lot o f m oney when the car leaked oil. (D) They d id n ’t have en ou gh m oney to buy another car.

Have you finished packing yet? You should call the porter to get the suitcases. What does the woman mean ?

(A) (B) (C)

(A) (B) (C)

(D)

It’s im portant to pack the suitcases. They n eed help carrying their bags. T he man should pack his suit in case he needs it. The suitcases are quite portable.

T O E F L E X E R C ISE 3: In th is e x e r c ise , liste n ca re fu lly to th e sh o r t c o n v e r sa tio n a n d q u e s­ tio n in th e r e c o r d in g p r o g r a m , a n d th e n c h o o s e th e b e st answ er to th e q u e s tio n . You sh o u ld b e c a refu l to a void answ ers w ith sim ila r so u n d s. N O W BEGIN THE RECORDING PROGRAM AT TOEFL EXERCISE 3.

1.

2.

(A) (B)

She has to wait for som e cash. T he waiter is bringing a glass o f water. (C) The lawn is too dry. (D) She n eed s to watch out for a crash.

(A) (B) (C) (D)

3.

(A) (B) (C) (D)

4.

(A) (B) (C) (D)

5.

The sweater’s the wrong size. The m an ’s feet aren ’t sweating. The sweater makes the m an seem fat. The sweet girl d o e sn ’t feel right. He has been regularly using a computer. He com m unicates with a Boston company. He regularly g oes to com m unities around Boston. H e has been traveling back and forth to Boston. He thought the lesson d id n ’t matter. H e c o u ld n ’t learn the lesson. H e learned a massive num ber o f details. He d id n ’t like m ost o f the lesson.

(A) Som e animals started the first fire. (B) Anim als are killed by forest fires. (C) In the first frost, animals die. (D) Frost can kill animals.

6.

(A) (B) (C) (D)

Twenty pairs o f sh oes are on sale. The sh oe salesclerk spent twenty dollars on pears. The shoes cost twenty dollars. The shoes could be repaired for twenty dollars.

7.

Tom tended to dislike biology lab. Attendance wasn’t necessary at biology lab. (C) Tom w ent to biology lab. (D) There was a tendency to require biology lab.

8.

(A) (B)

(A) (B)

(C) (D) 9.

(A)

10.

(A) (B)

The m eal will be served at noon. The m ales should be driven there by noon. H e ’s expecting the ice to m elt before noon. The letters ough t to be delivered at 12:00.

T he w eather will probably get worse later. (B) The newspaper headlines described a bad storm. (C) There was news about a headstrong man. (D) He had a new bed.

(C) (D)

If she could do the grocery shopping. If she prefers cooked vegetables or salad. If she could help prepare the salad. If she minds sh opp in g for vegetables.

LISTENING COMPREHENSION

T O E F L E X E R C ISE (S k ills 1 -3 ): In th is e x e r c is e , listen carefu lly to th e sh o rt c o n v ersa tio n a n d q u e s tio n in th e r e c o r d in g p ro g ra m , a n d t h e n c h o o s e th e b e st answ er to th e q u estio n .

D

N O W BEGIN THE RECORDING PROGRAM AT TOEFL EXERCISE (SKILLS 1-3).

(A) (B) (C) (D) 2.

She would prefer a sunny day. The park is too crowded. She would like a place that is not so loud. (D) She cannot walk because sh e’s too old.

(D)

H e should open an account. He should take a ride on a ship. H e should try to keep the cost cheap. H e should try som ething m onoton ous to get to sleep.

The departm ent is not changing the requirements. (B) H e hasn’t heard anything about the change. (C) The changes are believable. (D) What has happened is incredible to him.

(A)

8.

(A)

9.

(A)

10.

(A)

(A)

The wait has taken close to an hour. (B) They were stranded in their car. (C) Most o f the people have been in line for hours. (D) They made a line in the sand.

5.

(A)

6.

(A)

The instructor is selecting several passages. (B) T he conductor is fair to the passengers. (C) The stamp collector is conducting his business. (D) The riders are paying for the train trip.

The managers will take the train to the program. (B) A program to develop new managers will com m ence soon. (C) The new m anagem ent program is very weak. (D) The program will be maintained to the letter.

7.

(A) (B) (C)

(A) (B) (C)

4.

He would like som e iced coffee. H e wants to stop drinking coffee. A drink seem s like a good idea. H e needs to drink som ething to stop his coughing.

The fire started to attack the building. (B) T he firefighter stared at the attacker. (C) The fire probably began at the top o f the building. (D) The firefighter started to attack the fire. H e assured the woman that he knew the truth. (B) He is sure that it isn’t new. (C) He thought that the woman was aware o f what happened. (D) H e soon will know the truth. The art professor is not one of his fans. (B) His drawings were amazing. (C) The catches that h e made were fantastic. (D) His sketches showed a fantasy world.

LISTENING PARTA

W H O ,W H A T , W H E R E -----------------------------------------------------------------S k ill

4:

D R A W C O N C L U S IO N S A B O U T WHO, WHAT, W H ER E

It is c o m m o n in L iste n in g P art A to ask y o u to draw s o m e k in d o f c o n c lu s io n . In th is type o f q u e s tio n th e an sw er is n o t clea rly stated ; in ste a d y o u m u st draw a c o n c lu s io n b a sed o n c lu e s g iv e n in th e c o n v e rsa tio n . O n e k in d o f c o n c lu s io n th a t is c o m m o n in this p art o f th e test is to ask y o u to d e te r m in e who th e sp ea k er is, b a sed o n c lu e s g iv e n in th e c o n v e rsa tio n .

Exam ple On the recording, you hear: (woman) (man) (narrator)

Can you tell me w hat assignm ents I m issed when I was absent from your class? You m issed one hom ework assignm ent and a quizWho is the man?

In your test book, you read: (A) (B) (C) (D)

A newspaper editor. A police officer. A teacher. A student.

T h e c lu e s y o u r class, homework, a n d q u iz in th e c o n v e r sa tio n te ll y o u th a t th e m a n is p r o b a ­ bly a teach er. A n sw er (C ) is th e r e fo r e th e c o r r e c t answer. A n o th e r type o f c o n c lu s io n th a t is c o m m o n in L iste n in g P art A is to d e te r m in e w h a t w ill p rob ab ly h a p p e n n e x t, b a sed o n c lu e s g iv e n in th e c o n v e rsa tio n .

Exam ple On the recording, you hear: (woman) (man) (narrator)

Are you going to read those books here in the library? I think I'd rather check them out now and take them home. What w ill the m an probably do next?

In your test book, you read: (A) (B) (C) (D)

Sit down in the library. Look for some more books. Return the books to the shelves. Go to the circulation desk.

T h e m an says th a t h e w o u ld lik e to check the books out now. S in c e th e circulation desk is w h e r e y o u g o to c h e c k b o o k s o u t fr o m a library, th e m an w ill p ro b a b ly g o to th e c ir c u la tio n d e sk n e x t. T h e c o r r e c t an sw er is th e r e fo r e an sw er (D ).

LISTENING COMPREHENSION

A fin a l type o f c o n c lu s io n th a t is c o m m o n in L iste n in g Part A is to d e te r m in e where th e c o n v e r sa tio n p ro b a b ly takes p la c e , b a sed o n c lu e s g iv e n in th e c o n v e rsa tio n .

Exam ple On the recording, you hear: (woman) (man) (narrator)

Are you going into the water, or are you just going to lie there on the sand? I think I need to p u t on som e suntan lotion. Where does this conversation probably take place?

In your test book, you read: (A) (B) (C) (D)

At a beauty salon. At the beach. In a sandbox. At an outdoor restaurant.

T h e c lu e s water, sa n d , a n d s u n ta n lotion in th e c o n v e r sa tio n tell y o u th at th is c o n v e rsa tio n p r o b a b ly takes p la c e at th e beach. A n sw er (B ) is th e r e fo r e th e c o r r e c t answer. T h e fo llo w in g ch a rt o u tlin e s th e key p o in t th a t y o u sh o u ld r e m e m b e r a b o u t th is type o f q u estio n : CONCLUSIONS ABOUT WHO, WHAT, WHERE It is common for you to be asked to draw one of the following conclusions in Listening Part A: 1.

WHO is probably talking?

2.

WHAT will s/he probably do next?

3.

W H ERE does the conversation probably take place?

E X E R C ISE 4: In th is e x e r c ise , r ea d th e sh o r t c o n v e r sa tio n an d q u e s tio n , u n d e r lin e the c lu e s th a t h e lp y o u answ er th e q u e s tio n , a n d th e n c h o o s e th e b e st answer. You w ill have to draw c o n c lu s io n s a b o u t who, what, a n d where. 1.

(m an) (wom an) (narrator) (wom an) (m an) (narrator)

I ’d like to deposit thừ check in my account, please. Would you like any cash back ? Who is the woman?

(A) (B) (C) (D)

H ave you deposited your paycheck yet ? No, but that’s next on my list of errands. What will the m an probably do next?

(A) (B)

A store clerk. A bank teller. An accountant. A waitress.

Earn his paycheck. Write a check for a deposit on an apartment. (C) Go to a bank. (D) Make a list o f errands to run.

LISTENING PARTA

(man) (woman) (narrator)

Did you get the bread, eggs, and milk? Now we need to stand in line at the checkout counter. Where does this conversation probably take place?

(A) In a restaurant. (B) At a bakery. (C) On a farm. (D) In a market.

T O E F L E X E R C ISE 4: In th is e x e r c is e , liste n ca refu lly to th e sh o r t c o n v e r s a tio n a n d q u e s ­ tio n in th e r e c o r d in g p ro g ra m a n d th e n c h o o s e th e b e s t an sw er to th e q u e s tio n . You w ill have to draw c o n c lu s io n s a b o u t who, w hat, a n d where. n

N O W BEGIN THE RECORDING PROGRAM AT TOEFL EXERCISE 4.

1.

(A) (B) (C) (D)

In In In In

2.

(A) (B) (C)

a photography studio. a biology laboratory. an office. the library.

6.

(A) On a playground. (B) In a parking lot. (C) At a zoo. (D) In a ph oto studio.

H e ’s a pilot. H e ’s a flight attendant. H e ’s a m em ber o f the grounds crew. H e works clearing land.

7.

(A) (B) (C) (D)

3.

Wash the dishes immediately. Use as many dishes as possible. Wash the dishes for as lon g as possible. (D) Wait until later to clean up.

8.

(A) In an airplane. (B) In a police car. (C) In a theater. (D) At a fireworks exhibit.

9.

4.

(A) In a (B) In a (C) At a (D) In a

(A) (B) (C)

5.

(A) (B) (C) (D)

(D) (A) (B) (C)

A A A A

bank. restaurant. service station. beauty salon.

salesclerk in a shoe store. sh oe repairperson. party caterer. salesclerk in a fixtures departm ent.

(D) 10.

R espond to the mail. Put the letters in a file. It depend s o n where the file is. File the answers she received to the letters.

Take care o f Bob. Invite Bob to dinner. Let Bob know that they accept his invitation. R espond to the w om an’s question.

(A) A pharmacist. (B) A dentist. (C) A teacher. (D) A business manager.

LISTENING COMPREHENSION

S k ill

5:

LISTEN FOR W HO A N D WHAT IN PASSIVES

It is so m e tim e s d iffic u lt to u n d e rsta n d who or w h a t is d o in g th e a c tio n in a passive se n ­ te n c e . T h is p r o b le m is o fte n te sted in L iste n in g P art A.

Exam ple On the recording, you hear: (man) (woman) (narrator)

Did Sally go to the bank this morning? Yes, she did. She got a new checking account. What does the w om an imply?

In your test book, you read: (A) (B) (C) (D)

Sally wrote several checks. Sally wanted to check up on the bank. A new checking account was opened. Sally checked on the balance in her account.

In this co n v ersa tio n , th e w om an u ses an active sta te m en t th at m ea n s that Sally opened a check­ in g account. T h e c o rrect answ er u ses th e passive stru ctu re th at a checking account was opened to exp ress th e sam e idea. T h e r e fo r e , the b est answ er to th e q u estio n above is answ er (C ). You sh o u ld n o te th e fo llo w in g a b o u t passive s e n t e n c e s in L iste n in g P art A: PASSIVE STATEMENTS

___________________________

1. If the conversation contains a passive statement, the answer to the question Is often an oct/ve statement. 2. If the conversation contains an active statement, the answer to the question is often a passive statement. NOTE: Check carefully who or what is doing the action in these questions.

E X E R C ISE 5: In th is e x e r c ise e a c h o f th e c o r r e c t answ ers is e ith e r a passive r e sta te m e n t o f an active se n te n c e or a n active r e sta te m e n t o f a passive se n te n c e . R ead th e sh o r t c o n ­ v ersa tio n an d u n d e r lin e th e key active o r passive sta te m e n t. T h e n read th e q u e stio n and c h o o s e th e b e st answ er to th e q u e stio n . B e c a re fu l a b o u t who an d w hat w ith th e se passives. 1.

(wom an) (m an) (narrator)

Alice needs to pay her tuition today. But her tuition has alreadybeen paid. What does the man imply T

(A) (B) (C) (D)

A lice’s education has paid off. A lice’s tuition needs to be paid. Alice has already paid her fees. Alice has already received the money.

LISTENING PARTA

(man) (woman) (narrator)

Have you been taking good care o f the lawn ? I watered it only this morning. What does the woman mean ?

(A) (B) (C) (D)

(man)

Did you hear the news about the child who was lost in the park ? (woman) Yes, and I heard that she was ju st found! (narrator) What does the woman mean ?

(A) (B) (C) (D)

She drank som e water on the lawn this m orning. She waited for him on the lawn this m orning. T he lawn has already b een watered today. She wanted a new lawn this morning. Som eone located the girl. She heard about the new park from the child. T he child found her lost pet. The child was the last o n e in the park.

T O E F L E X ER C ISE 5: In th is e x e r c is e , liste n c a re fu lly to th e sh o r t c o n v e r s a tio n a n d q u e s ­ tio n in th e r e c o r d in g p r o g r a m , a n d th e n c h o o s e th e b e st an sw er to th e q u e s tio n . You sh o u ld be p a rticu la rly c a refu l o f p assives. N ow (A) (B) (C) (D) 2.

3.

If the restaurant is on the corner. If the man would like to go to the restaurant. If the vegetables are fresh. If vegetarian food can be obtained.

H e admittfcd that h e wanted to go to law school in the fall. (B) T h e law school accepted him as a student. (C) T h e law professor adm itted that he would be a student in the fall semester. (D) H e would be adm itted to law school after the fall semester.

5.

(A) (B) (C) (D)

(A)

(A) (B) (C) (D)

4.

BEGIN THE RECORDING PROGRAM AT TOEFL EXERCISE 5.

Mark’s plants were cared for in his absence. Mark’s plan was to be out o f town. Mark was careful about his plans for the out-of-town trip. She was careful while Mark was gone.

T h e lights in the trees were destroyed in the storm. (B) T h e storm dam aged the trees. (C) T h e falling trees destroyed a store. (D) In the light the destruction o f the storm could be seen.

6.

(A)

7.

(A)

T he road the horses took was long and hard. (B) It was hard to find the hidden houses. (C) T he riders worked the horses too m uch. (D) It was hard for p eop le to ride the horses for long.

(B) (C) (D)

(A)

8.

She was broke from skiing. She w ent skiing in spite o f her accident. H er leg was hurt on a skiing trip. H er skis were broken in the m ountains.

(A)

H e d id n ’t want the coffee that the wom an ordered. H e wasn’t sure if the wom an wanted coffee. H e assum ed the wom an had ordered coffee. H e was unaware that coffee had already b een ordered.

T he car was in the left parking lo t at the airport. (B) T he friends parked their car at the airport. (C) The airport c o u ld n ’t hold a lot o f cars. (D) T here were a lot o f cars to the left o f the parking lot.

LISTENING COMPREHENSION

9.

(A) (B)

The students p ointed at Mac. Mac was present when the other students m ade the appointm ent. (C) The class representative suggested Mac to the other students. (D) Mac was chosen by his classmates to represent them.

S k ill

6:

10.

(A)

After the earthquake, the insurance com pany came out to inspect the damage. (B) The insurance company insisted that the building be repaired to m eet earthquake safety standards. (C) The inhabitants paid their premiums after the earthquake. (D) T he insurance company paid for the earthquake damage.

LISTEN FOR W HO A N D WHAT W IT H MULTIPLE N O U N S

W h e n th e r e is m o r e th a n o n e n o u n in a s e n te n c e in L iste n in g P art A , it is c o m m o n for th e answ ers to c o n fu s e w h ic h n o u n d o e s w hat.

Exam ple On the recording, you hear: (man) (woman) (narrator)

Do you know w ho is in the band now? I heard that Mara replaced Robert in the band. What does the w om an say about the band?

In your test book, you read: (A) (B) (C) (D)

Robert became a new member of the band. Robert took Mara's place in the band. Mara didn’t have a place in the band. Mara took Robert’s place in the band.

In th e w o m a n ’s r e s p o n se to th e m a n ’s q u e s tio n , sh e talks a b o u t tw o p e o p le (M ara a n d Robert), a n d th e se tw o p e o p le are c o n fu s e d in th e answ ers. B e ca u se M ara replaced Robert, th is m e a n s th at M ara is in th e b a n d an d R o b er t is n o t. T h e b e st answ er is th e r efo r e an sw er (D ). T h e fo llo w in g ch art o u tlin e s th e key p o in t th a t y o u sh o u ld r e m e m b e r a b o u t q u estio n s w ith m u ltip le n ou n s:

LISTENING PARTA

EX E R C ISE 6: In th is e x e r c ise , u n d e r lin e th e c o n fu s in g n o u n s in e a c h sh o r t c o n v e rsa ­ tio n . T h e n read th e q u e s tio n a n d c h o o s e th e b e st an sw er to th a t q u e s tio n . R e m e m b e r to th in k very ca re fu lly a b o u t w h o is d o in g w h at. (m an) (wom an) (narrator)

(m an) (woman) (narrator)

Why is Bill not at work thừ week? H is doctor made him take a week off. What does the woman mean ?

(A)

Why is P aul going back home this summer? H e ’s returning to Vermont fo r his sister’s wedding. What does the woman mean 1

(A)

T he doctor d ecid ed to take som e rime o ff from work. (B) The doctor told Bill he w asn’t too weak to work. (C) Bill was mad w hen the doctor took som e time off. (D) Bill took a vacation on his d octor’s orders.

(B) (C) (D)

3.

(man) (woman) (narrator)

D id you hear that John's uncle died? Yes, and John was named beneficiary in his uncle’s will. What does the woman mean ?

Paul is gettin g m arried this summer. Paul’s sister is returning from Verm ont to g e t married. Paul will be there when his sister gets m arried this summer. Paul’s sister is com in g to his wedding in Vermont.

(A) John received an inheritance when his un cle died. (B) It’s a b enefit that J o h n ’s nam e is the sam e as his u n cle’s. (C) John knows that his un cle will com e to the benefit. (D) J o h n ’s uncle gave him a b eneficial name.

T O E F L E X E R C ISE 6: In th is e x e r c is e , liste n ca refu lly to th e sh o r t c o n v e r s a tio n a n d q u e s ­ tio n in th e r e c o r d in g p r o g r a m , a n d th e n c h o o s e th e b e st an sw er to th e q u e s tio n . Y ou s h o u ld b e particu larly c a re fu l o f w h o is d o in g w hat.

n

N ow

BEGIN THE RECORDING PROGRAM AT T o e f l E x e r c is e 6.

(A) (B)

2.

The passenger waited at the corner. T he passenger looked for a taxi at the corner. (C) T he cab driver waited for the passenger. (D) T he passenger cornered the waiting taxi driver.

3.

(A)

4.

It was hard for her to hear Jane last night. (B) Jane gave a harp recital last night. (C) Jane was playing hard while she was hurt. (D) She played the harp last night for Jane.

(A) (B) (C) (D) (A) (B) (C) (D)

The baby sister w ent to bed quite early. The children w ere forced to go to bed early. The baby-sitter m ade the bed after the children g o t up. The baby-sitter did not stay up late. The m an taught his son about football. The boy is receiving the ball from his dad. T he ball is being tossed in to the air by the boy. The m an is playing with the ball in the sun.

LISTENING COMPREHENSION

The chairman decid ed that Tony would serve on the board for another year. (B) The chairman elected the board. (C) The board decided Tony could be chairman after o n e year. (D) Tony becam e chairman for one more year.

(A)

The students were told to go listen to the speaker. ' (B) T he professor attended that even in g’s lecture. (C) The students were given directions to the lecture. (D) The professor was directed to the lecture hall.

8.

(A)

T he m anager went to the supply room. (B) The clerk set supplies on the floor (C) The clerk went to the supply room at the m anager’s request. (D) T he clerk backed into the manager in the supply room.

9.

(A) (B)

10.

(A)

(A)

T he librarian was quite reserved with the students for two days. (B) W ithin two days the librarian had the books for the students. (C) T he librarian asked the students for the books. (D) T he students put the books on hold for two days. (A)

The ju d ge defended the murderer. The ju d ge tried to protect the defendant from the murderer. (C) The ju d ge said that the defendant was a criminal. (D) The defense cou ld n ’t make a jud gm en t about the criminal. The woman should announce the nam es o f the com m ittee members. (B) H e is thankful to be appointed to the com m ittee. (C) H e is sure about the tim e o f the appointm ent with the com mittee. (D) The woman will serve on the com mittee.

T O E F L E X E R C ISE (S k ills 4 - 6 ) : In th is e x e r c ise , listen carefu lly to th e sh o r t co n v e rsa tio n a n d q u e s tio n in th e r e c o r d in g p rogram , an d th e n c h o o s e th e b e st answ er to th e q u estio n . N o w BEGIN THE RECORDING PROGRAM AT TOEFL EXERCISE (SKILLS 4-6).

(A) In a departm ent store. (B) In a stationery store. (C) At the post office. (D) At the airport.

4.

(A) A delivery man. (B) A famous chef. (C) A clerk in a fast-food restaurant. (D) An airline steward.

T he teacher gave the students a hand. (B) T he term papers were turned in. (C) The students got the papers from the office. (D) T he teacher handed the papers to the students.

5.

(A)

6.

(A) (B)

2.

(A)

3.

(A)

The attendant checked the oil in Mark’s car. (B) Mark checked to see if he had en ou gh oil in his car. (C) Mark checked with the servicc station attendant. (D) Mark wrote a check to pay for the oil.

They n eed new print for the additional copies. (B) They can make extra copies if necessary. (C) Printers are n eeded for the additional copies. (D) Additional copies are needed immediately. The professor bought two books. The students had to purchase two books. (C) T he students sold two books to the professor. (D) The students were required to read two books by the professor.

LISTENING PARTA

7.

8.

(A) T he doctor returned to the office. (B) Jim asked the doctor to com e to the office. (C) T he doctor will not return until n ext week. (D) Jim was told to com e back. (A) (B) (C) (D)

Go to work in the lab. Sample the work from the lab. Have the sam ples delivered. Send a n ote to the lab.

9.

(A) (B) (C) (D)

10.

Mary becam e the new class president. Sue took her place as class president. In place o f Mary, Sue becam e senior class president. T he senior class president replaced Sue and Mary.

(A)

The panel was analyzed on the television program. (B) A com m ittee evaluated recent political events. (C) T he program featured a psychoanalyst. (D) T he panel discussed the television program.

T O E F L R EV IEW E X E R C ISE (S k ills 1 -6 ): In th is e x e r c ise , listen ca re fu lly to th e s h o r t c o n v e r s a tio n a n d q u e s tio n in th e r e c o r d in g p r o g r a m , an d th e n c h o o s e th e b e s t a n sw er to th e q u e s tio n . N O W BEGIN THE RECORDING PROGRAM AT TOEFL REVIEW EXERCISE (SKILLS 1-6).

1.

(A) (B) (C) (D)

2.

(A)

H e seem ed to be rather hungry. She was quite angry at him. H e was trying to hang the posters. She believes he was mad.

T h e parents are g oin g to stay up late. (B) T h e parents have given H annah her allowance. (C) Lately, the parents have not been so loud. (D) H annah does n ot have to go to bed early.

3.

(A) At a departm ent store. (B) At a service station. (C) At a collection agency. (D) In a delivery room.

4.

(A) She just broke som e eggs. (B) They n eed to eat fast. (C) She is serious about the boat. (D) H e has a ch oice to make.

5.

(A) (B) (C) (D)

It was urgent that Ellen do her best. H e really urged Ellen to do more. H e was encouraged by Ellen to try harder. Ellen told him that she was trying to do better.

6.

(A) T he car stalled on the road. (B) Som eo.ie took the car. (C) Rob sold his car. (D) Rob heard som eon e steal his car.

7.

(A) Buying the bigger container. (B) Putting the milk in the cart. (C) Taking a carton that is smaller. (D) G etting the milk tom orrow instead.

8.

(A)

9.

T h e police officer was stationed near the tourist. (B) T he tourist was forced to accom pany the p olice officer. (C) T he tourist becam e m ad at the police station. (D) T h e tourist stated that the p olice officer never cam e.

10.

T he receptionist w elcom ed the businesspeople. (B) T he man created a shipping and receiving business. (C) T he businesspeople were rather greedy. (D) T he businesspeople greeted the receptionist. (A)

(A) H e hasn’t seen her ideas. (B) It was a terrible deal. (C) H e d oesn ’t like the idea. (D) It sounds m agnificent to him.

LISTENING COMPREHENSION

N EGATIVES S k i l l 7:

LISTEN FOR NEG ATIVE EXPRESSIONS

N e g a tiv e e x p r e s sio n s are very c o m m o n in L iste n in g P art A, a n d th e m o st c o m m o n kin d o f c o r r e c t r e s p o n s e to a n e g a tiv e sta te m e n t is a p o sitiv e sta te m e n t c o n ta in in g a w ord w ith an o p p o s ite m e a n in g . Exam ple On the recording, you hear: (man) (woman) (narrator)

How did they get to their grandmother's house in Maine in only five hours? They didn't drive slowly on the trip to Maine. What does the w om an say about the trip?

In your test book, you read: (A) (B) (C) (D)

They drove rather quickly. They couldn’t have driven more slowly. They wanted to travel slowly to Maine. They didn’t drive to Maine.

T h e c o r r e c t an sw er is answ er (A ). I f th ey d id not d rive slowly to M ain e, th is m e a n s that th ey d r o v e ra th er quickly. N o tic e th a t th e c o r r e c t an sw er u ses quickly, th e o p p o s ite o f slowly. T h e answ ers th a t u se slowly are n o t co rr ec t. T h e fo llo w in g c h a r t o u tlin e s th e types o f n e g a tiv e e x p r e ssio n s th a t y o u sh o u ld be c a r e fu l of: TYPES OF NEGATIVE EXPRESSIONS Expression

Example

Correct Answer

Regular negative: not or n't

Tom is not sad about the results.

not sod = happy

Other negatives: nobody, none,

Nobody arrived on time. Sal never works hard.

nobody . . . on time = late never works hard = lazy

The patient was insane. ■ ' '

insane = not sane = crazy

nothing, never

■ Negative prefixes: un-, in-, dis­ ' . .. -

-

:

.

y

E X E R C ISE 7: In th is e x e r c ise , u n d e r lin e th e n e g a tiv e in th e se c o n d lin e o f e a c h sh o rt c o n v e r sa tio n . T h e n read th e q u e stio n an d c h o o s e th e b e st answ er to th a t q u estio n . R e m e m b e r th a t th e b e st answ er is o n e th at u se s an o p p o s ite m e a n in g . (m an) (wom an) (narrator)

I can’t seem to get the door unlocked. That isn't the right key fo r the door. What does the woman mean ?

(A) (B)

The key in the drawer is on the right. The man should write the message on the door. (C) The man has the wrong key. (D) The right key isn’t in the drawer.

LISTENING PARTA

2.

(m an) (woman) (narrator)

(woman) (m an) (narrator)

Were you pleased with last week's convention ? Nothing went as planned. What does the woman mean ?

T he convention was disorganized. She d id n ’t plan to attend the convention. (C) She planned the convention last week. (D) She wasn’t pleased with the last w eek o f the convention.

Are you planning to go to college next year? I ’m really unsure about the idea. What does the m an mean ?

(A)

(A) (B)

H e definitely wants to g o to college. (B) H e is certain about his plans. (C) H e ’s hesitant about attending college. (D) His idea is to go to college.

T O E F L E X E R C ISE 7: In th is e x e r c is e , lis te n carefu lly to th e sh o r t c o n v e r s a tio n a n d q u e s tio n in th e r e c o r d in g p r o g r a m , a n d th e n c h o o s e th e b e st an sw er to th e q u e s tio n . You sh o u ld b e p articu larly c a refu l o f n e g a tiv e e x p r e s sio n s.

o

N ow

BEGIN THE RECORDING PROGRAM AT TOEFL EXERCISE

(A) (B) (C)

She is very busy. She has lots o f free time. It is not necessary to take ou t the trash. (D) She will do it if she has time. 2.

(A) (B)

The service satisfied her. T he food was worse than the service. (C) She thought the service was bad. (D) N either the food nor the service was satisfying.

(A) (B) (C)

T h e interview is very im portant. H e is w orried about the interview. What h e ’s wearing to the interview is important. (D) H e is not concerned about the interview.

(A) (B) (C) (D)

(A) H e has alm ost all the notes. (B) H is attendance was perfect. (C) H e went to all the lectures but one. (D) H e missed m ore than one psychology class. 4.

(A)

6 :00 . T h e library opens at 6:00 in the summer. (C) T h e library closes at 6:00. (D) You can’t check out m ore than six books in the summer.

9.

(A) (B) (C)

Water the plants on ce a day. Give the plants no m ore water. Water the plants often while the m an is gone. (D) Give the plants a lim ited am ount o f water.

The project will take all their effort. They have n o other work to do. It’s im possible to finish. They aren’t even close to finishing the project.

(A) (B)

She d oesn ’t m ind an hou r m ore. S h e’d rather stay m ore than an hour. It’s better to stay than go. She prefers to leave.

(C) (D) 10.

H e told his kids to leave. H e seriously wanted the woman to g°He was joking when he told the wom an to leave. He left with the woman.

(A) (B) (C) (D)

T hey passed the library at

(B)

5.

7.

(A)

T he service at the hotel wasn’t too good. (B) This hotel gav< excellen t service. (C) T he service at the h otel could have b een improved. (D) This h o tel’s service was the sam e as the service at other hotels.

LISTENING COMPREHENSION

LISTE N FOR DOUBLE N EG A TIVE EXPRESSIONS

S k i l l 8:

It is p o s sib le fo r tw o n e g a tiv e id e a s to a p p ea r in o n e s e n te n c e , an d th e r esu lt can b e q u ite c o n fu s in g . E x am p le On the recording, you hear: (man) (woman) (narrator)

I c a n ’t believe the news that I heard about the concert. Well, it isn't impossible for the concert to take place. What does the wom an say about the concert?

In your test book, you read: (A) (B) (C) (D)

There’s no possibility that the concert will take place. The concert will definitely not take place. The concert might take place. The concert can't take place.

T h e c o r r e c t an sw er to th is q u e s tio n is answ er (C ). I f it i s n ’t impossible fo r th e c o n c e r t to take p la c e , th e n it is possible, a n d th e m o d a l m ight in d ic a te s possibility. T h e fo llo w in g ch art o u tlin e s th e situ a tio n s w h e r e d o u b le n eg a tiv es c a n occu r: DOUBLE NEGATIVES Meaning

Example

Situation negative word (e.g., not, no, none) and a negative prefix (e.g., in-, un-, dis-)

He didn’t like the unclean office.

did not like unclean office = liked clean office

two negative verbs

It isn't snowing, so they aren't going to the mountains.

implies that they would go if it were snowing

neither or not... either

Sue didn’t like the movie, and neither did Mark.

both did not like the movie

E X E R C IS E 8: In this e x e r c ise , u n d e r lin e th e tw o n e g a tiv es in th e se c o n d lin e o f each sh o r t c o n v e r s a tio n . T h e n read th e q u e s tio n a n d c h o o s e th e b est answ er to that q u estio n . R e m e m b e r th a t tw o n e g a tiv e s m ak e th e s e n te n c e p o sitiv e. (m an) (woman) (narrator)

Paula, you worked so hard setting up the field trip. I hope no one’s unhappy with the arrangements. What does Paula mean ?

(A)

She hopes everyone will be pleased. (B) She knows no on e is happy with what she has done. (C) Sh e’s arranged to take a trip because she’s unhappy. (D) Everyone’s happy with the condition o f the field.

LISTENING PARTA

2.

(wom an) (m an) (narrator)

How was your history exam? I d id n ’t study enough, so I didn ’t do well. What does the man mean?

(A) (B) (C) (D)

3.

(m an) (wom an) (narrator)

Were your friends able to gel tickets fo r the concert ? Mark couldn't gel tickets fo r the concert, and neither could Paul. What does the woman mean?

(A) (B) (C) (D)

H e studied a lot and passed. H e failed in spite o f his effort. H e got a g ood grade even though he d id n ’t study. His grade was poor because o f inadequate preparation. A lthough Mark c o u ld n ’t get both tickets, Paul did. Both were unable to obtain tickets. N either Mark nor Paul wanted to go to the concert. Mark tried to get tickets, but Paul didn’t.

T O E F L E X E R C IS E 8: In th is e x e r c ise , lis te n carefu lly to th e sh o r t c o n v e r sa tio n a n d q u e s­ tio n in th e r e c o r d in g p ro g ra m , a n d th e n c h o o s e th e b e st answ er to th e q u e s tio n . You s h o u ld b e p articu larly c a refu l o f d o u b le n e g a tiv es.

n 1.

2.

N O W BEGIN THE RECORDING PROGRAM AT TOEFL EXERCISE 8.

(A) (B) (C) (D)

H e ’ll definitely be elected. T he election is now com plete. She has high hop es for his chances. It may happen.

(A) (B) (C)

Both parts o f his game were bad. H e served better than he volleyed. Som e parts o f his gam e were better than others. He played rather well.

(D) 3.

(A) (B) (C) (D)

4.

(A) (B) (C) (D)

5.

(A) (B) (C) (D)

6.

(B) (C) (D) 7.

It is a surprise that he was prepared. H e was not ready, as usual. H e prepared a really big surprise. His strong preparation cam e as no surprise. She felt good enough to go out. She w ent out to get som e m edicine. She felt like dancing, so she went out with everyone. She stayed hom e because she was sick. She has problem s that others aren’t aware of. O thers aren’t aware o f her problem s. She knows sh e’s b een a problem . She d o e sn ’t have a care in the world.

(A)

(A) (B) (C) (D)

8.

(A) (B) (C) (D)

Steve wanted to finish his paper, and so did Paul. Both Steve’s and Paul’s papers were incom plete. Steve and Paul were busy doin g their term papers. W hen Steve wasn’t able to finish his paper, Paul c o u ld n ’t help. It wasn’t G eorge’s responsibility to pay the bill. Bill was irresponsible about paying G eorge’s rent. G eorge acted carelessly by not taking care o f the bill. George took responsibility for the unpaid bill. It’s fortunate that he was accepted. It’s good that he wasn’t adm itted. Fortunately, the university d id n ’t admit him. It’s too bad he was rejected.

LISTENING COMPREHENSION

9.

T h e first essay was better than the second. (B) T h e first and second drafts co u ld n ’t b e better. (C) T h e second draft o f the essay was m uch better than the first. (D) Both versions were poorly written.

(A)

S k i l l 9:

10.

Roger has been bothered. Roger wasn’t the least bit disturbed. (C) T he problem s have had little i on Roger. (D) Roger hasn’t b een disturbed.

(A) (B)

E XPRESSIONS LISTE N FOR “A L M O S T N E G A TIV E ” 1

C e rta in e x p r e s sio n s in E n g lish have “a lm o st n e g a tiv e ” m e a n in g s. T h e s e e x p r e s sio n s are c o m m o n o n th e T O E F L te st a n d n e e d to b e r ev iew ed .

Exam ple On the recording, you hear: (woman) (man) (narrator)

Were you able to pay the electric bill? I had barely enough money. What does the m an imply?

In your test book, you read: (A) (B) (C) (D)

He had plenty of money for the bill. He did not have enough money for the bill. He paid the bill but has no money left. He was unable to pay the bill.

In th e m a n ’s sta te m e n t, th e w ord enough in d ic a te s th a t th e r e was enough, so h e d id pay the bill. H ow ever, it was barely e n o u g h , so h e a lm o st d id n o t have e n o u g h a n d certain ly has no money left. T h e c o r r e c t answ er is th e r e fo r e a n sw er (C ) . T h e fo llo w in g c h a r t o u tlin e s c o m m o n “a lm o st n e g a tiv e ” ex p r essio n s: COMMON "ALMOST NEGATIVE” EXPRESSIONS Meaning

Expression

Example

almost none

hardly, barely, scarcely, only

There is hardly any food in the refrigerator.

almost never

rarely, seldom

He rarely drives to work. ■

E X E R C ISE 9: In th is e x e r c ise , u n d e r lin e th e “a lm o st n e g a tiv e ” e x p r e ssio n in th e se c o n d lin e o f e a c h sh o r t c o n v e rsa tio n . T h e n read th e q u e s tio n a n d c h o o s e th e b e st answer. R e­ m e m b e r th a t th e b e s t answ er is o n e th a t m e a n s th a t it is true b u t it is almost not tru e. (m an) (woman) (narrator)

I hear that M ona’s been offered the manager’s job. B ut she has hardly any ivork experience! What does the woman say about M ona?

(A) M ona hasn’t worked hard. (B) M ona’s experience has been hard. (C) M ona’s job as manager is hard. (D) M ona hasn’t worked for very long.

LISTENING PARTA

2.

(w o m a n )

(m a n ) (n a rra to r)

3.

(woman) (man) (narrator)

How much time did Sam spend on his paper for economics class? Sam has seldom taken so much time on a research paper. What does the m an mean ?

(A)

Sam usually spends this m uch tim e on his schoolwork. (B) Sam has rarely worked so hard. (C) Sam took too m uch tim e on this paper. (D) Sam sh ould’ve worked harder o n this paper.

Does Steve usually park his car there? Only once has he parked his car in that lot. What does the m an mean ?

(A) (B) (C) (D)

H e parks his car there o n ce in a while. H e’s parked his car there a lot. H e only leaves his car there for short periods o f time. H e left his car there on ju st one

T O E F L E X E R C ISE 9: In th is e x e r c is e , lis te n ca refu lly to th e sh o r t c o n v e r s a tio n a n d q u e s­ tio n in th e r e c o r d in g p r o g r a m , a n d t h e n c h o o s e th e b e st an sw er to th e q u e s tio n . Y ou sh o u ld b e p articu larly c a refu l o f “a lm o st n eg a tiv e" e x p r e ssio n s. N O W BEGIN THE RECORDING PROGRAM AT TOEFL EXERCISE 9.

1.

(A) (B) (C) (D)

2.

(A)

3.

4.

T h ere’s litde rain in July. In July it never rains. It rains hard in July. W hen it rains in July, it rains hard.

T h e university accepted three students. (B) N on e o f the students is goin g to the university. (C) John was n ot accepted. (D ) Two were n ot adm itted. A lthough he did pass, Mark’s exam grade wasn’t too good . (B) Mark failed his history exam. (C) T he highest grade on the history exam w ent to Mark. (D) Professor Franks d id n't pass Mark on the history exam.

5.

(A)

6.

(A) (B) (C) (D)

(A)

H e often has lon g waits in Dr. Roberts’ office. (B ) He must wait p auendy for Robert. (C) Dr. Roberts is generally punctual. (D) He d o e sn ’t m ind waiting for Dr. Roberts.

Betty o ften tr.kes vacations in winter. ;' Then, she did read it. Then, he has gone there.

Meaning 1 thought he was not here. 1thought you could not go. 1thought you did not play tennis. 1 thought she had not read it. 1thought he had not gone there.

LISTENING COMPREHENSION

E X E R C ISE 13: In th is e x e r c ise , u n d e r lin e th e e x p r e s sio n o f e m p h a tic su rp rise in e a c h sh o r t c o n v e r s a tio n . T h e n read th e q u e s tio n a n d c h o o s e th e b e st answ er to th a t q u e stio n . R e m e m b e r th a t th e b e st answ er is o n e th a t sh ow s su rp rise. (man) (wom an) (narrator)

2.

(wom an) (man) (narrator)

(man) (wom an) (narrator)

I ju st got 600 on the TOEFL test! Then you did pass. What had the woman assumed?

(A) (B) (C) (D)

Would you like to go skiing this weekend ? So you can ski! What had the man assumed?

(A) (B)

I ju st got this letter from my sister. So the mail has come already. What had the woman assumed?

(A)

The man had n ot passed. T he man would pass easily. The man had already passed. The man got the score he was expected to get.

The woman was a good skier. T he woman was going skiing this weekend. (C) The woman did n ot know how to ski. (D) T he woman did n ot intend to go skiing. The m an’s sister never wrote to him. (B) T he mail had not yet arrived. (C) The mail always cam e early. (D) The mail had already arrived.

T O E F L E X ER C ISE 13: In th is e x e r c ise , listen carefu lly to th e sh o rt c o n v e rsa tio n a n d q u e s tio n in th e r e c o r d in g p rogram , an d th e n c h o o s e th e b e st answ er to th e q u e s tio n . You sh o u ld b e p articu larly carefu l o f e x p r e s sio n s o f e m p h a tic su rp rise. o

N O W BEGIN THE RECORDING PROGRAM AT TOEFL EXERCISE I 3.

Greg always com es to parties. Greg would com e to the party later. Greg was unable to attend the party. (D) Greg would stay at the party for only a m om ent.

(A) (B) (C)

T he woman always rode her motorcycle to school. (B) The woman was not com ing to school today. (C) T he woman was an expert m otorcycle rider. (D) T he w oman did not know how to ride a motorcycle.

(A)

5.

(A) (B) (C)

6.

(A) (B) (C) (D)

(A)

T he man was n ot a very good cook. The man never invited friends over for dinner. (C) T he man would never invite him over for dinner. (D) T he man was an excellen t cook.

(A) (B)

The woman had run more than three miles. (B) T he woman always got lots o f exercise. (C) The woman ran for three hours in the morning. (D) The woman had not gotten much exercise.

4.

H e had been som ewhere else. He had been in the library. H e had been working on his research project. (D) H e would start working on his project in five hours. He had changed apartments. He did not like his new apartment. H e was still in his old apartment. H e had moved from a house to an apartm ent

LISTENING PARTA

T he wom an did not like desserts. T he woman ate sweets regularly. T he w om an would n ot share her chocolate cake. (D) The woman had eaten his p iece o f cake.

(A) (B) (C)

9.

(A) (B) (C)

10.

(A) (B) (C)

(D)

T he man was goin g to study hard. T h e man already had a driver’s license. (C) T he man would n ot take the test. (D) The man had already taken the test (A) (B)

She had registered in physics. She would go to physics class later. She had already taken a physics class. She had not enrolled in physics.

T he pipes were n ot clear. The plum ber w ould be late. T he plum ber had already cleared the pipes. (D) T he pipes did n o t n eed to be cleared.

T O E F L EXERCISE (Sk ills 1 1 -1 3 ): In th is ex er cise , listen carefully to th e sh o rt co n v e rsa tio n a n d q u estio n in th e r ec o r d in g p rogram , a n d th e n c h o o s e th e b est answ er to th e q u estio n .

n

3.

4.

N O W BEGIN THE RECORDING PROGRAM AT TOEFL EXERCISE (SKILLS I 1-13).

(A) She plans to talk a lot this m onth. (B) She has a lo t to say about the p h on e bill. (C) T he bill is high because she has a lot to say. (D) She agrees with the m an.

6.

(A) He would be glad to say it over again. (B) H e would like the woman to repeat what she said. (C) H e says that h e w ould like to take the class again. (D) H e ’s happy the class is over, too.

(A) Bill had never really b een sick. (B) Bill was too sick to com e to class. (C) Bill was sick o f calculus class. (D) Bill had forgotten about the calculus class that m orning.

7.

(A) H e finifned all the problem s. (B) H e d o esn ’t believe what the wom an said. (C) H e was able to finish som e o f the problem s. Both he and the w oman were unsuccessful o n the math problem s.

8.

(A) T he man had m ailed the package. (B) T he man had forgotten to go to the post office. (C) T he m an had given the package to the wom an to mail. T he m an rem em bered the package ' after he went to the p ost office.

9.

(A) They should take both cars. (B) T he woman sh ould try n o t to be afraid. (C) The woman should buy a bigger car. (D) They should go together in his car.

10.

(A) H e wants to know if the m uffins taste good. (B) H e thinks the m uffins were recently prepared. (C) T he m uffins are not really fresh. (D) H e’s sure that the m uffins were ju st m ade.

(A) (B)

T he man sh ould go o u t tonight. T he man sh ould stay h om e and relax. (C) T h e man should work on the paper tonight. (D) T h e man should go ou t M onday instead. (A)

T he cafeteria was op en in the m o r n in g .

(B) (C) (D) 5.

(A)

T h e cafeteria did not serve breakfast. T h e breakfast in the cafeteria was n ot very tasty. T he woman never ate breakfast in the cafeteria.

H e believes that it is acceptable to park there. (B) T h e parking lot is too far from their destination. (C) H e knows that they w on ’t g et a ticket. (D) H e knows w here the parking lo t is.

LISTENING COMPREHENSION

T O E F L REVIEW EXERCISE (Skills 1-13): In this exercise, listen carefully to the short conver­ sation an d q u estion in the record in g program , a n d th e n c h o o se the b est answer to the question. N O W BEGIN THE RECORDING PROGRAM AT TOEFL R e v ie w E x e r c is e (S k ills I -

1.

(A) Write a message to the man. (B) Make som e p h one calls. (C) R espond to the m an’s questions. (D) Get a new phone installed.

2.

(A) (B) (C) (D) (A) (B) (C)

4.

(A) (B) (C) (D) (A) (B) (C) (D)

6.

H e couldn't find Paula’s ph one number, so he d id n ’t call her. (B) H e couldn’t give Paula the list over the phone. (C) W hen he went to call Paula, he cou ld n ’t find the list. (D) H e co u ld n ’t recollect the number that was on the list.

8.

(A) She co u ld n ’t take her luggage to the store. (B) She stored her luggage at the train station. (C) She carried her luggage from the train station to the store. (D) There were no lockers for her bags.

9.

(A)

H e barely rode the bicycle. H e d id n ’t have enough money. The bicycle did n’t need to be paid for. (D) H e paid for the bicycle.

3.

5.

S he’s not sure if sh e’s free. S h e’s marked it on her calendar. She’ll write a check for the calendar. H er calendar says she has to have a m eeting at 3:00.

7.

She fixed the television. Bob m ade the television work. The woman looked at Bob on television. Bob works for the woman. H e helped her say what she co u ld n ’t say. She was unable to say anything about him. He hasn't helped her very much. What he said was very helpful.

The man should spend more time on registration. (B) The man should walk m ore quickly through registration. (C) The m an should send in his registration materials. (D) The man should try to avoid registering next semester.

(A)

(A)

T he woman had taken a different major. (B) T he woman had chosen psychology as a major. (C) T he woman was uninform ed. (D) The woman n eeded to see a psychiatrist

10. (A)

She would like the man to repeat what he said. (B) She thinks the exam could have been a litde m ore difficult. (C) She shares the same opinion o f the exam as the man. (D) She believes that the exam was easy.

C O N TR A R Y M E A N IN G S _____ S k ill

14:

LISTEN FOR W ISH ES

C o n v er sa tio n s a b o u t w ish es can a p p ear in L iste n in g P art A. T h e im p o r ta n t id e a to re­ m e m b e r a b o u t w ish es is th at a w ish im p lie s h at the opposite o f the w ish is true.

LISTENING PARTA

E x am p le On the recording, you hear: (woman) (man) (narrator)

It's too bad that y o u have to stay here and work during the school break. I really w ish I could go w ith you and the others to Palm Springs. What does the m an mean?

In your test book, you read: (A) (B) (C) (D)

Maybe he will go with the others on the trip. He is unable to go on the trip. He's happy to be going on the trip. He’s going on the trip, but not with the others.

In th is c o n v e r sa tio n th e m a n wishes th a t h e could go w ith th e o th e r s o n th e trip , so th e im ­ p lie d m e a n in g is th a t h e is unable to go. T h e c o r r e c t an sw er is th e r e fo r e an sw er (B ). T h e fo llo w in g c h a r t o u tlin e s th e key p o in ts th at y o u sh o u ld k n o w a b o u t w ishes: KEY INFORMATION ABOUT WISHES Point • An affirmative wish implies a negative reality. ‘ A negative wish implies an affirmative reality.

Example

Meaning .••M.K.-.-.&ir.:,’ • = no time to help

1 wish 1 had time to help. .

: ’ ‘ • ■' v v 1wish 1 did not have time to help. • ' ■ ■• 1 wish he were at hom e*

• A past tense verb implies a present reality. A . • A past perfect tense verb implies a past reality.

;

. ... . .•■■ .

■. : 1 wish he had been at home. ; \ . - ... .ft:

' =^ time ' to

ii". help

■ = is not at home ..•’v. -v.. . = was not at home : -J . ^.. .'■ '

•Remember that were is used instead of was in wishes: “I wish I were going." E X E R C ISE 14: In th is e x e r c is e , u n d e r lin e th e w ish in e a ch sh o r t c o n v e r sa tio n . T h e n read th e q u e s tio n a n d c h o o s e th e b est an sw er to th a t q u e s tio n . R e m e m b e r th a t th e b e st an sw er is o n e th at im p lie s th e o p p o s ite o f w h a t is sa id . (man) (woman) (narrator)

(woman) (m an) (narrator)

Do you think we’U be able to have the picnic today ? / wish the sky weren 't so cloudy. What does the woman mean ?

T he sky is n ot very cloudy. T he sky yesterday was clou dier than it is today. (C) T he sky is too cloudy. (D) T h e sky is rather clear.

Did you enjoy the Thanksgiving dinner? I wish I h a d n ’t eaten so much. What does the man mean ?

(A) (B) (C) (D)

(A) (B)

He He He He

d id n ’t eat very m uch. plans on eating a lot. thinks he is eating a lot. ate too much.

LISTENING COMPREHENSION

3.

(man) (wom an) (narrator)

(A) She is com ing to the party. (B) She m ight com e to the party. (C) She will try to com e to the party. (D) She is n o t com in g to the party.

Are you coming to the party tonight? I wish I could. What does the woman mean?

T O E F L E X ER C ISE 14: In th is e x e r c ise , listen carefu lly to th e sh o r t c o n v e r s a tio n an d q u e s tio n in th e r e c o r d in g p rogram , a n d th e n c h o o s e th e b e st a n sw er to th e q u e s tio n . You s h o u ld r e m e m b e r th a t a w ish im p lie s a n o p p o s ite m e a n in g . N O W BEGIN THE RECORDING PROGRAM A T TOEFL EXERCISE 14.

1.

(A) (B)

The line is short. There are not very many p eople in front o f them. (C) The line in front o f them is too long. (D) N ot many p eople want to get tickets to the concert.

2.

(A)

3.

(A)

4.

(A)

The woman told him about the ticket. (B) He wanted the woman to get a ticket. (C) H e was happy to find ou t about the ticket. (D) The woman did not tell him about the ticket. She is not working too many hours next week. (B) She d oesn ’t have enough hours n ext week. (C) She is working too many hours n ext week. (D) She likes working so m uch.

6.

(A)

7.

(A)

8.

(A)

9.

(A) (B)

10.

(A)

T he departm ent did n ot change the requirements. (B) She likes the new requirem ents. (C) She changed her apartment just b e f o r e g ra d u a tio n .

(D )

S h e d o e s n o t lik e th e c h a n g e s th a t th e d e p a r t m e n t m a d e .

5.

H e is going to the theater. He d o esn ’t have enough money. He isn’t afraid to go. (D) H e d o esn ’t want to spend the money.

(A) (B) (C)

Harry did not prepare e n ou gh for the exam. (B) Harry studied hard for the exam. (C) He has n ot heard anything about Harry. (D) He had a bet with Harry. The algebra course that she is taking is n ot her favorite. (B) She d oesn ’t n eed to take the algebra course. (C) She has a good schedule o f courses this semester. (D) She’s good at m ath, but sh e ’s taking the algebra course anyway. H e was able to find a cheap apartment. (B) His apartment is too expensive. (C) H e d oesn ’t like th e apartm ent’s location. (D) The apartm ent is cheap because o f its location. He arrived early at the auditorium. He got o n e o f the best seats in the auditorium. (C) H e was n ot early en ou gh to get a seat at the front. (D) H e prefers sitting at the back. H e’d like to work on his social skills at the game. (B) He wishes he could work on his term paper for sociology. (C) H e can’t attend the gam e because o f his schoolwork. (D) Sociology is less im portant to him than football this weekend.

LISTENING PARTA

S k ill

15:

LIS TE N FOR U N T R U E C O N D IT IO N S

C o n v er sa tio n s c o n ta in in g c o n d itio n s c a n a p p e a r in L iste n in g P art A. T h e im p o r ta n t id e a to r e m e m b e r a b o u t c o n d itio n s is th a t a c o n d itio n im p lie s th a t th e opposite o f the condition is true.

E x am p le On the recording, you hear: (man) (woman) (narrator)

Do you think that you '11be able to go to the party? I f I had time, I w ould go. What does the w om an say about the party?

In your test book, you read: (A) (B) (C) (D)

Maybe she'll go. She has time, so she'll go. She is goiag even if she doesn’t have time. It's impossible to go.

_________________________________________________ l In th is q u e s tio n , th e c o n d itio n i f I h a d tim e im p lie s th a t th e o p p o s ite is true: T h e w o m a n d o e s n o t h ave tim e fo r th e party, so sh e can n o t go. T h e r e fo r e , th e b e st an sw er to this q u e s ­ tio n is an sw er (D ). T h e fo llo w in g b o x o u tlin e s th e key p o in ts th a t y o u sh o u ld k n ow a b o u t u n tr u e c o n d itio n s: KEY INFORMATION ABOUT UNTRUE CONDITIONS . *.

Point



r

•An affirmative condition implies a negative reality. • A negative condition implies an affirmative reality.

Example If she were at home, she could do it.* If she weren't «h om e, she could do it.

" ■ ■.... ■■ ■;..... * - : " , ' • . If 1 had money, 1would buy it. • A past tense implies a present reality. . • A past perfect verb implies a If 1 had had money, 1 would past reality. ' have bought it ; ■ ‘• . ■■ ■ ■ ~ ■: : :■ ... ■■ jv ■■ ■ •Had can be used without if Had 1hod money, 1 would :■ ■'■Z v > r ' have bought it.**

Meaning



= not at home , = at home

= do not have money ■■ ■ ■ = did noc have money . .. . = did not have money

•Remember that were is used instead of was in untrue conditions: "If I were there, I would help.” ’" T h is has the same meaning as “If I had had m o n e y .. .." Note that the subject and "had" are inverted.

LISTENING COMPREHENSION

E X E R C IS E 15: In th is e x e r c ise , u n d e r lin e th e c o n d itio n in e a c h sh o r t c o n v e rsa tio n . T h e n , r e a d th e q u e s tio n a n d c h o o s e th e b e st an sw er to th a t q u e s tio n . R e m e m b e r th a t th e b e st an sw er is o n e th a t im p lie s th e o p p o s ite o f w h at is said. 1.

2.

(m an)

Are you going to have something to eat ? (wom an) I f the food looked fresh, I would eat some. (narrator) What does the woman mean ? (wom an) (m an) (narrator)

3.

The flight must have taken longer than usual. H ad the flight left on time, we would not have arrived so late. What does the man say about the flight ?

(m an)

Are you sure you want to go o u t? You do not seem to be feeling very well. (wom an) I f there were some aspirin in the medicine cabinet, I would not need to go to the drugstore. (narrator) What does the woman mean?

(A) (B) (C) (D)

She is not going to cat. T he food looks fresh. She d oesn ’t like fresh food. She already ate som ething.

(A) (B) (C) (D)

It arrived early. It was unusually short. It left on time. It departed late.

(A) She really is feelin g fine. (B) There is plenty o f aspirin in the m edicine cabinet. (C) It is necessary to get som e aspirin. (D) She does not n eed to go out.

T O E F L E X E R C ISE 15: In th is e x e r c ise , listen carefu lly to th e sh o rt c o n v e rsa tio n an d q u e s tio n in th e r e c o r d in g p rogram , an d th e n c h o o s e th e b e st answ er to th e q u e s tio n . You s h o u ld b e particu larly carefu l o f u n tr u e c o n d itio n s . N O W BEGIN THE RECORDING PROGRAM AT TOEFL EXERCISE 15.

T he woman did not need to call him. T he woman called to let him know about the m eeting. (C) H e ’s n ot glad that the woman called. (D) H e already knew about the m eeting w hen the woman called. (A) (B)

2.

3.

T he m an often drives too quickly. T he police do not stop the man too m uch. (C) T he man drove rather slowly. (D) T he police should not stop the man so often.

(A) (B)

She’s so happy they d o n ’t have to work on Friday. (B) It would be nice if they could finish their work on Friday. (C) She wonders if the man would be nice enough to com e in to work in her place on Friday. (D) It’s too bad they must work on Friday.

(A)

She did not put en ou gh postage on the letter. (B) T he letter arrived last week. (C) T he letter did not n eed more postage. (D) She did not put any postage on the letter.

4.

(A)

5.

(A) H e has a dog. (B) H e d oesn ’t pay attention to dogs. (C) H e wishes he had a dog. (D) D ogs do not n eed m uch attention.

6.

(A)

They knew they had to prepare for the exam. (B) They did n’t prepare for the exam. (C) As soon as they knew about the exam, they began to prepare for it. (D) They knew that the preparation for the exam would take a lot o f time.

LISTENING PARTA

7.

(A) (B) (C) (D)

It costs too m uch for him to go. H e agrees to go with them. H e is unworried about the cost o f the restaurant. T h e restaurant is rather inexpensive.

9.

(A) (B) (C) (D)

10.

W hen J o e saw the car com ing, he tried to get out o f the way. (B) Joe was able to get out o f the way because he saw the car com ing. (C) Joe ju m p ed out o f the way o f the o n com in g car. (D) Because J o e did n’t see the car com ing, he co u ld n ’t get ou t o f the way.

(A)

(A)

(B)

(C) (D)

T h e woman d id n ’t com e. T h e woman wanted to be there. T h e woman was g oin g to leave immediately. T h e woman was n ot really there. Kathy d id n ’t work as hard as possible because she d id n ’t know what the reward was. Kathy c o u ld n ’t have put m ore effort into the project to win the prize. Kathy won first prize because o f her hard work on the art project. Kathy worked so hard that she knew first prize was hers.

T O E F L E X ER C ISE (Skills 1 4 -1 5 ): In this ex er cise , listen carefully to th e sh o r t con versation an d q u estio n in th e r ec o r d in g program , a n d th e n c h o o s e th e b est answ er to th e q u estio n . n

N ow 1.

(A) (B) (C) (D)

BEGIN THE RECORDING PROGRAM AT TOEFL EXERCISE (SKILLS 14-15).

She enjoys violent movies. She would have preferred a more violent movie. She thinks the film was too violent. She enjoyed the movie.

2.

(A) H e left the windows open. (B) The rain did not get in. (C) H e forgot to close the windows. (D) T he rain got into the house.

3.

(A)

(D)

Her family is unable to com e to graduation. It is possible that her family will com e. H er parents are com ing to the cerem onies. She is n o t graduating this year.

4.

(A) (B) (C) (D)

He He He He

5.

(A) (B)

H e enjoys chem istry lab. H e d o e sn ’t have chem istry lab this afternoon. H e isn’t taking chem istry class. H e has to go to the lab.

(B) (C)

(C) (D)

is g o in g to miss the conference. will take his vacation n ext week. will attend the conference. w on ’t miss his vacation.

6.

(A)

T hey filled up the gas tank at the last service station. (B) A lthough they filled up the tank, they still ran out o f gas. (C) Even thou gh they d id n ’t stop at the service station, they d id n ’t run out o f gas. (D) They ran ou t o f gas because they d id n ’t stop at the gas station.

(A) (B) (C) (D) 8.

(A) (B) (C) (D)

H is schedule is not really heavy. H e needs to add a few more courses. H e enrolled in m ore courses than he really wants. H e will register for a lot o f courses next semester. She never took the bus to work. She regularly takes the bus. She d oesn ’t know how to get to work. She gets lost on the bus.

LISTENING COMPREHENSION

9.

(A) (B) (C) (D)

She bought som e eggs at the store. She doesn't have any eggs to lend him. H e can borrow som e eggs. She did n’t go to the store.

10.

(A) (B)

Teresa is feeling a lot better. The doctor did n’t prescribe the m edicine. (C) Teresa d id n ’t follow the doctor’s orders. (D) Teresa did exacdy what the doctor said.

T O E F L REVIEW EXERCISE (Skills 1-15): In this exercise, listen carefully to the sh ort conver­ sation a n d q u estion in the record in g program , a n d th en c h o o se th e b est answer to th e question.

o

N ow

BEGIN THE RECORDING PROGRAM AT TOEFL REVIEW EXERCISE (SKILLS

1.

(A) (B) (C) (D)

2.

(A) In a bus station. (B) In a store. (C) In a restaurant. (D) In a theater.

3.

(A) (B) (C) (D)

4.

5.

Drinking the hot tea. Making more tea in a few minutes. Letting the tea cool o ff a bit. Having the tea immediately.

H e’s unhappy to end the semester. H e’s glad to be finishing .chool. H e co u ld n ’t be happier to begin the semester. The end o f the sem ester is making him feel sad.

The storm destroyed the houss. The house blocked the trees. The stormy weather caused the trees to fall. (D) During the storm, som eone knocked on the door o f the house.

6.

The team hasn’t won often. H e usually d o esn ’t pay attention to the football team. (C) It’s out o f the ordinary for the team to lose. (D) H e usually hears about the football gam es.

H e went to the office every m orning. .(B) H e was not working. (C) He had to arrive at work earlier than 8 o ’clock. (D) H e had a job. (A)

7.

H e did not enjoy his vacation as much as possible. (B) H e got lost on his vacation. (C) The vacation was really enjoyable. (D) H e did not really lose his passport.

8.

(A)

9.

(A) The laboratory assistant com pleted one experim ent. (B) The laboratory assistant cou ld n ’t finish one experim ent. (C) The laboratory assistant did n’t want to do m ore experim ents. (D) N on e o f the experim ents could be com pleted.

(A) (B) (C)

(A) (B)

I—IS).

(A)

It will take eight hours to get to Riverdale on the bus. (B) He believes he knows the correct bus. (C) H e d o e sn ’t know where Riverdale is. (D) H e assures the woman that he knows the way to Riverdale.

10. (A)

She would like the man to repeat what h e said. (B) The sem ester is really over! (C) The sem ester will never end. (D) She has the same wish as the man.

LISTENING PARTA

ID IO M A T IC LA N G U A G E _____________________________________ S k il l

16:

LISTEN F O R T W O -A N D THREE-PART VERBS

Two- a n d th ree-p art verb s a p p ea r in s o m e q u e s tio n s in L iste n in g Part A. T h e s e verb s are e x p r essio n s th a t in c lu d e a verb aijd o n e o r m o r e p a r ticles (su ch as in, on, o r a t) ; th e parti­ c le c h a n g e s th e m e a n in g o f th e verb. Q u e s tio n s in v o lv in g two- a n d th r e e-p a r t v e rb s can b e d iffic u lt for stu d e n ts b e c a u se th e a d d itio n o f th e p a r ticle c h a n g e s th e m e a n in g o f th e verb in an id io m a tic way.

E xam ple On the recording, you hear: (man) (woman) (narrator)

What tim e does the meeting start? - Didn't you hear that it was called o ff by the director? What does the w om an say about the meeting?

In your test book, you read: (A) The director called a meeting. (B) The director phoned her about the meeting. (C) The director called the meeting to order. (D) The director canceled the meeting.

In this q u e s tio n , th e tw o-part verb called o f f h a s a d iffe r e n t m e a n in g th a n th e verb call, w h ich m ea n s phone. T h e tw o-part v e rb call o f f m e a n s cancel, so th e b e st an sw er is answ er (D ).

NOTE: A list of common two- and three-part verbs and exercises using these verbs appear in Appendix B. You may want to study these two- and three-part verbs before you try the following exercises.

EX ER C ISE 16: In th is e x e r c ise , u n d e r lin e th e two- o r th r e e-p a r t verb in e a c h s h o r t c o n ­ v e rsa tio n . T h e n read th e q u e s tio n a n d c h o o s e th e b e st an sw er to th a t q u e s tio n . R e m e m ­ b e r th a t the b e st answ er is o n e th a t is r e la te d to th e m e a n in g o f th e two- o r th ree-p a rt verb an d m ig h t n o t se e m to b e r e la te d to th e m e a n in g o f th e verb w ith o u t t h e p a r ticle. 1.

(man) (woman) (narrator)

2.

(woman) (man) (narrator)

Did you have your history exam today f No, the professor p u t it offfor another week. What does the woman say about the exam ? Do we have any more soap ? We’ve run out o f it. Someone will have to go to the store. What does the man mean ?

(A) (B) (C) (D)

(A) (B) (C) (D)

She would like to pu t it o u t o f her mind. The professor canceled it. It was m oved to anoth er location. It was delayed.

H e will run to the store. H e n eed s soap to wash him self after running. There is no m ore soap. They have a store o f soap at hom e.

LISTENING COMPREHENSION

3.

(man) (woman) (narrator)

(A) (B) (C) (D)

1 need to take the written test to renew my driver's license. Then, you '11 have to brush up on the laws. What does the man need to do ?

Reapply for his driver’s license. Sweep around the lawn. Learn the laws for the first time. Review the inform ation that will be on the test.

T O E F L E X ER C ISE 16: In th is e x e r c ise , liste n carefu lly to th e sh o rt c o n v e rsa tio n a n d q u e s tio n in th e r e c o r d in g p rogram , a n d th e n c h o o s e th e b e st answ er to th e q u e stio n . You sh o u ld b e p articu larly carefu l o f two- a n d th ree-p art verbs. N O W BEGIN THE RECORDING PROGRAM AT TOEFL EXERCISE I 6.

o 1.

(A) (B)

P hone their neighbors. Call to their neighbors over the fence. (C) H elp the neighbors m ove in. (D) Visit their neighbors.

2.

(A)

The course is becom ing more interesting. (B) T he course used to be more interesting. (C) T he course is about the same as it was. (D) H e ’s not as bored in the class as the woman.

6.

(A) (B)

She gets along with lots o f people. She gets back at people who cross her. (C) She gets rid o f people she doesn’t want to spend time with. (D) She tries to get ahead o f everyone else.

7.

(A) (B)

He must try to find the children. It is necessary for him to clean up after the children. (C) The children n eed to be watched. (D) H e’s going to see what the children have done.

3.

(A) (B)

H er headache is getting worse. She felt better this m orning than now. (C) She seem s to be feelin g better now. (D) She is just getting another headache now.

8.

(A) They are going on strike. (B) They are lying down on the job. (C) They are being released from their jobs. (D) They are relaxing too m uch at the factory.

4.

(A)

9.

(A)

10.

(A)

5.

(A)

The man should stop breaking his cigarettes in half. (B) The man should decrease the( num ber o f cigarettes he smokes. (C) T he man should cut the ends o ff his cigarettes. (D) The man should stop smoking com pletely. T he client presented his case to the lawyer. (B) T he client was upset about the lawyer’s rejection. (C) The client was annoyed because the lawyer returned the suitcase. ( D ) T he client m ade the lawyer unhappy about the case.

H e is betting that the football team will win. (B) He really wants to succeed. (C) It is n ot so difficult to play on the football team. (D) He pulled a m uscle w hile playing football. She’s unsure why she tolerates the man. (B) She doesn't know where she put her keys. (C) She is actually the on e who put the keys in the car. (D ) She can't understand why the man did what he did.

LISTENING PARTA

S k i l l 17:

LISTE N FOR ID IO M S

Id io m s a p p e a r in s o m e q u e s tio n s in L iste n in g P art A. Id io m s are sp e c ia l e x p r e s sio n s in a la n g u a g e th a t all sp ea k er s o f th e la n g u a g e kn ow ; th e se sp ec ia l e x p r e s s io n s d e sc r ib e o n e situ a tio n in life b u t a r e a p p lie d to m a n y d if fe r e n t areas o f life . Id io m q u e s tio n s c a n b e d if­ fic u lt fo r stu d e n ts b e c a u se th e y s e e m to b e d e sc r ib in g o n e situ a tio n w h e n th e y are really d e sc r ib in g a d iffe r e n t situ a tio n . E x am p le On the recording, you hear: (man) (woman) (narrator)

Tom is a full-tim e student and is holding down a full­ tim e job. He's really burning the candle at both ends. What does the w om an say about Tom?

In your test book, you read: (A) (B) (C) (D)

He's lighting a candle. He’s holding the candle at the top and the bottom. He’s doing too much. He’s working as a firefighter.

In th is q u e s tio n , th e id io m b u rn in g the candle a t both ends h as n o th in g to d o w ith c a n d le s a n d n o th in g to d o w ith b u r n in g o r fires, so an sw ers (A ), (B ), a n d (D ) a r e n o t c o r r e c t. In ­ stead , th is id io m is a n e x p r e s sio n th a t is u s e d in a situ a tio n w h e n s o m e o n e is tr y in g to d o m o r e than h e o r sh e really c a n do; a fter all, a c a n d le u su ally o n ly b u r n s at o n e e n d , s o ac a n d le th a t b u r n s at tw o e n d s is d o in g more th a n it can. T h e r e fo r e , th e b e st a n sw er to th e q u e s tio n a b o v e is a n sw er ( C ) . NOTE: A list of common idioms and exercises using chese idioms appear in Appendix C. You may want to study these idioms before you try the following exercises. E X E R C ISE 17: In th is e x e r c ise , u n d e r lin e th e id io m in e a c h sh o r t c o n v e r s a tio n . T h e n r e a d th e q u e s tio n a n d c h o o s e th e b e s t a n sw er to th a t q u e s tio n . R e m e m b e r th a t th e b e st an sw er is o n e th a t m ig h t n o t s e e m to b e r e la te d to th e id io m in th e s e c o n d lin e . I have to take Advanced Biology from Professor Stanton next semester. (woman) D o n ’t worry about it. It's a piece o f cake. (narrator) What does the woman mean ? (man)

(wom an) (man) (narrator)

Thanks fo r changing the oil AND putting air in the tires. I t ’s all in a day's work. What does the m an mean ?

T he m an sh ould try a p iece o f cake. (B) T he m an sh ould worry about the course. (C) T he man sh o u ld n ’t take part in the course. (D) T he course is easy. (A)

It will take him a w hole day to do the job. (B) This is a regular part o f his job . (C) H e can do the work at the en d o f the day. (D) H e’s too busy today to d o the work. (A)

87

LISTENING COMPREHENSION

3.

(m an) (wom an) (narrator)

What was it like while the president was giving his speech ? You could hear a pin drop. What does the woman mean ?

(A) (B) (C) (D)

T he president dropped his pen. T he audience was very quiet. T he speech contained several puns. T he president discussed dropping a bomb.

T O E F L E X E R C ISE 17: In this e x e r c ise , liste n ca refu lly to th e sh o r t c o n v e rsa tio n and q u e s tio n in th e r e c o r d in g p rogram , a n d th e n c h o o s e th e b e st an sw er to th e q u e stio n . You sh o u ld b e particu larly carefu l o f id iom s.

n

N ow (A) (B) (C) (D) (A) (B) (C) (D) (A) (B) (C) (D)

BEGIN THE RECORDING PROGRAM AT TOEFL EXERCISE 17.

T h e m an’s never late. It’s good that the man was fifteen m inutes late. It’s never good to be late for class. It’s good that the man went to class, on time or not.

6.

(A) (B) (C)

T he w om an’s work is all in her head. T h e woman has to do two experim ents rather than one. It’s a good idea to work together. T h e biology experim ent concerns two-headed animals.

7.

(A)

Abbie used a feather in his art project. (B) H e was knocked down. (C) H e was really surprised. (D) A bbie’s father knocked on the door.

8.

(A)

9.

(A) (B)

She has n o time to work now. She d oesn ’t want to work on the report either. It’s best to get it over with now. T here’s no time to present the report now.

(A)

S h e’s very lucky to get the last book. (B) S h e’s sorry she can ’t get the book today. (C) She always has good luck with books. (L>) She just wanted to look at the book. (A) (B) (C) (D)

T he man d oesn ’t like eating in restaurants. She d oesn ’t really like that restaurant. Each o f them has his own restaurant. Everyone has different tastes.

10.

Sh e’ll do it immediately. It is not possible to do it. T he man should have told her sooner. (D) She would have d on e it if the man had asked.

They are taking a boat trip together. (B) T he six chapters are all about the boat. (C) Everyone has to do the same thing. (D) T he man will read while h e’s on the boat. She is taller than die others. She put her science project on top o f the others. (C) She has a really good head on her shoulders. • (D) S h e’s the best o f them all. T h e man needs to improve his penm anship. (B) T he man d o esn ’t really n eed to apply for the scholarship. (C) T he man needs to fill out the application with dots and crosses. (D) T he man needs to pay attention to every detail. (A)

LISTENING PARTA

T O E F L EX ER C ISE (Skills 1 6 -1 7 ): In this e x er cise , listen carefully to th e sh o r t co n v ersa tio n a n d q u estio n in th e r ec o r d in g p rogram , a n d th e n c h o o se th e b est answ er to th e q u estio n . N O W BEGIN THE RECORDING PROGRAM A T T O E R EXERCISE (SKILLS 16-17).

n 1.

(A) (B) (C) (D)

2.

She gets lots o f take-out dinners. She and her room m ate alternate cooking responsibilities. H er room m ate cooks m ore often than she does. H er room m ate does the cooking while she does other chores.

(A) (B) (C) (D)

H e resem bles his father. H e has a chippcd tooth. H e lives o n e block from his father. H e and his father were playing a gam e with blocks.

3.

(A) (B) (C) (D)

Sh e’s go in g som ewhere else. She does n ot like football. She has a lot o f work to do. She is gettin g sick.

4.

(A)

H e put his foot w here h e should n ot have. H e put the food that the teacher gave him into his m outh. H e said som ething embarrassing. H e told the teacher that his foot was hurt.

(B) (C) (D) 5.

(A) (B)

(C)

(D)

S h e’d like the man to delay his trip. She prefers that the man leave a few m inutes earlier than he planned. She wants to know if the m an will stay in the market for only a few m inutes. Sh e’d like to talk to the m an for a few m inutes.

6.

T h e man m ight start a fire in the park. (B) T he man parked his car near the fire. (C) T h e m an’s thinking o f doin g som ething dangerous. (D) T he m an’s playing a gam e in the park.

7.

(A) (B)

T h e m achines do n o t act very well. T h e m achines d o n ’t really bother her. (C) She would like them to stop the noise. (D) She wishes the m achines would cut the wood.

8.

(A) (B)

9.

(A)

Fred has Fred has tree. (C) Fred has (D) Fred has (A) (B) (C) (D)

a d og that barks a lot. hidden the m oney in a backed into a tree. m ade a mistake.

She will give him any help he needs. H e has to give away what he d o e sn ’t need. H e should not give up. H e should give back what he borrowed.

10. (A)

She'd rather go swimm ing than do the homework. (B) T he chem istry hom ework is really difficult. (C) S h e’s d oin g the hom ework by the swimming pool. (D) T h e stream is drying up.

TO EFL REVIEW EXERCISE (Skills 1-17): In this exercise, listen carefully to th e sh ort conver­ sation a n d q u estion in th e r eco rd in g program , an d th en c h o o se the b est answer to the q u estio n o

N ow (A) (B)

BEGIN THE RECORDING PROGRAM At TOEFI REVIEW EXERCISE (SKILLS 1-17).

T here’s no m ore w ood inside. The w ood in the fireplace should be pu t outside. (C) T here’s a fire outside. (D) H e n eed s to bring som e wood outside.

2.

(A) (B) (C) (D)

She worked late at a conference. H er m eeting was canceled. She called a con ference at work. She was late to a con ference.

LISTENING COMPREHENSION

3.

(A) In a hospital. (B) At a police station. (C) At the beach. (D) In a locker room .

4.

(A)

There was too m uch room on the dance floor. (B) H e enjoyed the room w here they w ent dancing. (C) T he dance floor was too crowded. (D) The club n eeded more rooms for dancing.

5.

(A)

H e could not understand the fax m achine. (B) H e wrote the letter that was sent. (C) T h e fax m achine was easy for him to use. (D) H e was not very good with figures.

6.

(A) (B) (C) (D)

7.

(A)

The T he The T he

woman woman woman woman

hit her head on a nail. hit his new car. was exactly right. bought the new car.

H e would like the woman to help him find his paper. (B) H e wants the woman to pat the paper away. (C) H e needs the woman to review the paper. (D) H e would like the woman to write the paper for him.

8.

(A) Inform ation about the problem is unavailable. (B) N o o n e has been inform ed. (C) Everybody knows what is going on. (D) Nobody is aware that the problem is serious.

9.

(A) (B) (C) (D)

H e did not sleep well. H e never woke up this morning. The alarm failed to go off. He n eed ed a loud alarm to wake up-

10.

(A) The pilot made an em ergency landing. (B) T he pilot was forced to leave the plane in a hurry. (C) The pilot fielded questions about the forced landing. (D) The plane was dam aged when it landed forcefully.

LISTENING PART B

-------------T H E L IS T E N IN G PART B Q U E S T IO N S ------------Part B o f th e L iste n in g C o m p r e h e n s io n se c tio n o f th e T O E F L test c o n sists o f two lo n g c o n ­ versa tio n s, e a c h fo llo w e d by a n u m b e r o f q u estio n s. You will h e a r th e c o n v e r sa tio n s an d th e q u e s tio n s o n th e r eco rd in g ; th ey are n o t w ritten in you r te st b o o k . Y ou m u st c h o o s e th e b e st an sw er to e a c h q u e s tio n from t h e fo u r c h o ic e s th a t are w ritten in y o u r te st b o o k . T h e c o n v e r sa tio n s a r e o ft e n a b o u t s o m e a sp e c t o f s c h o o l life (h o w d iffic u lt a class is, h o w to w rite a r esea r ch p ap er, h o w to r e g iste r fo r a c o u r s e ) . T h e c o n v e r s a tio n s c a n also b e a b o u t to p ic s c u r re n tly in th e new s in th e U n ite d S tates (d e s a lin a tio n o f th e w ater su pp ly, r ec y c lin g o f u s e d p r o d u cts, d a m a g e fr o m a sto rm o r s o m e o th e r type o f n atu ral p h en o m en o n ).

E xam ple On the recording, you hear: (narrator) (man) (woman)

(man)

(woman) (man) (woman)

Q u estio n s I th ro u g h 4. Listen to a conversation between a professor and a student. Hello, Professor Denton. Are you free for a m om ent? Could I have a word w ith you? Come on in, Michael. O f course I have som e time. These are m y office hours, and this b the right tim e for you to come and ask questions. Now, how can I help you? Well, I have a quick question fo r you about the hom ework assignm ent for tomorrow. I thought the assignm ent was to answer the first three questions at the top o f page 67 in the text, but when I looked, there weren't any questions there. I'm confused. The assignm ent was to answer the first three questions at the top o f page 76, not 67. Oh, now I understand. I ’m glad I came in to check. Thanks for yo u r help. No problem. See you tomorrow.

Questions: 1. On the recording, you hear: (narrator) In your test book, you read:

Who is the man? (A) (B) (C) (D)

A professor. An office worker. Professor Denton’s assistant. A student.

2. On the recording, you hear: (narrator) In your test book, you read:

When does the m an come to see Professor Denton? (A) (B) (C) (D)

During regular class hours. Just before class time. As soon as class is finished. During office hours. (continued on next page)

LISTENING COMPREHENSION

3. On the recording, you hear: (narrator) In your test book, you read:

Why does the man come to see Professor Denton? (A) (B) (C) (D)

To turn in an assignment. To ask a question. To pick up a completed test. To explain why he did not attend class.

4. On the recording, you hear: (narrator)

What incorrect information did the man have?

In your test book, you read:

(A) The date the assignment was due. (B) The page number of the assignment. (C) The length of the assignment. (D) The numbers of the assignment questions.

T h e first q u e s tio n asks y o u to d e te r m in e w h o th e m an is. S in ce th e m an o p e n s th e c o n ­ v e rsa tio n w ith Professor D enton a n d h e asks a b o u t th e p a g e n u m b e r o f an a ssig n m e n t fo r to­ m orrow , h e is p rob ab ly a stu d e n t. T h e b e st answ er to th is q u e s tio n is th e r e fo r e answ er ( D ). T h e s e c o n d q u e stio n asks a b o u t w h en th e m an c o m e s to se e th e professor. T h e p ro­ fe sso r says th a t these are my office hours, so th e b est answ er to this q u e s tio n is answ er (D ). T h e th ird q u e s tio n asks w h y th e m a n c o m e s to se e th e p rofessor. S in c e th e m a n says I have a quick question f o r you, th e b e st an sw er to th is q u e s tio n is answ er (B ). T h e last q u estio n asks w h a t in c o r r e c t in fo r m a tio n th e m an h a d . T h e m an th o u g h t th a t th e a ssig n m e n t was o n p a g e 6 7 a n d n o t o n p a g e 76, so h e was m ista k en a b o u t th e page num ber o f th e assign ­ m e n t. T h e b e st answ er to th is q u e s tio n is answ er (B ).

_________________ STRATEGIES FORTHE LISTENING PART B QUESTIONS_________________ 1. If you have tim e , p review th e answ ers t o th e L istening Part B q u estio n s. While you are looking at the answers, you should try to do the following: •Anticipate the to p ic s of the conversations you will hear. •Anticipate the q u estio n s for each of the groups of answers. 2.

L isten carefully to th e first line o f th e con versation . The first line of the conversation often contains the main idea, subject, or topic of the conversation, and you will often be asked to answer such questions.

3. A s you listen to th e con versation , draw co n clu sion s a b o u t th e situation o f the con versation : w h o is talking, w h er e th e con versation ta k es p lace, o r w h en it tak es p lace. You will often be asked to make such inferences about the conversation. 4. A s you listen to the conversation, follow along w ith th e answers in your te st book and try t o determ ine th e correct answers. Detail questions are generally answered In order in the conversation, and the answers often sound the same as what is said in the recording program. 5.

You should g u ess even if you are n ot su re. Never leave any answers blank.

6.

U se any rem aining tim e to look ahead a t th e answ ers to th e q u estio n s th a t follow.

LISTENING PART B

T h e fo llo w in g sk ills w ill h e lp y o u to im p le m e n t th e s e stra teg ies in P art B o f th e L iste n in g C o m p r e h e n s io n s e c tio n o f th e T O E F L test.

BEFORE LIS T E N IN G S k il l

18:

A N T IC IP A T E T H E TO P IC S

It is very h e lp fu l to y o u r o v era ll c o m p r e h e n s io n i f y o u k n ow w h at to p ic s to e x p e c t in Lis­ te n in g P art B. You sh o u ld th e r e fo r e try to a n tic ip a te th e to p ic s y o u w ill b e h e a r in g . For e x a m p le , are th e c o n v e r sa tio n s a b o u t s o m e a sp e c t o f s c h o o l life, o r s o m e typ e o f so c ia l is­ su e , o r a trip s o m e o n e is p la n n in g ? A h e lp fu l strategy is th e r e fo r e to l o o k b riefly at th e a n ­ sw ers in th e test b o o k , b e fo r e y o u actu a lly h e a r th e c o n v e r sa tio n s in th e r e c o r d in g p ro g ra m , a n d try to d e te r m in e th e to p ic s o f th e c o n v e r sa tio n s th a t y o u w ill hear.

E X E R C ISE 18: L o o k at th e an sw ers to th e five q u e s tio n s to g e th er , a n d try to a n tic ip a te th e to p ic o f th e c o n v e r s a tio n fo r th o s e five q u e s tio n s. ( O f co u r se , y o u c a n n o t always d e ­ te r m in e e x a ctly w h a t th e to p ic is, b u t y o u o fte n ca n g e t a g e n e r a l id e a .) Q u e s tio n s 1 th r o u g h 5 h ave b e e n a n sw e re d fo r y ou . 1.

(A) Find work on campus. (B) Work in the employment office. (C) H elp students find jobs. (D) Ask the woman questions.

4.

(A) Every m orning. (B) A fternoons and w eekends. (C) W hen h e ’s in class. (D) Weekdays.

2.

(A) (B) (C) (D)

5.

(A) (B)

3.

(A) N o m ore than ten. (B) At least twenty. (C) N ot m ore than twenty. (D) U p to ten.

In In In In

the library. a classroom. a cam pus office. an apartment.

(C) (D)

Fill out a form. Give her som e additional inform ation. Tell her som e news. P hone her.

W hat is th e to p ic o f th e c o n v e r s a tio n fo r q u e s tio n s 1 th r o u g h 5? looking for a job on campus________________ You can g u e ss th is b e c a u se o f th e fo llo w in g clu es: • work on cam pus • em ploym ent office • stu d en ts • jobs

LISTENING COMPREHENSION

6.

(A) Just before a vacation. (B) Just after the end o f a school semester. (C) At the end o f the summer. (D) Just after a break from school.

7.

(A) (B)

8.

A trip to visit the Eskimos. A trip the woman is planning to take. (C) A trip the man has already taken. (D) A cam ping trip the man and woman took.

9.

(A) (B)

Sleeping outside on the ground. Spending time in a sauna or hot tub. (C) Relaxing at the lodge. (D) Enjoying excellent food.

10.

(A) (B) (C) (D)

She’d be scared, but she’d like to try. She can’t wait. It would be quite exciting for her. Sh e’d prefer not to try.

(A) T hree hours. (B) Three com plete days. (C) T hree classes. (D) T hree weeks.

W h at is th e to p ic o f th e c o n v e rsa tio n fo r q u e s tio n s 6 th r o u g h 10?

All kinds o f pollution. How acid rain has harm ed the earth. (C) P ollution from cars and factories. (D) T he causes and possible effects o f acid rain.

(A) (B)

(A) (B) (C) (D)

N uclear power. Electricity. Burning coal and oil. Solar power.

From sulfur dioxide and water vapor. (B) From sulfur dioxide and nitrogen oxide. (C) From nitric acid and sulfur dioxide. (D) From water vapor and nitric acid.

(A)

Only in North America. At the North and South Poles. In parts o f several northern continents. (D) In equatorial areas.

14.

(A) (B) (C)

15.

(A)

She should protect herself from the rain. (B) She should clean up the water supply. (C) She should read a novel. (D) She should get m ore information about acid rain.

W hat is th e to p ic o f th e c o n v e rsa tio n fo r q u e s tio n s 11 th r o u g h 15?

LISTENING PART B

S k il l

19: A N T IC IP A T E T H E Q U E S T IO N S

It is very h e lp fu l to you r ab ility to an sw er in d iv id u a l q u e s tio n s in L iste n in g P art B if y ou c a n a n tic ip a te w h a t th e q u e s tio n s w ill b e a n d lis te n sp ec ific a lly fo r th e a n sw ers to th o se q u e s tio n s.

E x am p le In your test book, you read: (A) (B) (C) (D)

In the In the In the In the

airport. library. dormitory. travel agent's office.

You try to anticipate the question: Where does the conversation probably take place?

In th is e x a m p le , y o u can b e q u ite c e r ta in th a t o n e o f th e q u e s tio n s w ill b e a b o u t w h er e th e c o n v e r sa tio n takes p la c e. S in c e y o u are su re th a t th is is o n e o f th e q u e s tio n s , y o u can liste n carefu lly fo r c lu e s th a t w ill give y o u th e answ er. T h is e x a m p le sh o w s th a t a h e lp fu l strategy is th e r e fo r e to lo o k b riefly at th e answ ers in th e test b o o k , b e fo r e y o u actu ally h e a r th e c o n v e r sa tio n s in th e r e c o r d in g p ro g ra m , a n d try to d e te r m in e th e q u e s tio n s that y o u w ill b e a sk ed to answer. E X E R C ISE 19: Stu dy th e fo llo w in g answ ers an d try to d e te r m in e w h at th e q u e s tio n s w ill b e . (You sh o u ld n o te th at p e r h a p s y ou will o n ly b e a b le to p r e d ic t p art o f a q u e s tio n , rather th an th e c o m p le te q u e s tio n .) I f y o u c a n n o t p r e d ic t th e q u e stio n in a sh o r t p e r io d o f tim e, th e n m o v e o n to th e n e x t g r o u p o f answ ers. Q u e stio n 1 has b e e n a n sw ered fo r y ou. 1. Question: What does (someone) want to do?___________________________________________ (A) Find work on campus. (B) Work in the em ploym ent office. (C) H elp students find jobs. (D) Ask the woman questions. 2.

Q uestion:__________________________________________________ _____________________________ (A) In the library. (B) In a classroom. (C) In a cam pus office. (D) In an apartment.

3.

Question: _____________________________________________________ __________ ________________ (A) N o m ore than ten. (B) At least twenty. (C) N ot m ore than twenty. (D) U p to ten.

4.

Q u estion :______ ________________________________________________ ______________ __________ (A) Every m orning. (B) A fternoons and weekends. (C) W hen h e ’s in class. (D) Weekdays.

LISTENING COMPREHENSION

5.

Q u estio n :______________________________________ (A) Fill ou t a form. (B) Give her som e additional inform ation. (C) Tell h er som e news. (D) P hone her.

6.

Q u e stio n :__________________________— -------------(A) Just before a vacation. (B) Just after the end o f a school semester. (C) At the end o f the summer. (D) Just after a break from school.

7.

Q u e stio n :______________________________________ (A) A trip to visit the Eskimos. (B) A trip the woman is planning to take. (C) A trip the man has already taken. (D ) A cam ping trip the man and woman took.

8.

Q u estio n :______________________________________ (A) Three hours. (B) Three com plete days. (C) T hree classes. (D) Three weeks.

9.

Q u estio n :_____________________________________ (A) Sleep in g outside on the ground. (B) Sp en ding tim e in a sauna or h ot tub. (C) R elaxing at the lodge. (D) Enjoying excellen t food.

10.

Q u e stio n :___________________ _________________ _ (A) S h e’d be scared, but sh e’d like to try. (B) She ca n ’t wait. (C) It w ould be quite exciting for her. (D ) Sh e’d prefer n ot to try.

11.

Q u estio n :___________________________ -_________ (A) All kinds o f pollution. (B) How acid rain has harm ed the earth. (C) P ollution from cars and factories. (D ) The causes and possible effects o f acid rain.

12.

Q u estio n :_____________________________ ________ (A) N uclear power. (B) Electricity. (C) B urning coal and oil. (D) Solar power.

13.

Q u e stio n :__________________________________ — (A) From sulfur dioxide and water vapor. (B) From sulfur dioxide and nitrogen oxide. (C) From nitric acid and sulfur dioxide. (D ) From water vapor and nitric acid.

14.

Q u estio n :___________________________________ — (A) Only in North America. (B) At the North and South Poles. (C) In parts o f several northern continents. (D) In equatorial areas.

LISTENING i’ART B

15.

Q u estion :___________________________ — -------------------(A) She should protect herself from the rain. (B) She should clean up the water supply. (C) She sh ould read a novel. (D) She should get m ore inform ation about acid rain.

W H IL E LIS T E N IN G S k il l

20:

D E TER M IN E T H E T O P IC

As y o u listen to e a c h c o n v e r sa tio n in L iste n in g Part B, y o u s h o u ld b e th in k in g a b o u t th e to p ic (su b je ct) o r m a in id e a fo r e a c h c o n v e r sa tio n . S in c e th e first o n e o r tw o s e n t e n c e s g e n e r a lly give th e to p ic , y o u sh o u ld b e ask in g y o u r se lf w h at th e to p ic is w h ile y o u are lis te n in g ca refu lly to th e first p art o f th e co n v e rsa tio n .

E x am p le On the recording, you hear: (man) (woman) (man)

You can't believe w hat I ju st got! I bet you got that new car you've always wanted. Now, how in the world did you figure that out?

You think: The topic of the conversation is the new car that the man just got.

EX ER C ISE 20: L iste n to th e first p a r t o f e a c h o f th e co n v e rsa tio n s, a n d d e c id e o n th e to p ic o f e a c h c o n v e r sa tio n . N O W BEGIN THE RECORDING PROGRAM AT EXERCISE 20.

1.

What is the topic o f Conversation 1?

2.

What is the topic o f Conversation 2?

3.

What is the topic o f Conversation 3?

LISTENING COMPREHENSION

S k il l

21: D R A W C O N C L U S IO N S A B O U T W HO , WHAT, W H EN , W H ERE

A s y o u listen to e a c h c o n v e r sa tio n in L iste n in g P art B, y o u sh o u ld b e tryin g to se t th e situ­ a tio n in you r m in d . You sh o u ld b e th in k in g th e fo llo w in g th o u g h ts: • • • •

W ho is talking? W hen does the conversation probably take place? W here does the conversation probably take place? W h a t is the source o f info rm a tio n fo r the conversation ?

E xam ple On the recording, you hear: (man) (woman) (man)

Why do you have so m any books? I need them for my paper on George Washington. Do you know how I can check them out? Yes, you should go downstairs to the circulation desk and fill out a card for each book.

You think: Who is probably talking? (two students) Where are they? (in the library) What course are they discussing? (American History)

E X E R C ISE 21: L isten to th e first part o f e a c h o f th e c o n v e rsa tio n s an d try to im a g in e th e situ a tio n . T h e n answ er th e q u e s tio n s in th e te x t. N ow

BEGIN THE RECORDING PROGRAM AT EXERCISE 2 1.

Conversation 1 1. W ho is probably talking? 2.

W here does the conversation take place?

Conversation 2 1. W ho is probably talking? 2.

W hen does the conversation take place?

3.

What is the source o f the m an’s information?

Conversation 3 1. W ho is probably talking? 2.

W hen does the conversadon take place?

3.

What is the source o f the information?

LISTENING PART B

S k i l l 22:

LISTEN FOR A N S W E R S IN ORDER

T h e r e are two p o ssib le m e th o d s to u se w h ile y o u listen to a c o n v e r sa tio n in th e L iste n in g Part B o f th e T O E F L test. • You can j u s t listen to the conversation (a n d ignore the answers). • You can fo llo w along w ith the answers w hile you listen. S o m e stu d e n ts p r e fe r to ju s t liste n to th e c o n v e r sa tio n w h ile it is b e in g s p o k e n , a n d i f th a t m e th o d w orks w ell fo r y ou , th e n th at is w h at y o u sh o u ld d o . O th e r stu d e n ts fin d that th ey can answ er m o r e q u e s tio n s c o rr ec tly i f th e y r e a d a lo n g w ith th e answ ers w h ile th e c o n v e r ­ sation is b e in g sp o k e n . B e ca u se th e d e ta il q u e s tio n s are a n sw e re d in ord er, it is p o s sib le to read a lo n g w h ile y o u listen to th e c o n v e r sa tio n in th e r e c o r d in g p ro g ra m .

Exam ple On the recording, you hear: (man) (woman) (man)

(woman) (man)

(woman)

In your test book, you read (same time):

Can I help you? I ’m Interested in opening an account. Well, we have several different types o f accounts: checking accounts, savings accounts, money market accounts, time deposit accounts. I t ’s a checking account that I am interested in.

1.

(A) A checking account. (B) A savings account. (C) A money market account. (D) A time deposit account.

I can help you w ith that. First, you have to fill out a form, and then I need to see som e identification. That's about all there is to it. That sounds easy enough. Thanks for your help.

2.

(A) (B) (C) (D)

A form. An account. A piece of identification. A check.

On the recording, you hear: (narrator)

1.

What type o f account does the w om an want?

2.

What does the m an need for her to show him?

W h en y o u read th e an sw ers to th e first q u e s tio n , you can a n tic ip a te th a t th e first q u e s tio n is: W hat type o f account ? As y o u listen , y o u d e te r m in e th a t th e w o m a n w an ts a checking ac­ count. T h e r e fo r e , y o u c a n a n tic ip a te th a t th e b e st answ er to th e first q u e s tio n is (A ) . W h e n y o u r ea d th e an sw ers to th e s e c o n d q u e s tio n , y o u c a n .a n ticip a te th a t th e se c ­ o n d q u e s tio n is g o in g to ask W hat th in g . . . ? In th e c o n v e r sa tio n , th e m an asks h e r to fill o u t a. fo r m a n d sh o w s o m e identification, so as y o u are lis te n in g y o u can a n tic ip a te th at th e c o r r e c t answ er to th e s e c o n d q u e s tio n is e ith e r (A) o r (C ). W h e n y o u h e a r th e q u e s tio n , y o u can d e te r m in e th a t th e b e st an sw er is an sw er (C ).

LISTENING COMPREHENSION

T O E F L E X E R C IS E 22: L isten to e a c h c o m p le te c o n v e r sa tio n a n d an sw er th e q u e s tio n s th a t follow . N O W BEGIN THE RECORDING PROGRAM AT TOEFL EXERCISE 22.

o 1.

(A) Find work on campus. (B) Work in the em ploym ent office. (C) H elp students find jobs. (D) Ask the woman questions.

2.

(A) (B) (C) (D)

3.

(A) N o m ore than ten. (B) At least twenty. (C) N ot m ore than twenty. (D) U p to ten.

4.

5.

In In In In

the library. a classroom. a cam pus office. an apartment.

(A) Every m orning. (B) A fternoons and weekends. (C) W hen h e ’s in class. (D) Weekdays. Fill out a form. Give her som e additional inform ation. (C) Tell her som e news. (D) Phone her. (A) Just before a vacation. (B) Just after the end o f a school semester. (C) At the end o f the summer. (D) Just after a break from school.

7.

(A) (B)

A trip to visit the Eskimos. A trip the woman is planning to take. (C) A trip the man has already taken. (D) A cam ping trip the man and woman took. (A) (B) (C) (D)

T hree T hree T hree T hree

hours. com plete days. classes. weeks.

(A) Sleeping outside on the ground. (B) Spending time in a h ot tub. (C) Relaxing at the lodge. (D) Enjoying excellent food.

10.

S h e’d b e scared, but she’d like to try. (B) She can ’t wait. (C) It w ould be quite exciting for her. (D) She’d prefer not to try.

11.

(A) (B)

12.

(A) Nuclear power. (B) Electricity. (C) Burning coal and oil. (D) Solar power.

13.

(A)

14.

(A) (B) (C)

15.

(A)

(A) (B)

6.

8.

9.

(A)

All kinds o f pollution. How acid rain has harm ed the earth. (C) Pollution from cars and factories. (D) T he causes and possible effects o f acid rain.

From sulfur dioxide and water vapor. (B) From sulfur dioxide and nitrogen oxide. (C) From nitric acid and sulfur dioxide. (D) From water vapor and nitric acid. Only in North America. At the N orth and South Poles. In parts o f several northern continents. (D) In equatorial areas. She should protect herself from the rain. (B) She should clean up the water supply. (C) She should read a novel. (D) She should get m ore information about acid rain.

LISTENING PART B

T O E F L REV IEW E X E R C ISE (S k ills 1 8 -2 2 ): In this e x e r c ise , y o u will u s e all o f th e in fo r ­ m a tio n th at y o u le a r n e d in S k ills 18 th r o u g h 22. B e fo re th e r e c o r d in g p ro g ra m b e g in s, y o u sh o u ld read over th e an sw ers to q u e s tio n s 1 th r o u g h 15 a n d d o th e fo llow in g: • A n ticip a te the topics you w ill hear. • A n ticip a te the questions. W h ile y o u are lis te n in g to th e c o n v e r sa tio n s, y o u sh o u ld d o th e follo w in g : • L isten fo r the topic in the fir s t lines. • D ra w conclusions about the situ a tio n (w h o, w hat, w h e n , w h er e). • L isten fo r the answers in order. N O W BEGIN THE RECORDING PROGRAM AT TOEFL REVIEW EXERCISE SKILLS

n

(A) (B) (C) (D) 2.

To To To To

(A) Laramie. (B) Devil’s Tower N ational M onum ent. (C) Old Faithful. (D) Wyoming.

a concert. a rehearsal. a lecture. the library.

(A) O ne. (B) Two. (C) Three. (D) Four.

( 18 -22).

9.

(A) (B)

(C) (D)

Hear again about Yellowstone. Take a trip to Yellowstone. Get a jo b in a national park. Move to Yellowstone.

How and when we celebrate Thanksgiving. (B) The traditional Thanksgiving dinner. (C) W hen Thanksgiving began. (D) Abraham Lincoln.

3.

(A)

T he bus does not go directly to the Music Building. (B) T he bus goes very slowly to the Music Building. (C) The bus som etim es does not com e. (D) T he bus will not arrive for a while.

10.

(A)

4.

(A) Walk. (B) Wait for the bus. (C) Miss the lecture. (D) T hink o f another plan.

11.

(A) (B)

5.

(A) Boring. (B) Fantastic. (C) Lengthy. (D) Faithful.

12 .

(A) (B) (C)

13.

(A) At many different times. (B) In July. (C) Any tim e in November. (D) On a Thursday in Novem ber.

6. (A) By car. (B) By plane. (C) By train. (D) By bicycle. 7. (A) She went directly to Yellowstone. (B) She spent a few weeks in Laramie. (C) She stopped at the Devil's Tower National M onum ent. (D) She m ade a few stops before going on to Yellowstone.

With colonists in Massachusetts. Alone and thinking abou t how Thanksgiving developed. (C) With a big Thanksgiving dinner. (D) In an untraditional manner. The terrible winter. The corn harvest. The develop m en t o f Thanksgiving Day. (D) For getting the w hole family together.

LISTENING COMPREHENSION

T H E LIS TE N IN G PART C Q U E S T IO N S P art C o f th e L iste n in g C o m p r e h e n s io n s e c tio n o f th e T O E F L test c o n sists o f th r e e talks, e a c h fo llo w e d by a n u m b e r o f q u estio n s. You w ill h e a r th e talks a n d th e q u e s tio n s o n a rec o r d in g ; th ey a re n o t w ritten in y o u r test b o o k . Y ou m u st c h o o s e th e b e st a nsw er to each q u e s tio n fro m th e fo u r c h o ic e s th a t are w ritten in y o u r te st b o o k . L ike th e c o n v e rsa tio n s in L iste n in g Part B, th e talks are o fte n a b o u t so m e a sp e c t o f s c h o o l life o r to p ic s cu rren tly in th e new s. It is a lso very c o m m o n fo r th e talks to b e s h o r te n e d v ersio n s o f le c tu r e s from c o u r se s ta u g h t in A m e ric a n c o lle g e s an d u n iv er sitie s.

Exam ple On the recording, you hear: (narrator) (woman)

Q uestions I th ro u g h 4. Listen to a talk about the settlement o f America. The settling o f the vast farm lands in central North America was delayed at least partly because o f an error by one man. In the early nineteenth century, Lieutenant Zebulon Pike o f the U.S. Army w as'sent out to explore and chart the huge expanses o f land in the center o f the continent. When he returned from his explorations, he wrote a report in which he erroneously stated that the vast plains in the central part o f the continent were desertlike, comparable to the Sahara in Africa. In reality, however, these vast plains contained some o f the m ost fertile farmland in the world. Because o f Pike’s mistake, the m aps o f the day depicted the central part o f what is today the United States as a vast desert rather than the excellent and available farm land that it was. This mistaken belief about the nature o f those lands caused settlers to avoid the central plains for years.

Questions: 1. On the recording, you hear: (narrator) In your test book, you read:

2.

What is the topic o f this talk? (A) Zebulon Pike’s career. (B) A mistake that influenced the settlement of America. (C) A report for the army. (D) The farmlands.

On the recording, you hear: (narrator) In your test book, you read:

How did Pike describe the area that he explored? (A) As a desert. (B) As usable for army purposes. (C) As located in the Sahara. (D) As available for farmland. (continued on next page)

LISTENING PART C

3.

On the recording, you hear: (narrator) In your test book, you read:

4.

W hat was this area really like? (A) (B) (C) (D)

It was It was It was It was

a vast desert. covered with farms. excellent farmland. similar to the Sahara.

On the recording, you hear: (narrator) In your test book, you read:

This talk w ould probably be given in which o f the following courses? (A) (B) (C) (D)

Agricultural Science. American History. Geology of the United States. Military Science.

T h e first q u e s tio n asks a b o u t th e to p ic o f th e talk. T h e to p ic o f th e talk is f o u n d in th e first s e n te n c e o f th e talk: T he settling o f the vast fa r m la n d s in central N o rth A m erica was delayed at least partly because o f a n error by one m an. T h e r e fo r e , th e b e st an sw er to th e q u e s tio n is (B ). T h e s e c o n d q u e s tio n is a d e ta il q u e s tio n th at asks h o w P ike d e sc r ib e d th is area. It is sta te d in th e talk th a t P ike wrote a report in w hich he erroneously stated th a t the v a st p la in s in the central p a r t o f the continent were desertlike. . . . T h e r e fo r e , th e b e st an sw er to th is q u e s tio n is ( A ) . T h e th ird q u e s tio n is a n a d d itio n a l d e ta il q u e s tio n th a t asks w h a t th e a r ea was really lik e. B e ca u se th e talk in d ic a te s th a t in reality . . . these vast p la in s contained some o f the most fertile fa r m la n d in the world, th e b e st an sw er to th is q u e s tio n is (C ) . T h e fo u r th q u e s tio n is an in fe r e n c e q u e s tio n . It asks in w h ic h c o u r se this le c tu r e w o u ld p ro b a b ly b e g iv e n . T h e w o r d probably in d ic a te s to y o u th at th e q u e s tio n is n o t a n sw ered d ir ec tly in th e talk. You m u s t draw a c o n c lu s io n fro m th e in fo r m a tio n in th e talk to an sw er th is q u e s tio n . B e ca u se th is talk r efe rs to the early nineteenth century a n d d isc u sse s th e settling o f the v a st fa r m la n d s in central N o rth A m erica, it w o u ld p ro b a b ly b e g iv e n in a n A m e ric a n H is to r y c o u r se . T h e b e st an sw er to th is q u e s tio n is ( B ) .

_________________ STRATEGIES FORTHE LISTENING PART C QUESTIONS_________________ 1. If you have tim e , p review th e an sw ers t o th e L istening Part C q u estio n s. While you are looking at the answers, you should try to do the following: •Anticipate the to p ic s of the talks you will hear. •Anticipate the q u estio n s for each of the groups of answers. 2. Listen carefully to th e first lin e o f th e talk. The first line of the talk often contains the main idea, subject, or topic of the talk, and you will often be asked this type of question. 3. A s you listen to th e talk, draw c o n c lu sio n s a b o u t th e situ a tio n o f th e talk: w h o is talking, w h ere o r w h en th e talk ta k es p la ce, w h ich c o u r se th is lec tu r e m ig h t be given in. You will often be asked to make such inferences about the talk. 4. A s you listen to th e talk, follow alon g w ith th e an sw ers in your t e s t b o o k r.nd try to d e te r m in e th e c o r r e c t an sw ers. Detail questions are generally answered in order In the talk, and the answers often sound the same as what Is said on the recording. 5. You sh ould g u e ss even if you are n o t su re . Never leave any answers blank. 6. U se any rem aining tim e to look ahead a t th e answ ers to th e q u estio n s th a t follow .

LISTENING COMPREHENSION

T h e fo llo w in g sk ills w ill h e lp y o u to im p le m e n t th e s e str a teg ie s in P art C o f th e L iste n in g C o m p r e h e n s io n se c tio n o f th e T O E F L test.

BEFORE L IS T E N IN G ________________________________________ S k i l l 23:

A N TIC IPA TE TH E TOPICS

It is very h e lp fu l to you r overall c o m p r e h e n sio n if y o u k n ow w h at to p ic s to e x p e c t in Lis­ te n in g P art C. You sh o u ld th e r efo r e try to a n tic ip a te th e to p ics th a t you w ill be h e a rin g (as y o u d id in L iste n in g P art B ). F or e x a m p le , are th e talks a b o u t A m erica n history, o r lit­ era tu re, o r so m e a sp ec t o f sc h o o l life? A h e lp fu l strategy is th e r efo r e to lo o k b riefly at the answ ers in th e test b o o k , b e fo r e y o u a ctu ally h ea r th e talks o n th e r e c o r d in g , a n d try to d e te r m in e th e to p ic s o f th e talks th a t you w ill hear. EX ER C ISE 23: L o o k at th e answ ers to th e five q u e stio n s to g e th er , a n d try to a n ticip a te th e to p ic o f th e talk for th o s e five q u estio n s. (O f c o u r se , y o u c a n n o t always d e te r m in e e x ­ actly w h a t th e to p ic is, b u t y o u o fte n can g e t a g e n e r a l id e a .) Q u e stio n s 1 th r o u g h 5 have b e e n a n sw ered fo r you. During a biology laboratory session. (B) In a biology study group. (C) On the first day o f class. (D) Just before the final exam.

(A)

4.

(A) Room assignments. (B) Exam topics. (C) Reading assignments. (D) T he first lecture.

5.

(A) (B) (C)

(A) O nce a week. (B) Two times a week. (C) T hree times a week. (D) For fifteen hours.

Exams and lab work. Reading and writing assignments. Class participation and grades on examinations. (D) Lecture and laboratory attendance.

To do the first laboratory assignment. (B) To take the first exam. (C) To study the laboratory manual. (D) To read o n e chapter o f the text.

(A)

W h a t is th e to p ic o f th e talk for q u e s tio n s 1 th r o u g h 5? the requirements of a biology class You can g u e ss this b e c a u se o f th e fo llo w in g clues: • biology • fir s t day o f class • reading assignm ents • exams • lab work

LISTENING PART C

6.

(A) What caused the Ring o f Fire. (B) The volcanoes o f the Ring o f Fire. (C) Hawaiian volcanoes. (D) D ifferent types o f volcanoes.

7.

(A) (B)

8.

The Ring o f Fire. The characteristics o f volcanoes in the Ring o f Fire. (C) The volcanoes o f Hawaii. (D) Mauna Loa.

(A) (B) (C) (D)

9.

10.

They are not so violent. They are located alon g the Ring o f Fire. (C) They contain a lo t o f gas. (D) They contain thick lava.

(A) (B)

(A) A volcano on the Ring o f Fire. (B) An island in Hawaii. (C) A long, low volcanic m ountain. (D) An explosive volcano.

In Hawaii. In the U nited States. A long the Ring o f Fire. Within the Ring o f Fire.

W hat is th e to p ic o f th e talk fo r q u e s tio n s 6 th r o u g h 10?

11.

(A) (B) (C) (D)

An artist. A tour guide. An Indian. Orville Wright.

12.

(A) (B) (C) (D)

Several. Sixty thousand. Sixteen m illion. M illions and millions.

(A)

The National Air and Space Museum. The Museum o f Natural History. The American History Museum. The Sm ithsonian Arts and Industries Building.

13.

(B) (C) (D)

14.

(A) (B)

T he American H istory M useum. T he Sm ithsonian Arts and Industries Building. (C) T he W ashington M useum. (D) The National Air and Space Museum.

15.

(A) (B) (C) (D)

To the White H ouse. To the Sm ithsonian. To the mall. To various other m useum s.

W hat is th e to p ic o f th e talk for q u e s tio n s 11 th r o u g h 15?

LISTENING COMPREHENSION

S k il l

24:

A N TIC IP A TE T H E Q U E S T IO N S

It is very h e lp fu l to y o u r ability to an sw er in d iv id u a l q u e stio n s in L iste n in g P art C i f y ou ca n a n tic ip a te w h at th e q u e s tio n s w ill b e a n d listen sp ec ific a lly fo r th e an sw ers to th o s e q u e s tio n s (as you d id in L iste n in g P art B ).

Exam ple In your test book, you read: (A) For three weeks. (B) For three days. (C) For three months. (D) For three hours. You try to anticipate the question: How long does (something) last?

In th is e x a m p le, y o u can b e q u ite c e r ta in th a t o n e o f th e q u e stio n s w ill b e a b o u t h o w lo n g so m e th in g lasts. S in c e you are su re th a t th is is o n e o f th e q u e s tio n s, y o u c a n liste n ca re­ fully for c lu e s th a t w ill give y o u th e answ er. T h is e x a m p le sh ow s th a t a h e lp fu l strategy is th e r e fo r e to lo o k b riefly at th e an sw ers in th e test b o o k , b e fo r e y o u actu ally h e a r th e talks o n th e rec o rd in g , a n d try to d e te r m in e th e q u e s tio n s that y o u will b e ask ed to answer.

EXER CISE 24: Study th e fo llo w in g answ ers a n d try to d e te r m in e w h a t th e q u e s tio n s will be. (You sh o u ld n o te th at p e r h a p s y o u w ill o n ly be ab le to p r e d ict p art o f a q u e stio n , rather th an th e c o m p le te q u e s tio n .) If y o u c a n n o t p r e d ict th e q u estio n in a sh o rt p e r io d o f tim e, th e n m ove o n to th e n e x t g r o u p o f answ ers. Q u e stio n 1 has b e e n an sw ered fo r you.

1

rw stin n (A) (B) (C) (D)

When does the talk probably take place?_________________________________ •

During a biology laboratory session. In a biology study group. O n the first day o f class. Just before the final exam.

2.

Q uestion:_______________________________ ________________________________________________ (A) O nce a week. (B) Two times a week. (C) Three tim es a week. (D) For fifteen hours.

3.

Q uestion:______________________________ _________________________________________________ (A) To do the first laboratory assignment. (B) To take the first exam. (C) To study the laboratory manual. (D) To read o n e chapter o f the text.

4.

Q uestion:______ ___________________________ ______________________________________________ (A) Room assignments. (B) Exam topics. (C) Reading assignments. (D) T he first lecture.

LISTENING PART C

5.

Q u estion :_____________________________________________ (A) Exams and lab work. (B) Reading and writing assignm ents. (C) Class participation and grades on exam inations. (D) Lecture and laboratory attendance.

6.

Q uestion:_____________________________________________ (A) What caused the Ring o f Fire. (B) The volcanoes o f the Ring o f Fire. (C) Hawaiian volcanoes. (D) D ifferent types o f volcanoes.

7.

Q uestion:_____________________________________________ (A) The Ring o f Fire. (B) The characteristics o f volcanoes in the Ring o f Fire. (C) The volcanoes o f Hawaii. (D) Mauna Loa.

8.

Q u estion :_____________________________________________ (A) In Hawaii. (B) In the U nited States. (C) A long the Ring o f Fire. (D) Within the Ring o f Fire.

9.

Q uestion:_____________________________________________ (A) They are not so violent. (B) They are located along the Ring o f Fire. (C) They contain a lot o f gas. (D) They contain thick lava.

10.

Q u estion :_____________________________________________ (A) A volcano on the Ring o f Fire. (B) An island in Hawaii. (C) A long, low volcanic mountain. (D) An explosive volcano.

11.

Q u estion :_____________________________________________ (A) An artist. (B) A tour guide. (C) An Indian. (D) Orville Wright.

12.

Q uestion:_____________________________________________ (A) Several. (B) Sixty thousand. (C) Sixteen million. (D) M illions and m illions.

13.

Q u estion :_____________________________________________ (A) T he National Air and Space Museum. (B) The M useum o f Natural History. (C) T he American History M useum. (D) T he Sm ithsonian Arts and Industries Building.

107

LISTENING COMPREHENSION

14.

Q u estion :_________________________________ ______ (A) The American History Museum. (B) The Sm ithsonian Arts and Industries Building. (C) The Washington Museum. (D) The National Air and Space Museum.

15.

Q u estion :________________________________________ (A) To the White House. (B) To the Smithsonian. (C) To the mall. (D) To various other museums.

W H IL E LIS TE N IN G S k i l l 25:

DETERM INE TH E TO PIC

As you listen to ea ch talk in L isten in g Part C, y o u sh o u ld b e th in k in g a b o u t th e to p ic (su b ­ je c t) o r m a in id e a fo r th e talk (as you d id in L iste n in g Part B ). S in ce th e first se n te n c e is g e n e ra lly a to p ic se n te n c e , y o u sh o u ld b e ask in g y o u r se lf w hat th e to p ic is w h ile you are lis te n in g ca refu lly to th e first p art o f th e talk.

Exam ple On the recording, you hear: (man)

The major earthquake that occurred east o f Los Angeles in 1971 is still affecting the economy o f the area today.

You think: The topic of the talk is the effect of the 1971 earthquake on Los Angeles today.

E X ER C ISE 25: L isten to th e first part o f e a c h o f th e talks, a n d d e c id e o n th e to p ic o f e a ch talk. N O W BEGIN THE RECORDING PROGRAM AT EXERCISE 25.

1.

What is the topic o f Talk 1?

2. What is the topic o f Talk 2?

3. What is the topic o f Talk 3?

LISTENING PART c

S k i l l 26:

D R A W C O N C L U S IO N S A B O U T W HO , WHAT, W H EN , W H ER E

As y o u listen to e a c h talk in L iste n in g Part C , y o u sh o u ld b e tr y in g to se t th e situ a tio n in you r m in d (a s y o u d id in L iste n in g Part B ). You s h o u ld b e th in k in g th e fo llo w in g th ou gh ts: • • • • •

W ho is talkin g ? W hen does the talk probably take place? W here does the talk probably take place? W h a t course is the talk concerned w ith? W h a t is the source o f in fo rm a tio n fo r the talk?

Exam ple On the recording, you hear: (woman)

The next stop on our tour o f Atlanta will be the original hom e o f Coca-Cola, at 107 Marietta Street. Coca-Cola was manufactured at this location un til early in September o f 1888.

You think: (a tour guide) (in Atlanta) (in the middle of a tour)

Who is probably talking? Where are they? When does the talk take place?

EX ER C ISE 26: L iste n to th e first p art o f e a c h o f th e talks a n d try to im a g in e th e situ a ­ tio n . T h e n an sw er th e q u e s tio n s in th e text. o

N O W BEGIN THE RECORDING PROGRAM AT EXERCISE

Talk 1 1. W ho is probably talking?

26. _____________________________________________

2.

W here does the talk probably take p l a c e ? _____________________________________________

3.

W hen does the talk probably take place?

_____________________________________________

4.

What course is b ein g discussed?

_____________________________________________

Talk 2 1. W ho is probably talking?

_____________________________________________

2.

W here does the talk probably take place?

_____________________________________________

3.

W hen does the talk probably take place?

_____________________________________________

4.

What course is b ein g discussed?

---------------------------------------------------------------------

Talk 3 1. W ho is probably talking?

---------------------------------------------------------------------

2.

W here does the talk take place?

_____________________________________________

3.

W hen d oes the talk take place?

_____________________________________________

LISTENING COMPREHENSION

S k i l l 27:

LISTEN FOR ANSW ERS IN ORDER

T h e r e are tw o p o ssib le m e th o d s to u se w h ile y o u liste n to th e talks in L iste n in g Part C. • You can j u s t listen to the talk (a n d ignore the answers). * You can follow along w ith the answers w hile you listen. S o m e stu d e n ts p refer to ju s t listen to th e talk w h ile it is b e in g sp o k e n , a n d i f th a t m e th o d w orks w ell fo r y o u , th e n th at is w h at y o u sh o u ld d o . O th e r stu d e n ts fin d th a t th ey can a n ­ sw er m o r e q u e s tio n s co rrectly i f they read a lo n g w ith th e answ ers w h ile th e talk is b e in g g iv en . B e ca u se th e d etail q u e stio n s are a n sw ered in ord er, it is p o ssib le to read a lo n g w h ile y o u lis te n to th e talk in th e r e c o r d in g p rogram .

Exam ple On the recording, you hear:

In your test book, you read (same time):

(woman)

1.

(A) (B) (C) (D)

2.

(A) The dry weather prior to the fire made it worse. (B) It happened during the summer. (C) Chicago’s winds made it worse. (D) It killed many people.

The Great Chicago Fire began on October 8, 1871, and, according to legend, began when a cow knocked over a lantern in Mrs. O ’Leary’s bam . No matter how it began, it was a disastrous fire. The preceding sum m er had been exceedingly dry in the Chicago area, and the extreme dryness accompanied by Chicago’s infam ous winds created an inferno that destroyed 18,000 buildings and killed more than 300 people before it was extinguished the following day.

In a bam. In Mrs. O’Leary’s home. In a cow pasture. In a lantern factory.

On the recording, you hear: (narrator)

Ỉ. According to legend, where did the Great Chicago Fire begin ? 2.

Which o f the following is not true about the Great Chicago Fire?

W h en you read th e answ ers to th e first q u e stio n , y o u can an ticip a te th a t th e first q u estio n is: Where did som ething happen'? As you listen , y o u d e te r m in e th at th e fire b e g a n in M rs. O 'Leary's barn. T h e r e fo r e , y o u can a n tic ip a te that th e b est answ er to th e first q u estio n is (A ). I f you rea d th e answ ers to th e se c o n d q u e s tio n w h ile y o u listen to th e talk, y o u can d e ­ te r m in e th at answ ers (A ), (C ), an d (D ) are tru e. A n sw er (B ) is n o t true: th e fire d id n o t b e g in in th e su m m er, it b e g a n in October, w h ich is in th e a u tu m n . T h e r e fo r e , answ er (B ) is th e b e st answ er to th e q u e stio n Which o f the fo llo w in g is n o t true about the Great Chicago F ire?

LISTENING PART C

T O E F L EXER CISE 27: L isten to e a c h c o m p le te talk a n d answ er th e q u e s tio n s th a t follow , n

N O W BEGIN THE RECORDING PROGRAM AT TOEFL EXERCISE 27.

1.

During a biology laboratory session. (B) In a biology study group. (C) On the first day o f class. (D) Just before the final exam.

(A)

2.

(A) (B) (C) (D)

3.

To do the first laboratory assignment. (B) To take the first exam. (C) To study the laboratory m anual. (D) To read o n e chapter o f the text.

4.

(A) Room assignments. (B) Exam topics. (C) Reading assignments. (D) T he first lecture.

5.

Exams and lab work. Reading and writing assignments. Class participation and grades on exam inations. (D) Lecture and laboratory attendance.

6.

(A) What caused the Ring o f Fire. (B) T h e volcanoes o f the Ring o f Fire. (C) Hawaiian volcanoes. (D) Different types o f volcanoes.

7.

(A) (B)

8.

O nce a week. Two times a week. Three times a week. For fifteen hours.

(A)

9.

(C) (D)

They are not so violent. They are located along the Ring o f Fire. They contain a lot o f gas. They contain thick lava.

10.

(A) (B) (C) (D)

A volcano on the Ring o f Fire. An island in Hawaii. A long, low volcanic m ountain. An explosive volcano.

11.

(A) (B) (C) (D)

An artist. A tour guide. An Indian. Orville Wright.

12.

(A) (B) (C) (D)

Several. Sixty thousand. Sixteen m illion. M illions and m illions.

13.

T he National Air and Space Museum. (B) T he Museum o f Natural History. (C) T he Am erican H istory M useum. (D) T he Sm ithsonian Arts and Industries Building.

14.

(A) (B)

15.

(A) (B) (C) (D)

(A) (B) (C)

T h e Ring o f Fire. T h e characteristics o f volcanoes in the Ring o f Fire. (C) T he volcanoes o f Hawaii. (D) Mauna Loa. (A) In Hawaii. (B) In the U n ited States. (C) A long the Ring o f Fire. (D) Within the Ring o f Fire.

(A) (B)

(A)

T he Am erican H istory M useum. The Sm ithsonian Arts and Industries Building. (C) The W ashington M useum. (D) T he National Air and Space Museum. To To To To

the W hite H o u « . the Sm ithsonian. the mall. various other m useum s.

LISTENING COMPREHENSION

T O E F L REV IEW E X E R C ISE (S k ills 2 3 -2 7 ): In th is e x e r c ise , y o u w ill u se all o f th e in fo r­ m a tio n th a t y o u le a r n e d in Skills 2 3 th r o u g h 27. B e fo r e th e r e c o r d in g p rogram b e g in s, y o u s h o u ld read over th e answ ers to q u e stio n s 1 th r o u g h 12 a n d d o th e follo w in g : • A n tic ip a te the topics you w ill hear. • A n tic ip a te the questions. W h ile y o u are lis te n in g to th e talks, you sh o u ld d o th e follow in g: • L isten fo r the topic in the fir s t sentence. • D raw conclusions about the situ a tio n (w ho, w h at, w h e n , w h er e). • Listen f o r the answers in order. n

N O W BEGIN THE RECORDING PROGRAM AT TOEFL REVIEW EXERCISE (SKILLS 23-27).

O ther librarians. Undergraduate students. Students who are not in the business department. (D) Graduate business students.

(A) (B) (C)

2.

(A) (B) (C) (D)

It opens at 7:00 A.M. It closes at 7:00 P.M. It closes at midnight. It is always open.

3.

(A)

Com puter area and business materials. Magazines and newspapers. Business departm ent and library staff offices. First and second floors o f the library.

(B) (C) (D)

Go hom e. Return to class. Work on the computers. Tour the library.

4.

(A) (B) (C) (D)

5.

(A) A student in health services. (B) A drug abuse lecturer. (C) A derm atologist. (D) A representative o f the tobacco industry.

6.

How to reduce nicotine and other addictions. (B) How stress affects the skin. (C) The effects o f alcohol on health. (D) How to achieve optim al health. (A)

7. (A) Alcohol. (B) Nicotine. (C) Caffeine. (D) A reduced supply o f blood. g (A)

It increases the flow o f blood to the skin. (B) It causes increased consum ption of alcohol. (C) It prevents the skin from receiving enough nourishm ent. (D) It causes stress.

9. (A) (B) (C)

Before the Civil War. At the end o f the Civil War. At the beginning o f the twentieth century. (D) W ithin the last decade.

10. (A) (B)

The Civil War ended. T he U.S. governm ent issued a large am ount o f paper currency. (C) T he price o f gold plum m eted. (D) T he value o f gold becam e inflated.

11.

(A) (B) (C) (D)

The The The T he

president. president’s brother. president’s brother-in-law. president’s wife.

12.

(A) Issue greenbacks. (B) Sell gold. (C) Corner the gold markeL (D) H old its gold reserves.

l D l D l D l D l D l D l D l TOEFL POST-TEST SECTION 1 LISTENING COMPREHENSION T im e — a p p r o x im a te ly 3 5 m in u te s ( in c lu d in g th e r e a d in g o f t h e d ir e c t io n s fo r e a c h p a r t) In this section of the test, you will have an opportunity to demonstrate your ability to understand conversations and talks in English. There are three parts to this section, with special direotions for each part. Answer all the questions on the basis of what is stated or im plied by the speakers you hear. Do not take notes or write in your test book at any time. Do not turn the pages until you are told to do so.

P art A D irections: In Part A you will hear short conversations between two people. After each conversation, you will hear a question about the conversation. The conversations and questions will not be repeated. After you hear a question, read the four possible answers in your test book and choose the best answer. Then, on your answer sheet, find the number o f the question and fill in the space that corresponds to the letter of the answer you have chosen. Listen to an exam ple.

Sam p le Answer

® CD

On the recording, you w ill hear: (m an) (w om an) (narrator)

©

That exam was just awful. Oh, it could have been worse. What does the w om an mean?

In your test book, you w ill read:

(A) (B) (C) (D)

The exam was really awful. It was the worst exam she had ever seen. It couldn't have been more difficult. It wasn't that hard.

You learn from the conversation that the man thought the exam was very difficult and that the woman disagreed with the man. The best answer to the question, "What does the wom an mean?" is (D), "It wasn’t that hard.’’ Therefore, the correct choice is (D).

TOEFL* test directions an d form at are rep rin ted by perm ission o f ETS, the copyright owner. However, all exam ples and test questions are provided by P earson Education, Inc.

LISTENING COMPREHENSION POST-TEST

□I 1. (A) (B) (C) (D)

d

I

d

I

Hell correct the exams this afternoon. The exam will be at noon. He will collect the exams at 12:00. The tests have not yet been graded.

2. (A) Martha applied for a visa last month. (B) Martha's visa will last for only a month. (C) Martha arrived last month without her visa. (D) Martha’s visa was already delivered. 3. (A) The professor described what the students should do. (B) There was a long line to register for the required class. (C) The professor required an outline. (D) The professor lined up for retirement. 4. (A) (B) (C) (D)

Chuck had improved. This visit was better than the last. Chuck looked at him in the hospital. Chuck didn't seem to be doing very well.

5. (A) She thinks the tuition should be increased. (B) The semester’s tuition is quite affordable. (C) It costs too much. (D) She has more than enough for tuition. 6. (A) He thinks he got a good grade. (B) The history grades were all C or above. (C) No one got history grades. (D) All the grades were C or lower. 7. (A) The parking lots were full before 10:00. (B) It was impossible to start class by 10:00. (C) He parked the car before class at 10:00. (D) The possibility of finding a place to park increased.

d

I

d

I

d

I

8. (A) (B) (C) (D)

She’s found a new ring. She would like a hug. She’s shopping for a carpet. She’s thankful she has a rag.

9. (A) (B) (C) (D)

In a department store. In a bank. In an accounting firm. In a checkout line.

d

10. (A) Jane usually visits San Francisco for her vacations. (B) Jane’s cousin often visits San Francisco. (C) Whenever there’s a holiday, Jane's cousin goes to San Francisco. (D) Whenever there’s a holiday, Jane leaves San Francisco. 11. (A) (B) (C) (D)

He wishes he had something tc eat. He hopes he won’t eat for weeks. He wishes he hadn't eaten so much. He wishes he weren’t eating.

12. (A) Traffic should not be allowed. (B) She thinks that the traffic should stay outside. (C) She agrees that the traffic is noisy. (D) She’ll stay outside with the man. 13. (A) The headings for today's reading assignment. (B) The chance to make the headlines. (C) Her reading ability. (D) The daily newspaper. 14. (A) The bus trip is only five minutes long. (B) The man missed the bus by five minutes. (C) The man doesn’t have time to waste. (D) The bus was five minutes late. 15. (A) (B) (C) (D)

It’s not possible to pass the class. She'll definitely fail. It's always possible. She shouldn't say anything about the class.

GO ON TO THE NEXT PAGE

I 14

LISTENING COMPREHENSION POST-TEST

lDlDlnlnlnlnlnl 16. (A) (B) (C) (D)

She gave Tom money to pay the rent. She was given money for the rent. Tom borrowed money for the rent. She had som e money to lend.

17. (A) (B) (C) (D)

The cake is extremely good. He never tasted the cake. He wished he hadn't tasted the cake. The cake has never been very good.

18. (A) At the com er she ran into another car. (B) She ran to Carl because she cared. (C) She unexpectedly met one of her relatives. (D) Carl was running from place to place. 19. (A) She shouldn't leave her purse here. (B) She’s probably in the apartment. (C) Her purse must not be in the apartment. (D) She left without taking her purse. 20. (A) The landlord failed to collect rent on the first of last month. (B) The tenants absolutely must pay rent by the first o f the month. (C) The landlord will not fail to collect your rent on the first o f next month. (D) It is important to call the landlord about rent on the first of the month. 21. (A) (B) (C) (D)

Taking the car out for a test drive. Listening to the noises. Fixing the car herself. Getting the car repaired.

22. (A) (B) (C) (D)

Martha’s jobs are easy. It’s easy to hold two jobs. It’s better for Martha to have two jobs. Martha should slow down.

23. (A) The plane took off just after he arrived. (B) He arrived just after the plane took off. (C) He wasn’t in time to catch the plane. (D) He arrived too late to catch the plane.

24. (A)

He agrees with the wom an’s suggestion. (B) Parking is not free on the weekend. (C) It is not necessary for them to park. (D) He thinks they don’t have to pay.

25. (A) He is eager to leave his job. (B) He is unhappy at the thought of retiring. (C) He couldn’t be unhappier about retiring. (D) He is retiring too soon. 26.

(A) He got the car he really wanted. (B) He didn't get a new car. (C) The car that he got was not his first choice. (D) He didn’t really want a new car.

27. (A)

Mr. Drew pointedly asked the president about the committee. (B) The president pointed to Mr. Drew’s head. (C) Mr. Drew became head of the new commission. (D) Mr. Drew was committed to the president’s appointments.

28. (A) She felt inferior. (B) She wasn’t furious. (C) She felt there should have been more fairness. (D) She was extremely angry. 29. (A) (B) (C) (D) 30.

The The The The

man would do the dishes. plates did not need to be washed. man would not be ready to go. dishes would not be done.

(A) He knew that grapes were cheaper than cherries. (B) He didn't know that grapes were cheaper than cherries. (C) He bought grapes because they were cheaper than cherries. (D) He didn't buy either grapes or cherries because of the price.

LISTENING COMPREHENSION POST-TEST

I 15

i D l D l D l D l ü l ü l ü l Part B D ire ctio n s: In this part of the test, you will hear longer conversations. After each conversation, you will hear several questions. The conversations and questions will not be repeated. After you hear a question, read the four possible answers in your test book and choose the best answer. Then, on your answer sheet, find the number o f the question and fill in the space that corresponds to the letter of the answer you have chosen. Remember, you are not allowed to take notes or write in your test book.

31. (A) (B) (C) (D)

Attend a football game alone. Go to a sporting event. Eat in the cafeteria and study. See a play.

32. (A) It’s the final game of the season. (B) It's better than the drama departments play. (C) It’s a very important game. (D) It’s close to the cafeteria.

35. (A) Trash orbiting Earth. (B) A trip by an astronaut to the Moon. (C) The overabundance of garbage on Earth. (D) Becoming space scientists. 36. (A) (B) (C) (D)

From a lecture. In a magazine article. In a book. On a television program. 17,000 pounds. 3,000 tons. 3,000 pounds. 300 tons.

33. (A) (B) (C) (D)

A play. A game. A study group meeting. Dinner in the cafeteria.

37. (A) (B) (C) (D)

34. (A) (B) (C) (D)

Saturday night. After dinner in the cafeteria. Sunday afternoon. Maybe next weekend.

38. (A) She will be able to travel in space. (B) The problem will take care of itself. (C) Scientists will find solutions to the problem. (D) The junk will fall to Earth.

GO ONTO THE NEXT PAGE

I 16

LISTENING COMPREHENSION POST-TEST

TOEFL® test directions and form at are reprinted by perm ission of ETS, the copyright owner. However, all exam ples and test questions are provided by Pearson Education, Inc.

i D l D l D l D l D l D l D l Part C D ire ctio n s: In this part of the test, you will hear several talks. After each talk, you will hear some questions. The talks and questions will not be repeated. After you hear a question, you will read the four possible answers in your test book and choose the best answer. Then, on your answer sheet, find the number of the question and fill in the space that corresponds to the letter of the answer you have chosen. H e re is a n ex am p le. O n th e re c o rd in g , yo u w ill h e ar: (n a rr a to r) (m an )

Listen to an instructor talk to his class about painting. Artist Grant Wood was a guiding force in the school o f painting know n as American regionalist, a style reflecting the distinctive characteristics o f art from rural areas o f the United States. Wood began drawing animals on the family farm at the age o f three, and w hen he was thirty-eight one o f his paintings received a remarkable a m ount o f public notice and acclaim. This painting, caller. “American Gothic, " is a starkly simple depiction o f a serious couple staring directly cut at the viewer.

Now listen to a sample question. (n a rra to r)

Sam p le Answer

What style o f painting is know n as American regionalist?

(B)

© I n y o u r te s t b o o k , you w ill re a d :

(A) (B) (C) (D)

Art from America s inner cities. Art from the central region of the U.S. Art from various urban areas in the U.S. Art from rural sections of America.

The best answer to the question, "What style o f painting is known as American regionalist?” is (D), "Art from rural sections of America.” Therefore, the correct choice is (D). Now listen to another sample question. (n a rra to r)

Sam p le Answer

What is the nam e o f Wood's m ost successful painting?

I n y o u r te st b o o k , you w ill re a d :

(A) (B) (C) (D)

“American Regionalist.” "The Family Farm in Iowa.” "American Gothic." "A Serious Couple.”

• (B)

The best answer to the question, "What is the name of Wood's most successful painting?” is (C), "American Gothic.” Therefore, the correct choice is (C). Remember, you are n o t allowed to take notes or write in your test book.

(w a it) TOEFL® test directions an d form at are reprinted by perm ission of ETS, th e copyright owner. However, all exam ples and test questions are provided by P earson Education, Inc.

LISTENING COMPREHENSION POST-TEST

I 17

i D l D l D l D l D l D l D l 39. (A) (B) (C) (D)

On the first day of class. In the middle of the semester. At the end o f class. In the final week of the semester.

46. (A) (B) (C) (D)

Preparing for a trip. Writing a report about the weather. Beginning a study of the weather. Buying warm clothes for a trip.

40. (A) (B) (C) (D)

Later today. By Friday o f this week. In two weeks. In three weeks.

47. (A) (B) (C) (D)

Modem American Authors. United States History. American Democracy. Nineteenth-Century American Literature.

41. (A) (B) (C) (D)

Journal and magazine articles. Books from outside the library. Books listed in student journals. Both books and journals.

48. (A) (B) (C) (D)

The death o f Abraham Lincoln. The beauty of American democracy. The raising of plants. The maturity of poetry.

42. (A) (B) (C) (D)

TVvo. Three. Five. Seven.

43. (A) (B) (C) (D)

In winter. In spring. In summer. In fall.

49. (A) It’s a poem about the author. (B) It’s a poem about Abraham Lincoln. (C) It’s a collection of twelve poems that remained unchanged. (D) It’s a volume of poetry that grew with its author.

44. (A)

Seasonable, with warm summers and cold winters. (B) Fairly constant and moderate. (C) Very humid. (D) Extremely hot year-round.

45. (A) (B) (C) (D)

50. (A) “Leaves of Grass." (B) "Song of Myself.” (C) “When Lilacs Last in the Dooryard Bloomed.” (D) “American Democracy."

They come from the southwest. They come most days of the year. They are the hardest during the night. They increase the humidity. This is the end of Section 1. Stop work on Section 1. Turn off the recording.

[st o p]

[st o p ]

[s t o p ]

STOP

[s t o p ]

[sto pJ

Isto pJ

W h e n y o u fin ish th e test, y o u m ay d o th e fo llow in g: • T u rn to th e D ia g n o stic C h art o n p a g e s 5 8 3 -5 8 4 , a n d cir cle th e n u m b e rs o f th e q u e s tio n s th a t y o u m issed . • T u rn to S c o r in g I n fo r m a tio n o n p a g e s 5 8 1 -5 8 2 , an d d e te r m in e your T O E FL sc o r e. • T u rn to th e P ro g r ess C hart o n p a g e 5 9 1 , a n d a d d y o u r sc o r e to th e charL

I 18

LISTENING COMPREHENSION POST-TEST

SECTION TW O

STRUCTURE AND WRITTEN EXPRESSION

120

2

»

*

2

*

2

2

*

2

«

»

2

2

«

2

DIAGNOSTIC PRE-TEST SECTION 2 STRUCTURE AND WRITTEN EXPRESSION T im e — 2 5 m in u t e s (in c lu d in g th e r e a d in g o f th e d ir e c t io n s ) N o w s e t y o u r c lo c k fo r 2 5 m in u te s . This section is designed to measure your ability to recognize language that is appropriate for standard written English. There are two types of questions in this section, with special directions for each type.

S tru c tu re D irections: These questions are incomplete sentences. Beneath each sentence you will see four words or phrases, marked (A), (B), (C), and (D). Choose the o n e word or phrase that best completes the sentence. Then, on your answer sheet, find the number o f the question and fill in the space that corresponds to the letter of the answer you have chosen. Look at the following examples. E x am p le I

S a m p le A nsw er

The president.

. the election by a landslide.

• ®©(§)

(A) won (B) he won (C) yesterday (D) fortunately The sentence should read, "The president won the election by a landslide.” Therefore you should choose answer (A). ’ E xam ple II When _ (A) (B) (C) (D)

S a m p le A nsw er . the conference?

© • © ( g )

the doctor attended did the doctor attend the doctor will attend the doctors attendance

The sentence should read, "When did the doctor attend the conference?” Therefore you should choose answer (B).

GO ON TO THE NEXT PAGE TOEFL» test directions and form at are reprinted by perm ission of ETS. the copyright owner. However, all exam ples and test questions are provided by Pearson E ducation. Inc.

f STRUCTURE AND WRITTEN EXPRESSION PRE-TEST

12 1

2

*

2

*

2

*

2

*

1. The North P ole _____ a latitude of 90 degrees north. (A) (B) (C) (D)

has is having which is having it has

2. _____ greyhound, can achieve speeds up to thirty-six miles per hour. (A) (B) (C) (D)

The The fastest The fastest dog The fastest dog, the

3. The Mayflower was bound for Virginia, but a hurricane_____ off course. (A) (B) (C) (D)

blew it to blow it it blew blowing it

4. The greenhouse effect occu rs--------heat radiating from the Sun. (A) when does the Earth’s atmosphere trap (B) does the Earths atmosphere trap (C) when the Earth’s atmosphere traps (D) the Earth's atmosphere traps 5. The Rose B ow l,______ place on New Year’s Day, is the oldest postseason collegiate football game in the United States. (A) (B) (C) (D)

takes which takes it takes took

6. Experim ents_______represent a giant step into the medicine of the future. (A) (B) (C) (D)

I 22

using gene therapy use gene therapy they use gene therapy uses

STRUCTURE AND WRITTEN EXPRESSION PRE-TEST

2

*

2

*

2

*

2

7. _____ off the Hawaiian coastline are living, others are dead. (A) (B) (C) (D)

Coral reefs Some types o f coral reefs There are many types o f coral reefs While some types of coral reefs

8. People who reverse the letters of words to read suffer from dyslexia. (A) (B) (C) (D)

if they tried when trying when tried if he tries

9. Featured at the Henry Ford M useum -------of antique cars dating from 1865. (A) (B) (C) (D)

an exhibit is an exhibit is an exhibit which is an exhibit

10. Rubber_____ from vulcanized silicones with a high molecular weight is difficult to distinguish from natural rubber. (A) (B) (C) (D)

is produced producing that produces produced

11. _____ in scope, romanticism was a reaction against neoclassical principles. (A) (B) (C) (D)

Mainly literary It was mainly literary The main literature was The literature was mainly

12. The Central Intelligence Agency (CIA) _____ came about as a result of the National Security Act of 1947. (A) (B) (C) (D)

what it was was what it was what

2

»

2

*

2

*

2

13. Oil shale is a soft, fine-grained sedimentary rock-------- oil and natural gas are obtained. (A) (B) (C) (D)

from is from is which from which

14. --------- appears considerably larger at the horizon than it does overhead is merely an optical illusion. (A) (B) (C) (D)

»

2

»

2

»

2

»

2

15. According to the World Health Organization,_____ there to be an outbreak of any o f the six m ost dangerous diseases, this could be cause for quarantine. (A) (B) (C) (D)

were they were there were were they

The Moon That the Moon When the Moon The Moon which

STRUCTURE AND WRITTEN EXPRESSION PRE-TEST

123

2

*

2

*

2

*

2

*

*

2

2

*

2

«

2

W r it t e n E x p r e s s io n D irections: In these questions, each sentence has four underlined words or phrases. The four underlined parts of the sentence are marked (A), (B), (C), and (D). Identify the o n e underlined word or phrase that must be changed in order for the sentence to be correct. Then, on your answer sheet, find the number of the question and fill in the space that corresponds to the letter of the answer you have chosen. Look at the following examples. Exam ple I

Sam ple Answer

The four string on a violin are tuned A B C D in fifths. The sentence should read, "The four strings on a violin are tuned in fifths.” Therefore, you should choose answer (B). Exam ple II The research for the book Roots taking A B C

Sam ple Answer

® d ) # ®

Alex Haley twelve years. D The sentence should read, "The research for the book Roots took Alex Haley twelve years." Therefore, you should choose answer (C).

124

STRUCTURE AND WRITTEN EXPRESSION PRE-TEST

TOEFL* test directions an d form at are reprinted by perm ission o f ETS, the copyright owner. However, all examples an d test questions arc provided by Pearson Education, Inc.

2

* 16.

2

*

Segregation

2

*

2

*

2

in public schools was declare ~A B

*

2

*

2

*

2

unconstitutionalby the Supreme Courtin C D

1954.

17.

Sirius, the Dog Star, is the most brightest star in the sky with an absolute magnitude A B about twenty-three times that of the Sun. —C D~

18.

Killer whales tend to wander in family clusters that hunt, play, and resting together. “A B C D

19.

Some of the

20.

The community of Bethesda, Maryland, was previous known as Darcy’s Store. A B C D

21.

J. H. Pratt used group therapy early in the past century when he brought A B C

m ost useful resistor material are carbon, metals, and metallic alloys. A B C D

tuberculosis patients together to discuss its disease. TT 22. 23.

Alloys of gold and copper have been widely using in various types of coins. ~~K~ B C

D

The United States has import all carpet w ools in recent years because dom estic A B C wools are too fine and soft for carpets. D

24.

Banks are rushing to merge because consolidations enable them to slash theirs costs A B C~ and expand. D

25.

That water has a very high specific heat means that, without a large temperature ~~A~ B change, water can add or lose a large number of heat. C D

26.

Benny Goodman was equally talented as both a jazz performer as well ac a classical A B C musician.

b

s

s

b

}

STRUCTURE AND WRITTEN EXPRESSION PRE-TEST

I25

2

« 27.

2

*

2

»

2

*

2

*

2

«

2

*

No longer satisfied with the emphasis of the Denishawn School, Martha Graham A B C is moving to the staff of the Eastman School in 1925. D

28.

Irving Berlin wrote "Oh, How I Hate to Get Up in the Morning" while serving in a A E C U.S. Army during World War I. D

29.

Shortly before the Allied invasion of Normandy, Ernest Hemingway has gone to A B C London as a war correspondent for Collier's. D

30.

During the 1960s, the Berkeley campus of the University o f California came to A national attention as a result its radical political activity. “I T C D

31.

Because of the flourish with which John Hancock signed the Declaration of A Independence, his name become synonymous with signature. IT ' C D

32.

On the floor of the Pacific Ocean is hundreds of flat-topped mountains more than a ~~A~ B C D mile beneath sea level.

33.

William Hart was an act best known for his roles as western heros in silent films. “A B ~C 17

34.

Prior to an extermination program early in the last century, alive wolves roamed A B C across nearly all of North America. D

35.

The state seal still used in Massachusetts designed by Paul Revere, who also ~~A~ B ' “ Cdesigned the first Continental currency. D

36.

Artist Gutzon Borglum designed the Mount Rushmore Memorial and worked on A project from 1925 until his death in 1941. B —C~~ TT

126

STRUCTURE AND WRITTEN EXPRESSION PRE-TEST

2

2

» 37.

*

2

2

*

2

»

2

«

2

«

2

*

2

It is proving less costly and more profitably for drugmakers to market directly to A B C D patients.

38.

Sapphires weighing as much as two pounds have on occasion mined. A B C D

39.

Like snakes, lizards can be found on all others continents except Antarctica. A B C D

40.

Banks, savings and loans, and finance companies have recently been doing hom e A B equity loans with greater frequency than ever before. C D

This is the end of the Structure and Written Expression Pre-Test.

W h e n y o u fin ish th e test, y o u m ay d o th e fo llow in g: • T u r n to th e D ia g n o s tic C h art o n p a g e s 5 8 5 —5 8 7 , an d c ir c le th e n u m b e r s o f th e q u e s tio n s th a t y o u m issed . • T u r n to S c o r in g I n fo r m a tio n o n p a g e s 5 8 1 -5 8 2 , an d d e te r m in e y o u r T O E F L sco re. • T u r n to th e P r o g r e s s C h art o n p a g e 5 9 1 , a n d a d d y o u r sc o r e to th e chart.

STRUCTURE AND WRITTEN EXPRESSION PRE-TEST

I 27

±

STRUCTURE AND WRITTEN EXPRESSION T h e s e c o n d se c tio n o f th e T O E FL te st is th e S tru ctu re a n d W ritten E x p r e ssio n se c tio n . T h is se c tio n c o n sists o f forty q u e stio n s (so m e tests m ay b e lo n g e r ). You have twenty-five m in u te s to c o m p le te th e forty q u e s tio n s in th is se c tio n . T h e r e are tw o types o f q u e stio n s in th e S tru ctu re a n d W ritten E x p r e ssio n se c tio n o f th e T O E F L test: 1.

2.

S tru ctu re (q u e s tio n s 1 -1 5 ) c o n sists o f fifte e n s e n te n c e s in w h ic h part o f th e s e n ­ te n c e h a s b e e n r e p la c e d w ith a blan k. E ach s e n te n c e is fo llo w e d by fo u r answ er c h o ic e s . You m u st c h o o s e th e answ er th a t c o m p le te s th e s e n te n c e in a gram m ati­ cally c o r r e c t way. W ritten E x p r e ssio n (q u e stio n s 1 6 -4 0 ) c o n sists o f tw enty-five s e n te n c e s in w h ich fo u r w ord s o r g r o u p s o f w ords have b e e n u n d e r lin e d . You m u st c h o o s e th e u n d e r ­ lin e d w ord o r g r o u p o f w ords th a t is not co rr ec t.

GENERAL STRATEGIES

1. Be familiar with the directions. The directions on everyTOEFL test are the same, so it is not necessary to spend time reading the directions carefully when you take the test.You should be completely familiar with the directions before the day of the test.

2. Begin with questions I through 15. Anticipate that questions I through S will be the easiest. Anticipate that questions I I through 15 will be the most difficult. D o not spend too much time on questions I I through 15. There will be easier questions that come later.

3. Continue with questions 16 through 40. Anticipate that questions 16 through 20 will be the easiest. Anticipate that questions 36 through 40 will be the most difficult. Do not spend too much time on questions 36 through 40. 4. If you have tim e, return to questions I I through 15. You should spend extra time on questions I I through 15 only after you spend all the time that you want on the easier questions.

5. Never leave any answers blank on your answer sh eet. Even if you are not sure of the correct response, you should answer each question. There is no penalty for guessing.

128

STRUCTURE

T H E S TR U C TU R E Q U E S T IO N S In th e T O E F L test, q u e s tio n s 1 th r o u g h 15 o f th e S tru ctu re a n d W ritten E x p r e ssio n se c ­ tio n test you r k n o w le d g e o f th e c o r r e c t str u c tu r e o f E n g lish se n te n c e s . T h e q u e s tio n s in this se c tio n are m u ltip le -c h o ic e q u e s tio n s in w h ic h you m u st c h o o s e th e le tte r o f th e answ er that b e st c o m p le te s th e s e n te n c e .

Exam ple is taking a trip to N ew York. (A) (B) (C) (D)

They When The woman Her

In th is e x a m p le , y o u sh o u ld n o tic e im m e d ia te ly th a t th e s e n te n c e h a s a verb (is ta k in g ), a n d th a t th e verb n e e d s a su b ject. A n sw ers (B ) a n d (D ) are in c o r r e c t b e c a u se w hen a n d her are n o t su b jects. In an sw er (A ), they is a su b je ct, b u t they is p lu ral a n d th e v e rb is ta k in g is sin gu lar. T h e c o r r e c t an sw er is a n sw e r ( C ) ; the w o m a n is a sin g u la r su b je ct. You sh o u ld th e r e fo r e c h o o s e a n sw er ( C ) .

____________________ STRATEGIES FOR THE STRUCTURE QUESTIONS___________________

1. First study th e sen ten ce. Your purpose is to determine what is needed to complete the sentence correctly. 2. Then study each answer based on how well it co m p letes th e sen ten ce. Eliminate

answers that do not complete the sentence correctly.

3. Do not try to elim inate incorrect answers by looking only at the an sw ers. The incorrect answers are generally correct by themselves.The incorrect answers are generally incorrect only when used to complete the sentence.

4. N ever leave any answers blank. Be sure to answer each question even if you are unsure of the correct response.

5. Do not spend to o much tim e on th e Structure questions. Be sure to leave adequate time for the Written Expression questions.

T h e fo llo w in g sk ills w ill h e lp y o u to im p le m e n t th e se str a teg ie s in th e S tr u c tu r e s e c tio n o f th e T O E F L test.

STRUCTURE AND WRITTEN EXPRESSION

SENTENCES W IT H O N E CLAUSE___________________________ S o m e s e n te n c e s in E n g lish have ju s t o n e su b ject a n d verb , a n d it is v ery im p o r ta n t fo r you to fin d th e su b je ct an d verb in th e se se n te n c e s . In so m e s e n te n c e s it is easy to fin d th e su b je ct a n d verb . H ow ever, c e r ta in stru ctu res, su ch as o b je c ts o f p r e p o sitio n s, a p p o sitiv es, a n d p a r ticip le s, can c a u se c o n fu s io n in lo c a tin g th e su b je ct a n d verb b e c a u s e e a c h o f th e se str u c tu r es can lo o k lik e a su b je c t o r verb. T h e o b je c t o f th e p r e p o s itio n c a n b e m is­ ta k en fo r a su b ject. ■ T h e r e fo r e , y o u sh o u ld b e a b le to d o th e fo llo w in g in s e n te n c e s w ith o n e su b je c t a n d verb: (1 ) b e su re th e s e n te n c e h a s a su b ject a n d a verb , (2 ) b e c a refu l o f o b je c ts o f p r e p o ­ sitio n s a n d a p p o sitiv es w h e n y o u are lo o k in g for th e su b ject, a n d (3 ) b e c a r e fu l o f p r e se n t p a r ticip le s an d p ast p a r ticip le s w h e n y o u are lo o k in g fo r th e verb .

S k ill

I:

BE SURE T H E SEN TE N C E H A S A SUBJECT A N D AVERB

You k n o w th at a s e n te n c e in E n g lish s h o u ld have a su b je ct an d a verb . T h e m o st c o m m o n types o f p r o b le m s th at y o u will e n c o u n te r in th e S tru ctu re se c tio n o f th e T O E F L te st have to d o w ith su b jects an d verbs: p e r h a p s th e s e n te n c e is m issin g e ith e r th e su b je ct o r th e verb o r b o th , o r p e r h a p s th e s e n te n c e h a s an ex tra su b je ct o r verb .

Exam ple I _ was backed up for miles on the freeway. (A) Yesterday (B) In the morning (C) Traffic (D) Cars In th is e x a m p le y o u s h o u ld n o tic e im m e d ia te ly th at th e r e is a verb (w a s), b u t th e r e is n o su b ject. A n sw er (C ) is th e b est answ er b e c a u se it is a sin g u la r su b je c t th a t a g ie e s w ith th e sin g u la r verb was. A n sw er (A ) , yesterday, a n d an sw er ( B ) , in the m orning, are n o t su bjects, so th e y are n o t c o rr ec t. A lth o u g h answ er ( D ), cars, c o u ld b e a su b ject, it is n o t c o r r e c t b e ­ cau se cars is p lu ral an d it d o e s n o t a g r ee w ith th e sin g u la r verb was.

Exam ple II E ngineers_____ for work on the new space program. (A) necessary (B) are needed (C) hopefully (D) next month In th is e x a m p le y o u s h o u ld n o t ic e im m ed ia te ly th a t th e s e n te n c e h as a su b je c t (engineers), a n d th a t th e r e is n o verb . B e c a u se an sw er (B ), are needed, is a verb , it is th e b e st answer. A n sw ers (A ), ( C ) , a n d (D ) are n o t verb s, so th e y are n o t co rr ec t.

STRUCTURE

E xam ple III The b o y _____ going to the movies with a friend. (A) he is (B) he always was (C) is relaxing (D) will be T h is s e n te n c e h a s a su b je c t (boy) a n d h as p a rt o f a verb (going) ; to b e c o r r e c t, s o m e fo rm o f th e verb be is n e e d e d to m ak e th e s e n te n c e c o m p le te . A n sw ers (A) a n d (B ) are in c o r ­ rect b e c a u se th e s e n te n c e alread y h a s a su b je c t (boy) a n d d o e s n o t n e e d th e e x tra su b ject he. A n sw er (C ) is in c o r r e c t b e c a u se relaxing is an ex tra verb part th a t is u n n e c e ss a r y b e ­ c a u se o f going. A n sw er (D ) is th e b e st answ er; w ill be to g e th e r w ith g o in g is a c o m p le te verb. T h e fo llo w in g c h a r t o u tlin e s w h at y o u sh o u ld r e m e m b e r a b o u t su b je cts a n d verbs: SUBJECTS ANDVERBS A sentence in English must have at least one subject and one verb.

E X E R C ISE 1: U n d e r lin e th e su b je cts o n c e a n d th e verbs tw ice in e a c h o f th e fo llo w in g se n te n c e s . T h e n in d ic a te i f th e s e n te n c e s are c o r r e c t (C ) o r in c o r r e c t (I ). 1

1

Last w eek went fishing for trout at the nearby m ountain lake.

C

9

A schedu le o f the day’s events can be obtained at the front desk.

3.

A jo b o n the day shift or the night shift at the plant available.

4

T he new com puter program has provides a variety o f helpful applications.

5.

T he b ox can be op en ed only with a special screwdriver.

6.

T he assigned text for history class it contains m ore than twenty chapters.

7.

T he papers in the wastebasket should be em ptied into the trash can outside.

8.

Departure before dawn on a boat in the m iddle o f the harbor.

9.

Yesterday found an interesting article on pollution.

10.

S k ill

2:

T he new m achine is processes 50 percent m ore than the previous m achine.

BE C A R EFU L OF OBJECTS OF P R EPO SITIO N S

A n o b je c t o f a p r e p o s itio n is a n o u n o r a p r o n o u n th a t c o m e s a fter a p r e p o s itio n , su c h as in , at, of, to, by, behind, c n, a n d so o n , to fo r m a p r e p o sitio n a l p h ra se. (/ fter his exams) Tom will take a trip (by boat). T h is s e n t e n c e c o n ta in s tw o o b je c ts o f p r e p o sitio n s. E xa m s is th e o b je c t o f th e p r e p o sitio n after a n d boat is th e o b je c t o f th e p r e p o s itio n by.

STRUCTUREANDWRITTEN EXPRESSION

A n o b je c t o f a p r e p o sitio n can c a u se c o n fu s io n in th e S tru ctu re se c tio n o f th e T O E F L te st b e c a u se it can b e m ista k en fo r th e su b ject o f a s e n te n c e .

Exam ple With his friend

found the movie theater.

(A) has (B) he (C) later (D) when In th is e x a m p le y o u s h o u ld lo o k first for th e su b ject a n d th e verb. You sh o u ld n o tic e th e verb fo u n d a n d sh o u ld also n o tic e th a t th e r e is n o su bject. D o n o t th in k th a t fr ie n d is th e subject; frie n d is th e o b je c t o f th e p r e p o sitio n with, a n d o n e n o u n c a n n o t b e b o th a su b je ct a n d an o b je c t at th e sa m e tim e. B e ca u se a su b ject is n e e d e d in this se n te n c e , an sw er ( B ) , he, is th e b e st answer. A n sw ers (A ), (C ), a n d (D ) are n o t c o r r e c t b e c a u se th ey c a n n o t be su bjects. T h e fo llo w in g ch art o u tlin e s th e key in fo r m a tio n th a t y o u sh o u ld r e m e m b e r a b o u t o b je c ts o f p rep o sitio n s: OBJECTS OF PREPOSITIONS A preposition is followed by a noun or pronoun that is called an object o f the preposition.

If a word is an object o f a preposition, it is not th e subject.

NOTE; A lengthy list of prepositions and practice in recognizing prepositions can be found in Appendix D at the back of the text. You may want to complete these exercises before continuing with Exercise 2.

E X ER C ISE 2: E ach o f th e fo llo w in g s e n te n c e s c o n ta in s o n e o r m o r e p r e p o sitio n a l p h rases. U n d e r lin e th e su b jects o n c e a n d th e verbs tw ice. C ircle th e p r e p o sitio n a l p h ra ses th a t c o m e b e fo r e th e verb . T h e n in d ic a te i f th e s e n te n c e s are c o r r e c t (C ) o r in ­ c o r r e c t (I). C

1. The interviews (by radio broadcasters) were carried live by the station.

^

2. (in the last possible m om ent)(before takeoff) took his seat in the airplane.

_____ 3. At the neighb orh ood flower shop, flowers in quantities o f a dozen or a h a lf dozen can be delivered for free. _____ 4. The progressive reading m ethods at this school are given credit for the im proved test scores. _____ 5. For the last three years at various hospitals in the county has b een practicing m edicine. _____ 6. In the past a career in politics was n ot considered acceptable in som e circles. _____ 7. Shopping in the downtown area o f the city it has improved a lot in recent years. _____ 8. At the building site the carpenters with the m ost experience were given the m ost intricate work.

STRUCTURE

-------- 9.

For the fever and headache took two aspirin tablets.

-------- 10.

T h e report with com plete docum entation was delivered at the conference.

S k i l l 3:

BE CAREFUL OF APPO SITIVES

A p p o sitiv es c a n c a u se c o n fu s io n in th e S tru ctu re s e c tio n o f th e T O E F L te st b e c a u se an a p p o sitiv e c a n b e m ista k e n fo r th e su b je ct o f a s e n te n c e . A n a p p o sitiv e is a n o u n th at c o m e s b e fo r e o r a fte r a n o th e r n o u n a n d h as th e sa m e m e a n in g . I I Sally, the best student in the class, got an A on the exam. In th is e x a m p le S ally is th e su b ject o f th e s e n te n c e a n d the best stu d e n t in the class c a n easily b e r e c o g n iz e d as a n a p p o sitiv e p h ra se b e c a u se o f th e n o u n stu d e n t a n d b e c a u se o f th e c o m m a s. T h e s e n t e n c e says th at Sally a n d the best stu d e n t in the class are th e sa m e p e r so n . N o te th at i f y o u le a v e o u t th e a p p o sitiv e p h ra se, th e s e n t e n c e still m a k es se n s e (Sally got a n A on the exam ). T h e fo llo w in g e x a m p le sh ow s h o w an a p p o sitiv e c a n b e c o n fu s e d w ith th e su b je c t o f a se n te n c e in th e S tr u c tu r e se c tio n o f th e T O E F L test.

Exam ple I , George, is attending the lecture. (A) (B) (C) (D)

Right now Happily Because of the time My friend

In this e x a m p le y o u sh o u ld r e c o g n iz e from th e c o m m a s that George is n o t th e su bject o f th e se n te n c e . George is a n ap p ositive. B eca u se this s e n te n c e still n e e d s a su bject, th e b est answ er is ( D ) , my frien d . A n sw ers (A ) , (B ) , a n d (C) are in c o r r e c t b e c a u se th ey are n o t subjects. T h e n e x t e x a m p le sh ow s th at an a p p o sitiv e d o e s n o t always c o m e a fter th e su b ject; an a p p o sitiv e c a n a lso c o m e at th e b e g in n in g o f th e s e n te n c e .

Exam ple II , Sarah rarely misses her basketball shots. (A) An excellent basketball player (B) An excellent basketball player is (C) Sarah is an excellent basketball player (D) Her excellent basketball play In th is e x a m p le y o u c a n te ll th a t Sarah is th e su b je ct a n d misses is th e verb b e c a u se th e r e is n o c o m m a se p a r a tin g th e m . In th e sp a ce y o u s h o u ld p u t a n a p p o sitiv e fo r S arah, an d Sarah is a n excellent basketball player, so an sw er (A) is th e b e st answ er. A n sw ers (B ) a n d (C) are n o t c o r r e c t b e c a u s e th e y e a ch c o n ta in th e verb is, a n d a n a p p o sitiv e d o e s n o t n e e d a verb . A n sw e r (D ) c o n ta in s a n o u n , play, th a t c o u ld p o ssib ly be an a p p o sitiv e, b u t play is n o t th e sa m e as Sarah, so th is an sw er is n o t co rr ec t.

STRUCTURE AND WRITTEN EXPRESSION

T h e fo llo w in g ch art o u tlin e s th e key in fo r m a tio n th a t y o u sh o u ld r e m e m b e r a b o u t a p p ositives: APPOSITIVES An ap Positive is a noun that comes before or after another noun and is generally set off from the noun ivith commas. If a word is an appositive, it is not th e subjcct. The following appositive struct jres are both possible in English: S, Tom,

APP.

v

a really good mechanic.

APP, A really good mechanic,

Is fixing

S

V

Tom

Is fixing

the car.

the car.

E X E R C ISE 3: E a c h o f th e fo llo w in g s e n te n c e s c o n ta in s an a p p ositive. U n d e r lin e th e su b­ j e c t s o n c e a n d th e verb s tw ice. C ircle th e a p p o sitiv e p h rases. T h e n , in d ic a te if th e se n ­ te n c e s are c o r r e c t (C ) o r in c o r r e c t ( I ) . C

1. (The son o f the previous owner,) the new owner is undertaking som e fairly broad changes in m anagem ent policy.

I

2.

Last semester, (a friend,) graduated cum laude from the university.

_____ 3.

Valentine's Day, February 14, is a special holiday for sweethearts.

_____ 4.

At long last, the ch ief executive officer, has decided to step down.

_____ 5.

T onight’s supper, leftovers from last night, did not taste any better tonight than last night.

_____ 6.

T he only entrance to the closet, the door was kept locked at all times.

_____ 7.

In the cold o f winter, a wall heating unit, would not turn on.

_____ 8.

The new tile pattern, yellow flowers on a w hite background, really brightens up the room.

_____ 9.

The high-powered com puter the m ost powerful m achine o f its type, was finally readied for use.

_____ 10.

A longtim e friend and confident, the psychologist was often invited over for Sunday dinner.

S kill

4:

BE CAREFUL OF PRESENT PARTICIPLES

A p r e s e n t p a r ticip le is th e -ing fo r m o f th e verb ( talkin g , p la yin g ). In th e S tru ctu re se c tio n o f th e T O E F L te st a p r e se n t p a r ticip le can ca u se c o n fu s io n b e c a u se it can b e e ith e r a part o f th e verb or an ad jective. It is p art o f th e verb w h e n it is p r e c e d e d by so m e form o f the v e r b be. T h e m an is talking to his friend. -

VERB

In th is s e n te n c e ta lk in g is part o f th e verb b e c a u se it is a c c o m p a n ie d by is.

STRUCTURE

A p r e s e n t p a r tic ip le is an a d jectiv e w h e n it is n o t a c c o m p a n ie d by s o m e fo r m o f th e v e rb be. T h e man talking to his friend has a beard. ADJECTIVE

In th is s e n te n c e ta lk in g is an a d jectiv e a n d n o t p art o f th e verb b e c a u se it is n o t a c c o m p a ­ n ie d by s o m e fo r m o f be. T h e verb in th is s e n t e n c e is has. T h e fo llo w in g e x a m p le sh ow s h o w a p r e se n t p a r ticip le can b e c o n fu s e d w ith th e verb in th e S tr u c tu r e s e c tio n o f th e T O E F L test.

Exam ple playing in the yard is my son.

The child (A) now (B) is (C) he (D) was

In th is e x a m p le , i f y o u lo o k at o n ly th e first w ord s o f th e s e n te n c e , it a p p ea r s th a t child is th e su b je c t an d p la y in g is p a rt o f th e verb . If y o u th in k th at p la y in g is p art o f th e verb , you m ig h t c h o o s e a n sw er (B ), is, o r an sw er ( D ) , was, to c o m p le te th e verb . H ow ever, th e s e two an sw ers are in c o r r e c t b e c a u se p la y in g is n o t p art o f th e verb . You s h o u ld r e c o g n iz e that p la y in g is a p a r ticip ia l a d jectiv e ra th e r th a n a verb b e c a u se th e r e is a n o th e r verb in th e s e n t e n c e (is). In th is s e n te n c e th e r e is a c o m p le te su b ject (child) a n d a c o m p le t e v e rb (is), so th is s e n te n c e d o e s n o t n e e d a n o th e r su b je c t o r verb. T h e b e st an sw er h e r e is ( A ) . T h e fo llo w in g c h a r t o u tlin e s w h a t y o u s h o u ld r e m e m b e r a b o u t p r e s e n t p a rticip les: -

!

"

.. .'.

PRESENT PARTICIPLES

A present participle is the -ing form of the verb. T h e present particip le can be ( I) p art o f th e verb o r (2 ) an adjective. It is part of the verb when it is accompanied by some form of the verb be. It is an adjective when it is not accompanied by some form of the verb be. .. • - . • . . . . . . . _. * 1. The boy is standing in the comer.

*

2. The boy standing in the comer was naughty.

E X E R C ISE 4: E a c h o f th e fo llo w in g s e n t e n c e s c o n ta in s o n e o r m o r e p r e s e n t p a r ticip le s. U n d e r lin e th e su b je cts o n c e a n d th e verb s tw ice. C ircle th e p r e s e n t p a r tic ip le s an d la b e l th e m as a d je c tiv e s o r verb s. T h e n in d ic a te i f th e s e n te n c e s are c o r r e c t (C ) or in c o r r e c t (I ). £

1. The com panies (offering) the lowest prices will have the m ost customers. ADJ.

I

2. Those travelers are (completing") their trip on Delta should report to Gate T hree. VERB

_____

3. T he artisans were dem onstrating various handicrafts at booths throughout the fair.

_____

4. The fraternities are giving the wildest parties attract the m ost new pledges.

_____

5. The first team winning four games is awarded the championship.

STRUCTURE AND WRITTEN EXPRESSION

6.

T he speaker was trying to make his point was often interrupted vociferously.

7.

T he fruits were rotting because o f the m oisture in the crates carrying them to market.

8.

Any students desiring official transcripts should com plete the appropriate form.

9.

The advertisem ents were an n ou n cin g the half-day sale received a lot o f attention.

10. T he spices flavoring the meal were quite distinctive.

S k i l l 5:

BE CAREFUL OF PAST PARTICIPLES

P ast p a r tic ip le s can cau se c o n fu s io n in th e S tru ctu re se c tio n o f th e T O E F L test b e c a u se a p ast p a r tic ip le can b e e ith e r an ad jective or a p art o f a verb . T h e p a st p a rticip le is th e fo r m o f th e verb th a t ap p ea rs w ith have o r be. It o fte n e n d s in -ed, b u t th e r e are also m a n y irr eg u la r past p a r ticip le s in E n glish . (S e e A p p e n d ix F fo r a list o f irreg u la r past p a r tic ip le s.) The family has purchased a television. VERB

The poem was written by Paul. v e rb

.

In th e first s e n te n c e th e p ast p a rticip le purchased is part o f th e verb b e c a u se it is a c co m p a ­ n ie d by has. In th e s e c o n d s e n te n c e th e past p a r ticip le w ritten is p art o f th e verb b e c a u se it is a c c o m p a n ie d by was. A p a st p a r tic ip le is an ad jective w h e n it is n o t a c c o m p a n ie d by so m e fo r m o f be o r have. T he television purchased yesterday was expensive. a d je c tiv e

The poem urritten by Paul appeared in the magazine. a d je c tiv e

In th e first s e n t e n c e purchased is an ad jective ra th er than a verb b e c a u se it is n o t a c co m p a ­ n ie d by a fo r m o f be o r have (a n d th e r e is a verb , was, later in th e s e n te n c e ). In th e se c o n d s e n t e n c e w ritten is an ad jective rath er th an a v erb b e c a u se it is n o t a c c o m p a n ie d by a form o f be o r have, (a n d th e r e is a verb, appeared, later in th e s e n t e n c e ). T h e fo llo w in g e x a m p le sh ow s h o w a past p a r ticip le c a n b e c o n fu s e d w ith th e verb in th e S tr u c tu r e se c tio n o f th e T O E F L test.

Exam ple The packages (A) have (B) were (C) them (D) just

mailed at the post office will arrive Monday.

STRUCTURE

In th is e x a m p le , i f y o u lo o k o n ly a t th e first few w ord s o f th e s e n te n c e , it a p p ea r s th a t p ack­ ages is th e su b je ct a n d m ailed is e ith e r a c o m p le te verb o r a past p a r tic ip le th a t n e e d s a h e lp in g verb . B u t i f y o u lo o k fu r th e r in th e s e n te n c e , y o u w ill se e th a t th e verb is w ill ar­ rive. You w ill th e n r e c o g n iz e th at m ailed is a p a rticip ia l ad jective a n d is th e r e fo r e n o t part o f th e verb . A n sw ers (A ) a n d (B ) are in c o r r e c t b e c a u se m ailed is an a d jectiv e an d d o e s n o t n e e d a h e lp in g verb su c h as have o r were. A n sw er (C ) is in c o r r e c t b e c a u se th e r e is n o n e e d fo r th e o b je c t them. A n sw e r (D ) is th e b e st an sw er to th is q u e s tio n . T h e fo llo w in g c h a r t o u tlin e s w h a t y o u sh o u ld r e m e m b e r a b o u t p ast p articip les: PAST PARTICIPLES

A past participle often ends In -ed, but there are also many Irregular past participles. For many verbs, including -ed verbs, the simple past and the past participle are the same and can be easily confused. The -ed form o f the verb can be ( I) the sim ple past, (2 ) the past p articip le of a verb, o r (3) an adjective.

1. 2. 3.

She pain ted this picture. She has painted this picture. The picture pain ted by Karen is now in a museum.

E X E R C ISE 5: E ach o f th e fo llo w in g s e n te n c e s c o n ta in s o n e o r m o r e p ast p a r ticip le s. U n ­ d e r lin e th e su b je cts o n c e a n d th e verb s tw ice. C ircle th e p ast p a r ticip le s a n d la b e l th e m as a d jectiv es o r verb s. T h e n in d ic a te i f th e s e n te n c e s are c o r r e c t (C ) o r in c o r r e c t (I ). — !—

1. T he m oney was (offered) by the client was not (accepted) VERB



2.

VERB

T he car (jisteti) in the advertisem ent had already (stalled) a d j.

v erb

--------

3.

T he chapters were taught by the professor this m orning will be on next week's exam.

--------

4.

The loaves o f bread were baked in a brick oven at a low temperature for many hours.

--------

5.

T he ports were reached by the sailors were under the control o f a foreign nation.

--------

6. T hose suspected in the string o f robberies were arrested by the police.

--------

7. T he pizza is served in this restaurant is the tastiest in the county.

--------

8. The courses are listed on the second page o f the brochure have several prerequisites.

--------

9. All the tenants were invited to the In d epend en ce Day barbecue at the apartm ent com plex.

--------

10. Any bills paid by the first o f the m onth will be credited to your account by the next day.

STRUCTURE AND WRITTEN EXPRESSION

E X E R C IS E (S k ills 1 -5 ): U n d e r lin e th e su b jects o n c e an d th e verbs tw ice in e a c h o f th e fo llo w in g s e n te n c e s . T h e n in d ic a te if th e s e n te n c e s are c o r r e c t (C) o r in c o r r e c t (1). _____

1.

For three weeks at the beginn in g o f the sem ester students with fewer than the m axim um num ber o f units can add additional courses.

_____

2.

O n her lunch hour went to a nearby departm ent store to purchase a w edding gift.

_____

3.

T h e fir trees were grown for the holiday season were harvested in November.

_____

4.

In the grove the overripe oranges were falling on the ground.

_____

5.

T h e papers being delivered at 4:00 will contain the announcem ent o f the president’s resignation.

_____

6.

A specialty shop with various blends from around the world in the shopping mall.

_____

7.

T h e portraits exhibited in the H ouston M useum last m onth are now o n display in Dallas.

_____

8.

W ith a sudden jerk o f his hand threw the ball across the field to one o f the other players.

_____

9.

Construction o f the housing developm ent it will be underway by the first o f the m onth.

_____

10.

T hose applicants returning their com pleted forms at the earliest date have the highest priority.

T O E F L E X E R C ISE (S k ills 1 -5 ): C h o o se th e le tte r o f th e w o rd o r g r o u p o f w ord s th a t b est c o m p le te s th e se n te n c e . 1. The North Platte River_ Wyoming into Nebraska. (A) (B) (C) (D)

it flowed flows flowing with flowing water

. Biloxi received its name from a Sioux word meaning "first people.” (A) (B) (C) (D)

The city of Located in It is in The tour included

_up to forty lions. 3. A pride of lions . including one to three males, several females, and cubs. (A) (B) (C) (D)

can contain it contains contain containing

. tea plant are small and white.

.from (A) (B) (C) (D)'

The On the Having flowers the The flowers of the

. antibiotics, are 5. The tetracyclines. used to treat infections. (A) (B) (C) (D)

are a family of being a family a family of their family is

6. Any possible academic assistance from taking stim ulants_____ marginal at best. (A) (B) (C) (D)

it is there is is as

STRUCTURE

7. Henry Adams, b om in B oston ,-------famous as a historian and novelist. (A) (B) (C) (D)

(A) (B) (C) (D)

became and became he was and he became

8. The major c a u se_____ the pull of the Moon on the Earth. (A) (B) (C) (D)

9. Still a novelty in the late nineteenth century,_____ limited to the rich.

the ocean tides are of ocean tides is of the tides in the ocean the oceans’ tides

was was photography it was photography photography was

10. A computerized map of the freeways using information gathered by sensors embedded in the pavem ent--------on a local cable channel during rush hours. (A) (B) (C) (D)

airs airing air to air

SEN TEN CES W IT H M ULTIPLE CLAUSES----------------------------------M any se n te n c e s in E n g lish h a v e m o r e than o n e c la u se. (A c la u se is a g r o u p o f w ords c o n ­ ta in in g a su b je ct a n d a verb .) W h e n e v e r you fin d a s e n t e n c e o n th e T O E F L test w ith m o r e th an o n e c la u se , y o u n e e d to m ak e su re that ev ery su b je ct h a s a verb a n d ev er y verb h a s a su b ject. N e x t y o u n e e d to c h e c k th a t th e variou s cla u se s in th e se n te n c e are co rrectly j o in e d . T h e r e are v ariou s ways to j o in cla u se s in E n g lish . C erta in p a tte rn s a p p ea r fr eq u e n tly in E n g lish a n d o n th e T O E F L test. You sh o u ld b e very fa m ilia r w ith th e se p a ttern s.

S kill

6:

USE C O O R D IN A T E C O N N E C TO R S CORRECTLY

W h e n y o u h a v e tw o cla u se s in an E n g lish s e n te n c e , y o u m u st c o n n e c t th e tw o cla u se s c o r ­ rectly. O n e way to c o n n e c t tw o c la u ses is to u se and, but, or, so, or yet b e tw e e n th e cla u ses. Tom is singing, and Paul is dancing. Tom is tall, but Paul is short. Tom must write the letter, or Paul will do it. Tom told a jo k e, so Paul laughed. Tom is tired, yet he is n ot going to sleep. In e a c h o f th e se e x a m p le s , th e r e are two cla u se s th a t are c o rr ec tly j o i n e d w ith a c o o r d i­ n a te c o n ju n c tio n a n d , but, or, so, o r yet, a n d a c o m m a ( ,) .

I

STRUCTURE AND WRITTEN EXPRESSION

T h e fo llo w in g e x a m p le sh ow s h o w th is s e n te n c e p a ttern c o u ld b e te ste d in th e Struc­ ture se c tio n o f th e T O E F L test.

Exam ple A power failure occurred,

the lamps went out.

(A) then (B) so (C) later (D) next In th is e x a m p le y o u sh o u ld n o tic e q u ick ly th at th e r e are two c la u se s, a power fa ilu r e oc­ curred a n d the lam ps w ent out. T h is s e n te n c e n e e d s a c o n n e c to r to j o in th e tw o clau ses. Then, later, an d next are n o t c o n n e c to r s, so answ ers (A ), (C ), an d (D ) are n o t c o rr ec t. T h e b e st an sw er is an sw er (B ) b e c a u se so can c o n n e c t two clau ses. T h e fo llo w in g ch a rt lists th e c o o r d in a te c o n n e c to r s an d th e s e n te n c e p a tte r n u se d w ith th em : COORDINATE CONNECTORS and

but

S

V,

She laughed,

. so

or (coordinate connector) but

S

yet

V

she w anted to cry.

E X E R C ISE 6: E ach o f t h e fo llo w in g s e n te n c e s c o n ta in s m o r e th a n o n e c la u se . U n d e r lin e th e su b jects o n c e an d th e verb s tw ice. C ircle th e c o n n e c to r s. T h e n in d ic a te i f th e se n ­ te n c e s are c o r r e c t (C ) o r in c o r r e c t (I). C

1.

T he software should be used on an IBM computer, (and) this com puter is an IBM. = ^ = ^ == T he rain clouds can be seen in the distance, (b u t)n o has fallen. They are trying to sell their house, it has been on the market for two months. So the quality o f the print was not good, I changed the toner cartridge. T h e lifeguard will warn you about the riptides, or she may require you to get out o f the water.

6.

You should have finished the work yesterday, yet is not close to being finished today.

7.

The ph on e rang again and again, so the receptionist was n ot able to get much work done.

8.

T he missing wallet was found, but the cash and credit cards had been rem oved.

9.

Or you can drive your car for another 2,000 miles, you can get it fixed.

10. T h e chem ist was awarded the N obel Prize, he flew to Europe to accept it.

STRUCTURE

S kill

7:

USE ADVERB TIM E A N D CAUSE C O N N E C T O R S CO RRECTLY

S e n te n c e s w ith ad verb c la u se s h ave two b asic p a tte rn s in E n g lish . S tu d y th e c la u se s a n d c o n n e c to r s in th e fo llo w in g se n te n c e s: I will sign the check before you leave.

In ea ch o f th e se e x a m p le s , th e r e are two clau ses: y o u leave a n d I m i l sign the check, a n d th e c la u se you leave is an ad verb tim e c la u se b e c a u se it is in tr o d u c e d w ith th e c o n n e c t o r before. In th e first e x a m p le th e c o n n e c to r before c o m e s in th e m id d le o f th e s e n te n c e , a n d n o c o m m a (,) is u sed . In th e s e c o n d e x a m p le th e c o n n e c to r before c o m e s at th e b e g in n in g o f th e se n te n c e . In th is p a tte rn , w h e n th e c o n n e c to r c o m e s at th e b e g in n in g o f th e s e n ­ te n c e , a c o m m a (,) is r e q u ir e d in th e m id d le o f th e se n te n c e . T h e fo llo w in g e x a m p le sh ow s h o w th is s e n te n c e p a tte rn c o u ld b e te s te d in th e S tru c­ tu re se c tio n o f th e T O E F L test. E x am p le --------was late, I missed the appointment. (A) I (B) Because (C) The train (D) Since he

In this e x a m p le y o u sh o u ld r e c o g n iz e ea sily th at th e r e is a verb , was, th a t n e e d s a su b ject. T h e r e is a lso a n o th e r c la u se , I m issed the appointm ent. If y o u c h o o s e an sw er (A ) o r an sw er (C ), y o u w ill h a v e a su b je c t for th e verb rvas, b u t y o u w ill n o t have a c o n n e c t o r to j o i n th e two cla u ses. B e c a u se y o u n e e d a c o n n e c t o r to j o in two c la u se s, a n sw ers (A ) a n d (C ) are in c o r r e c t. A n sw er (B ) is in c o r r e c t b e c a u se th e r e is n o su b je ct fo r th e v e r b was. A n sw er (D ) is th e b e st a n sw er b e c a u se th e r e is a su b ject, he, for th e verb was, a n d th e r e is a c o n ­ n ecto r, since, to j o i n th e tw o cla u ses. T h e fo llo w in g c h a r t lists ad verb tim e a n d cause c o n n e c to r s a n d th e s e n te n c e p a tte r n s u sed w ith them :

STRUCTURE AND WRITTEN EXPRESSION

E X E R C ISE 7: E ach o f th e fo llo w in g s e n te n c e s c o n ta in s m o r e th an o n e c la u se . U n d e r lin e th e su b jects o n c e an d th e verbs tw ice. C ircle th e c o n n e c to r s. T h e n in d ic a te if th e se n ­ te n c e s are 1.

C

c o r r e c t (C ) o r in c o r r e c t (I). (Since) the bank closes in less than an hour, the deposits need to be tallied immediately.

I

2. Their backgrounds are thoroughly investigated (before) are adm itted to the organization.

_____

3.

The citizens are becom ing m ore and m ore incen sed about traffic accidents whenever the accidents occur at that intersection.

_____

4.

T he ground had been prepared, the seedlings were carefully planted.

_____

5.

We can start the conference now that all the participants have arrived.

_____

6.

The building quite vulnerable to damage until the storm windows are installed.

____

7.

O nce the address label for the package is typed, can be sent to the mail room.

_____

8.

Because the recent change in work shifts was not posted, several workers missed their shifts.

_____

9.

The m other is going to be quite upset with her son as long he misbehaves so much.

_____

10.

Inasmuch as all the votes have n ot yet been counted the outcom e o f the election cannot be announced. »

S

8:

USE OTH ER ADVERB C O N N E C T O R S CORRECTLY

kill

A d verb c la u se s ca n e x p r ess th e id e a s o f tim e a n d c a u se , as y o u saw in Skill 7; adverb c la u se s c a n also ex p r ess a n u m b e r o f o th e r id ea s, su ch as c o n tra st, c o n d itio n , m an n er, a n d p la c e. B e ca u se th e se cla u ses are adverb c la u se s, th ey have th e sam e stru ctu re as th e tim e a n d ca u se cla u ses in Skill 7. Study th e fo llo w in g ex a m p les: I will leave at 7:00 i f I am ready. Although I was late, I managed to catch the train. In e a c h o f th e se e x a m p le s, th e r e are two cla u ses th a t are c o rr ec tly j o in e d w ith adverb c o n ­ n e c to rs. In th e first se n te n c e th e adverb c o n d itio n c o n n e c to r i f c o m e s in th e m id d le o f th e se n te n c e . In th e s e c o n d s e n te n c e th e adverb c o n tr a st c o n n e c to r although c o m e s at th e b e g in n in g o f th e se n te n c e , an d a c o m m a (,) is u s e d in th e m id d le o f th e s e n te n c e . T h e fo llo w in g e x a m p le sh ow s a way th at th is s e n te n c e p a tte rn can b e te sted in th e S tru ctu re se c tio n o f th e T O E F L test.

E xam ple You will get a good grade on the exam provided (A) (B) (C) (D)

studying study to study you study

STRUCTURE

In th is e x a m p le y o u sh o u ld q u ick ly n o tic e th e adverb c o n d itio n c o n n e c to r provided. T h is c o n n e c to r c o m e s in th e m id d le o f th e s e n te n c e ; b e c a u se it is a c o n n e c to r , it m u st be fo l­ low ed by a su b je ct a n d a verb. T h e b e s t an sw er to th is q u e s tio n is an sw er ( D ) , w h ic h c o n ­ tains th e su b je c t a n d verb yo u study. T h e fo llo w in g c h a r t lists th e a d v e r b c o n tr a st, c o n d it io n , m a n n er , a n d p la c e c o n n e c ­ tors a n d th e s e n t e n c e p a tte rn s u s e d w ith th em : OTHER ADVERB CONNECTORS CONDITION

CONTRAST

MANNER

PLACE

If

although even though though while whereas

as in that

where wherever

in case provided providing unless whether

S

V

(adverb connector)

Bob w e n t to school (adverb connector) Even th o u g h

S

even th o u g h S

V,

Bob fe lt sick,

V

h e fe lt sick. S

V

he w e n t to school.

NOTE: A comma is often used in the middle of the sentence with a contrast connector. The Smith family arrived at 2:00, while the Jones family arrived an hour later.

E X ER C ISE 8: E ach o f th e fo llo w in g s e n t e n c e s c o n ta in s m o r e th an o n e c la u se . U n d e r lin e th e su b jects o n c e a n d th e verb s tw ice. C ircle th e c o n n e c to r s . T h e n in d ic a te i f th e s e n ­ te n c e s are c o r r e c t (C ) o r in c o r r e c t (I ). __ 0—

1.

It is im possible to enter that program (Tf)you lack experien ce as a teacher.

__ !__

2.

T he commandant left strict orders about the passes, several soldiers left the post anyway.

_____

3.

N o on e is adm itted to the academy unless he or she the education requirem ents.

_____

4.

W hile m ost students turned the assignm ent in on tíme, a few asked for an extension.

_____

5.

I will take you wherever n e e d to go to com plete the registration procedures.

_____

6.

I will wait here in the airport with you w hether the plane leaves on time or not.

_____

7.

Providing the envelope is postm arked by this Friday, your application still acceptable.

_____

8.

As the nurse already explain ed all visitors m ust leave the hospital room now.

_____

9.

This exam will be m ore difficult than usual in that it covers two chapters instead o f one.

_____

10.

T hough snow had b een falling all day long, everyone g o t to the church on time for the wedding.

STRUCTURE AND WRITTEN EXPRESSION

EX ER C ISE (S k ills 6 - 8 ) : U n d e r lin e th e su b jects o n c e a n d th e verbs tw ice in e a c h o f th e fo llo w in g se n te n c e s. C ircle th e c o n n e c to r s . T h e n in d ic a te if th e s e n te n c e s are c o r r e c t (C ) or in c o r r e c t (I). 1.

Until the registrar makes a decision about your status, you must stay in an unclassified category.

2.

Or the bills can be paid by mail by the first o f the month. T he parents left a p h on e num ber with the baby-sitter in case a problem with the children.

4.

The furniture will be delivered as soon it is paid for.

5.

W henever you want to hold the m eeting, we will schedule it.

6.

T he governm ent was overthrown in a revolution, the king has not returned to his hom eland.

7. Whereas m ost o f the docum ents are com plete, this form still needs to be notarized. 8.

Trash will be collected in the m orning, so you should put the trash cans o u t tonight.

9.

It is im possible for the airplane to take o ff while is snowing so hard.

10. We did not go out to dinner tonight eventhough I would have preferred n ot to cook.

T O E F L EXERCISE (S k ills 6 -8 ) : C h o o se th e le tte r o f th e w ord o r g r o u p o f w ord s th a t b e st c o m p le te s th e s e n te n c e . 1. The president of the U. S. appoints the cabinet m embers,_____ appointments are subject to Senate approval. (A) (B) (C) (D)

their with their because their but their

2. The prisoners were prevented from speaking to reporters because_____ (A) not wanting the story in the papers, (B) the story in the papers the superintendent did not want (C) the public to hear the story (D) the superintendent did not want the story in the papers Like Thomas Bergers fictional character Little Big Man, Lauderdale managed to find himself w here_____ of important events took place. (A) (B) (C) (D)

it was an extraordinary number there was an extraordinary number an extraordinary number an extraordinary number existed

. sucked groundwater from below, some parts of the city have begun to sink as much as ten inches annually. (A) (B) (C) (D)

Pumps have As pumps have So pumps have With pumps

5. Case studies are the target o f much skepticism in the scientific community, _____ used extensively by numerous researchers. (A) (B) (C) (D)

they are are yet they yet they are

6. According to the hypothesis in the study, the monarchs pick up the magnetic field of th e _____ migrate by following magnetic fields. (A) (B) (C) (D)

target monarchs target since monarchs target since monarchs are target

STRUCTURE

7. _____ show the relations among neurons, they do not preclude the possibility that other aspects are important. (A) (B) (C) (D)

Neural theories A neural theory Although neural theories However neural theories

8. _____or refinanced, the lender will generally require setting up an escrow account to ensure the payment of property taxes and homeowner’s insurance. (A) (B) (C) (D)

A home is A home is bought When a home When a hom e is bought

T O E F L R EV IEW E X E R C ISE (S k ills 1 -8 ): th a t b e st c o m p le te s th e s e n te n c e . 1. The three basic chords i n _____ the tonic, the dominant, and the subdominant. (A) (B) (C) (D)

functional functional functional functional

harmony harmony is harmony are harmony they are

2. --------Hale Telescope, at the Palomar Observatory in southern California, scientists can photograph objects several billion light years away. (A) (B) (C) (D)

The With the They use the It is the

3. Without the proper card installed inside the com puter,_____ impossible to run a graphical program. (A) (B) (C) (D)

is definitely because o f it is is

4. The charter for the Louisiana lottery was coming up for renew al,_____ spared no expense in the fight to win renewal. (A) (B) (C) (D)

the lottery committee so the lottery committee and so the lottery committee the lottery committee made

9.

If ultraviolet radiation enters the Earth's atm osphere,_____ generally blocked by the ozone concentrated in the atmosphere. (A) (B) (C) (D)

it it is so it is then it

10. Among human chromosomes, the Y chromosome is un usual_____ most of the chromosome does not participate in m eiotic recombination. (A) (B) (C) (D)

in so and in that

th e le tte r o f th e w o rd o r g r o u p o f w ords

5. While in reality Alpha Centauri is a triple star,--------to the naked eye to be a single star. (A) (B) (C) (D)

it appears but it appears appears despite it

6. The Sun’s gravity severely distorted the path of the c o m et_____ entered its wildly erratic orbit around Jupiter. (A) (B) (C) (D)

it when after the com et came into it once the comet

7. Each object-------- Jupiter's magnetic field is deluged with electrical charges. (A) (B) (C) (D)

enters it enters entering enter

8. As its name suggests, the Prairie Wetlands Resource Center_____ the protection of wetlands on the prairies o f the Dakotas, Montana, Minnesota, and Nebraska. (A) (B) (C) (D)

it focuses focuses on focusing to focus on

STRUCTURE AND WRITTEN EXPRESSION

9. One of the largest and most powerful birds of prey in the w orld,_____ a six-foot wingspan and legs and talons roughly the size of a man’s arms and legs. (A) (B) (C) (D)

10 .

creation of such a community was a desirable step, the requisite political upheaval had to be accepted.

so the harpy has the harpy having with the harpy having the harpy has

(A) (B) (C) (D)

Since the The Later, the It was the

MORE SENTENCES W IT H M ULTIPLE CLAUSES----------------------A s w c saw in S k ills 6 th r o u g h 8 , m an y s e n te n c e s in E n g lish have m o r e th a n o n e c la u se . In Sk ills 9 th r o u g h 12, w e w ill se e m o r e p a tte rn s for c o n n e c tin g th e cla u ses in s e n te n c e s with m u ltip le cla u ses. B e ca u se th ese p attern s a p p ear fr eq u e n tly in E n g lish a n d o n th e T O EFL test, y o u s h o u ld b e very fam iliar w ith th e m .

S k ill

9:

USE N O U N CLAUSE C O N N E C T O R S CORRECTLY

A n o u n c la u se is a clau se th at fu n c tio n s as a n o u n ; b e c a u se th e n o u n c la u se is a n o u n , it is u s e d in a s e n te n c e as e ith e r an o b je c t o f a verb , an o b je c t o f a p r e p o sitio n , or th e su b ject o f th e se n te n c e . I know when he will arrive. NOU N CLAUSE AS OBJECT OF VERB

I am concerned about when he will arrive. N O U N CLAUSE AS OBJECT O F PREPOSITION

When he will arrive is not important. N OU N CLAUSE AS SUBJECT

In th e first e x a m p le th e r e are tw o cla u se s, I know an d he w ill arrive. T h e se two cla u ses are j o in e d w ith th e c o n n e c to r when. W hen c h a n g e s th e c la u se he w ill arrive in to a n o u n clau se th a t fu n c tio n s as th e o b je c t o f th e verb know. In th e s e c o n d e x a m p le th e tw o c la u se s I am concerned an d he w ill arrive are also j o in e d by th e c o n n e c to r when. W hen c h a n g e s th e c la u se he w ill arrive in to a n o u n c la u se th a t fu n c­ tio n s as th e o b je c t o f th e p r e p o sitio n about. T h e th ird e x a m p le is m o r e d ifficu lt. In th is e x a m p le th e r e are two clau ses, b u t they are a little h a r d e r to r e c o g n iz e . H e w ill arrive is o n e o f th e cla u ses, a n d th e c o n n e c to r when c h a n g e s it in to a n o u n clau se th at fu n c tio n s as th e su b je ct o f th e se n te n c e . T h e o th e r c la u se h a s th e n o u n cla u se when he w ill arrive as its su b ject an d is as its verb. T h e fo llo w in g e x a m p le sh ow s h o w th e se s e n te n c e p a tte rn s c o u ld b e te sted in th e S tr u c tu r e se c tio n o f th e T O E F L test.

STRUCTURE

E x am p le was late caused many problems. (A) (B) (C) (D)

That he The driver There Because

In th is e x a m p le th e r e are tw o verb s ( was a n d caused) , a n d e a c h o f th e s e v e rb s n e e d s a su b ­ j e c t. A n sw er (B ) is w r o n g b e c a u se the driver is o n e su b ject, a n d tw o su b je cts are n e e d e d . A n sw ers (C ) a n d (D ) are in c o r r e c t b e c a u se there a n d because a re n o t su b jects. T h e b e st an­ sw er is an sw er (A ). I f y o u c h o o s e an sw er (A ), th e c o m p le te d s e n te n c e w o u ld be: T h a t he was late caused m a n y problems. In th is s e n te n c e he is th e su b je c t o f th e verb was, a n d th e n o u n c la u se that he was late is th e su b je ct o f th e verb caused. T h e fo llo w in g c h a r t lists th e n o u n c la u se c o n n e c to r s a n d th e s e n te n c e p a tte r n s u se d w ith them : NOUN CLAUSE CONNECTORS • what, when, where, why, how

• whatever, whenever

whether, if

• that

N O U N CLAUSE AS OBJECT

S V

(noun connector) S V what

/ know

you did.

N O U N CLAUSE AS SUBJECT

(noun connector) W h at

S

V

V

you did

. •'

/

"

was wrong.

EX E R C ISE 9: E a ch o f th e fo llo w in g s e n te n c e s c o n ta in s m o r e th an o n e c la u se . U n d e r lin e th e su b je cts o n c e a n d th e verb s tw ice. C ircle th e c o n n e c to r s . P u t b o x e s a r o u n d th e n o u n cla u ses. T h e n in d ic a te i f t h e s e n t e n c e s are c o r r e c t (C ) o r in c o r r e c t (I). C

(W hen)the season starts is determ ined by the weather. 2.

T he manual (how)the device should be built.

3.

The schedule indicated if the teams would be playing in the final gam e.

4.

He refused to en ter a plea could not be determ ined by the lawyer.

5.

Talked about where we should go for lunch.

6. Why the condition o f the patient deteriorated so rapidly it was n ot explained. 7.

W hether or n o t the new office w ould be built was to be determ ined at the m eeting.

8. That the professor has n o t yet decided when the paper is due. 9.

T he contract will be awarded is the question to be answered at the m eeting.

10. H e always talked with w hom ever he pleased and did whatever he wanted.

STRUCTURE AND WRITTEN EXPRESSION

S k il l

10:

USE N O U N CLAUSE CO NNECTO R/SUBJECTS CORRECTLY

In Sk ill 9 w e saw th a t n o u n c la u se c o n n e c to r s w ere u s e d to in tr o d u c e n o u n su b ject cla u ses o r n o u n o b je c t clau ses. In Skill 10 w e will se e th a t in so m e c a se s a n o u n cla u se c o n n e c to r is n o t ju s t a c o n n e c to r ; a n o u n clau se c o n n e c to r can also b e th e su b je ct o f th e cla u se at th e sa m e tim e.

N OU N CLAUSE AS SUBJECT

In th e first e x a m p le th e r e are two clauses: I do not kn o w an d w hat is in the box. T h e se two c la u se s are j o in e d by th e c o n n e c to r w hal. It is im p o r ta n t to u n d e r sta n d that in this se n ­ te n c e th e w o rd w hat se rv es two fu n c tio n s. It is b o th th e su b je ct o f th e verb is a n d th e c o n ­ n e c to r that j o in s th e two clau ses. In th e s e c o n d e x a m p le th e r e are two cla u ses. In th e first c la u se we is th e su b ject o f are. In th e s e c o n d cla u se who is th e su b ject o f w ill do. W ho also se rv es as th e c o n n e c to r th a t jo in s th e tw o clau ses. T h e n o u n c la u se who w ill do the work fu n c tio n s as th e o b je c t o f th e p r e p o sitio n about. In th e last e x a m p le th ere are also two clau ses: whoever is th e su b ject o f th e verb is com­ ing, a n d th e n o u n cla u se whoever is com ing to the party is th e su b ject o f m ust bring. T h e w ord whoever se rv es two fu n c tio n s in the se n te n c e : It is th e su b ject o f th e verb is coming, an d it is th e c o n n e c t o r th at jo in s th e two clau ses. T h e fo llo w in g e x a m p le sh ow s h o w this s e n te n c e p a ttern c o u ld b e te sted in th e Struc­ tu re se c tio n o f th e T O E F L test.

E xam ple was on television made me angry. (A) (B) (C) (D)

It The story What When

In th is e x a m p le y o u sh o u ld n o tic e im m e d ia te ly th at th e r e are two verb s, was a n d made, a n d e a c h o f th o s e verb s n e e d s a su bject. A n sw ers (A) a n d (B) are in c o r r e c t b e c a u se it an d the story c a n n o t b e th e su b ject fo r b o th was a n d m ade a t th e sa m e tim e. A n sw er (D ) is in ­ c o r r e c t b e c a u se w hen is n o t a su bject. In answ er (C ) w hal s e r v es as b o th th e su b ject o f th e verb w as a n d th e c o n n e c to r th a t jo in s th e tw o c la u se s to g e th er ; th e n o u n cla u se w hat was on television is th e su b ject o f th e verb made. A n sw er (C ) is th e r e fo r e th e b e st answer.

STRUCTURE

T h e fo llo w in g c h a r t lists th e n o u n c la u se c o n n e c to r /s u b j e c ts a n d th e s e n te n c e pat­ te rn s u sed w ith th em : NOUN CLAUSE CONNECTOR/SUBJECTS who whoever

what whatever

which whichever

'■ ............. .

■■ ■ ■

N O U N CLAUSE AS OBJECT

S

V

| (noun connector/subject)

/ know

w hat

V happened.

NO U N CLAUSE AS SUBJECT

| (noun connector/subject) W hat

V]

happened

V was great.

EX E R C ISE 10: E ach o f th e fo llo w in g s e n te n c e s c o n ta in s m o r e th a n o n e c la u se . U n d e r ­ lin e th e su b jects o n c e a n d th e verb s tw ice. C ircle th e c o n n e c to r s. P u t b o x e s a r o u n d th e n o u n cla u ses. T h e n in d ic a te i f th e s e n te n c e s are c o r r e c t (C ) o r in c o r r e c t (I ). C

_____

3.

The em ployee was unhappy about what was added to his jo b description.

_____

4.

W hoever wants to take the desert tour during spring break signing up at the office.

_____

5.

The m otorist was unable to discover who he had struck his car.

_____

6.

The voters should elect w hichever o f the candidates seem s best to them .

_____

7.

It was difficult to distinguish what was on.sale and what was m erely o n display.

--------

8.

You sh ould buy whatever the cheapest and m ost durable.

_____

9.

What was written in the letter angered him beyond belief.

---------

10.

You can spend your time with whoever im portant to you.

S kill

11:

USE ADJECTIVE CLAUSE C O N N E C TO R S CORRECTLY

A n a d jectiv e cla u se is a cla u se th a t d e sc r ib e s a n o u n . B e ca u se th e cla u se is a n a d jectiv e, it is p o s itio n e d d irectly a fter th e n o u n th a t it d escrib es. T he woman is filling the glass that sh e put on the table. ADJECTIVE CLAUSE

r

T he glass that she put on the table contains milk. ADJECTIVE CLAUSE

STRUCTURE AND WRITTEN EXPRESSION

In th e first e x a m p le th e r e are tw o clau ses: w om an is th e su b je ct o f th e v e rb is fillin g , an d she is th e su b ject o f th e verb pu t. T h a t is th e ad jective c la u se c o n n e c to r th a t j o in s th e se two c la u se s, a n d th e a d jectiv e c la u se that she p u t on the table d e sc r ib e s th e n o u n glass. In th e s e c o n d e x a m p le th e r e are also two clauses: glass is th e su b je ct o f th e verb con­ tains, a n d she is th e su b je ct o f th e verb p u t. In th is s e n te n c e a lso , th a t is th e a d jectiv e cla u se c o n n e c to r th at j o in s th e se two c la u se s, a n d th e ad jective c la u se th a t she p u t on the table d e ­ scrib es th e n o u n glass. T h e fo llo w in g e x a m p le sh ow s h o w th e se s e n te n c e p a tte rn s c o u ld b e te ste d in th e S tru ctu re s e c d o n o f th e T O E F L test.

E xam ple The g ift_____ selected for the bride was rather expensive. (A) because (B) was (C) since (D) which we

In th is e x a m p le y o u s h o u ld n o tic e q u ick ly th at th e r e are two c la u se s, g ift is th e su b je ct o f th e verb was, a n d th e verb selected n e e d s a su b ject. B e ca u se th e r e are tw o c la u se s, a c o n ­ n e c to r is also n e e d e d . A nsw ers (A ) a n d (C ) have c o n n e c to r s, b u t th e r e are n o su b jects, so th e se answ ers are n o t c o rr ec t. A n sw er (B ) c h a n g e s selected in to a passive verb; in th is case th e s e n te n c e w o u ld have o n e su b je ct a n d two verbs, so answ er (B ) is n o t c o r r e c t. T h e b est answ er to th is q u e s tio n is answ er ( D ) . T h e c o r r e c t s e n te n c e sh o u ld say: T he g ift w hich we se­ lected fo r the bride was rather expensive. In th is se n te n c e g ift is th e su b ject o f th e verb was, we is th e su b ject o f th e verb selected, a n d th e c o n n e c to r which j o in s th e s e two c la u se s. T h e fo llo w in g ch a rt lists th e ad jective c la u se c o n n e c to r s a n d th e s e n te n c e p attern s u se d w ith them : ADJECTIVE CLAUSE CONNECTORS whom

which

that

(for people)

(for things)

(for people o r things)

S

V

|(agiective^ gggggg>T) S

I liked the book S

The book

which

you recommended.

|(adjective connector)

which

V

S V |V

you recommended

wot Interesting.

NOTE: The adjective connectors can be omitted.This omission Is very common In spoken English or in casual written English. It is not as common in formal English or in the Structure section of the TOEFL test.

STRUCTURE

E X ER C ISE 11: E ach o f th e fo llo w in g s e n te n c e s c o n ta in s m o r e th an o n e c la u se . U n d e r ­ lin e th e su b je cts o n c e a n d th e verb s tw ice. C ircle th e c o n n e c to r s. P u t b o x e s a r o u n d th e ad jective cla u se s. T h e n in d ic a te if th e s e n t e n c e s are c o r r e c t (C ) o r in c o r r e c t (I). C

3.

I ju st finished reading the novel whom the professor suggested for my book report.

4.

T h e plane that h e was schedu led to take to Hawaii was delayed.

5.

T h e movie which we watched on cable last nigh t it was really frightening.

6.

I m ade an appointm ent with the doctor whom you recom m ended.

7.

T he enthusiasm with which he greeted m e m ade me fe el welcom e.

8.

T he story that you told m e about Bob.

9.

T h e m en with w hom were having the discussion did n ot seem very friendly.

10. I’m not really sure about taking part in the plans that we m ade last night.

S k i l l 12:

USE ADJECTIVE CLAUSE CONNECTOR/SUBJECTS CORRECTLY

In Sk ill 11 w e saw th a t a d je c tiv e c la u se c o n n e c to r s w e re u sed to in tr o d u c e c la u se s th a t d e ­ scrib e n o u n s . In Sk ill 12 w e w ill se e th a t in s o m e c a ses a n a d jectiv e c la u se c o n n e c to r is n o t ju s t a c o n n e c to r ; an ad je c tiv e c la u se c o n n e c t o r can a lso b e th e su b je c t o f th e c la u se a t th e sa m e tim e.

ADJECTIVE CLAUSE

In th e first e x a m p le th e r e a re tw o clau ses: w om an is th e su b je c t o f th e verb is fillin g , a n d that is th e su b je c t o f th e v e rb is. T h e s e tw o c la u se s are j o in e d w ith th e c o n n e c t o r that. N o ­ tice th a t in th is e x a m p le th e w o rd th a t se r v es tw o fu n c tio n s a t th e sa m e tim e: it is th e su b ­ j e c t o f th e verb is, a n d it is th e c o n n e c to r th a t j o in s th e two c la u se s. T h e a d je c tiv e c la u se th a t is on the table d e sc r ib e s th e n o u n glass. In th e s e c o n d e x a m p le , th e r e a re also tw o clau ses: glass is th e su b je c t o f th e verb con­ tains, a n d th a t is th e su b je c t o f th e v e rb is. In th is e x a m p le th a t also se r v e s tw o fu n c tio n s : it is th e su b je c t o f th e v e rb is, a n d it is th e c o n n e c to r th a t jo in s th e two c la u se s. B e c a u se th a t is on the table is an a d je c tiv e c la u se d e sc r ib in g th e n o u n glass, it d irectly fo llo w s glass.

STRUCTURE AND WRITTEN EXPRESSION

T h e fo llo w in g e x a m p le sh ow s h o w th e se s e n te n c e p a ttern s c o u ld b e te sted in th e S tru ctu re s e c tio n o f th e T O E F L test.

E xam ple is on the table has four sections. (A) The notebook (B) The notebook which (C) Because the notebook (D) In the notebook

In th is e x a m p le you sh o u ld n o tic e im m e d ia te ly that th e s e n te n c e has two verbs, is an d has, a n d e a c h o f th e m n e e d s a su b ject. (You k n ow th a t table is n o t a su b ject b e c a u se it follow s th e p r e p o sitio n on; table is th e o b je c t o f th e p r e p o sitio n .) T h e on ly answ er th a t has two su b jects is an sw er (B ), so answ er (B ) is th e c o r r e c t answer. T h e c o r r e c t s e n te n c e sh o u ld say: T he notebook which is on the table has fo u r sections. In this se n te n c e notebook is th e su b ject o f th e verb has, a n d which is th e su b ject o f th e verb is. W hich is also th e c o n n e c to r that jo in s th e two clau ses. T h e fo llo w in g ch a rt lists th e a d jecu v e c la u se c o n n e c t o r /s u b je c ts an d th e s e n te n c e p a tte rn s u se d w ith them : ADJECTIVE CLAUSE CONNECTOR/SUBJECTS who (for people) S

which (for things) V

|(adiective connector/subject)

She needs a secretary S A secretary

that (for people or things) VI

who

types fast.

|(adjective connector/subiect) V I who

V

types fast is invaluable.

EX ER C ISE 12: E ach o f th e fo llo w in g s e n te n c e s c o n ta in s m o r e than, o n e c la u se. U n d e r ­ lin e th e su b jects o n c e a n d th e verbs tw ice. C ircle th e c o n n e c to r s. P u t b o x e s a r o u n d th e ad je c u v e cla u ses. T h e n in d ic a te if th e s e n te n c e s are c o r r e c t (C ) o r in c o r r e c t (I). C

1.

The ice cream (that) is served in the restaurant has a sm ooth, creamy texture.

2.

T he cars are trying to enter the freeway system are lined up for blocks.

3.

I have great respect for everyone who on the D ean’s List.

4.

It is going to be very difficult to work with the man which just began working here.

5.

The door that leads to the vault it was tightly locked.

6.

T he neighbors reported the man who was trying to break into the car to the police.

STRUCTURE

7.

T hese plants can only survive in an environm ent is extrem ely hum id.

8.

T he boss m eets with any production workers w ho they have surpassed their quotas.

9.

The salesclerk ran after the woman w ho had left her credit card in the store.

10. T h e shoes which m atched the dress that was on sale. EX ER C ISE (S k ills 9 - 1 2 ) : E ach o f th e fo llo w in g s e n te n c e s c o n ta in s m o r e th a n o n e c la u se . U n d e r lin e th e su b je cts o n c e a n d th e verb s tw ice. C ircle th e c o n n e c to r s . P u t b o x e s a r o u n d th e cla u ses. T h e n in d ic a te if th e s e n te n c e s are c o r r e c t (C ) o r in c o r r e c t (I). --------

1.

N o on e explained to m e w hether was com ing or not.

--------

2.

T he part o f the structure that has already b een built needs to be torn down.

--------

3.

T h e girl who she ju st jo in e d the softball team is a great shortstop.

--------

4. I have no idea about when the m eeting is supposed to start.

--------

5.

We have b een told that we can leave w henever want.

--------

6.

T h e racquet with w hom I was playing was too big and too heavy for me.

--------

7.

I will never understand that he did.

--------

8. H e was still sick was obvious to the entire m edical staff.

--------

9.

--------

10.

W'hat is m ost im portant in this situation it is to finish on time. T h e newspapers that were piled up on the front porch were an indication that the residents had not b een h om e in som e time.

T O E F L E X E R C ISE (S k ills 9—12): C h o o s e th e lette r o f th e w o rd o r g r o u p o f w ord s th a t b e st c o m p le te s th e s e n te n c e . 1. Dolphins form extremely complicated allegiances a n d -------- continually change. ,., ... , (A) enmities that . ... (B) that are enm ities (C) enmities that are (D) that enmities 2. Scientists are now beginning to conduct experiments o n _____ trigger different sorts of health risks. ,., . .... (A) noise pollution can in • pollution li .• (B)\ .u that. noise • (C ) how noise pollution (D) how noise pollution can

3. The Apollo I I astronauts_____ of the Earth’s inhabitants witnessed on the famous first moonwalk on July 20, 1969, .. „ .. .. were Neil Armstrong and Buzz Aldnn. (A) (B) (C) (D)

whom whom millions were some whom some were

4. At the end of the nineteenth century, Alfred Binet developed a test for measuring . . , . 6 r intelligence--------served as the basis of , . m odem IQ tests. (A) (B) (C) (D)

has it has and which has

STRUCTURE AND WRITTEN EXPRESSION

5 . _____ have at least four hours of hazardous materials response training is mandated by federal law. (A) (B) (C) (D)

All police officers All police officers must That all police officers For all police officers

6. A clouds reservoir of negative charge extends upward from the altitude a t -------the freezing point. (A) (B) (C) (D)

temperatures hit hit temperatures which temperatures hit which hit temperatures

7. In a 1988 advanced officers' training program, Sampson developed a plan to incorporate police in enforcing environmental protection laws whenever feasible. (A) (B) (C) (D)

it is is has it has

8. _____ will be carried in the next space shuttle payload has not yet been announced to the public. (A) (B) (C) (D)

It What When That

9. During free fa ll,_____ up to a full minute, a skydiver will fall at a constant speed of 120 m.p.h. (A) (B) (C) (D)

it is which is being is

10. The fa ct_____ the most important ratings period is about to begin has caused all three networks to shore up their schedules. (A) (B) (C) (D)

is that of that what

T O E F L REVIEW EX ER C ISE (S k ills 1 -1 2 ): C h o o se th e lette r o f th e w o rd o r g r o u p o f w ord s th a t b e st c o m p le te s th e s e n t e n c e . 1 . _____ loom high above the north and northeastern boundaries of the expanding city of Tucson. (A) The Santa Catalina mountains (B) Because the Santa Catalina mountains (C) The Santa Catalina mountains are (D) That the Santa Catalina mountains 2. Radioactive_____ provides a powerful way to measure geologic time. (A) (B) (C) (D)

it dates dating can

3. _____ contained in the chromosomes, and they are thought of as the units of heredity. (A) (B) (C) (D)

Genes which are Genes are When genes Because of genes

4. The benefit_____ the study is that it provides necessary information to anyone who needs it. (A) (B) (C) (D)

of which that because

5. The same symptoms that occu r-------occur with cocaine. (A) (B) (C) (D)

amphetamines can with amphetamines can so amphetamines with amphetamines they

6. Many companies across the country have molded the concepts_____ describes into an integrated strategy for preventing stress. (A) (B) (C) (D)

and Wolf that Wolf what Wolf so Wolf

STRUCTURE

7.

in the first draft of the budget will not necessarily be in the final draft. (A) (B) (C) (D)

Although it appears It appears What appears Despite its appearance

8. If a food label indicates that a food is mostly carbohydrate, it does not mean is a good food to eat. (A) (B) (C) (D)

9. A need for space law to include commercial concerns has been recognized inasm uch_____ been expanding drastically in recent years. (A) the commercial launch industry (B) the commercial launch industry has (C) as has the commercial launch industry (D) as the commercial launch industry has 10. The report on the nuclear power plant indicated that when the plant had gone on line ___ _ unsafe.

and it and that it when

(A) (B) (C) (D)

and it had been it had been had been that it had been

SENTENCES W IT H R ED U C ED CLAUSES. It is p o ssib le in E n g lish fo r a c la u se to a p p e a r in a c o m p le te fo r m o r in a r e d u c e d form . My friend sh ould be o n the train lafych ^ arriving at the station now. Although

w^s not realty difficult, the exam took a lot o f tirae.

T h e first s e n t e n c e sh o w s an a d je c tiv e c la u se in its c o m p le te fo r m , w hich is a rr iv in g at the station now, a n d in its r e d u c e d fo r m , a r r iv in g a t the sta tio n now. T h e s e c o n d s e n t e n c e sh ow s an adverb c la u se in its c o m p le te fo r m , although it was n o t really difficu lt, a n d its r e d u c e d fo r m , although n o t really difficu lt. T h e two typ es o f c la u se s th a t c a n r e d u c e in E n g lish are: (1) a d je c tiv e c la u se s a n d (2) adverb c la u se s. It is im p o r ta n t to b e c o m e fa m ilia r w ith th e se r e d u c e d c la u se s b e c a u s e th ey a p p e a r fr e q u e n tly o n th e T O E F L test. S k i l l 13:

USE RED U C ED AD JEC TIVE CLAUSES CORRECTLY

A d jective c la u se s c a n a p p e a r in a r e d u c e d fo r m . In th e r e d u c e d fo r m , th e a d je c tiv e c la u se c o n n e c to r a n d th e ¿«-verb th a t d ir e c d y fo llo w it are o m itte d . T h e wom an ntyo if waving to us is the tour guide. T h e letter whfch likis written last week arrived today. T h e pitcher t\a t

on the table is full o f iced tea.

E ach o f th e se s e n t e n c e s m ay b e u s e d in th e c o m p le te fo r m o r in th e r e d u c e d fo r m . In th e r e d u c e d fo r m th e c o n n e c to r who, which, o r th a t is o m itte d a lo n g w ith th e ¿>e-verb is o r was. If th e r e is n o be-\erb in th e a d je c tiv e c la u se , it is still p o ssib le to h ave a r e d u c e d fo r m . W h en th e r e is n o ¿»e-verb in th e a d je c tiv e c la u se , th e c o n n e c to r is o m itte d a n d th e verb is c h a n g e d in to th e -¿n gform .

STRUCTURE AND WRITTEN EXPRESSION

appearing I d o n ’t understand the article wftych apfcars in today’s paper. In th is e x a m p le th e r e is n o fte-verb in th e ad jective c la u se which appears in today's paper, so th e c o n n e c to r w hich is o m itte d a n d th e m ain verb appears is c h a n g e d to th e -in g form appearing. It sh o u ld b e n o te d th a t n o t all ad jective c la u se s can a p p ea r in a r e d u c e d fo r m . A n ad­ je c tiv e c la u se ca n a p p ea r in a r e d u c e d form o n ly i f th e ad jective c la u se c o n n e c to r is fo l­ lo w e d d irectly by a verb . In o th e r w ords, an ad jective cla u se c a n o n ly b e r e d u c e d if th e c o n n e c to r is a lso a su b ject. T he woman that I ju st met is the tour guide. T he letter which you sent me arrived yesterday.

(does not reduce) (does not reduce)

In th e s e two e x a m p le s th e ad jective cla u ses c a n n o t b e r e d u c e d b e c a u se th e adjective c la u se c o n n e c to r s th a t a n d which are n o t d irectly fo llo w e d by verbs; th a t is directly fo l­ lo w ed by th e su b ject I, a n d which is d irectly fo llo w e d by th e su b ject you. A fin al p o in t to n o te is th at so m e ad jective cla u ses are set o f f from th e rest o f th e s e n ­ te n c e w ith c o m m a s, a n d th e se ad jective cla u ses can also b e r e d u c e d . In a d d itio n , w h en an ad jective c la u se is se t o f f w ith c o m m a s, th e r e d u c e d ad jective cla u se can a p p ea r at th e fr o n t o f th e s e n te n c e . T he W hite H ouse, which is located in Washington, is the hom e o f the president. T he W hite H ouse, heated in Washington, is the hom e o f the president. Located in Washington, the W hite House is the hom e o f the president. T he president, who is now preparing to give a speech, is m eeting with his advisors. T he president, now preparing to give a speech, is m eeting with his advisors. Now prefmring to give a speech, the president is m eeting with his advisors. In th e s e two e x a m p le s , th e ad jective cla u ses are set o f f from th e rest o f th e s e n te n c e w ith c o m m a s, so e a c h s e n te n c e can b e str u c tu r ed in th r e e d iffe r e n t ways: ( 1 ) w ith th e c o m ­ p le te c la u se , (2 ) w ith th e r ed u c e d c la u se fo llo w in g th e n o u n th at it d e sc r ib es, an d (3) w ith th e r e d u c e d c la u se at th e b e g in n in g o f th e s e n te n c e . T h e fo llo w in g e x a m p le sh ow s h o w r e d u c e d ad jective c la u se s c o u ld b e te sted in the S tru ctu re se c tio n o f th e T O E F L test.

Exam ple _____ on several different television programs, the witness gave conflicting accounts of what had happened. (A) He appeared (B) Who appeared (C) Appearing (D) Appears

In th is e x a m p le , answ er (A) is in c o r r e c t b e c a u se th ere are two clau ses, H e appeared. . . an d the w itness g a v e . . . , a n d th ere is n o c o n n e c to r to jo in th e m . A nsw er (B) is in c o r r e c t b ecau se

STRUCTURE

an ad jective c la u se su c h as w ho appeared . . . c a n n o t a p p ea r at th e b e g in n in g o f a s e n te n c e (u n le ss it is in a r e d u c e d fo r m ). A n sw er (C ) is th e c o r r e c t an sw er b e c a u se it is th e re­ d u c e d fo r m o f th e c la u se w ho appeared, a n d th is r e d u c e d fo r m can a p p e a r at th e fr o n t o f th e s e n te n c e . A n sw er (D ) is n o t th e r e d u c e d fo rm o f a verb; it is m e r e ly a verb in th e p r e ­ s e n t ten se; a verb su c h as appears n e e d s a su b je c t a n d a c o n n e c to r to b e c o r r e c t. T h e fo llo w in g c h a r t lists th e str u c tu r e fo r r e d u c e d ad jective c la u se s a n d r u le s fo r h o w a n d w h e n r e d u c e d fo r m s c a n be u sed : REDUCED ADJECTIVE CLAUSES with a be-verb in the adjective clause

(ADJECTIvrtrONNECÎOR/SyBiEGT)— " __ __——fwfio which tfrat)~—

with no be-verb in the adjective clause

(AOJECTTvTtTONNtCIgR^yBJECT)---^'''^ ” -— (wfio which Vrai)'—

• • • •

(VERB + INC)

To reduce an adjective clause, omit the adjective clause connector/subject and the be-verb. If there is no be-verb, omit the connector/subject and change the main verb to the -ing form. Only reduce an adjective clause if the connector/subject is directly followed by the verb. If an adjective clause is set off with commas, the reduced clause can be moved to the front of the sentence.

E X E R C ISE 13: E ach o f th e fo llo w in g s e n te n c e s c o n ta in s a n a d je c tiv e c la u se , in a c o m ­ p le te o r r e d u c e d fo r m . U n d e r lin e th e ad jective cla u se s. T h e n in d ic a te i f th e s e n te n c e s are c o r r e c t (C ) o r in c o r r e c t (I). __ —_

1. We will have to return the m erchandise purchased yesterday at the Broadway.

__ L___

2. T he children sat in the fancy restaurant found it difficult to behave.

_____

3. Serving a term o f four years, the mayor o f the town will face reelection n ext year.

_____

4. T h e brand new Cadillac, purchasing less than two weeks ago, was destroyed in the accident.

_____

5.

___

T he fans who supporting their team always com e out to the gam es in large num bers.

6. T he suspect can be seen in the photographs were just released by the police.

_____

7. T h e food placing on the picnic table attracted a large num ber o f flies.

_____

8. Impressed with everything she had heard about the course, Marie sign ed her children up for it.

_____

9. T h e passengers in the airport waiting room , heard the an n ou n cem en t o f the canceled flight, groaned audibly.

_____

10. Dissatisfied with the service at the restaurant, the meal really war not enjoyable.

STRUCTUREANDWRITTEN EXPRESSION

S kill

14:

USE REDUCED ADVERB CLAUSES CORRECTLY

A d verb c la u se s can also ap p ea r in a r e d u c e d fo r m . In th e r e d u c e d fo r m , th e adverb c o n ­ n e c to r rem a in s, b u t th e su b ject a n d ¿e-verb are o m itte d . Although hp k rather unwell, the speaker will take part in the seminar. W'hen \\m

ready, you can begin your speech.

T h e s e tw o e x a m p le s m ay b e u sed in e ith e r th e c o m p le te o r r e d u c e d fo r m . In th e r ed u c ed fo r m , th e ad verb c o n n e c to r s although a n d when rem ain ; th e su b jects he a n d you as w ell as th e ¿e-verbs is a n d are are o m itted . I f th e r e is n o ¿e-verb in th e adverb c la u se , it is still p o s sib le to h ave a r e d u c e d form . W h e n th e r e is n o ¿e-verb in th e adverb c la u se, th e su b je ct is o m itte d a n d th e m a in verb is c h a n g e d in to th e -¡r e fo r m . feeling Although \e fetys rather sick, the speaker will take part in the seminar. giving When yhu gfyeyour speech, you should speak loudly and distinctly. In th e first e x a m p le th e adverb c la u se although he feels rather sick d o e s n o t in c lu d e a ¿everb; to r e d u c e th is c la u se , th e su b ject he is o m itte d a n d th e m a in verb feels is c h a n g e d to feeling. In th e s e c o n d e x a m p le th e adverb c la u se w hen you give y o u r speech also d o e s n o t in c lu d e a ¿e-verb; to r e d u c e th is c la u se , th e su b ject you is o m itte d a n d th e m a in verb give is c h a n g e d to g iving. T h e fo llo w in g e x a m p le sh ow s h ow this s e n te n c e p a tte r n c o u ld b e te sted in th e Struc­ tu re s e c tio n o f th e T O E F L test. E xam ple W hen_____ , you are free to leave. (A) the finished report (B) finished with the report (C) the report (D) is the report finished

In th is e x a m p le you sh o u ld n o tic e th e adverb c o n n e c to r when, a n d y o u sh o u ld k n ow th at th is tim e w ord c o u ld b e fo llo w e d by e ith e r a c o m p le te cla u se o r a r e d u c e d c la u se. A n ­ sw ers (A ) a n d (C ) c o n ta in th e su b jects the fin ish e d report an d the report a n d n o verb, so th e se an sw ers are in c o r r e c t. In answ er (D ) th e su b je ct a n d verb are in v e rte d , a n d this is n o t a q u e s tio n , so an sw er (D ) is in c o r r e c t. T h e c o r r e c t answ er is answ er (B ); th is answ er is th e r e d u c e d fo r m o f th e cla u se when you are fin ish e d w ith the report. It sh o u ld b e n o te d th at n o t all adverb c la u se s can a p p ea r in a r e d u c e d fo r m , an d a n u m b e r o f ad verb c la u se s can o n ly b e r e d u c e d i f th e verb is in th e passive form . Once you submit your thesis, you will graduate. (active — does not reduce)

Once ^ submitted., your thesis will be reviewed, (passive — does reduce)

In th e first e x a m p le , th e adverb clau se once you subm it y o u r thesis d o e s n o t r e d u c e b e c a u se c la u se s in tr o d u c e d by once o n ly r e d u c e i f th e verb is passive, a n d th e verb subm it is active. In th e s e c o n d e x a m p le , th e adverb cla u se once it is subm itted d o e s r e d u c e to once subm itted b e c a u se th e c la u se is in tr o d u c e d by once an d th e verb is subm itted is passive.

STRUCTURE

T h e fo llo w in g c h a r t lists th e str u c tu r es for r e d u c e d adverb c la u se s a n d w h ic h adverb c la u se c o n n e c to r s c a n b e u s e d in a r e d u c e d form : R E D U C E D A D V ER B C lA U S E S

w ith a be-verb in

(a d verb

c o n n ecto r)

X

the ad verb clause w ith no be-verb in

(VERB + ING)

(ADVERB CONNECTOR)

the adverb clause C o n d itio n

C o n tra s t

before

unless

though

since while when

whether

once until when

if unless whether

T im e reduces in

reduces in

a c t iv e

pa ssiv e

Place

M a n n er

where wherever

as

although

after

although though

whenever • To red u ce an ad verb clause, o m it th e subject and the be-verb fro m the ad verb clause. • If th e re is no be-verb, then o m it th e subject and change the ve rb to the -ing form .

E X ER C ISE 14: E ach o f th e fo llo w in g s e n te n c e s c o n ta in s a r e d u c e d ad verb c la u se . C ircle th e ad verb c o n n e c to r s . U n d e r lin e th e r e d u c e d c la u se s. T h e n in d ic a te i f th e s e n t e n c e s are c o r r e c t (C ) o r in c o r r e c t (I). ——

1 . (if) not com pletely satisfied, you can return the product to the manufacturer.

— !—

2. Steve has had to learn how to cook and clean (since) left hom e.

--------

3. T h e ointm ent can be applied where needed.

--------

4.

Tom began to look for a job after com pleting his master’s degree in engineering.

--------

5. A lthough n e t selectin g for the team, he attends all o f the gam es as a fan.

--------

6. W hen purchased at this store, the buyer gets a guarantee on all items.

--------

7.

T h e m edicine is not effective unless taken as directed.

--------

8.

You should negotiate a lot before buy a new car.

--------

9.

O n ce purchased, the swimsuits cannot be returned.

--------

10.

T h ou gh located near the coast, the town does not get m uch o f an ocean breeze.

STRUCTURE AND WRITTEN EXPRESSION

EX ER C ISE (Sk ills 1 3 -1 4 ): E ach o f th e fo llo w in g s e n te n c e s c o n ta in s a r e d u c e d c la u se. U n d e r lin e th e r e d u c e d c la u se s. T h e n in d ic a te i f th e s e n te n c e s are c o r r e c t (C ) o r in c o r ­ r e c t (I). --------

1. T hough was surprised at the results, she was pleased with what she had don e.

_ —

2. Wearing only a light sweater, she stepped out into the pouring rain.

--------

3. T he family stopped to visit m any relatives while driving across the country.

_____

4. T he com pany president, n eed ed a vacation, boarded a plane for the Bahamas.

_____

5. When applying for the jo b , you should bring your letters o f reference.

_____

6. She looked up into the dreary sky was filled with dark thunderclouds.

____ _

7. Feeling weak after a long illness, Sally wanted to try to get back to work.

_____

8. Before decided to have surgery, you should get a second opinion.

_____

9.

_____

10.

T he construction material, a rather grainy type o f wood, gave the room a rustic feeling. The application will at least be reviewed if submitted by the fifteenth o f the m onth.

T O E F L EXER CISE (S k ills 1 3 -1 4 ): C h o o s e th e le tte r o f th e w ord o r g r o u p o f w ord s th at b e st c o m p le te s th e s e n te n c e . 1. W hen_____ nests during spring nesting season, Canadian geese are fiercely territorial. (A) building (B) are building (C) built (D) are built 2. In 1870, Calvin, along with Adirondack hunter Alvah Dunning, made the first known ascent of Seward M ountain,_____ far from roads or trails. (A) (B) (C) (D)

a remote peak it is a remote peak a remote peak is which a remote peak

3. Kokanee salmon begin to deteriorate and die so o n _____ at the age o f four. (A) (B) (C) (D)

they spawn after spawning spawn spawned the salmon

4. --------- benind government secrecy for nearly half a century, the Hanford plant in central Washington produced plutonium for the nuclear weapons of the Cold War. (A) (B) (C) (D)

It is hidden Hidden Which is hidden The plant is hiding

5. U n til--------incorrect, astronomers had assumed that the insides of white dwarfs were uniform. (A) (B) (C) (D)

they their proof the astronomers recently proven recently proven

6. --------- artifacts from the early Chinese dynasties, numerous archeologists have explored the southern Silk Road. (A) (B) (C) (D)

They were searching for It was a search for Searched for Searching for

STRUCTURE

7. In Hailey, th e best-know n lectu rer was w om en’s rights activist Abigail Scott Duniway o f P ortland, Oregon, w ho could usually be persu ad ed to sp e a k _____ tow n visiting h e r son. (A) (B) (C) (D)

she w as in while in why le she was was in

8. The N ational R e sta u ra n t_____ W ashington, says th a t federal efforts to regulate w orkplace sm oking w ould lim it re stau ra n ts’ ability to respond to the desires of th e ir patrons. (A) (B) (C) (D)

Association in Association is in A ssociation w hich is in Association, based in

T O E F L R EV IEW E X E R C ISE (S k ills 1 -1 4 ): w ord s th a t b e st c o m p le t e s th e s e n te n c e . 1. In the United S tates_____ approximately four million miles of roads/streets, and highways. there (B) is (C) they (D) there are (A )

2 . _____ twelve million immigrants entered the United States via Ellis Island. More than (B) There were more than (C) Of more than (D) The report of (A )

3. The television,_____ so long been a part o f our culture, has an enormous influence. has (B) it has (C) which (D) which has (A )

4. Psychologists have traditionally maintained that infants cannot formulate long-term m emories u n til_____ the age of eight or nine months. they (B) they reach (C) to reach (D) reach (A )

9.

in N orth Am erican w aterw ays less th an a decade ago, zebra m ussels have already earned a nasty rep u tatio n for th eir expensive h abit of clogging w ater pipes in the Great Lakes area. (A) (B) (C) (D)

The first sighting Although first sighted Zebra m ussels w ere first sighted First sighting

10. Sm all com panies m ay take th eir goods abroad for trad e show s w ithout paying foreign value-added taxes by acquiring an ATA carnet. (A) (B) (C) (D)

a docum ent calls a docum ent called calls a docum ent called a docum ent

C h o o se th e lette r o f th e w o r d o r g r o u p o f

5. --------a cheese shop has since grown into a small conglomerate consisting of a catering business and two retail stores. (A) (B) (C) (D)

In the beginning of It began as Its beginning which was What began as

6. Primarily a government contractor,_____ preferential treatment from government agencies as both a minority-group member and a woman. (A) (B) (C) (D)

receives Weber Weber receives the reception o f Weber according to Weber's reception

7. Because the project depends o n _____ at the federal level, the city and county may have to wait until the budget cutting ends. (A) (B) (C) (D)

it happens which happening what happens that it happens

STRUCTURE AND WRITTEN EXPRESSION

8.

definitive study of a w estern h a rd ­ rock m ining com m unity cem etery appears to have been done is in Silver City, Nevada. (A) (B) (C) (D)

10.

_ early approaches for coping with w orkplace stress dealt w ith the problem only after its sym ptom s had appeared. (A) (B) (C) (D)

Most The m ost W here m ost W here the m ost

Although well intending Although it is a good intention Although a good intention Although well intended

One of the areas of m ultim edia th a t is growing quickly is sound. (A) (B) (C) (D)

yet is easily overlooked is easily overlooked it is easily overlooked th at is easily overlooked

SENTENCES W IT H INVERTED SUBJECTS A N D VERBS----------S u b jects a n d verb s are in v e rte d in a variety o f situ a tio n s in E n g lish . In v erted su b jects an d verb s o c c u r m o st o fte n in th e fo r m a tio n o f a q u e s tio n . T o fo r m a q u e s tio n w ith a h e lp in g verb (be, have, can, could, will, would, e t c .) , th e su b ject a n d h e lp in g v erb are in verted . H e can go to th e movies. C an h e go to th e movies? You w ould tell m e the tru th . W ould you tell m e th e tru th ? She was sick yesterday. Was she sick yesterday? To fo r m a q u e s tio n w h e n th e r e is n o h e lp in g verb in th e se n te n c e , th e h e lp in g verb do is u sed . H e goes to the movies. D oes h e go to th e movies? You to ld m e th e tru th . D id you tell m e th e tru th ? T h e r e are m a n y o th e r situ a tio n s in E n g lish w h e n su b je cts a n d verb s are in v e rte d , b u t if y o u j u s t r e m e m b e r th is m e th o d o f in v e rtin g su b jects a n d verb s, y o u w ill b e a b le to h a n d le th e o th e r situ a tio n s. T h e m o st c o m m o n p r o b le m s w ith in v e rte d su b jects an d verb s o n th e T O E F L te st o c c u r in th e fo llo w in g situ ation s: (1 ) w ith q u e s tio n w ord s su ch as what, when, where, why, a n d how; (2) a fter so m e p la c e e x p r essio n s; (3) a fter n e g a tiv e ex p ressio n s; (4) in s o m e c o n d itio n a ls; an d (5 ) after s o m e c o m p a r iso n s.

STRUCTURE

S k ill

15:

IN V E R T T H E SUBJECT A N D V E R B W IT H Q U E S T IO N W O R D S

T h e r e is s o m e c o n fu s io n a b o u t w h e n to in v e rt th e su b ject a n d verb a fter q u e s tio n w ord s su c h as what, when, where, why, a n d how. T h e se w ords can have two v e ry d if fe r e n t fu n c tio n s in a s e n te n c e . First, th e y c a n in tr o d u c e a q u e s tio n , a n d in th is case th e su b je c t a n d verb th a t fo llo w a r e in v e rte d .

What is the hom ew ork? When can I leave? Where a re you g o in g ? A lso, th e se w o rd s c a n j o in to g e th e r tw o cla u ses, a n d in th is c a se th e su b je c t a n d verb th a t fo llo w are n o t in v e rte d . I d o n o t know what the hom ew ork is. When I can leave, I will take th e first train. D o you know where you. a re g o in g ? In e a c h o f th e s e e x a m p le s th e r e are tw o cla u ses j o in e d by a q u e s tio n w ord . N o t ic e th a t th e su b jects a n d verb s th a t fo llo w th e q u e s tio n w ord s w hat, when, a n d where are n o t in v e r te d in th is case. T h e fo llo w in g e x a m p le sh ow s h o w this s e n te n c e p a tte rn c o u ld b e te s te d in th e S tru c­ tu re se c tio n o f th e T O E F L test.

E x am p le it.

The law yer asked the client why (A) did he do (B) did he (C) he did (D) did

*

In th is e x a m p le th e q u e s tio n w o rd why is u s e d to c o n n e c t th e two c la u se s, so a su b je c t a n d verb are n e e d e d a fter th is c o n n e c to r ; this is n o t a q u e s tio n , so th e su b je c t a n d verb s h o u ld n o t b e in v e r te d . T h e b e st an sw er is th e r e fo r e answ er (C ). T h e fo llo w in g c h a r t lists th e q u e s tio n w ord s a n d th e ir s e n t e n c e p a ttern s: INVERTED SUBJECTS AND VERBS WITH QUESTION WORDS when —

When the questii

---------- T T - r r

:





:

introduces a question, the subject and verb are invei mestion woi

W hen the questii

:onnects tw o clauses, the subject and verb that follow are not ii question won I know ..

-

STRUCTURE AND WRITTEN EXPRESSION

E X E R C IS E 15: E ach o f th e fo llo w in g s e n te n c e s c o n ta in s a q u e s tio n w ord. C ircle th e q u es­ tio n w ord s. U n d e r lin e th e su b jects o n c e a n d th e verb s tw ice. T h e n In d ic a te i f th e se n ­ te n c e s are c o r r e c t (C ) o r in c o r r e c t (I).

C

1.

The p h on e company is n ot certain (When) will the new directories be ready.

2.

The professor does not understand (why) so many students did poorly on the exam.

3.

How new students can get inform ation about parking?

4.

W here is it cheapest Ld get typeset copies printed?

5.

Only the pilot can tell you how far can the plane go on one tank o f fuel.

6.

What type o f security does he prefer for his investments?

7.

N ot even the bank president knows when the vault will be opened.

8.

H ow lon g it has been since you arrived in the U nited States?

9.

T he jury doubts what the witness said under cross-examination.

10. Do you know why he wants to take an extended leave o f absence?

S k ill

16:

INVERT TH E SUBJECT A N D VERB W IT H PLACE EXPRESSIONS

A fte r id e a s e x p r e s sin g p la c e , th e su b ject a n d th e verb so m e tim e s in v ert in E n g lish . T h is ca n h a p p e n w ith sin g le w ord s e x p r e s sin g p la c e, su ch as here, there, o r nowhere. Here is the book that you lent me. There are the keys that I thought I lost. Nowhere have I seen such beautiful weather. In th e fir s t e x a m p le th e p la c e w ord here c a u ses th e su b ject book to c o m e a fter th e verb is. In th e s e c o n d e x a m p le th e p la c e w ord there c a u ses th e su b je ct keys to c o m e after th e verb are. In th e last e x a m p le th e p la c e w ord nowhere ca u se s th e su b ject I to c o m e after th e verb have. T h e su b je c t a n d verb can a lso b e in v erted a fter p r e p o sitio n a l p h ra ses e x p r e ssin g p la c e. In the closet are the cloth es that you want. Around the comer is Sam’s house. Beyond the mountains lies the town where you will live. In th e first e x a m p le th e p r e p o sitio n a l p h ra se o f p la c e in the closet c a u se s th e su b je ct clothes to c o m e a fter th e verb are. In th e s e c o n d e x a m p le th e p r e p o sitio n a l p h ra se o f p lace a ro u n d the com er c a u se s th e su b ject house to c o m e after th e verb is. In th e la st e x a m p le th e p r e p o sitio n a l p h r a se o f p la c e beyond the m o u n ta in s ca u se s th e su b je c t tow n to c o m e after th e verb lies.

STRUCTURE

It is im p o r ta n t (a n d a b it d iffic u lt) to u n d e r sta n d th a t th e su b je ct a n d verb w ill in vert a fter p la c e e x p r e s s io n s at th e b e g in n in g o f a s e n te n c e o n ly w h e n th e p la c e e x p r e s s io n is necessary to c o m p le te th e s e n te n c e . S tu d y th e fo llo w in g ex a m p les: In the forest are many exotic birds. In the forest I walked for many hours. In th e first e x a m p le th e su b je c t birds a n d verb are are in v e rte d b e c a u se th e p la c e e x p r e s­ s io n in the forest is n e e d e d to c o m p le te th e id e a m a n y exotic birds are. . . . In th e s e c o n d e x ­ a m p le th e su b je ct / a n d th e verb w alked a r e n o t in v e r te d b e c a u se th e id e a I w alked f o r m any hours is c o m p le t e w ith o u t th e p la c e e x p r e s sio n in the forest; th e p la c e e x p r e s sio n is th e r e ­ fo r e n o t n e e d e d to c o m p le te th e s e n te n c e . T h e fo llo w in g e x a m p le sh ow s h o w th is s e n te n c e p a tte r n c o u ld b e te ste d in th e S tru c­ tu re s e c tio n o f th e T O E F L test.

Exam ple On the second level of the parking lot (A) is empty (B) are empty (C) som e empty stalls are (D) are some empty stalls T h is e x a m p le b e g in s w ith th e p la c e e x p r e s sio n on the second level o f the p a rk in g lot, w h ich c o n sists o f two p r e p o sitio n a l p h ra ses, on the second level a n d o f the p a rk in g lot. T h is s e n te n c e n e e d s a su b je ct a n d a v e rb to b e c o m p le te , a n d th e two answ ers th a t c o n ta in b o th a su b ­ j e c t , stalls, a n d verb , are, are an sw ers (C ) a n d (D ). T h e su b ject a n d verb sh o u ld b e in ­ v e rte d b e c a u s e th e p la c e e x p r e s sio n is n e c e s sa r y to c o m p le te th e id e a some empty stalls are. . . . T h e b e st an sw er is th e r e fo r e an sw er (D ). T h e fo llo w in g ch art lists th e s e n t e n c e p a tte rn s u s e d w ith p la c e e x p r essio n s: INVERTED SUBJECTS AND VERBS WITH PLACE EXPRESSIONS W h e n a place exp ression at th e fro n t o f the sentence is necessary to co m p lete th e sentence, th e subject and v e rb that fo llo w are inverted. ( PLACE (n e c e s s a ry ))

V

S

In the classroom were some old desks. W h e n a place expression at th e fro n t o f th e sen ten ce contains extra inform ation th a t is not needed to co m p le te th e sen ten ce, the subject and ve rb that fo llo w are not inverted.

( PLACE (extra) )

S

V

In the classroom,I studied very hard.

STRUCTURE AND WRITTEN EXPRESSION

E X E R C ISE 16: E a ch o f th e fo llo w in g s e n te n c e s c o n ta in s an e x p r e s sio n o f p la c e at th e b e­ g in n in g o f th e se n te n c e . C ircle th e e x p r e s sio n s o f p la c e. L o o k at th e cla u ses th a t im m e d i­ a tely fo llo w th e p la c e e x p r e ssio n s a n d u n d e r lin e th e su b jects o n c e a n d th e verb s tw ice. T h e n in d ic a te i f th e s e n te n c e s are c o r r e c t (C ) o r in c o r r e c t (I).

c

1 . (In front o f the house) were som e giant trees.

1

2. (There) a big house is on the corner. 3.

In the cave was a vast treasure o f gem s and jewels.

4.

To the north the stream is that the settlers will have to cross.

fi.

Around the corner are the offices that you are trying to Find.

6. At the Italian restaurant was the food too spicy for my taste. V.

Now here in the world farmers can grow such delicious food.

8. In the backyard the two trees are that need to be pruned. 9. Around the recreation hall and down the path are the tents where we will be staying this week.

j.___

10. In the apartment next to m ine, a family was that had a lot o f pets.

S k ill

17:

IN VER T T H E SUBJECT A N D VERB W IT H NEGATIVES

T h e su b je c t a n d verb can also b e in v e rte d a fter c er ta in n e g a tiv es a n d r ela ted ex p r essio n s. W h e n n e g a tiv e e x p r e ssio n s, su ch as no, not, o r never, c o m e at th e b e g in n in g o f a se n te n c e , th e su b je ct a n d verb are in v erted . Not once did I miss a question. Never has Mr. Jones taken a vacation. A t no time can the woman talk on the telephone. In th e first e x a m p le th e n e g a tiv e e x p r e ssio n not once c a u se s th e su b ject I to c o m e after the h e lp in g verb did. In th e s e c o n d e x a m p le th e n e g a tiv e w ord never ca u se s th e su b ject Mr. Jones to c o m e after th e h e lp in g verb has. In th e last e x a m p le th e n e g a tiv e e x p r e ssio n at no tim e ca u se s th e su b je ct w om an to c o m e after th e h e lp in g verb can. C erta in w ord s in E n g lish , su ch as hardly, barely, scarcely, a n d only, act lik e n egatives. I f o n e o f th e s e w o rd s c o m e s at th e b e g in n in g o f a s e n te n c e , th e su b je ct a n d verb are also in v e rte d . Hardly ever does he take tim e off. (This means that he almost never takes time off.) Only once did the manager issue overtim e paychecks. (This means that the manager almost net;«-issued overtime paychecks.)

STRUCTURE

I n th e first e x a m p le th e “a lm o st n e g a tiv e ” e x p r e s sio n hardly ever c a u se s th e su b je c t he to c o m e a fter th e h e lp in g verb does. In th e s e c o n d e x a m p le th e “a lm o st n e g a tiv e ” e x p r e s sio n only once c a u se s th e su b je ct m anager to c o m e after th e h e lp in g verb did. W h e n a n e g a tiv e e x p r e s sio n a p p ea r s in fr o n t o f a su b je ct a n d verb in t h e m id d le o f a s e n t e n c e , th e su b je c t an d verb are also in v e rte d . T h is h a p p e n s o fte n w ith th e n e g a tiv e w o rd s neither a n d nor. I do n o t want to go, and neither does Tom . T he secretary is n ot attending the m eeting, nor is her boss. In th e first e x a m p le th e n e g a tiv e neither ca u se s th e su b je ct Tom to c o m e a fte r th e h e lp in g v e rb does. In th e s e c o n d e x a m p le th e n e g a tiv e nor c a u se s th e su b ject boss to c o m e a fter th e v erb is. T h e fo llo w in g e x a m p le sh ow s h o w th is s e n te n c e p a tte rn c o u ld b e te ste d in th e S tru c­ tu re s e c tio n o f th e T O E F L test.

Exam ple stopped.

Only in extremely dangerous situations (A) will be the printing presses (B) the printing presses will be (C) that the printing presses will be (D) will the printing presses be

In th is e x a m p le y o u s h o u ld n o tic e th a t th e s e n t e n c e b e g in s w ith th e n e g a tiv e only, so an in v e r te d su b je c t a n d verb are n e e d e d . A n sw er (D ) c o n ta in s a c o r r e c tly in v e r te d su b je c t a n d verb, w ith th e h e lp in g verb w ill, th e su b ject p r in tin g presses, a n d th e m a in v e rb be, so an sw er (D ) is th e b e st answ er. T h e f o llo w in g c h a r t lists th e n e g a tiv e e x p r e s sio n s a n d th e s e n te n c e p a tte r n u s e d w ith th em : IN V ER T ED S U B JE C T S A N D V E R B S W IT H N EG A T IVES

no

not

never

barely

hardly

only

neither rarely

nor scarcely

■ '______ •______;_____ ■■ ____________;______ . ■. ._____ ■

-

_____

seldom

-____________

W hen a negative expression appears in front of a subject and verb (at the beginning of a sentence or in the middle of a sentence), the subject and verb ore inverted. (negative expression) Rarely .

'

' '





• •

.

.

_

V

S

were they so happy. _____ . -

• .

-

'

.

■ - ■’ •

STRUCTURE AND WRITTEN EXPRESSION

E X E R C ISE 17: E ach o f th e fo llo w in g s e n te n c e s c o n ta in s a n e g a tiv e o r “a lm o st negative" e x p r e s s io n . C ircle th e n e g a tiv e e x p r essio n s. L o o k at th e c la u se s th a t fo llo w a n d u n d e r lin e th e su b je cts o n c e a n d th e verb s tw ice. T h e n in d ic a te i f th e s e n te n c e s a r e c o r r e c t (C) o r in ­ c o r r e c t (I). — !—

1. (N ever) the boy wrote to his sisters.

— ——2. (O n no occasion) did they say that to me. --------

3.

Steve did not win the prize, nor did he expect to do so.

--------

4.

Only once in my life gone I have to New York City.

_____

5.

Did he go out o f the house at no time.

_____

6.

Seldom their secretary has made such mistakes.

--------

7.

N o sooner had she hung up the ph one than it rang again.

--------

8.

Sheila did not arrive late for work, nor she left early.

_____

9.

Barely had he finished the exam when the graduate assistant collected the papers.

--------

10.

T he police did not arrive in time to save the girl, and neither did the paramedics.

S k ill

18:

IN V E R T T H E SUBJECT A N D VERB W IT H C O N D IT IO N A L S

In c e r ta in c o n d itio n a l stru ctu res, th e su b ject a n d verb m ay a lso be in v e rte d . T h is can o c ­ cu r w h e n th e h e lp in g verb in th e c o n d itio n a l c la u se is had, should, o r were, an d th e c o n d i­ tio n a l c o n n e c t o r i f is o m itte d . I f he had taken more time, the results would have been better. Had he taken more tim e, the results would have been better. I would help you i f I were in a position to help. I would help you were 1 in a position to help. I f you should arrive before 6:00, just give me a call. Should you arrive before 6:00, just give m e a call. In e a c h o f th e s e e x a m p le s y o u can s e e th at w h e n i f is in c lu d e d , th e su b ject a n d verb are in t h e r eg u la r o r d e r ( i f he h a d taken, i f I were, i f you should arrive). It is also p o ssib le to o m it if, in this c a se , th e su b ject a n d verb are in v erted ( h a d he taken, were I, should y o u a rrive).

STRUCTURE

T h e fo llo w in g e x a m p le sh o w s h o w th is s e n te n c e p a ttern c o u ld b e te ste d in th e S tr u c ­ ture se c tio n o f th e T O E F L test.

Exam ple The report would have been accepted (A) (B) (C) (D)

in checking its accuracy.

if more care more care had been taken had taken more care had more care been taken

In th is e x a m p le a c o n n e c to r ( if) a n d a su b je ct a n d verb are n e e d e d , b u t ¡ /c o u ld b e o m it­ ted a n d th e su b je ct a n d verb in v e r te d . A n sw er (A) is in c o r r e c t b e c a u se it c o n ta in s th e c o n n e c to r i /a n d th e su b je ct care b u t n o verb. A n sw er (B ) js in c o r r e c t b e c a u se it c o n ta in s th e su b je ct care a n d th e verb had. been taken b u t d o e s n o t have a c o n n e c to r . In an sw ers (C ) an d (D ), i f h as b e e n o m itte d . B e c a u se it is c o r r e c t to in vert th e su b ject more care a n d th e h e lp in g verb had, a n sw er (D ) is c o r r e c t. T h e fo llo w in g c h a r t lists th e c o n d itio n a l verb s th a t m ay in vert a n d th e s e n te n c e p at­ tern s u se d w ith them :

INVERTED SUBJECTS AND VERBS WITH CONDITIONALS had

should

were

When the verb in the conditional clause is had, should, or were, it is possible to omit if and invert the subject and verb. (omitted if)

V

S

W ere he

here, he would help.

It is also possible to keep if! Then the subject and verb are not inverted. if If

S

V

he were

here, he would help.

EX ER C ISE 18: E ach o f th e fo llo w in g s e n te n c e s c o n ta in s a c o n d itio n a l (w ith a sta te d o r im p lie d i f ) . C ircle th e c o n d itio n a ls , o r p u t an asterisk (*) w h er e ¡ / h a s b e e n o m itte d . L ook at th e c la u se s th a t fo llo w an d u n d e r lin e th e su b jects o n c e a n d th e verb s tw ice. T h e n in d ic a te if th e s e n te n c e s are c o r r e c t (C ) o r in c o r r e c t (I). — ^L_

1. *Were our neighbors a bit m ore friendly, it would be somewhat easier to get to know them .

— !—

2. There are plenty o f blankets in the closet if should you get cold during the night.

--------

3. Has he en ou gh vacation days left this year, he will take two full weeks o f f in December. 4.

Had we b een inform ed o f the decision, we m ight have had som ething to say about it.

STRUCTURE AND WRITTEN EXPRESSION

5. I would like to know could you help m e pack these boxes. 6. H e would have b een in big trouble had n ot he rem em bered the assignm ent at the last m inute. 7. If your friends com e to visit, will they stay in a hotel or at your house? 8. H e m ight be a little m ore successful today was he a little m ore willing to do som e hard work. 9.

Should you ever visit this town again, I would be delighted to show you around.

10. D o you think that she would give the speech were she asked to do so?

S k ill

19:

IN VER T T H E SUBJECT A N D VERB W IT H COMPARISONS

A n in v e r te d su b je c t a n d verb m ay o c c u r also a fter a c o m p a r iso n . T h e in v e rsio n o f a su b ­ j e c t a n d verb a fter a c o m p a r is o n is o p tio n a l, ra th er th an r e q u ir e d , an d it is a ra th er for­ m al str u c tu r e. T h e r e h ave b e e n a n u m b e r o f in v e rte d c o m p a r iso n s o n r e c e n t T O E FL tests, so y o u s h o u ld b e fa m ilia r w ith th is stru ctu re. My sister spends more hours in the office

T he report you are looking for could be in the file (or) on the desk.

3.

She works very hard but usually gets below-average grades.

4.

T he speaker introduced him self, told several interesting anecdotes, and finishing with an em otional plea.

5.

You should know w hen the program starts and how many units you must com plete.

6.

T he term paper he wrote was rather short but very impressive.

7.

She suggested taking the plane this evening or that we go by train tomorrow.

8 . T he dean or the assistant dean will inform you o f when and where you should apply for your diplom a. 9. 10.

S kill 2 5 :

T here are papers to file, reports to type, and those letters should be answered. T he m anager n eed ed a quick but thorough response.

USE PARALLEL STR U C TU R E W IT H PAIRED C O N J U N C T IO N S

T h e p a ir e d c o n ju n c tio n s both . . . a nd, e ith e r. . . or, n e ith e r . . . nor, a n d n ot only . . . but also req u ire p a ra llel stru ctu res. I know both where you went and what you did. Either Mark orSue has the book. T h e tickets are neither ir^my pocket nor in my purse. H e is not only an excellen t student but also an outstanding athlete. T h e fo llo w in g is n o t p a r a lle l a n d m u st b e c o rr ec ted : H e wants either to go by train or by plane*. It is n o t c o r r e c t b e c a u se to go by train is n o t p a ra llel to by plane. It can b e c o r r e c te d in sev­ eral ways. H e wants either to g o by train or to g o by plane. H e wants to go eitherby train or by plane. H e wants to go by either train or plane. W h en y o u are u s in g th e s e p a ir e d c o n ju n c tio n s , b e su re that th e c o r r e c t parts are u se d to­ g eth er. T h e fo llo w in g a re in c o rr ec t: I want both this book or* that one. F.itherSam nor* Sue is taking the course. T h e se s e n te n c e s are in c o r r e c t b e c a u se th e w r o n g parts o f th e p a ir ed c o n ju n c tio n s are u sed to g e th er . In th e first e x a m p le , a n d sh o u ld b e u se d w ith both. In th e s e c o n d e x a m p le , or sh o u ld b e u s e d w ith either. •

STRUCTURE AND WRITTEN EXPRESSION

T h e fo llo w in g c h a r t o u tlin e s th e u se o f p a r a lle l stru ctu re w ith p a ir e d c o n ju n ctio n s: PARALLEL STRUCTURE WITH PAIRED CONJUNCTIONS ’ . :'

both either

neither .: " ••• : not only •

(same structure) "

r

and or nor but also

(same structure) •

- . • .-. .; •

.

.....

v.. . . . .

EX E R C ISE 25: E ach o f th e fo llo w in g s e n te n c e s c o n ta in s w ord s o r g r o u p s o f w ords that sh o u ld b e p arallel. C ircle t h e w ord o r w ord s th a t in d ic a te th a t th e s e n te n c e sh o u ld have p arallel parts. U n d e r lin e th e parts th a t sh o u ld b e p a ra llel. T h e n in d ic a te if th e s e n te n c e s are c o r r e c t (C ) o r in c o r r e c t (I). __ l _

c

1.

According to the syllabus, you can (either) write a paper (or)you can take an exam .

2.

It would be (jaoth) noticed (and) appreciated if you could finish the work before you leave.

_____

3.

She would like neither to see a movie or to go bowling.

4.

Either the manager or her assistant can help you with your refund.

5.

She wants not only to take a trip to Europe but she also would like to travel to Asia.

6.

H e could correct neither what you said nor you wrote.

7.

Both the tailor or the laundress could fix the dam age to the dress.

8.

H e not only called the police departm ent but also called the fire departm ent.

9.

You can graduate either at the end o f the fall sem ester or you can graduate at the end o f the spring semester.

10.

S k il l 2 6 :

T he m ovie was neither am using nor was it interesting.

USE PARALLEL S TR U C TU R E W IT H COM PARISONS

W h e n y o u m a k e a c o m p a r is o n , y o u p o in t o u t th e sim ila r itie s o r d iffe r e n c e s b e tw e e n two th in g s , a n d th o s e sim ila r itie s o r d iffe r e n c e s m u st b e in p a ra llel fo r m . You c a n r e c o g n iz e a c o m p a r is o n sh o w in g h o w tw o th in g s are d iffe r e n t fro m th e - e r . . . th a n or th e more . . . than. My school is farther than your sch ool. To be rich is better than to be poor. W hat is written is more easily understood than what is spoken.

WRITTEN EXPRESSION

A c o m p a r iso n sh o w in g h o w tw o th in g s are th e sam e m ig h t c o n ta in as . . . as o r e x p r e s­ sio n s su c h as the sam e as o r sim ila r to. T heir car is as big as a small house. Renting those apartments costs about the same as leasing them. T he work that I did is similar to the work that you did.

T h e fo llo w in g c h a r t o u tlin e s th e u se o f p a ra llel str u c tu r es w ith c o m p a r iso n s:

PARALLEL STRUCTURE WITH COMPARISONS ■v : .■

.■

-:

'•

- V- - V: .' '

'

(same structure) ‘

; ' - -•V-

.

more ... than ■ -er... than less ... than 'v. ! -, b as ...as the same... as ■d similar ...to

'

: -■ . -

--

- ; ■:

(same structure) --

EXERCISE 26: E a ch o f th e fo llo w in g s e n t e n c e s c o n ta in s w ord s o r g r o u p s o f w o r d s th a t sh o u ld b e p a r a lle l. C ircle th e w o rd o r w ord s th a t in d ic a te th a t th e s e n t e n c e s h o u ld h ave p a r a lle l parts. U n d e r lin e th e parts th a t s h o u ld b e p arallel. T h sn in d ic a te i f e a c h s e n t e n c e is c o r r e c t (C ) o r in c o r r e c t (I). __ 0— 1.

His research for the thesis was (more useful than) hers.

__ !__

2.

D inin g in a restaurant is (m ore fun than) to eat at hom e.

_____

3.

I want a new secretary who is as efficient as the previous one.

_____

4.

What you do today sh ould be the same as did yesterday.

_____

5.

This lesson is m ore difficult than we had before.

_____

6.

You have less hom ework than they do.

_____

7.

What you do has more effect than what you say.

_____

8.

Music in your country is quite similar to my country.

_____

9.

T h e collection o f foreign journals in the university library is m ore extensive than the high school library.

_____

10.

How to buy a used car can be as difficult as buying a new car.

188

STRUCTURE AND WRITTEN EXPRESSION

EXERCISE (Skills 2 4 -2 6 ): Circle the word or words that indicate that the sentence should

have parallel parts. Underline the parts that should be parallel. Then indicate if the sen­ tences are correct (C) or incorrect (I). _____ 1.

After retirem ent he plans on traveling to exotic locations, dine in the finest restaurants, and playing a lo t o f golf.

_____

2.

She was both surprised by and pleased with the seminar.

_____

3.

What cam e after the break was even m ore boring than had com e before.

_____

4.

H e would find the missing keys neither under the bed or b ehind the sofa.

_____

5.

D epend in g on the perspective o f the viewer, the film was considered laudable, mediocrity, or horrendous.

_____

6.

H e exercised not only in the m orning, but he also exercised every afternoon.

_____

7.

Working four days per w eek is much m ore relaxing than working five days per week.

_____

8.

Sam is always good-natured, generous, and helps you.

_____

9.

Either you have to finish the project, or the contract will be canceled.

_____

10.

T he courses that you are required to take are m ore im portant than the courses that you choose.

T OEFL EXERCISE (Skills 24 -2 6 ): Choose the letter of the word or group of words that

best completes the sentence. 1. Truman Capote’s In Cold Blood is neither journalistically accurate (A) (B) (C) (D)

a piece of fiction nor a fictitious work or written in a fictitious way nor completely fictitious

2. Vitamin C is necessary for the prevention a n d _____ of scurvy. (A) (B) (C) (D)

it cures cures cure for curing

3.

A baby’s development is influenced by both heredity and (A) (B) (C) (D)

by environmental factors environmentally the influence of the environment environment

4. Because bone loss occurs earlier in women th a n _____ , the effects of osteoporosis are more apparent in women. (A) (B) (C) (D)

men do in men as men similar to men

WRITTEN EXPRESSION

C h o o se th e le tte r o f th e u n d e r lin e d w o rd o r g r o u p o f w ord s th a t is n o t c o r r e c t.

5.

Fire extinguishers can contain liquefied gas, dry chemicals, or watery. A

B

C- - D

6.

The U.S. Congress consists of both the Senate as well as the House o f Representatives A B C D

7.

The prison population in this state, now at an all time high, is higher than anv state. A B C D

8. A well-com posed baroque opera achieves a delicate balance by focusing alternately A B C on the aural, visual, emotional, and philosophy elements. D 9.

-10.

Manufacturers may use food additives for preserving, to color, to flavor, or to fortifv A B —C ~ foods.

A bankruptcy may be either voluntary nor involuntary. A B C W

TOEFL REVIEW EXERCISE (Skills 1—26): C h o o se th e lette r o f th e w ord o r g r o u p o f w ord s th at b e st c o m p le te s th e s e n te n c e . 1. The growth of hair_____ cyclical process, with phases of activity and inactivity. (A) (B) (C) (D)

it is is a which is a regular

(A) (B) (C) (D)

2. The fire. . to have started in the furnace under the house. (A) (B) (C) (D)

is believed that is believed they believe that they believe

3. In Roman numerals, numeric values. (A) (B) (C) (D)

4. The legal systems of m ost countries can Ipe classified--------com m on law or civil law. as either either as either to to either

5. One difference between mathematics and language is that mathematics is precise (A) (B) (C) (D)

language is not while language is not but language not while is language

. symbols for

are letters of the alphabet letters of the alphabet are which uses letters of the alphabet in which letters of the alphabet are

6. Your criticism of the three short stories should not be less than 2,000*words, nor more than 3,000. (A) (B) (C) (D)

should it be it should be it is should be it

STRUCTURE AND WRITTEN EXPRESSION

C h o o se th e le tte r o f th e u n d e r lin e d w o rd o r g r o u p o f w o rd s th a t is n o t co rr ec t.

7.

In 1870, the attorney general was made head of the Department of Justice, given an A enlarged staff, and endow with clear-cut law-enforcement functions. B C D

8.

The General Sherman Tree, the largest of all the giant sequoias, are reputed to be the ------ Â B C worlds largest living thing. D

_ 9.

The skeleton of a shark is made of cartilage rather than having bone. ~fiT B C D

_10

At least one sample of each of the brands contains measurable amounts of aflatoxin, A B and there is three which exceed the maximum.

PROBLEMS W IT H COMPARATIVES A N D SUPERLATIVES. S e n te n c e s w ith in c o r r e c t co m p a ra tiv es a n d su p erla tiv es c a n a p p ea r o n th e T O E F L test. It is th e r e fo r e im p o r ta n t fo r y ou to k n ow h o w to d o th e fo llo w in g : ( 1 ) fo rm th e c om p arative a n d su p erla tiv e correctly; (2) u s e th e c o m p a r a tiv e an d su p erla tiv e correctly; a n d (3 ) u se th e irr eg u la r -er, -er stru ctu re th a t h as b e e n a p p e a r in g fr e q u e n tly o n th e T O E F L test.

S k i l l 27:

FORM COM PARATIVES A N D SUPERLATIVES CORRECTLY

T h e p r o b le m w ith so m e o f th e c o m p a r a tiv e a n d su p e rla tiv e s e n te n c e s o n th e T O E F L test is th a t th e c o m p a r a tiv e o r su p erlative is fo r m e d in co rrectly . You sh o u ld th e r e fo r e u n d e r ­ sta n d h o w to fo r m th e c o m p a r a tiv e a n d su p erla tiv e to an sw er su c h q u e s tio n s correctly. T h e co m p a r a tiv e is fo r m e d w ith e it h e r -er o r more a n d than. In th e co m p a r a tiv e , -er is u s e d w ith sh o r t a d jectiv es su ch as tall, an d more is u s e d w ith lo n g e r a d jectiv es su ch as

beautiful. Bob is taller than Ron. Sally is more beautiful than Sharon. T h e su p e rla tiv e is fo r m e d w ith the, e ith e r -est or most, a n d so m e tim e s in, of, o r a that-c la u se. In th e su p e rla tiv e, -est is u se d w ith sh o rt ad jectives su ch as tall, a n d most is u s e d w ith lo n g e r a d jectiv es su ch as beautiful.

,

Bob is the tall erf man in the room. Sally is the most beautiful o f all the w om en at the party. T he spider over there is (Aelargesi on e that I have ever seen. The fastest runner wins the race, (no in, of, or that)

WRITTEN EXPRESSION

T h e fo llo w in g c h a r t o u tlin e s th e p o s sib le fo r m s o f co m p a ra tiv es a n d su p erlatives:

THE FORM OF COMPARATIVESAND SUPERLATIVES more (long adjective) (short adjective) + er

COMWWATIVE

• than

r

,

SUPERLATIVE •

'

most (long adjective) [ (short adjective) + est

-,

J

.

, . , . maybe ,n. o f thot

• . '

EXERCISE 27: E ach o f th e fo llo w in g s e n te n c e s c o n ta in s a c o m p a r a tiv e o r su p e rla tiv e. C ircle th e co m p a r a tiv e o r su p e rla tiv e. T h e n in d ic a te i f th e s e n te n c e s are c o r r e c t (C ) or in c o r r e c t (I ). I

1.

O xygen is (abundanter than )nitrogen.

2.

T he directions to the exercise say to choose (the m ost appropriate) response.

3.

The lesson you are studying now is the most im portantest lesson that you will have.

4. Fashions this year are shorter and more colorful than they were last year. 5. T he professor indicated that A nthony’s research paper was m ore lon g than other students’ papers.

the

6. Alaska is the cold est than all the states in the U nited States. 7. T h e workers on the day shift are m ore rested than the workers on the nigh t shift. 8.

She was m ore happier this m orning than she had been yesterday.

9.

T he quarterback on this year’s football team is more versatile than the quarterback on last year’s team.

10. She always tries to do the best and m ost efficien tjob that she can do.

S k i l l 28:

USE CO M PARATIVES A N D SUPERLATIVES CORRECTLY

A n o th e r p r o b le m w ith th e c o m p a r a tiv e a n d su p erlative o n th e T O E F L te st is th a t th e y can b e u s e d in co rrectly . T h e c o m p a r a tiv e a n d su p erla tiv e have d iffe r e n t u ses, a n d y o u s h o u ld u n d e r s ta n d th e se d iffe r e n t u ses to answ er su c h q u e s tio n s correctly. T h e c o m p a r a tiv e is u s e d to c o m p a r e tw o e q u a l th in gs. T h e history class is larger than the math class. Mary is more intelligent than Sue. In th e first e x a m p le the history class is b e in g c o m p a r e d w ith the m ath class. In th e s e c o n d e x a m p le M a ry is b e in g c o m p a r e d w ith Sue.

STRUCTURE AND WRITTEN EXPRESSION

T h e su p erla tiv e is u s e d w h e n th e r e are m o r e th an two item s to c o m p a r e an d y o u w an t to sh ow th e o n e th a t is th e b est, th e b ig g e st, or in so m e way th e m o st o u tsta n d in g . T he history class is the largest in the school. Mary is the most intelligent o f all the students in the class. In th e first e x a m p le the history class is c o m p a r e d w ith all th e o th e r cla sses in th e s c h o o l, a n d th e h isto r y class is la rg er th a n e a c h o f th e o th e r classes. In th e s e c o n d e x a m p le , M a ry is c o m p a r e d w ith all th e o th e r stu d e n ts in th e class, a n d M ary is m o r e in te llig e n t th an e a c h o f th e o th e r stu d en ts. T h e fo llo w in g ch art o u tlin e s th e u ses o f co m p a ra tiv es a n d su perlatives: THE USES OF COMPARATIVES AND SUPERLATIVES The

c o m p a r a t iv e

The

s u p e r l a t iv e

Is used to compare two equal things.

is used to show which one of many is in some way the most outstanding.

EXERCISE 28: E ach o f th e fo llo w in g s e n te n c e s c o n ta in s a co m p a ra tiv e o r su p erla tiv e. C ircle th e com p arative o r su p erla tiv e. T h e n in d ic a te i f th e s e n te n c e s are c o r r e c t (C ) or in c o r r e c t (I). __ ~ r_

1.

__ !__

2. Rhonda is (more hard working) o f the class.

_____

3.

The engineers hired this year have more experience than those hired last year.

--------

4.

The graduate assistant inform ed us that the first exam is the most difficult o f the two.

_____

5. He bought the more powerful stereo speakers that he could find.

_____

6. T he afternoon seminar was m uch more interesting than the m orning lecture.

--------

7.

The food in this restaurant is the best o f the restaurant we visited last week.

--------

8.

The plants that have been sitting in the sunny window are far healthier than the other plants.

_____

9. The photocopies are the darkest that they have ever been.

--------

10. The first journal article is the longest o f the second article.

S

29:

k il l

Harvard is probably (the m ost prestigious) university in the U nited

States.

USE T H E IRREGULAR -ER, -ER STRUCTURE CORRECTLY

A n irr eg u la r c o m p a r a tiv e stru ctu re th a t has b e e n a p p ea r in g fr e q u e n tly o n th e T O E FL te st co n sists o f tw o p a ra llel c o m p a r a tiv e s in tr o d u c e d by the. The harder he tried, the further he fell behind. The older the children are, the more their parents expect from them.

WRITTEN EXPRESSION

the harder a n d the further. T h e sec­ the older a n d the more.

T h e firs t e x a m p le c o n t a in s th e tw o p a r a lle l c o m p a r a tiv e s o n d e x a m p le c o n t a in s t h e tw o p a r a lle l c o m p a r a tiv e s I n th is ty p e o f s e n te n c e ,

the a n d th e c o m p a r is o n c a n b e f o llo w e d b y a n u m b e r o f d if­

f e r e n t s tru c tu re s .

The more children you have, the bigger the house you need. The harder you work, the more you accom plish. The greater the experien ce, the higher the salary.

the more is f o llo w e d by th e n o u n children a n d t h e s u b je c t a n d v e r b you have, w h ile the bigger is f o llo w e d b y th e n o u n the house a n d th e s u b je c t a n d v e r b you need. In th e s e c o n d e x a m p le , the harder is f o llo w e d b y th e s u b je c t a n d v e rb you work, w h ile the more is fo llo w e d b y th e s u b je c t a n d v e r b you accomplish. I n th e t h ir d e x a m p le , the greater is fo l­ lo w e d o n ly b y th e n o u n the experience, w h ile the higher is f o llo w e d o n ly b y t h e n o u n the salary. \fou s h o u ld n o te t h a t th is la s t e x a m p le d o e s n o t e v e n c o n t a in a v e r b , y e t it is a c o r ­

I n th e firs t e x a m p le ,

r e c t s t r u c t u r e in E n g lis h . T h e f o llo w in g c h a r t o u tlin e s th is ir r e g u la r

-er, -er s tru c tu re :

THE-ER-ER STRUCTURE -er THE

more



....... ..

.. • -

.

(same structure),

• .

the

-er more

(same structure)

This type of sentence may o r may not include a verb.

-er, -er s tru c t u r e . the. U n d e r l in e th e p a rts th a t s h o u ld b e p a r a lle l. T h e n in ­

E X E R C I S E 29: E a c h o f t h e f o llo w in g s e n te n c e s c o n t a in s t h e i r r e g u la r C ir c le th e tw o c o m p a r is o n s w ith d ic a te

i f th e s e n te n c e s a re c o r r e c t ( C ) o r i n c o r r e c t

(I).

^

1.

(The hotter) the food is,(h ard ei) it is to eat.

C

2.

(T he warmer) the weather, (the greater) the attendance at the ou td oor concert.

_____

3.

The m ore you say, the worst the situation will be.

_____

4.

T he m ore time they have to play, the happier the children are.

_____

5.

The thicker the walls, the noise that com es through is less.

_____

6.If you run faster, the m ore quickly you’ll arrive.

_____

7.

T he m ore you use the p h on e, the higher the bill will be.

_____

8.

T he harder you serve, the easier it is to win the point.

_____

9.

T h e earliest you send in your tax forms, the sooner you will receive your refund.

_____

10.

The m ore p eople there are at the party, you ’ll have a good time.

STRUCTURE AND WRITTEN EXPRESSION

EXERCISE (Skills 27-2 9 ): C ircle th e com p a ra tiv es a n d su p erla tiv es in th e fo llo w in g se n ­ te n c e s. T h e n in d ic a te if th e s e n te n c e s are c o r r e c t (C ) o r in c o r r e c t (I). _____

1.

The coffee is m ore stronger today than it was yesterday.

_____

2.

The tree that was struck by lightning had b een the tallest o f the two trees we had in the yard.

_____

3.

H e will buy the m ost fuel-efficient car that h e can afford.

_____

4.

The closest it gets to summer, the longer the days are.

_____

5.

The business departm ent is bigger o f the departm ents in the university.

_____

6.

1 really do not want to live in the Southeast because it is on e o f the m ost hot areas in the U n ited States.

_____

7.

It is preferable to use the m ost efficient and m ost effective m eth od that you can.

_____

8.

T onight’s dinner was m ore filling than last nigh t’s.

_____

9.

The soon er the exam is scheduled, the less tim e you have to prepare.

_____

10.

The hou se is now the cleanest that it has ever been.

TOEFL EXERCISE (Skills 2 7 -2 9 ): C h o o se th e le tte r o f th e w ord o r g r o u p o f w ord s that b e st c o m p le te s th e s e n te n c e . 1. The speed of light i s _____ the speed of sound. (A) (B) (C) (D)

faster much faster than the fastest as fast

3. _____ in Stevenson’s landscapes, the more vitality and character the paintings seem to possess. (A) (B) (C) (D)

The brushwork is loose The looser brushwork The loose brushwork is The looser the brushwork is

2. The use of detail i s _____ method of developing a controlling idea, and almost all students employ this method. (A) (B) (C) (D)

more common common m ost common the most common

C h o o se t h e lette r o f th e u n d e r lin e d w ord o r g r o u p o f w ord s th at is n o t c o rr ec t. _____ 4.

_____ 5.

Certain types of snakes have been known to survive fasts more as a year long. A B ~C~ If The grizzly bear, which can grow up to eight feet tall, has been called a more A B C D dangerous animal of North America.

WRITTEN EXPRESSION

_ 6.

Climate,

soil type, and availability o f water A TT

are the m ost criticalfactors than ~C~

selecting the best type of grass for a lawn. D _ 7.

Peter Abelard, a logician and theologian, was the controversialest teacher o f his age. A ~B C D

_ 8.

Protein m olecules are the m ost complex than the molecules o f carbohydrates. A B ~C~ D

_ 9.

The leek,

a A

_10.

member of the lily family, has a B ~TT

mildest taste than the onion. D

The widely used natural fiber of all is cotton. A B~ ~TT D

TOEFL REVIEW EXERCISE (Skills 1—29): C h o o se th e lette r o f th e w o r d o r g r o u p o f w ord s th a t b e st c o m p le te s th e se n te n c e . 1 . ____ , a liberal arts college specifically for deaf people, is located in Washington, D.C (A) (B) (C) (D)

Gallaudet College Gallaudet College is About Gallaudet College Because o f Gallaudet College

2. --------varieties o f dogs at the show, including spaniels, poodles, and collies. (A) (B) (C) (D)

3. While the discovery that many migratory songbirds can thrive in deforested wintering sp ots-------- , the fact remains that these birds are dying at unusual rates. (A) (B) (C) (D)

it is heartening hearten heartening is heartening

The several Those Several There were several

C h o o se th e le tte r o f th e u n d e r lin e d w o rd o r g r o u p o t w ord s u ia t is n o t c o r r e c t. --------4.

The coyote is somewhat smaller in size that a timber wolf. A B ~C D­

--------5.

The weather reports all showed that there were a tremendous storm front m oving in. A B ~C~ D

--------6.

Seldom cactus plants are found outside o f North America. A ~B C ~D

--------7.

In a basketball game a player what is fouled receives one or two free throws. ~A BCD

-------- 8. Until recently, California was largest producer of oranges in the United States. A B C D

STRUCTURE AND WRITTEN EXPRESSION

9.

An understanding of engineering theories and problems are impossible until basic B A arithmetic is fully mastered. C D

.10.

The earliest the CVS (chorionic villus sampling) procedure in the pregnancy, the ■c A B greater the risk to the baby. D

PROBLEMS W IT H T H E FO R K OF T H E VERB It is c o m m o n in th e W ritten E x p r e ssio n p art o f th e T O E F L test fo r th e verb s to b e fo r m e d in correctly. T h e r e fo r e , you s h o u ld c h e c k th e fo r m o f th e verb carefully. You sh o u ld b e fa­ m iliar w ith th e fo llo w in g verb form s: th e b ase fo r m , th e p r e se n t te n se , th e p r e se n t p a rtici­ p le , a n d th e p a st p a r ticip le . T h e fo llo w in g are e x a m p le s o f e a c h o f th e se verb fo r m s as they are u sed in this text: BASE FORM* walk hear cook sing come

begin

PRESENT

PRESENT PARTICIPLE

walk(s) hear(s) cook(s) sing(s) come(s) begin(s)

walking hearing cooking singing coming beginning

PAST PARTICIPLE walked heard cooked sang came began

walked heard cooked

sung come begun

You sh o u ld b e particu larly aw are o f th e fo llo w in g th r e e p r o b le m a tic situ a tio n s w ith verb s b e c a u se th ey a re th e m o st c o m m o n a n d th e e a sie st to co rrect: ( 1 ) c h e c k w h at c o m e s a fter have; (2) c h e c k w hat c o m e s a fter be; a n d (3 ) c h e c k w h at c o m e s a fter will, w ould, a n d o th e r m od als. NOTE: A more complete list of verb forms and an exercise to practice their use are included at the back of the text in Appendix F. You may want to complete this exercise before you continue with skills 30 through 32.

S k i l l 30:

AFTER HAVE, U S E T H E PAST PARTICIPLE

W h e n e v e r you s e e th e verb h a v e in any o f its fo r m s (have, has, having, h a d ), b e su re th a t th e verb that fo llo w s it is in th e p ast p a r tic ip le form . They had walk* to school. We have see* the show. H e has took* the test. H a v in g a t^ , he went to school. She should have did* the work.

(should (should (should (should (should

be be be be be

had walked) have seen) has taken) H aving eaten) should have done)

WRITTEN EXPRESSION

In a d d itio n , y o u sh o u ld b e su re th a t i f y o u h ave a su b ject a n d a p ast p a r tic ip le , y o u a lso have th e verb have. T h is p r o b le m is p articu larly c o m m o n w ith th o s e v e rb s (su c h as sing, sang, su n g ) th a t c h a n g e fr o m p r e se n t to p ast to p a st p a r ticip le by c h a n g in g o n ly th e vow el. My friend sung* in the choir. H e become* angry at his friend. T he boat sunk* in the ocean.

(should be (should be (should be

sang or has sung) became o r has become) sank or has sunk)

T h e fo llo w in g c h a r t o u tlin e s th e u se o f verb fo r m s a fter have: VERB FORMS AFTER HAVE________________________ _____________________

.

HAVE ________

__

past participle

E X E R C ISE 30: E ach o f th e fo llo w in g s e n te n c e s c o n ta in s a v e rb in th e p a st o r a p ast p ar­ tic ip le . U n d e r lin e th e verb s o r p ast p a r ticip le s tw ice. T h e n in d ic a te i f th e s e n t e n c e s are c o r r e c t (C ) o r in c o r r e c t (I). '

1. T h e young girl drunk a glass o f milk.

C

2. Before she left, she had asked her m other for perm ission.

_____

3.

H aving finished the term paper, he began studying for the exam .

_____

4.

T h e secretary has broke her typewriter.

_____

5.

T he installer should have com pletes the task m ore quickly.

_____

6.

H e has often becom e angry during meetings.

_____

7.

She has rarely rode her horse in the park.

_____

8.

Having saw the film, he was quite disappointed.

_____

9. Tom has thought about taking that job.

_____

10. You m ight have respond m ore effectively.

S k i l l 31:

AFTER BE, USE T H E PRESENT PARTICIPLE O R T H E PAST PARTICIPLE

T h e verb be in an y o f its fo r m s (a m , is, are, was, were, be, been, being) can b e fo llo w e d by a n ­ o th e r verb . T h is verb s h o u ld b e in th e p r e se n t p a r ticip le or th e p ast p a r tic ip le fo r m . We are dai* our hom ework. T he hom ework was do* early. Tom is take* the book. T he book was take* by Tom.

(should (should (should (should

be be be be

are doing) was done) is taking) was taken)

T h e fo llo w in g c h a r t o u tlin e s th e u se o f verb fo r m s a fter be: VERB FORMS AFTER BE BE

+

(1) present participle (2) past parti cple

■ ■.

, ' ..

:



STRUCTURE AND WRITTEN EXPRESSION

E X ER C ISE 31: E ach o f th e fo llo w in g s e n te n c e s c o n ta in s a verb fo r m e d w ith be. U n d e r lin e th e verbs tw ice. T h e n in d ic a te if th e s e n te n c e s are c o r r e c t (C ) o r in c o r r e c t (I). I

1.

At 12:00 Sam is eat his lunch.

C

2.

We are m eeting them later today.

3.

The message was took by the receptionist.

4.

Being heard was extrem ely im portant to him.

5.

The Smiths are build their house on som e property that they own in the desert.

6.

It had been noticed that som e staff m embers were late.

7.

The report should have been subm it by noon-.

8.

Are the two com panies m erge into one?

9.

He could be taking four courses this semester.

10. The score inform ation has been duplicates on the back-up disk.

S k i l l 32:

AFTER W ILL.W O ULD , O R O TH ER M ODALS, USE T H E BASE FORM OF T H E VERB

W h e n e v e r y o u s e e a m o d a l, su c h as will, w ould, shall, should, can, could, may, m ight, o r must, y o u sh o u ld b e su re th at th e verb th a t fo llo w s it is in its base form . T he boat will leaving* at 3:00. T he doctor may arrivess* soon. T he students must taken* the exam .

(should be will leave) (should be may arrive) (should be must take)

T h e fo llo w in g ch a rt o u d in e s th e u se o f verb fo r m s after m odals: VERBS FORMS AFTER MODALS MODAJ.

+

main form o f th e verb

EXER CISE 32: E a ch o f th e fo llo w in g s e n te n c e s c o n ta in s a verb fo r m e d w ith a m o d a l. U n ­ d e r lin e th e verbs tw ice. T h e n in d ic a te i f th e s e n te n c e s are c o r r e c t (C) o r in c o r r e c t (I). —-—

1.

The salesclerk m ight lower the price.

— !—

2.

T he television movie will finishes in a few minutes.

3.

Should everyone arrive by 8:00?

4.

T he m ethod for organizing files can be improved.

WRITTEN EXPRESSION

_____

5.

The m achine may clicks o ff if it is overused.

_____

6.

Every m orning the plants must be watered.

_____

7.

The houses with ocean views could sell for considerably m ore.

_____

8.

Would anyone liked to see that movie?

_____

9.

I do n ot know w hen it will depart.

_____

10.

She will work on the project only if she can has a full-time secretary.

EX ER C ISE (S k ills 3 0 - 3 2 ) : U n d e r lin e th e verb s tw ice in th e fo llo w in g se n te n c e s . T h e n in ­ d ica te i f th e s e n te n c e s are c o r r e c t (C ) o r in c o r r e c t (I). _____

1.

I have gave you all the m oney that I have.

_____

2.

The articles were put in the newspaper before he was able to stop production.

_____

3.

All the tickets for the concert m ight already be sold.

_____

4.

H e was so thirsty that he drunk several large glasses o f water.

_____

5.

The deposit will has to be paid before the apartm ent can be rented.

_____

6.

H e objects to being held without bail.

____ _

7.

Having com pleted the first chapter o f the manuscript, she decid ed to take a break.

_____

8.

If Steve had really wanted to pass his exam , he would has studied m uch m ore.

_____

9.

H e thought that he sh ould have be invited to attend the conference.

_____

10.

Before the speaker finished, many guests had rose from their seats and started for the door.

T O E F L E X E R C ISE (S k ills 3 0 -3 2 ): C h o o se th e le tte r o f t h e u n d e r lin e d w o r d o r g r o u p o f w ords th a t is n o t c o rr ec t. _____ 1.

Alice in Wonderland, first published in 1865, has since being translated into thirty A B C D languages.

_____ 2. _____ 3 .

The Peace Corps was establish on March 1, 1961, by then President John F. Kennedy. A B C D The advisor told himself, while listening to the speech, that a dozen other reporters ~A~ B would has already asked that question. C D

STRUCTURE AND WRITTEN EXPRESSION

. 4.

At the start of the American Revolution, lanterns were hung in the Old North Church ~A~ B C as a signal that the British were came. D

. 5.

Before he died, Linus Pauling had wins two Nobel Prizes: the 1954 Nobel Prize ~A B C in Chemistry and the 1962 Nobel Peace Prize. D

. 6.

On the huge Ferris wheel constructed for a world exhibition in Chicago in 1893, ~A~ B each of the thirty-six cabs could held sixty people. C HET

.7 .

To overcome rejection of a skin graft, a system for matching donor and recipient A B C tissues has be developed.

TT

_ 8. Nails are commonly makeof steel butalso can contain substances ~A~ B “C D

such as aluminum

or brass. .9 .

A patient suffering from amnesia may had partial or total loss of memory. ~B C~ "D-

_10. Theidea o f using pure nicotine to help smokers stop was first tries in the mid-1980s ~A~ B — ~D ~ with the nicotine-laced chewing gum Nicotette.

TO EFL REVIEW EXERCISE (Skills 1-32): C h o o se th e le tte r o f th e w o rd o r g r o u p o f w ord s th a t b e st c o m p le te s th e s e n te n c e . . separates Manhattan’s Upper East Side from the Upper West Side. (A) (B) (C) (D)

Central Park Where Central Park Where is Central Park Central Park which

2. Bioluminescent anim als_____ the water or on land. (A) (B) (C) (D)

live are living either they are found in can be found in

3. The purpose of a labor union is to improve the working conditions,_____ , and pay of its members. (A) (B) (C) (D)

jobs are secure to be secure job security the job's security

4. W hen. _on July 4, 1789, the federal tariff, intended by the Founding Fathers to be the government’s primary source of revenue, was remarkably evenhanded. (A) (B) (C) (D)

was first enacted first enacted was enacted first it first

. inclined to push for such a reduction, it would probably not be successful. (A) (B) (C) (D)

The Office of Management The Office of Management was In the Office of Management Were the Office of Management

WRITTEN EXPRESSION

C h o o se th e le tte r o f th e u n d e r lin e d w o rd o r g r o u p o f w ords th a t is n o t c o r r e c t. -------- 6.

--------7.

-------- 8.

Helium has the most low boiling point of all substances. ~A~ B " C D There is twenty-six bones in the human foot, fourteen of them in the toes. A B ~C~ D Extension of the countdown hold to fourteen hours was order to give crews ~A~ B more time to repair wiring and clear away equipment. C D

--------9.

The study demonstrates that neither experience or awareness will improve chances A B ~C D of success.

--------10.

Som e of the eye movements used in reading is actually unnecessary. ~A~ B C D

PROBLEMS W IT H T H E USE O F T H E VERB___________________ M any d iffe r e n t p r o b le m s in u s in g th e c o r r e c t verb te n se are p o ssib le in E n g lish . H ow ever, fo u r sp e c ific p r o b le m s o c c u r fr eq u e n tly o n th e T O E F L test, so y o u n e e d to pay c a r e fu l at­ te n tio n to th e s e four: ( 1 ) k n o w in g w h e n to u se th e past w ith th e p r e se n t, ( 2 ) u s in g h a d a n d have correctly, (3 ) u s in g th e c o r r e c t te n se w ith tim e e x p r e s sio n s, a n d (4 ) u s in g th e c o r r e c t te n s e w ith w ill a n d would. S kill

33:

K N O W W H E N T O USE T H E PAST W IT H T H E PRESENT

O n e verb te n s e p r o b le m th a t is c o m m o n b o th in stu d e n t w ritin g a n d o n th e T O E F L te st is th e sw itch fro m th e p ast te n s e to th e p r e se n t te n se fo r n o p articu lar r e a s o n . O fte n w h e n a s e n te n c e h as b o th a p ast te n s e a n d a p r e se n t te n se , th e s e n te n c e is in c o r r e c t. H e took the m oney w hen he wants* it. T h is s e n te n c e says th at he took the money (in th e past) when he w a n ts it (in th e p r e s e n t). T h is m e a n in g d o e s n o t m a k e an y se n se; it is im p o ssib le to d o s o m e th in g in th e p a st as a r esu lt o f s o m e th in g y o u w a n t in th e p r e se n t. T h is s e n te n c e can b e c o r r e c te d in sev era l ways, d e ­ p e n d in g o n th e d e s ir e d m e a n in g . H e took the m oney when he wanted it. H e takes the m oney when he wants it. T h e first e x a m p le m e a n s th a t he took the money (in th e past) when he w an ted it (in th e p a st). T h is m e a n in g is lo g ic a l, a n d th e s e n t e n c e is co rr ec t. T h e s e c o n d e x a m p le m e a n s th a t he takes the money (h a b itu a lly ) w hen he w ants it (h ab itu ally). T h is m e a n in g is a lso lo g ic a l, a n d th e s e c o n d e x a m p le is also c o rr ec t.

202

STRUCTURE AND WRITTEN EXPRESSION

It is n e c e s sa r y to p o in t o u t, how ever, th a t it is p o ssib le fo r a lo g ic a l s e n te n c e in E n glish to have b o th th e p a st a n d th e p r e s e n t te n se . 1 know that he took the m oney yesterday. T h e m e a n in g o f th is s e n te n c e is lo g ica l: 1 know (r ig h t now , in th e p r e se n t) th a t he took the money (yesterday, in th e p a st). You can se e from th is e x a m p le that it is p o s sib le fo r an E n g­ lish s e n t e n c e to h ave b o th th e p ast a n d th e p r e se n t ten se. T h e err o r y o u n e e d to avoid is th e sw itch from th e p ast to th e p r e se n t fo r n o p articu lar rea so n . T h e r e fo r e , w h en y o u see a s e n te n c e o n th e T O E F L test w ith b o th th e p ast a n d th e p r e se n t te n se , y o u m u st c h e c k th e m e a n in g o f th e s e n t e n c e ca refu lly to se e if it is lo g ic a l in E n glish . T h e fo llo w in g ch a rt o u tlin e s th e u se o f th e p ast ten se w ith th e p r e se n t te n se in E nglish:

USING THE PASTWITH THE PRESENT 1. If you see a sentence with one verb in the past and one verb in the present, the sentence is probably incorrect. 2.

However, it is possible for a correct sentence to have both past and present together.

3. If you see the post and present together, you must check the meaning to determine whether or not the sentence is correct.

E X E R C ISE 33: E ach o f th e fo llo w in g s e n te n c e s h a s at lea st o n e verb in th e p ast a n d o n e verb in th e p r e se n t. U n d e r lin e th e verb s tw ice a n d d e c id e if th e m e a n in g s are lo g ica l. T h e n in d ic a te i f th e s e n te n c e s are c o r r e c t (C ) o r in c o r r e c t (I). 1

1.

I tell him the truth when he asked m e the question.

C

2.

1 understand that you were angry.

3.

W hen he was a child, he always goes to the circus.

4.

Last sem ester h e reads seven books and wrote five papers.

5. Steve wakes up early every m orning because he went to work early.

fi

Mark studied at the American University when he is in W ashington, D.C.

7.

H e is telling the teacher why he did not have time to finish his homework.

8.

H e put som e m oney in his account when h e goes to the bank.

9.

Tom keeps studying hard because he intended to go to dental school.

10

S kill 3 4 :

She is where she is today because she worked hard when she was a student.

USE HAVE A N D HAD CORRECTLY

T w o te n se s th a t are o fte n c o n fu s e d are th e p r e se n t p e r fe c t (have + past p a r tic ip le ) and th e past p e r fe c t (had + p ast p a r tic ip le ). T h e s e tw o te n se s have c o m p le te ly d iffe r e n t uses, a n d you s h o u ld u n d e r sta n d h o w to d iffe r e n tia te th e m .

WRITTEN EXPRESSION

T h e p r e se n t p e r fe c t ( have + p a st p a r ticip le ) refers to th e p e r io d o f tim e from the p ast

u n til the present. Sue has lived in Los A ngeles for ten years. T h is s e n te n c e m e a n s th a t S u e h as liv ed in L os A n g e le s for th e te n years u p to now . A c­ c o r d in g to this s e n te n c e , S u e is still liv in g in L os A n g e le s. B e ca u se th e p r e se n t p e r fe c t r efe rs to a p e r io d o f tim e from th e p ast u n til th e p r e se n t, it is n o t c o r r e c t in a s e n t e n c e th a t in d ic a te s p ast only. A t the start o f the nineteenth century, Thomas Jefferson has become* president o f the U nited States. Every tim e Jim worked on his car, he has improved* it. In th e first e x a m p le , th e p h ra se at the start o f the nineteenth century in d ic a te s th a t th e a c tio n o f th e verb was in th e p ast only, b u t th e verb in d ic a te s th e p e r io d o f tim e fr o m th e p ast u n ­ til th e p r e se n t. S in c e th is is n o t lo g ic a l, th e s e n te n c e is n o t c o r r e c t. T h e verb in th e first e x a m p le sh o u ld b e became. T h e s e c o n d e x a m p le in d ic a te s th a tjim worked o n h is ca r in th e past, b u t h e im p r o v e d it in th e p e r io d fr o m th e p ast u n til th e p r e se n t. T h is id e a a lso is n o t lo g ic a l. T h e verb in th e s e c o n d e x a m p le s h o u ld b e th e sim p le p a st improved. T h e p ast p e r fe c t ( had + past p a r tic ip le ) refers to a p e r io d o f tim e that started in the past and ended in the past, before something else happened in the past. Sue had lived in Los A ngeles for ten years when she moved to San D iego. T h is s e n t e n c e m e a n s th a t S u e liv e d in L os A n g e le s fo r ten years in th e p ast b e fo r e sh e m o v e d to San D ie g o in th e past. S h e n o lo n g e r lives in L os A n g e le s. B e c a u se th e p ast p e r fe c t b e g in s in th e past a n d e n d s in th e past, it is g e n e r a lly n o t c o r­ rect in th e sa m e s e n t e n c e w ith th e p r e se n t te n se . Tom had finished the exam when the teacher collects* the papers. T h is s e n t e n c e in d ic a te s th a t Tom finished the exam (in th e past) a n d th a t a c tio n e n d e d when the teacher collects the papers (in th e p r e se n t). T h is is n o t lo g ic a l, so th e s e n t e n c e is n o t c o r ­ rect. T o m fin is h e d th e e x a m (in th e p a st), a n d th e a c tio n o f fin is h in g th e e x a m e n d e d w h en th e te a c h e r c o lle c te d th e p a p ers. T h e r e fo r e , th e s e c o n d verb in th is e x a m p le s h o u ld b e in th e p ast te n se , collected. T h e fo llo w in g c h a r t o u tlin e s th e u ses o f th e p r e se n t p e r fe c t a n d th e p a st p e r fec t: USING (HAVE + PAST PARTICIPLE) AND (HAD + PAST PARTICIPLE) TENSE

*

present perfect ' ' ■ past perfect

FORM

MEANING

USE

hove + past participle

past up to now

not with a past tense**

had + past participle

before past up to past

not with a present tense

**Except when the time expression since is part of the sentence (see Skill 35).

203

STRUCTURE AND WRITTEN EXPRESSION

E X E R C ISE 34: E ach o f th e fo llo w in g s e n te n c e s c o n ta in s had o r have. U n d e r lin e th e verb s tw ice a n d d e c id e i f th e m e a n in g s are lo g ica l. T h e n in d ic a te i f th e s e n te n c e s are c o r r e c t (C ) o r in c o r r e c t (I). C

i

I have always liked the designs that are on the cover.

I

2.

Because her proposal had been rejected, she is depressed.

_____

3 . T h e students have registered for classes before the sem ester started.

_____

4. W hen she had purchased the car, she contacted the insurance agent.

_____

5.

H e said that he had finished the typing w hen you finish the reports.

_____

6.

She has enjoyed herself every time that she has gone to the zoo.

_____

7.

H e drove to the post office after he had finished preparing the package.

_____

8.

After the votes were counted, it had been determ ined that Steve was the winner.

_____

9.

Last night all the waiters and waitresses have worked overtime.

_____

10.

S k i l l 35:

H e had fastened his seat belt before the airplane took off.

USE T H E CORRECT TENSE W IT H TIM E EXPRESSIONS

O fte n in s e n te n c e s in th e W ritten E x p ressio n se c tio n o f th e T O E F L te st th e r e is a tim e e x ­ p r e ssio n th at clearly in d ic a te s w h at verb te n se is n e e d e d in th e se n te n c e . We m oved to New York in 1970. We had left there by 1980. We have lived in San Francisco since 1982. In th e first e x a m p le , th e tim e e x p r e ssio n in 1 9 7 0 in d ic a te s th a t th e verb s h o u ld b e in th e s im p le p ast (m oved). In th e s e c o n d e x a m p le , th e rim e e x p r e s sio n by 1 9 8 0 in d ic a te s th at th e v e rb s h o u ld b e in th e past p e r fe c t (h a d left). In th e th ird e x a m p le , th e tim e e x p r e ssio n since 1982 in d ic a te s th a t th e verb sh o u ld be in th e p r e se n t p e r f e c t (h a v e lived). S o m e a d d itio n a l tim e e x p r e ssio n s that clearly in d ic a te th e c o r r e c t te n se are ago, last, a n d lately. She got a job two years ago. She started working last week. She has worked very hard lately. In th e first e x a m p le , th e tim e e x p r e s sio n two years ago in d ic a te s th a t th e verb s h o u ld b e in th e sim p le p a st (got). In th e se c o n d e x a m p le , th e tim e e x p r e s sio n last week in d ic a te s th at th e verb sh o u ld b e in th e sim p le past (started). In th e third e x a m p le , th e tim e e x p r e ssio n lately in d ic a te s th a t th e verb sh o u ld b e in th e p r e se n t p e r fe c t (h a s w orked).

WRITTEN EXPRESSION

T h e fo llo w in g c h a r t lists tim e e x p r e s sio n s th a t in d ic a te th e c o r r e c t v e r b ten se: USING CORRECT TENSES WITH TIME EXPRESSIONS PAST PERFECT

by (1920) :

'

'

.r

. ■"

. ..

SIMPLE PAST

PRESENT PERFECT

(two years) ago

since (1920)

last (year) - ... .

-

-

■•••.........;•

V

.••••

.•

in (1920)

EXERCISE 35: E ach o f th e fo llo w in g s e n te n c e s c o n ta in s a tim e e x p r e s s io n . C ircle th e tim e e x p r e s s io n s a n d u n d e r lin e th e verb s tw ice. T h e n in d ic a te i f th e s e n te n c e s are c o r ­ rect (C ) o r in c o r r e c t (I ). C

1.

T he p h one rang incessantly (last night.)

__ I__

2.

They have finished contacting everyone(by 4:00 yesterday)

_____

3.

T h e Pilgrims have arrived in the New World in 1612.

_____

4.

Since the new law was passed, it has been difficult to estim ate taxes.

--------

5.

T he cashier put the m oney into the account two hours ago.

_____

6.

All the votes have been counted last week.

_____

7.

T h e students are writing many com positions lately.

----- --

8.

T h e Senate votes on the law to ban cigarette sm oking in public in 1990.

_____

9.

By the time the m ain course was served, all the guests had arrived and b een seated.

_____

10.

S k il l

36:

I had not d on e m uch m ore work since I talked to you on Wednesday.

USE T H E C O R R EC T TENSE W IT H W ILL A N D W O U LD

C ertain c o m b in a tio n s o f v erb s are very c o m m o n in E n g lish . O n e is th e c o m b in a tio n o f th e sim p le p r e se n t a n d will. I know that they will arrive soon. It is certain that he will graduate. A n o th e r c o m b in a tio n th a t is q u ite c o m m o n is th e c o m b in a tio n o f th e s im p le p ast a n d would. I knew that h e would arrive. It was certain that h e would graduate. It is im p o r ta n t to stress th a t in th e c o m b in a tio n d isc u s se d h e r e , th e p r e s e n t s h o u ld b e u sed w ith w ill a n d th e p ast s h o u ld b e u s e d w ith w ould; th e y g e n e r a lly s h o u ld n o t b e m ix e d .

20S

206

STRUCTURE AND WRITTEN EXPRESSION

T h e c o m m o n e rr o rs th a t m u st g e n e ra lly b e a v o id e d a re th e c o m b in a tio n o f th e p ast w ith

will a n d th e c o m b in a tio n o f th e p r e se n t w ith uiould. I know that he would* arrive soon. It was certain that h e m il* graduate. In th e first e x a m p le , th e p r e se n t, know, is illo g ic a l w ith w ould. It can b e c o r r e c te d in tw o d iffe r e n t ways. I knew that he would arrive soon. I know that he will arrive soon. In th e s e c o n d e x a m p le , th e p ast, was, is illo g ic a l w ith wilL It c a n also b e c o r r e c te d in two d iffe r e n t ways. It was certain that he would graduate. It is certain that he will graduate. T h e fo llo w in g c h a r t o u d in e s th e u se o f te n se s w ith w ill a n d would: USING CORRECT TENSES WITH W1U.AND W OULD USE

MEANING

VERB

will

after the present

do not use with past

would

after the past

do not use with present

NOTE: There is a different modal would that Is used to make polite requests.This type of would is often used with the present tense. 1 would like to know if you hare a pencil that 1 could borrow.

E X E R C ISE 36: E ach o f th e fo llo w in g s e n te n c e s c o n ta in s w ill o r w ould. U n d e r lin e th e verb s tw ice a n d d e c id e i f th e m e a n in g s are lo g ic a l. T h e n in d ic a te i f th e s e n te n c e s are co r ­ r ec t (C ) o r in c o r r e c t (I). I

1. H e knew that he will be able to pass the exam.

C

2. I think that I will leave tomorrow.

_____

3. Paula did not say when she will finish the project.

_____

4. Jake doubts that h e would have time to finish the project.

_____

5. I know that I will go if I can afford i t

_____

6.

T h e police officer indicated that he would write a ticket if he has the time.

_____

7.

Students will often study in the library before they go to classes or before they go hom e.

_____

8. H e told m e that he thought h e will get the jo b in spite o f his lack

_____

9. T h e executive vice president em phasizes at the conferences that the board would not change its position.

_____

10.

o f education.

Students will register for classes according to w ho has the highest num ber o f units.

WRITTEN EXPRESSION

EXERCISE (Skills 3 3 -3 6 ): U n d e r lin e th e verb s tw ice in e a c h o f th e fo llo w in g s e n te n c e s . T h e n in d ic a te i f th e s e n te n c e s are c o r r e c t (C ) o r in c o r r e c t (I). --------

1.

W hen he receives the m oney from the insurance com pany two days ago, he had already rebuilt the house.

--------.

2.

T h e position on the city council will be filled n ext week w hen the electorate votes.

--------

3.

T h e dentist fills the cavities every tim e the X-rays show that it was necessary.

--------

4.

W hen the bell rang, the students have left the class.

--------5.

T h e space shuttle would be launched next m onth if the weather is good.

--------

6.

T h e special delivery package has arrived by n oon yesterday.

--------

7.

It is probable that the students who were tested yesterday were quite successful.

--------

8.

After forty-five students had signed up for the class, the class was closed.

--------

9.

T h e parking at the arena was inadequate for the trem endous n u m ber o f drivers who will want to park there.

--------

10.

They have not returned to Rhode Island since they left in 1970.

TOEFL EXERCISE (Skills 3 3 -3 6 ): C h o o se th e lette r o f th e u n d e r lin e d w o r d o r g r o u p o f w ord s th a t is n o t c o r r e c t.

-------- 1■ In several of his paintings, Edward Hicks depicted the Quaker farm in Pennsylvania A where he spends his youth. B C ET -------- 2.

Florida has become the twenty-seventh state in the United States on March 3, 1845. A B C D

-------- 3.

After last week's meeting, the advertising department quickly realized that the A B product will need a new slogan. "C D~

-------- 4.

John F. Kennedy's grandfather, John F. Fitzgerald, serves two terms as the mayor of A B C Boston in the beginning of the twentieth century. D

--------5.

Fort Ticonderoga, a strategically important fortification during the Revolution, had “ A~ since been reconstructed and turned into a museum. B C D

-------- 6.

In making their calculations. Institute researchers assume that the least costly form A B o f energy would be used. C ~D ~

207

STRUCTURE AND WRITTEN EXPRESSION

7.

A twentv-one-vear-old man became the second casualty yesterday when he loses A— B C control of his truck.

8.

Most people had written with quill pens until pens with metal points becom e popular “A B C D in the middle of the nineteenth century.

9.

In a determined drive to pare its debt, Time Warner is launching a stock offering A B C plan that would potentially raise $2.8 billion. D

_10

The formula used in the study calls for either peroxide or metaldehyde, but ~JT B C metaldehyde was not always available. TT

TOEFL REMEW EXERCISE (Skills 1-36): C h o o se th e lette r o f th e word o r g r o u p o f w ord s th a t b e s t c o m p le te s th e s e n te n c e . 1 . _____ in the United States declined from twenty million in 1910 to nine million in the 1970s. (A) (B) (C) (D)

For a number of horses The number of horses When the number of horses That the number of horses

2. Because of his reservations about the issu e ,_____ refused to vote for it. (A) (B) (C) (D)

who and which the senator the senator'

3. Bats avoid running into objects b y . high-frequency sounds and listening for echoes. (A) (B) (C) (D)

the emission emitted emitting they emit

4. It has been estimated that if we intend to stay above the starvation level,--------the food supply. (A) (B) (C) (D)

so we will have to double and it m ust double which it must be doubled w e must double

Choose the letter of the underlined word or group of words that is not correct. 5

To determine an object’s force, the mass and speed of the object must be measure. -------- A-------B -CD

6.

The most common time for tornados to occur are in the afternoon or evening on a A B C hot, humid spring day. D

7.

Automakers Nissan and Ford and several aerospace research facilities in Great A Britain are working lately to apply active noise cancellation to entire cars and planes. ------- B------C D

WRITTEN EXPRESSION

When a country in an early stage of development, investments in fixed capital A B ~C~ are vital.

_ 9.

John Chapman became famous in American folklore as "Johnny Appleseed” after he A plants apple trees throughout the northeastern part of the United States. B C D

_10.

Inasmuch he kept mostly to himself, the author of The Treasure o f the Sierra Madre A B C was known as “the mysterious B. Treuen.” TT

PROBLEMS W IT H PASSIVE VERBS___________________________ S e n te n c e s in w h ic h th e err o r is an in c o r r e c t passive a re c o m m o n in th e W ritten E x p r e s­ sio n se c tio n o f th e T O E F L test. You th e r e fo r e n e e d to b e a b le to r e c o g n iz e th e c o r r e c t form o f th e passive a n d to b e a b le to d e te r m in e w h en a passive verb r a th e r th a n an active verb is n e e d e d in a s e n te n c e . T h e d iffe r e n c e b e tw e e n a n active a n d a passive verb is th a t th e su b je c t in a n active s e n ­ te n c e does th e a c tio n o f th e verb , a n d th e su b je ct in a passive s e n te n c e receives th e a c tio n o f th e verb. T o c o n v e r t a s e n te n c e fr o m active to passive, two c h a n g e s m u s t b e m a d e . (1 ) T h e su b ject o f th e active s e n t e n c e b e c o m e s th e o b je c t o f th e passive s e n t e n c e , w h ile th e o b je c t o f th e active s e n te n c e b e c o m e s th e su b je ct o f th e passive s e n te n c e . (2 ) T h e verb in th e passive s e n te n c e is fo r m e d by p u ttin g th e h e lp in g v e rb bein th e sa m e fo r m as th e v e fb in th e active s e n te n c e a n d th e n a d d in g th e p a st p a r tic ip le o f th is verb.

SUBJECT

OBJECT

T h e first e x a m p le is an active s e n te n c e . T o c o n v e r t th is active s e n te n c e to a p assive s e n ­ te n c e , y o u m u st first m ak e th e su b je ct o f th e active s e n te n c e , Margaret, th e o b je c t o f th e passive s e n te n c e w ith by. T h e o b je c t o f th e active s e n te n c e , letter, b e c o m e s t h e su b je c t o f th e passive s e n te n c e . N e x t, th e passive verb c a n b e fo r m e d . B e c a u se wrote is in th e p a st te n se in th e active s e n te n c e , th e p a st te n se o f be (was) is u s e d in th e p assive s e n te n c e . T h e n th e verb xvrote in th e active s e n te n c e is c h a n g e d to th e p ast p a r tic ip le written in th e passive s e n te n c e . It s h o u ld b e n o te d th a t in a passive s e n te n c e , by + object d o e s n o t n e e d to b e in c lu d e d to h a v e a c o m p le te s e n te n c e . T h e fo llo w in g are b o th e x a m p le s o f c o r r e c t s e n te n c e s . T he letter was written yesterday by Margaret. T he letter was written yesterday. N o tic e th a t th e s e passive s e n te n c e s a r e c o r r e c t i f by Margaret is in c lu d e d (as in th e first e x ­ a m p le ) o r i f by Margaret is o m itte d (as in th e s e c o n d e x a m p le ) .

STRUCTUREANDWRITTEN EXPRESSION

NOTE: Exercises to practice active and passive forms can be found in Appendix G at the back of the text. You may want to complete these exercises before you begin Skill 37.

S k i l l 37:

USE T H E CORRECT FORM OF T H E PASSIVE

O n e way th a t th e passive can b e te sted o n th e T O E F L te st is sim p ly w ith an in c o r r e c t fo rm o f th e p assive. T h e fo llo w in g are e x a m p le s o f passive e rro rs th a t m ig h t a p p e a r o n th e T O E F L test: T he portrait was p a in tin g by a fam ous artist. The project will fin is h e d by Tim. In th e first e x a m p le , th e passive is fo r m e d in c o rr ec tly b e c a u se th e p ast p a r tic ip le p a in ted sh o u ld b e u s e d rath er than th e p r e se n t p a rticip le p a in tin g . In th e s e c o n d e x a m p le , th e verb be h as n o t b e e n in c lu d e d , a n d so m e fo rm o f be is n e c essa ry for a passive verb . T h e verb in th e s e c o n d s e n te n c e sh o u ld b e w ill be fin ish ed . T h e fo llo w in g c h a r t o u tlin e s th e way to fo r m th e p assive correcd y:

THE FORM OFTHE PASSIVE BE

+

past participic (BY + object)

E X E R C ISE 37: E ach o f th e fo llo w in g s e n te n c e s has a passive m e a n in g . U n d e r lin e tw ice th e verb s th a t sh o u ld b e passive. T h e n in d ic a te i f th e s e n te n c e s are c o r r e c t (C ) o r in corr ect (I). 1

1.

T h e boy had never be stung by a bee.

C

2.

T h e suits were hung in the closet when they were returned from the cleaners.

3.

M oney is len ding by the credit union to those w ho want to buy hom es.

4.

T h e record had been chose by dancers near the jukebox.

fi

T h e topic for your research paper should have been approved by your advisor. T hat song has been playing over and over again by Steve.

_____ 6.

. ..

7.

T heir utility bills have been increased again and again.

8.

T he patients who are too sick to sit up are being assisted by the orderlies.

9.

T he offices were thoroughly clean last evening by the n igh t crew.

10.

T h e car that was struck in the intersection yesterday is b ein g repaired today.

WRITTEN EXPRESSION

R E C O G N IZ E A C T IV E A N D PASSIVE M EA N IN G S

S k i l l 38:

W h e n t h e r e is n o o b je c t (w it h o r w it h o u t by) a fte r a v e rb , y o u m u s t l o o k a t t h e m e a n in g o f th e s e n te n c e to d e t e r m in e i f t h e v e r b s h o u ld b e a c tiv e o r passive. S e n t e n c e s w it h a n in ­ c o r r e c t p a ssive v e r b a n d n o by + object to te ll y o u th a t th e v e r b s h o u ld b e p a ssive a r e th e m o st d if f ic u lt p a ssive e r r o r s to re c o g n iz e o n th e T O E F L test. S t u d y th e e x a m p le s :

We m ailed the package at the post office. T h e letter was m ailed by us today before noon. T h e letter was mailed today before noon. T h e letter m ailed* today before noon. T h e f ir s t th re e e x a m p le s a b o v e a r e c o r r e c t . T h e firs t e x a m p le h a s th e a c tiv e v e r b m ailed u sed w ith th e o b je c t package, t h e s e c o n d e x a m p le h as th e passive v e r b w as m ailed u s e d w ith

by usr, t h e t h ir d s e n t e n c e h a s th e p a ssive v e r b was m ailed u s e d w it h o u t a n o b je c t. T h e f o u r th e x a m p le is th e ty p e o f passive e r r o r th a t a p p e a rs m o s t o f te n o n th e T O E F L test. T h is ty p e o f s e n t e n c e h as th e fo llo w in g c h a ra c te ris tic s : (1 ) a n i n c o r r e c t p a s­ sive v e r b th a t lo o k s lik e a c o r r e c t a c tiv e v e rb , a n d (2 ) n o by + object to te ll y o u t h a t a p a ssive is n e e d e d . T o c o r r e c t t h e f o u r t h e x a m p le , th e a c tiv e v e rb n e e d s to b e c h a n g e d to th e p as­ sive was m ailed. T o d e t e r m in e th a t s u c h a s e n t e n c e is in c o r r e c t , y o u m u s t s tu d y t h e m e a n in g o f th e su b je ct a n d th e v e rb . Y o u m u s t a sk y o u r s e lf i f th e s u b je c t does t h e a c t io n o f th e v e r b (s o a n a ctive v e r b is n e e d e d ) o r i f t h e s u b je c t receives th e a c t io n o f th e v e r b (s o a p a ssiv e v e rb is n e e d e d ). I n th e i n c o r r e c t e x a m p le , y o u s h o u ld stu d y th e m e a n in g o f th e s u b je c t a n d v e rb ,

the letter m ailed. Y o u s h o u ld ask y o u r s e lf i f a letter m ails its e lf (t h e le t t e r does t h e a c t i o n ) o r i f s o m e o n e m ails a letter ( t h e le t t e r receives th e a c t io n o f b e in g m a ile d ). S i n c e a le t t e r d o e s n o t m a il itse lf, t h e p a ssive is r e q u ir e d in th is s e n te n c e . T h e f o llo w in g c h a r t o u d in e s th e d if f e r e n c e in m e a n in g b e tw e e n a c t iv e a n d p a ssive verbs:

ACTIVE AND PASSIVE MEANINGS

ACTIVE

The subject does the action of the verb.

PASSIVE

The subject receives the action of the verb.

E X E R C I S E 38: E a c h o f t h e f o llo w in g s e n te n c e s c o n t a in s a t le a st o n e a c t iv e v e r b ; h o w e v e r, so m e o f th e v e rb s s h o u ld b e p a ssive. U n d e r l in e th e v e rb s tw ice. T h e n in d ic a t e i f t h e se n ­ te n ce s a r e c o r r e c t ( C ) o r i n c o r r e c t ( I ) .

— !—

1. T h e car parked in a no-parking zone.

—_

2. T he physics exam began just a few m inutes ago.

--------

3. Everything to organize the picnic has already don e.

--------

4. T he police investigated him because o f his unusual actions. 5.

T he package containing the necessary sam ples has just sent.

STRUCTUREANDWRITTEN EXPRESSION

6.

The vacation to Europe will plan carefully before the scheduled departure date.

7.

The coffee turned bitter w hen it left on the stove for so long.

8.

The soccer gam e won in the closing m inutes.

9.

The clothes m ade to rival the latest fashions o f the season.

10.

W hen the roads are icy, the buses do not drive.

E X E R C I S E (S k ills 3 7 - 3 8 ): U n d e r l in e th e v e rb s tw ic e in th e f o llo w in g s e n te n c e s . T h e n in ­ d ic a te i f th e s e n te n c e s a re c o r r e c t ( C ) o r i n c o r r e c t ( I ) .

_____

1.

After the old radiator had be replaced, the travelers continued their cross-country trip.

_____

2.

During the lightning storm, he struck in the head by a falling tree.

_____

3.

W hile I am on vacation, the pets should be feeds every m orning and evening.

_____

4.

A book being written now by a team o f writers will be published in the fall.

_____

5.

I found out that the real estate agent had already been leased the condom inium .

_____

6.

The hou se that Mrs. Martin has always wanted to buy has just placed on the market.

_____

7.

The foundation should have been finishing by the construction workers before they left the construction site.

_____

8.

We m ust leave that m oney in the checking account because the bills pay o n the first o f the m onth.

_____

9.

The horses can ’t be taken out now because they have been rode for the past few hours.

_____

10.

It is being announced by a presidential aide that a lawyer from Virginia has b een nam ed attorney general.

T O E F L E X E R C I S E (S k ills 3 7 - 3 8 ): C h o o s e th e le t t e r o f th e w o r d o r g r o u p o f w o rd s th a t b e s t c o m p le te s th e s e n te n c e .

2.

_discussed by the board of directors when it was proposed again by the supervisors.

3. The X-ray tre a tm e n ts---up to the th at he w as dism issed from the hospital, (A) gavedaiIy

(A) The problem had already (B) The problem is already (C) The problem had already been (D) The problem has already

(B) were given daily (C) basically have given (D) daily had been given

M uch of the carnage of elephants, giraffes, and big c a ts _____ uncaring hunters. (A) (B) (C) (D)

m ust m ust m ust m ust

com m it by be com m itted have com m itted have been com m itted by

WRITTEN EXPRESSION

C h o o s e th e le t t e r o f t h e u n d e r lin e d w o r d o r g r o u p o f w o rd s t h a t is n o t c o r r e c t .

4.

Particular issues that concern teenagers were covering in the half-hour program. A B C D

5. Electrical impulses may also picked up by the optic nerve. A B C D 6. Workers training for a specific job have a strong possibility of being replace by a A B C D machine. 7.

On June 30, 1992, international timekeepers in Paris were added an extra second to ~A” B C the da D- ^

8.

The report could not be turned in on time because all the needed work lost. A B C D

. 9.

In English these questions have be formed by changing the word order of a A B C statement, whereas in som e languages the word order remains the same. D

.10.

He was not able to define the process by which the body had protected by the A B C D immunologic system.

T O E F L R E V IE W

E X E R C IS E

(S k ills 1-3 8): C h o o s e th e le t t e r o f t h e w o r d o r g r o u p o f

w o rd s th a t b e s t c o m p le te s t h e s e n te n c e .

1. _____ Big Dipper, a seven-star constellation in the shape of a cup, is part of Ursa Major. (A) (B) (C) (D)

The It is the With the That the

2. The Military Academy at West P oint-------on the west bank o f the Hudson River, north of New York City. (A) (B) (C) (D)

located is located which is located whose location is

3.

impressive chapter in the book was the chapter on Stuart’s scientific theories. (A) (B) (C) (D)

It was the most The most Most Most of the

214

STRUCTURE AND WRITTEN EXPRESSION

C h o o s e th e le t t e r o f th e u n d e r lin e d w o r d o r g r o u p o f w o rd s th a t is n o t c o r r e c t.

--------4.

The first fish have appeared on the earth approximately 500 million years ago. A B C D

-------- 5.

--------6.

Only rarely sound waves are of a single frequency encountered in practice. A B C D Cameos can be carved not only from onyx and sardonyx or from agate. A B ~C~ TF

-------- 7.

Although most of the wild horses in the western range have already been rounded A B up,

t h e r :O St r e m o t e t h e a r e a , t h e g r e a t e r t h e p o s s i b i l i t y t h a t w i l d h o r s e s c a n s t i l l b e

C

D

found.

8.

During this period, $206 was spend annually on food by families in the lower third ~~A B£ D income bracket.

_ 9.

The dangers of noise are, unfortunately, not as clear-cut than are those from A ~B C most other health hazards. D

_10.

In a recent survey of Americans, more than 75 percent expressed the view that the A B government it should take a more active role in health care.

PROBLEMS W IT H N O U N S . T h e sa m e types o f p ro b le m s w ith n o u n s a p p e a r o fte n in th e W r it t e n E x p re s s io n s e c tio n o f th e T O E F L test. Y o u s h o u ld b e f a m ilia r w ith th e s e p r o b le m s so th a t y o u w ill re c o g n iz e th e m easily. Y o u s h o u ld b e a b le to d o th e f o llo w in g : (1 ) use th e c o r r e c t s in g u la r o r p lu r a l n o u n , ( 2 ) d is tin g u is h c o u n ta b le a n d u n c o u n t a b le n o u n s , (3 ) re c o g n iz e i r r e g u la r s in g u la r a n d p lu r a l n o u n s , a n d (4 ) d is tin g u is h th e p e rs o n fro m th e th in g .

S k ill

39:

USE T H E CORRECT S IN G U LA R O R PLURAL N O U N

A p r o b le m th a t is c o m m o n in th e W r it t e n E x p re s s io n s e c tio n o f t h e T O E F L test is a sin ­ g u la r n o u n u s e d w h e r e 'a p lu r a l n o u n is n e e d e d , o r a p lu r a l n o u n u se d w h e r e a s in g u la r n o u n is n e e d e d .

-

On the table there were many dish*. The lab assistant finished every tots*.

I n th e f ir s t e x a m p le , m any in d ic a te s th a t th e p lu r a l dishes is n e e d e d . I n t h e s e c o n d e x a m ­ p le , every in d ic a t e s th a t t h e s in g u la r lest is n e e d e d .

WRITTEN EXPRESSION

I n t h e W r i t t e n E x p r e s s io n s e c tio n o f t h e T O E F L test, y o u s h o u ld w a tc h v e r y c a r e f u lly f o r k e y w o rd s , s u c h as each, every, a, one, a n d single, th a t i n d ic a t e t h a t a n o u n s h o u ld b e sin ­ g u la r. Y o u s h o u ld a lso w a tc h c a r e f u lly f o r s u c h k e y w o rd s a s m any, several, both, vario u s, a n d

two ( o r a n y o t h e r n u m b e r e x c e p t one) th a t in d ic a t e th a t a n o u n s h o u ld b e p lu r a l. T h e f o llo w in g c h a r t lists th e k e y w o rd s th a t in d ic a t e to y o u w h e t h e r a n o u n s h o u ld b e s in g u la r o r p lu r a l:

KEYWORDS FOR SINGULAR AND PLURAL NOUNS For Singular Nouns

each

every

single

one

a

For Plural Nouns

both

two

many

several

various

E X E R C I S E 39: E a c h o f t h e f o llo w in g s e n te n c e s c o n t a in s a t lea st o n e k e y w o r d to te ll y o u i f a n o u n s h o u ld b e s in g u la r o r p lu r a l. C ir c le th e k e y w o rd s . D r a w a rro w s to t h e n o u n s th e y d e s c rib e . T h e n in d ic a t e i f th e s e n te n c e s a re c o r r e c t ( C ) o r in c o r r e c t ( I ) .

I

1.

i y i y T h e autom otive shop stocked (man)) part for the (various) types o f H ondas.

2. (Every) receipt must be rem oved from the cashier’s drawer and tallied. 3.

T h e salesclerk dem onstrated various additional way that the m achine could be used.

4.

T h e woman fou n d it difficult to believe that both o f the piece o f jew elry had disappeared.

5.

T he unhappy man becam e m ore and m ore discouraged with each passing days.

6.

An exten d ed cruise w ould be a nice way to spend a vacation on e days.

7.

T he m anager was surprised that not a single worker was available on Tuesday.

8.

T h e h ou sekeep er cleaned the room and took two o f the occupant's dress to the laundry.

9.

W hen the first bill was defeated, the Senate im m ediately began work on a different bills.

,

10. T here were several b oxes in the cupboard, and each box contained a dozen glasses.

S

k il l

40:

D IS T IN G U IS H C O U N T A B L E A N D U N C O U N T A B L E NOUNS

I n E n g lis h n o u n s a re c la s s ifie d as c o u n ta b le o r u n c o u n t a b le . F o r c e r t a in q u e s tio n s o n th e T O E F L test, it is n e c e s s a r y to d is tin g u is h c o u n ta b le a n d u n c o u n t a b le n o u n s in o r d e r to u se th e c o r r e c t m o d if ie r s w ith th e m .

STRUCTURE AND WRITTEN EXPRESSION

As th e n a m e im p lie s, c o u n ta b le n o u n s are n o u n s th at c a n b e c o u n te d . C o u n ta b le n o u n s can c o m e in q u a n titie s o f o n e , o r two, o r a h u n d r e d , e tc . T h e n o u n book is c o u n t­ ab le b e c a u se y o u can have o n e b o o k o r several book s. U n c o u n ta b le n o u n s , o n th e o th e r h a n d , are n o u n s th a t c a n n o t b e c o u n te d b e c a u se th ey c o m e in so m e in d e te r m in a te q u an tity o r m ass. A n o u n su c h as m ilk o r happiness ca n ­ n o t b e c o u n te d ; you c a n n o t have o n e m ilk o r tw o m ilk s, a n d y o u c a n n o t fin d o n e h a p p i­ n e ss o r two h a p p in e ss e s . U n c o u n ta b le n o u n s are o fte n liq u id item s, su c h as water, oil, o r sham poo. U n c o u n ta b le n o u n s can also refer to abstract id ea s, su c h as security, frien d sh ip , o r hope. It is im p o r ta n t fo r you to r e c o g n iz e th e d iffe r e n c e b e tw e e n c o u n ta b le a n d u n c o u n t­ a b le n o u n s w h e n y o u c o m e across su ch key w ord s as m u ch a n d many. H e has seen much* foreign films. H e d id n ’t have many* f u n at the movies. In th e first e x a m p le , m uch is in c o r r e c t b e c a u se film s is c o u n ta b le . T h is s e n te n c e s h o u ld say m any foreig n film s . In th e s e c o n d e x a m p le , m any is in c o r r e c t b e c a u se f u n is u n c o u n ta b le . T h is s e n t e n c e sh o u ld say m uch fu n . T h e fo llo w in g c h a r t lists th e key w ord s th a t in d ic a te to y o u w h e th e r a n o u n sh o u ld b e c o u n ta b le o r u n c o u n ta b le :

KEYWORDS FOR COUNTABLE AND UNCOUNTABLE NOUNS For Countable Nouns

many

number

few

fewer

For Uncountable Nouns

much

amount

little

less

E X E R C ISE 40: E ach o f th e fo llo w in g s e n te n c e s c o n ta in s at lea st o n e k ey w ord to tell y o u i f a n o u n sh o u ld b e c o u n ta b le o r u n c o u n ta b le . C ircle th e key w ords. D raw arrow s to th e n o u n s th e y d e sc r ib e . T h e n in d ic a te i f th e s e n te n c e s are c o r r e c t (C ) o r in c o r r e c t (I). r — x—1.

. . H e received (little) notice that the bill would have to be paid in full. 2.

T he police had (few) opportunities to catch the thief who had com m itted a large (am ount) o f crimes.

--------

3.

You will have fewer problem s with your incom e taxes if you get professional help.

--------

4.

After the strike, the company dismissed many em ployees.

--------

5.

Because the bottom corner o f the pocket was torn, m uch coins fell out.

--------

6.

Since he bought the new adapter, he has had less trouble with the m achine.

--------7.

T here are much new items to purchase before leaving, and there is such a short am ount o f time.

WRITTEN EXPRESSION

8.

The less time you take on the assignm ent, the less pages you will com plete.

9.

A few soldiers w ho had been in heavy com bat were brought back for a little rest.

10. It is better to go sh opp in g in the late evening because there are less p eo p le in the market, and you can accom plish a num ber o f tasks in a short period o f time.

S k il l

4 1:

R E C O G N IZ E IRREGULAR PLURALS OF N O U N S

M any n o u n s in E n g lish h ave irreg u la r p lu rals, an d th e se irreg u la r fo r m s c a n c a u se c o n fu ­ sio n in th e W ritten E x p r e ssio n s e c tio n o f th e T O E F L test. T h e irr eg u la r fo r m s th a t are th e m o st p r o b le m a tic are p lu ral fo r m s th a t d o n o t e n d in 5. D ifferent criteria was* used to evaluate the performers. In this e x a m p le th e plu ral n o u n criteria lo o k s sin g u la r b e c a u se it d o e s n o t e n d in s; you m ig h t in c o r r e c tly a ssu m e th a t it is sin g u la r b e c a u se th e r e is n o fin al 5. H ow ever, criteria is a plu ral n o u n , so th e sin g u la r verb w as used is in c o r r e c t. T h e verb sh o u ld b e th e p lu ra l fo r m were used. T h e fo llo w in g c h a r t lists th e irr eg u la r plu rals th at y o u sh o u ld b e c o m e fa m ilia r with:

IRREGULAR PLURALS Vowel change

man / men woman / women

foot 1feet tooth 1teeth

Add -en

child / children

ox / oxen

Same as singular

deer / deer fish / fish

salmon / salmon sheep / sheep

trout / trout

-is ---- »--ES

analysis / analyses axis / axes crisis / crises

diagnosis / diagnoses hypothesis / hypotheses parenthesis 1parentheses

synthesis / syntheses thesis / theses

Ends in -A

bacterium / bacteria curriculum 1curricula

datum / data phenomenon / phenomena

criterion / criteria

-US —► -/

alumnus / alumni bacillus / bacilli cactus 1cacti

fungus / fungi nucleus / nuclei radius / radii

stimulus / stimuli syllabus / syllabi

goose / geese mouse / mice

NOTE: Additional exercises to practice these irregular plurals of nouns appear in Appendix H at the back of the text. You may want to complete these exercises before you begin Exercise 4 1.

217

18

STRUCTURE AND WRITTEN EXPRESSION

E X E R C ISE 41: Each o f th e fo llo w in g s e n te n c e s c o n ta in s at lea st o n e n o u n w ith an ir r e g u ­ lar plu ral. C ircle th e n o u n s w ith irreg u la r plurals. T h e n in d ic a te if th e s e n te n c e s are c o r­ r ec t (C ) o r in c o r r e c t (1). 1. (Parentheses)is n eed ed around that expression. <

2.

He wants to go on a fishing trip this weekend because he has heard that the (fish) are running.

3.

The syllabi for the courses is included in the packet o f materials.

4.

T he diagnosis that h e heard today were not very positive.

5.

T he crisis is not going to be resolved until som e o f the pressure is relieved.

6.

All o f the alumni are attending the reception at the president’s house.

7.

A flock o f geese were seen heading south for the winter.

8.

T he teeth in the back o f his m outh needs to be capped.

9.

The fungi has spread throughout the garden.

10. T he sheepdog is chasing after the sheep which are heading over the hill.

S k i l l 42:

D IS T IN G U IS H TH E PERSON FROM T H E T H IN G

N o u n s in E n g lish can r efe r to p e r so n s or th in g s. S o m e tim e s in th e W ritten E x p r e ssio n s e c tio n o f th e T O E F L test th e p e r so n is u s e d in p la c e o f th e th in g , o r th e th in g is u s e d in p la c e o f th e p e r so n . Ralph Nader is an authorization* in the field o f consum er affairs. T here are many jo b opportunities in accountant*. In th e first e x a m p le , authorization is in c o r r e c t b e c a u se authorization is a th in g an d R alph N a d e r is a p e r so n . T h e p e r so n authority sh o u ld b e u s e d in th is s e n t e n c e . In th e s e c o n d e x ­ a m p le , acco u n ta n t is in c o r r e c t b e c a u se accountant is a p e r so n a n d th e fie ld in w h ic h an ac­ c o u n ta n t w orks is accounting. T h e th in g accounting sh o u ld b e u se d in th is s e n te n c e . T h e fo llo w in g ch a rt o u tlin e s w h a t you sh o u ld r e m e m b e r a b o u t th e p e r s o n o r thing:

PERSON OR THING 1. It is common to confuse a person with s thing in the Written Expression section of the TOEFL test. 2. This type of question generally appears near the end of the Written Expression section.

♦ WRITTEN EXPRESSION

E X E R C ISE 42: S o m e o f th e fo llo w in g s e n te n c e s c o n ta in in c o rr ec tly u s e d persons o r things. C ircle th e in c o r r e c tly u s e d w ord s. T h e n in d ic a te if th e s e n t e n c e s are c o r r e c t (C ) o r in c o r ­ rect (I ). 1.

In the evening he relaxes in front o f the fire and writes lo n g (p o e ts)

2.

Service in the restaurant was slow because o n e cook had called in sick.

3.

T h e sculpture worked from sunrise until sunset o n his new project.

4.

She has received several awards for her research in engineer.

5.

T he econom ist's radical views were printed in a colum n in the Sunday newspaper.

6.

You must have remarkable looks to work as a m odel for Vogue.

7.

H e had several critics to offer about the new play.

8.

T h e gardener worked feverishly after the frost to save as many plants as possible.

9.

T h e com pany hired a statistic to prepare marketing studies for the new product.

10.

T h e fam ous acting has appeared in m ore than fifty Broadway plays.

E X E R C ISE (S k ills 3 9 - 4 2 ) : S tu d y th e n o u n s in th e fo llo w in g s e n te n c e s . T h e n in d ic a te i f th e s e n te n c e s are c o r r e c t (C ) o r in c o r r e c t (I). --------

1.

T h e professor d oes not give many exam in chem istry class, but the on es sh e gives are difficult.

--------

2.

His thesis includes an analyses o f the hypotheses.

--------

3.

It was his dream to be a musical in the New York Philharm onic.

--------

4.

For the reception, the caterers prepared a large amc unt o f food to serve a large num ber o f people.

--------

5.

Many job opportunities exist in the field o f nurse if you will accept a low-paying position.

--------

6.

For each business trip you make, you can choose from many d ifferen t airlines.

--------

7.

T h e stim ulus for his career change is his acknow ledgm ent that he is in a d ead-end job .

--------

8.

She wants to undergo a series o f treatments, but she thinks it costs a little too m uch money.

--------

9.

T he television producer that was shown last nigh t on the CBS network from 9:00 to 11:00 was o n e o f the best shows o f the season.

_____

10.

Various sight-seeing excursion were available from the tourist agency.

2 19

220

STRUCTURE AND WRITTEN EXPRESSION

T O E F L E X E R C ISE (S k ills 3 9 - 4 2 ) : C h o o se th e lette r o f th e u n d e r lin e d w ord o r g r o u p o f w ord s th a t is n o t c o rr ec t. _____ 1.

As a compilation of useful details, a weekly magazine commends itself in several -------- A-------B C respect. D

_____ 2.

Through aauaculture, or fish farming, more than 500 million tons of fish A B are produced each years. C ~ET"

_____ 3

The legal system has much safeguards to protect the right o f a defendant to an A B C impartial jury. D

_____ 4.

The mystery bookstore was largely a phenomena of the last decade. ~A~ B C D

_____ 5 . The Song o f Hiawatha, by Longfellow, tells the story o f the Indian heroism who A B C married Minehaha.

. 6.

Uranus is the seventh planets from the Sun. a ! c d

. 7.

The sycamore has broad leaves with a large amount of pointed teeth. A B C D

8.

The first of two such investigation requires the students to read continuously over a A B period of four hours. C D

_ 9.

A quantitative analysis, using both the computer and quantitative techniques, A B are used to optimize financial decisions. C D

_10.

To enter the FBI National Academy, an application must be between the ages of A B C D twenty-three and thirty-four.

WRITTEN EXPRESSION

T O E F L REV IEW E X E R C ISE (S k ills 1 -4 2 ): C h o o se th e lette r o f th e w o r d o r g r o u p o f w ord s th a t b e st c o m p le te s th e se n te n c e . 1. Presidential____held every four years on the first Tuesday after the first Monday in November. (A) (B) (C) (D)

electing elections are is elected elected and

2. Studies of carcinogenesis in animals can provide data o n --------in human susceptibility. (A) (B) (C) (D)

differences are that differences are differences have differences

3. Those who favor the new law say that the present law does not set spending limits on lobbyists' gifts to politicians, n o r -------statewide funds. (A) (B) (C) (D)

. The population of the earth is increasing at a tremendous rate a n d ------------------out of control. (A) (B) (C) (D)

they have become are soon going to be soon will be why it will be

Starting in 1811, traders and manufacturers were more easily able to send goods upriver in ______ provided the necessary power to counteract the flow of the waters. (A) (B) (C) (D)

steamboats which that that steamboats

it limits limits it does it limit does it

C h o o se th e le tte r o f th e u n d e r lin e d w o r d o r g r o u p o f w o rd s th at is n o t c o r r e c t.

_____ 6.

Temperature indicates on a bimetallic thermometer by the amount that the A B C D bimetallic strip bends.

_____ 7.

Many of the food consumed by penguins consists of fish obtained from the ocean. A B C D

_____ 8.

Before the newspaper became widespread, a town crier has walked throughout a A "B C" village or town singing out the news. D

_____ 9.

All of NASA's manned spacecraft project are headquartered at the Lyndon B. A B C D Johnson Space Center in Houston.

.10.

Fungi cause more serious plant diseased than do other parasites. A B C D

STRUCTURE AND WRITTEN EXPRESSION

PROBLEMS W IT H P R O N O U N S __________________ ____________ P r o n o u n s are w ord s, su ch as he, she, o r it, that take th e p la c e o f n o u n s. W h e n y o u s e e a p r o n o u n in th e W ritten E x p ressio n se c tio n o f th e T O E FL test, y o u n e e d to c h e ck th a t it serv es th e c o r r e c t fu n c tio n in th e s e n te n c e (as a su b ject o r o b ject, fo r e x a m p le ) a n d that it a g r ee s w ith th e n o u n it is rep la cin g . T h e fo llo w in g p r o n o u n p r o b le m s are th e m o st c o m m o n o n th e T O E FL test: (1) d istin g u ish in g su b ject a n d o b je c t p r o n o u n s , (2) d istin ­ g u ish in g p o sse ssiv e p r o n o u n s an d possessive adjectives, a n d (3 ) c h e c k in g p r o n o u n refer­ e n c e fo r a g r e e m e n t.

S k ill

43:

D IS T IN G U IS H SUBJECT A N D OBJECT PR O N O U N S

S u b ject a n d o b je c t p r o n o u n s can b e c o n fu se d o n th e T O E FL test, so y o u sh o u ld b e able to r e c o g n iz e th e se tw o types o f p ro n o u n s: SUBJECT

OBJECT me you him her it us them

/ you he she it we they

A su b je ct p r o n o u n is u sed as th e su b ject o f a verb. A n o b je c t p r o n o u n can b e u sed as the o b je c t o f a verb o r th e o b je c t o f a p r e p o sitio n . C o m p a re th e fo llo w in g two se n te n c e s . Sally gave the book to John.

\

s'S

She gave it to him.

In th e s e c o n d s e n te n c e th e su b ject p r o n o u n she is r e p la c in g th e n o u n Sally. T h e o b je c t o f th e verb it is r ep la c in g th e n o u n book, a n d th e o b je c t o f th e p r e p o sitio n h im is r ep la c in g th e n o u n Jo h n . *• T h e fo llo w in g are e x a m p le s o f th e types o f su b ject o r o b je c t p r o n o u n erro rs th a t you m ig h t se e o n th e T O E FL test. Him* and the girl are going shopping. T he gift was intended for you and I*. In th e first e x a m p le , th e o b je c t p r o n o u n him is in c o r r e c t b e c a u se th is p r o n o u n serv es as th e su b ject o f th e se n te n c e . T h e o b ject p r o n o u n him sh o u ld b e c h a n g e d to th e su b ject p r o n o u n he. It ca n b e d ifficu lt to r e c o g n iz e that him is th e su b ject b e c a u se th e verb are has a d o u b le su b ject, h im a n d girl. In th e se c o n d e x a m p le , th e su b ject p r o n o u n I is in c o r r e c t b e c a u se th is p r o n o u n serv es as th e o b ject o f th e p r e p o sitio n for. T h e su b je ct p r o n o u n I sh o u ld b e c h a n g e d to th e o b je c t p r o n o u n me. It ca n b e d ifficu lt to r e c o g n iz e th a t / is th e o b je c t o f th e p r e p o sitio n f o r b e c a u se th e p r e p o sitio n fo r h a s two objects: th e c o r r e c t o b ­ j e c t you a n d th e in c o r r e c t o b je c t I.

WRITTEN EXPRESSION

EX E R C ISE 43: E a ch o f t h e fo llo w in g s e n te n c e s c o n ta in s at least o n e su b je c t o r o b je c t p r o ­ n o u n . C ir cle th e p r o n o u n s . T h e n in d ic a te i f th e s e n t e n c e s are c o r r e c t (C ) o r in c o r r e c t (I). —

1.

T he worst problem with (it) is that (he) cannot afford (it)

— !—

2. (They) saw Steve and (T) at the m ovies last night after class.

--------

3. Perhaps you would like to g o to the seminar with they and their friends.

--------

4. T he m other took her son to the doctor’s office because h e was feelin g sick.

--------

5. I did not know that you and her were working together on the project.

--------

6. She did not buy the sweater because it had a small hole in it. 7.

T he man lead in g the sem inar gave me all the inform ation I n eed ed to make a decision.

8.

T h e cards connectin g the com puter to its printer need to be replaced before them wear down.

9.

H e is goin g to the party with you and me if you do not m ind.

10. You and her ou gh t to return the books to the library because they are already overdue.

S k ill

44:

D IS T IN G U IS H POSSESSIVE ADJECTIVES A N D P R O N O U N S

P ossessive a d jectiv es a n d p r o n o u n s b o th sh ow w h o o r w h at “o w n s ” a n o u n . H o w ev er, p o s­ sessive a d jectiv es a n d p o sse ssiv e p r o n o u n s d o n o t have th e sa m e fu n c t io n , a n d t h e s e tw o k in d s o f p o sse ssiv e s can b e c o n fu s e d o n th e T O E F L test. A p o sse ssiv e a d je c tiv e d e sc r ib e s a n o u n : it m u st b e a c c o m p a n ie d by a n o u n . A p ossessive p r o n o u n takes th e p la c e o f a n o u n : it c a n n p t b e a c c o m p a n ie d by a n o u n . 1 t T hey lent m e their book. ADJECTIVE

T hey lent m e theirs. PRO N O U N

N o tic e th a t in th e first e x a m p le th e p o ssessiv e ad jective their is a c c o m p a n ie d by th e n o u n book. In th e s e c o n d e x a m p le th e p o sse ssiv e p r o n o u n iheirs is n o t a c c o m p a n ie d b y a n o u n . T h e s e e x a m p le s sh o w th e typ es o f e rro rs th a t are p o s sib le w ith p o sse ssiv e a d je c tiv e s a n d p o sse ssiv e p r o n o u n s o n th e T O E F L test. Each m orning they read theirs* newspapers. C ould you give m e your*}

224

STRUCTURE AND WRITTEN EXPRESSION

In th e first e x a m p le , th e p ossessive p r o n o u n theirs is in c o r r e c t b e c a u se it is a c c o m p a n ie d by th e n o u n newspapers, an d a p o ssessiv e p r o n o u n c a n n o t b e a c c o m p a n ie d by a n o u n . T h e p o ssessiv e a d jectiv e their is n e e d e d in th e first e x a m p le . In th e s e c o n d e x a m p le , th e p o s­ sessive a d jectiv e y o u r is in c o r r e c t b e c a u se it is n o t a c c o m p a n ie d by a n o u n , a n d a p o sses­ sive a d jectiv e m u st b e a c c o m p a n ie d by a n o u n . T h e p o sse ssiv e p r o n o u n yours is n e e d e d in th e s e c o n d e x a m p le . T h e fo llo w in g c h a r t o u tlin e s th e p o sse ssiv e s an d th e ir uses:

POSSESSIVE ADJECTIVES

POSSESSIVE PRONOUNS

my your his her its our their

mine yours his hers

must be accompanied by a noun

cannot be accompanied by a noun

ours theirs

E X E R C ISE 44: E ach o f th e fo llo w in g s e n te n c e s c o n ta in s at le a s t o n e p o ssessiv e p r o n o u n or ad jective. C ircle th e p ossessives in th e se se n te n c e s . T h e n in d ic a te i f th e s e n te n c e s are c o r r e c t (C ) o r in c o r r e c t (I). I

1. If she borrows(ycmr) coat, then you should be able to borrow (her)

C

2. Each pot and pan in (hëî) kitchen has (its) own place on the shelf.

_____

3. Mary and Mark invited theirs parents to see their new apartment.

_____

4. W hen my room mate paid her half o f the rent, I paid m ine.

_____

5. All students need to bring theirs own pencils and answer sheets to the exam.

_____

6. All her secretaries are working late tonight to finish her report.

_____

7. T he horse trotting around the track won its race a few m inutes ago.

_____

8.

Before the report is finalized, the inform ation in their notes and our m ust be proofed.

_____

9.

She worked all day cooking food and making decorations for her son ’s birthday party.

_____

10.

T he weather in the m ountains this w eekend will be extrem ely cold, so please take yours heavy jackets.

WRITTEN EXPRESSION

S k ill

45:

C H E C K P R O N O U N REFERENCE FOR AG R EEM EN T

A fter y o u have c h e c k e d th a t th e su b je ct a n d o b je c t p r o n o u n s a n d th e p o sse ssiv e s are u sed correctly, y o u sh o u ld a lso c h e c k e a c h o f th e se p r o n o u n s a n d p o sse ssiv e s fo r a g r e e m e n t. T h e fo llo w in g are e x a m p le s o f e rro rs o f th is type th at y o u m ig h t fin d o n th e T O E F L test: T h e boys will cause trouble if you let him*. Everyone m ust give their* nam e. In th e first e x a m p le , th e sin g u la r p r o n o u n h im is in c o r r e c t b e c a u s e it r e fe r s to th e plu ral n o u n boys. T h is p r o n o u n s h o u ld b e r e p la c e d w ith th e p lu ra l n o u n them. In th e s e c o n d e x ­ a m p le , th e p lu ra l p o ssessiv e a d jectiv e their is in c o r r e c t b e c a u s e it r efe rs to th e sin gu lar everyone. T h is a d jectiv e s h o u ld b e r e p la c e d w ith th e sin g u la r his o r his or her. T h e fo llo w in g c h a r t o u tlin e s w h at y o u s h o u ld r e m e m b e r a b o u t c h e c k in g p r o n o u n r e fe r e n c e :

PRONOUN AGREEMENT 1. Be sure that every proncun and possessive agrees with the th< noun it refers to. A r,[l:

2. You generally check back in the sentence for agreement.

E X E R C ISE 45: E ach o f th e fo llo w in g s e n te n c e s c o n ta in s at le a s t o n e p r o n o u n o r p o sses­ sive. C ir cle th e p r o n o u n s a n d p o ssessiv es. D raw arrow s to th e n o u n s th e y r efe r to. T h e n in d ic a te i f th e s e n t e n c e s are c o r r e c t (C ) o r in c o r r e c t (I). — !— C

1■ If a person really wants to succeed, (they) must always work hard. 2.

If you see the students from the math class, could you return (their) exam papers to (them?)

3.

Som e friends and I w ent to see a movie, and afterwards we wrote a critique about them.

4.

If you have a problem , you are welcom e to discuss it with m e before you u y to resolve them.

5.

I know you had a terrible time last week, but you m ust try to forget about it.

6.

At the start o f the program , each student n eeds to see his advisor about his schedule.

7.

In spite o f its small size, these video recorders produce excellen t tapes.

8.

W hatever the situation, you should reflect profoundly abou t them before com ing to a decision.

9.

T he p eop le 1 admire m ost are those who m anage to solve their own problems.

10. If anyone stops by w hile I am at the m eeting, please take a m essage from them.

STRUCTURE AND WRITTEN EXPRESSION

E X ER C ISE (Sk ills 4 3 - 4 5 ) : C ircle th e p r o n o u n s a n d p ossessives in th e fo llo w in g se n ­ te n c e s. T h e n in d ic a te i f th e s e n te n c e s are c o r r e c t (C ) o r in c o r r e c t ( I ) . _____

1.

H elicopters are being used m ote and m ore in em ergency situadons because o f its ability to reach out-of-the-way places.

_____

2.

The worker was fired by the chem ical com pany because his refused to work with certain dangerous chemicals.

_____

3.

If you have car trouble while driving on the freeway, you should pull your car over to the side o f the freeway and wait for help.

_____

4.

The administration will n o t install the new security system because they cost so much.

_____

5.

Some parents prefer to send their children to private schools because they believe the children will be better educated.

_____

6.

The air traffic controller was not blam ed for the accident because he had strictly follow ed the correct procedures.

_____

7.

The new student has b een assigned to work on the project with you and I.

_____

8.

Many different kinds o f aspirin are on the market, but theirs effectiveness seem s to be equal.

_____

9.

You must bring a tent and a sleep in g bag for your trip to the Sierras.

_____

10.

Each o f the team m em bers had their new uniform.

T O E F L E X ER C ISE (S k ills 4 3 - 4 5 ) : C h o o se th e le tte r o f th e u n d e r lin e d w ord or g r o u p o f w ord s th at is n o t co rr ec t.

_____ 1.

_____ 2

Superman made their com ic debut in 1938 in Action Comics. A BC D Commercial letters of credit are often used to finance export trade, but them can A B C have other uses. D

_____ 3

When children experience too much frustration, its behavior ceases to be integrated. A B C D

_____ 4

On March 30, 1981, President Reagan was shot as his was leaving a Washington hotel. A B C D

_____ 5.

Although the destruction that it causes is often terrible, cyclones benefit a -----X----B much wider belt than they devastate.

WRITTEN EXPRESSION

6.

President Andrew Jackson had an official cabinet, but him preferred the advice o f A ~B C his informal advisors, the Kitchen Cabinet. D

7.

After Clarence Day’s book Life w ith Father was rewritten as a play, they ran for A B ~C~ six years on Broadway. D

8.

Almost half of the Pilgrims did not survive theirs first winter in the New World. A B ~C D-

9.

There was no indication from the Senate that he would agree with the decision made A B ~C ~D ~ in the House.

_10.

A baby learns the meanings of words as they are spoken by others and later uses him ~A~ B ~C~ ~D~ in sentences.

T O E F L REVIEW E X E R C ISE (S k ills 1 - 4 5 ) : C h o o se th e le tte r o f th e w o r d o r g r o u p o f w ord s th a t b e st c o m p le te s th e s e n t e n c e . 1. ____worst phase of the Depression, more than thirteen million Americans had no jobs. (A) (B) (C) (D)

It was in the During the While the The

3- Speech consists not m erely o f sounds but -------- that follow various structural patterns. (A) (B) (C) (D)

o f organized sound patterns organized sound patterns that sound patterns are organized *n organizing sound patterns

2. When reading a book, you must keep your point of view separate from the point o f view i n _____ you are studying. (A) (B) (C) (D)

that the material and the materials that the materials that are

C h o o se th e le tte r o f th e u n d e r lin e d w o rd o r g r o u p o f w o rd s th a t is n o t c o r r e c t.

_____ 4.

The latest medical report indicated that the patient's temperature was near normal A B and their lungs were partially cleared. ~C~ D-

5.

Most oxygen atom s have eight neutrons, but a small amount have nine or ten . A ~B ~ C D

6.

When Paine expressed his belief in independence, he praised by the public. _A~ B C 5

STRUCTURE AND WRITTEN EXPRESSION

____ 7.

A vast quantity of radioactive material is made when does a hydrogen bomb explode. A B C D

_____ 8.

Genes have several alternative form , or alleles, which are produced by mutations. A B C D

_____ 9.

A star that has used up its energy and has lost its heat became a black dwarf. ~A~ IT C D

_____ 10.

Each lines of poetry written in blank verse has ten syllables, which are alternately AB C D stressed and unstressed.

PROBLEMS W IT H ADJECTIVES A N D ADVERBS______________ M any d iffe r e n t p r o b le m s w ith a d jectiv es a n d adverbs are p o ssib le in th e W ritten E xp res­ sio n se c tio n o f d ie T O E F L test. To id e n tify th e s e p r o b le m s, y o u m u st first b e a b le to r e c ­ o g n iz e adjectives an d adverbs. O fte n adverbs are fo r m e d by a d d in g -ly to ad jectives, a n d th e se -ly adverbs are very easy to r e c o g n iz e . T h e fo llo w in g e x a m p le s sh o w ad verb s that are fo r m e d by a d d in g -ly to adjectives: ADVERB

ADJECTIVE recent

recently

public

publicly

evident

evidently

H ow ever, th e r e are m a n y ad verb s in E n g lish th a t d o n o t e n d in -ly. T h e s e ad verb s ca n b e r e c o g n iz e d fr o m th eir m e a n in g s. T h e y can d e sc r ib e w hen so m e th in g h a p p e n s (often, soon, later), how s o m e th in g h a p p e n s (fa st, hard, well), or where s o m e th in g h a p p e n s (here, there, nowhere). T h e r e are th r e e sk ills in v o lv in g a d jectiv es a n d adverbs th a t w ill h e lp y o u o n th e W rit­ te n E x p ressio n se c tio n o f th e T O E F L test: (1 ) k n o w in g w h e n to u se ad jectiv es an d ad­ verbs, (2) u s in g a d jectives ra th er th a n ad verb s after lin k in g verb s, a n d (3) p o s itio n in g ad jectives an d adverbs correctly.

S kill

46:

USE BASIC ADJECTIVES A N D ADVERBS CORRECTLY

S o m e tim e s in th e W ritten E x p r e ssio n se c tio n o f th e T O E F L test, ad jectives a re u s e d in p lace o f adverbs, o r adverbs are u s e d in p la c e o f adjectives. A d jectiv es a n d ad verb s h ave very d iffe r e n t u ses. A d jectives h ave o n ly o n e jo b : they d e sc r ib e n o u n s o r p r o n o u n s . She is a beautiful w om an. ADJ.

She is beautiful. PRO

ADJ

NOUN

WRITTEN EXPRESSION

In th e first e x a m p le , th e a d je c tiv e bea u tifu l d e sc r ib e s th e n o u n w om an. In th e s e c o n d e x ­ a m p le , th e a d je c tiv e b e a u tifu l d e sc r ib e s th e p r o n o u n she. A d verb s d o th r e e d iffe r e n t th in g s. T h e y d e sc r ib e verb s, a d jectiv es, o r o th e r adverb s. f

1

,

She sings beautifully. VERB

ADV.

I \ I t She is a beautifully dressed woman. ADV.

ADJ.

N OU N

I } I ^ I ^ She is a truly beautifully dressed woman. ADV.

ADV.

ADJ.

NOUN

In th e first e x a m p le , th e ad verb beautifully d e sc r ib es th e verb sings. In th e s e c o n d e x a m p le , th e adverb beautifully d e sc r ib e s th e a d jectiv e dressed (w h ic h d e sc r ib e s th e n o u n w om an). In th e th ird e x a m p le , th e ad verb truly d e sc r ib e s th e adverb beautifully, w h ic h d e sc r ib e s th e ad jective dressed (w h ic h d e sc r ib e s th e n o u n woman). T h e fo llo w in g are e x a m p le s o f in c o r r e c t s e n te n c e s as th ey m ig h t a p p e a r o n th e T O E F L test.

.

I

t

They were seated at a largely* table. NOUN

j I T h e child talked quick* to her mother. VERB

ADJ.

I t We read an extreme* long story. ADJ.

ADJ.

In th e first e x a m p le , th e ad verb largely is in c o r r e c t b e c a u se th e a d je c tiv e large is n e e d e d to d e sc r ib e th e n o u n table. In th e s e c o n d e x a m p le , th e ad jective quick is in c o r r e c t b e c a u se th e adverb quickly is n e e d e d to d e sc r ib e th e verb talked. In th e last e x a m p le , th e ad jective extreme is in c o r r e c t b e c a u se th e adverb extremely is n e e d e d to d e sc r ib e th e a d je c tiv e long. T h e fo llo w in g c h a r t o u tlin e s th e im p o r ta n t in fo r m a tio n th at y o u s h o u ld r e m e m b e r a b o u t th e b a sic u se o f a d jectiv es a n d adverbs:

BA SIC U SE O F A D JEC T IV ES A N D A D VERBS ADJECTIVES ACWER&S

'

A d je ctive s describe nouns o r pronouns. A d ve rb s d escribe verbs, adjectives, o r o th e r adverbs.

STRUCTUREANDWRITTEN EXPRESSION

EX E R C ISE 46: E ach o f th e fo llo w in g s e n te n c e s h a s at le a s t o n e a d je c tiv e o r ad verb . C ircle th e ad jectiv es a n d adverb s, a n d la b e l th e m . D raw arrow s to th e w o r d s th e y d e sc r ib e . T h e n in d ic a te i f th e s e n t e n c e s are c o r r e c t (C ) o r in c o r r e c t (I ).

1.

T h e m other was(pleasan ¿¡(surprised) w hen her daughter cam e to visit.

2.

i ^ T he salespeople (frequently) visit the East Coast for trade shows.

ADJ.

ADJ.

ADV.

VERB

3.

H e was driving an expensively sports car.

4.

T here is a special program on television this evening.

5.

She was chosen for the leading part because she sings so well.

6.

T he car was n o t com plete ready at 3:00.

7.

It was difficult to believe that what we read in the newspaper was a truly story.

8.

Points will be subtracted for each incorrect answered question.

9.

T he production m anager quietly requested a com pletely report o f the terribly incident.

10. The children finished their hom ework quickly so they could watch television.

S k ill

47:

USE ADJECTIVES AFTER L IN K IN G VERBS

G en er a lly an ad verb r a th e r th an an ad jective w ill c o m e d ir ec d y after a verb b e c a u se th e adverb is d e sc r ib in g th e verb.

f

1

She spoke nicely. VERB

ADV.

In th is e x a m p le , th e verb spoke is fo llo w e d by th e adverb nicely. T h is ad verb d e sc r ib e s th e v erb spoke. H ow ever, y o u m u st b e v e ry c a re fu l i f th e verb is a lin k in g verb . A lin k in g v erb is fo l­ lo w ed by an ad jective r a th e r th a n an adverb.

t

1

She looks nice. SUB.

ADJ.

In th is e x a m p le , th e lin k in g verb looks is fo llo w e d by th e ad jective nice. T h is ad jective d e ­ scrib es th e su b je c t she.

WRITTEN EXPRESSION

You sh o u ld b e su re to u se a n a d je c tiv e ra th er th an an adverb a fter a lin k in g verb . B e c a re fu l, h ow ever, b e c a u se th e a d jectiv e th a t g o e s w ith th e lin k in g verb d o e s n o t always d i­ rectly fo llo w th e lin k in g verb.

1

I

tl

H e seem s unusually nice. SUB.

adv.

ADJ.

In this e x a m p le , th e a d je c tiv e nice, w h ic h d e sc r ib e s th e su b je c t he, is its e lf d e s c r ib e d by th e adverb u n u su a lly. F rom th is e x a m p le , y o u sh o u ld n o tic e th a t it is p o s sib le to h a v e an a d ­ verb d irectly a fter a lin k in g verb , b u t o n ly i f th e ad verb d e sc r ib e s a n a d je c tiv e th a t follow s. T h e fo llo w in g c h a r t lists c o m m o n ly u s e d U n king verb s a n d o u tlin e s th e d iffe r e n t u ses o f ad jectives a n d ad verb s a fter reg u la r v erb s a n d lin k in g verbs:

ADJECTIVES AND ADVERBS AFTER VERBS (subject)

f

+

(regular verb)

+

'

(adverb)

A regular verb is followed by an adverb. The adverb describes the verb. f

(subject)

_

+

(linking verb)

+

' V (adjective)

A linking verb is followed by an adjective. The adjective describes the subject. f (subject)

+

' (linking verb)

+

I (adverb)

+

t I (adjective)

It is possible that a linking verb is followed by an adverb and an adjective. The adverb describes the adjective and the adjective describes the subject. UNKING VERBS:

appear be

feel look

become

prove

seem smell taste

EX E R C ISE 47: E a ch o f th e fo llo w in g s e n te n c e s c o n ta in s at lea st o n e a d je c tiv e o r adverb. C ircle th e a d jectiv es a n d ad verb s, a n d la b e l th e m . D raw arrow s to th e w o r d s th ey d e sc r ib e. T h e n in d ic a te i f th e s e n te n c e s are c o r r e c t (C ) o r in c o r r e c t (I).

J__

1.

f ^ 1 ___v T he parents seem (angrily) about the child’s report card. ADV.

c

2. T h e speaker talked (knowingly) about (prehistoric) fossils. ADV.

___ ___ 4.

ADJ.

3. After she drank the lem onade, the cake tasted too sweetly to her. T h ro u g h o u t d in n e r we w ere b o re d because h e spoke incessantly. 5. Sam felt terribly depressed after the accident.

STRUCTURE ANDW RITTEN EXPRESSION

6.

T h e n eighb or appeared calm in spite o f the fact that his house was on fire.

7.

H e looked quite unhappily at the thought o f leaving his job.

8.

Marla ju m p ed up quick w hen she heard the gunshot.

9.

Even though we were not really hungry, the food sm elled delicious.

10. T h e history course that I took last sem ester proved m ore difficultly than I had expected.

S k il l

48:

P O S IT IO N ADJECTIVES A N D ADVERBS CORRECTLY

A d jectiv es a n d a d v erb s can a p p ea r in in c o r r e c t p o s itio n s in th e W ritten E x p r e ssio n se c ­ tio n o f th e T O E F L test. T h e r e are tw o c o m m o n errors o f th is type th a t y o u sh o u ld bew are of: (1 ) th e p o s itio n o f a d jectives w ith th e n o u n s th ey d e sc r ib e , a n d (2 ) th e p o s itio n o f ad­ verb s w ith o b je c ts. In E n g lish it is c o r r e c t to p la c e a o n e-w o rd a d jectiv e in fr o n t o f th e n o u n it d escrib es. O n th e T O E F L test, h ow ever, an in c o r r e c t s e n te n c e m ig h t have an a d jectiv e after th e n o u n it d e sc r ib es. T he inform ation important* is on the first page. N OU N

ADJ.

In th is e x a m p le , th e a d jectiv e im p o rta n t sh o u ld c o m e b e fo r e th e n o u n inform ation, b e ­ c a u se im p o rta n t d e sc r ib e s inform ation. A s e c o n d p r o b le m you s h o u ld b e aware o f is th e p o s itio n o f ad verb s w ith o b jects o f verb s. W h e n a verb h a s an o b je c t, an adverb d e sc r ib in g th e v erb sh o u ld n o t c o m e b etw een th e verb a n d its o b je c t. H e has taken recently* an English course. ADV.

OBJECT

T h is e x a m p le is in c o r r e c t b e c a u se th e adverb recently c o m e s b e tw e e n th e verb has taken a n d its o b je c t a n E n g lish course. T h e r e are m a n y p o ssib le c o r r e c tio n s fo r th is s e n te n c e . Recently he has taken an English course. He has recently taken an English course H e has taken an English course recentiy. You ca n s e e fr o m th e s e e x a m p le s th a t th ere are m an y p o ssib le c o r r e c t p o s itio n s fo r th e ad verb . W h a t is im p o r ta n t fo r you to r em e m b e r is th a t an adverb th a t d e sc r ib es a verb c a n n o t c o m e b e tw e e n th e verb a n d its ob ject.

WRITTEN EXPRESSION

T h e fo llo w in g c h a r t o u tlin e s th e key p o in ts th a t y o u s h o u ld r e m e m b e r a b o u t th e p o si­ tio n o f a d jectiv es a n d adverbs: p • : ,■ • ■

THE POSITION OF ADJECTIVES AND ADVERBS

ADJECTIVES

A one-word adjecVve comes before the noun it describes. It does not come directly after.

ADVERBS

An adverb can appear in many positions. It cannot be used between a verb and its object.

E X E R C ISE R S: E a ch o f th e fo llo w in g s e n te n c e s c o n ta in s at least o n e a d je c tiv e o r adverb. C ircle th e a d jectiv es an d adverb s, a n d la b el th em . D raw arrow s to th e w ord s th ey d e sc r ib e . T h e n in d ic a te i f th e s e n t e n c e s are c o r r e c t (C ) o r in c o r r e c t (I).

I

--------

1.

f ~

I

^

The store op en ed with a sale (fantastic) ADJ.



2.

The pharm acist has (S l w ^ ) filled our order (quickly) ADV.

ADV.

--------

3.

The political candidates expressed their opposing views.

--------

4.

T he lawyer has selected carefully a new case.

--------

5.

Frequently the coffee has tasted bitter.

--------

6.

The w edding reception was held at a restaurant expensive.

--------

7.

The salesclerk has often traveled to New York.

--------

8.

Following the failure o f the first set o f plans, the manager has altered subsequently

9. --------

10.

The students had to study many hours daily during the program intensive. The naval officer was asked to transfer to a foreign country.

EX ER C ISE (S k ills 4 6 - 4 8 ) : C ircle th e ad jectives a n d adverbs in th e fo llo w in g s e n te n c e s hT cT rre«°(I) ^

W° rdS * * * d eSC db e' T h e n in d ic a te i f t h e s e n t e n c e s are c o r r e c t (C ) or

They wer= unable to see where their friends were sitting in the theater because o f the lights dim. 2.

After the com prehensive exam , she looked exhaustedly by the experience.

3.

The project was remarkable close to being finished.

4.

Mark always d oes his hom ework careful.

234

STRUCTURE AND WRITTEN EXPRESSION

_____

5

.

T h e p ro g ra m proved far m o re in te re stin g th a n I h a d im a g in ed it w ould be.

_____

6

.

T h e stu d e n t h a d a tte n d e d regularly all th e lectu res in the series.

_____

7.

_____

8

.

T h e g ra n d p a re n ts speak p roudly a b o u t all th e ir offspring.

_____

9

,

T h e m a n a g e r seem ed certainly th a t the p ro je c t w ould b e fin ish ed u n d e r budget.

_____

10.

T h e p a tie n t b e cam e h ealth y a fte r th e o p e ratio n .

T h e firefig h te rs w orked feverishly, a n d they p u t o u t im m ediately the fire.

T O E F L E X E R C IS E (S k ills 4 6 - 4 8 ) : C h o o se th e le tte r o f th e u n d e r lin e d w ord or g r o u p o f w o r d s th a t is n o t c o rr ec t. 1 M odem a rt is on display at the Guggenhein M useum , a building w ith an unusually A------T “ C D design. 2 Bv the beginning of the 1980s fifteen states had adopted already no-fault insurance ------- A------B C D laws. 3. H eart attacks are fatally in 75 percent of o ccu rren ces. A B C D In spite o f a trem endous am ount of electronic gadgetry, a ir traffic control still A B C

4

depends heavy on people. D 5

6

-

Only recently have Gooden’s industrially designers and engineers been able to -------- A B optim ize W atertred’s unusual tread patterns for m ass p ro d u c tio n . C ' D . A baboon’s arm s appear as lengthily as its legs. A B C D

7

A serious problem is how to com m unicate reliable w ith ---------- A---------B C

a subm erged subm arine. D

8

A m ericans are destroying rapidly wetlands, faster th an ------ A------------ B C

a n acre every two m inutes. D

9. The c entral banking system A B

JO. Telegraph service A

of the U nited States consists of twelve C

across the Atlantic was successful established in B C D

b a rk s d istric t. D 1866.

WRITTEN EXPRESSION

TOEFL REVIEW EXERCISE (Skills 1 -4 8 ): C h o o se th e le tte r o f t h e w o rd o r g r o u p o f w ord s th at b e st c o m p le te s th e s e n te n c e . 1. Patty Berg, the top tournament winner in women's g o lf,-------- eighty-three golf tournaments from 1935 through 1964. (A) (B) (C) (D)

she won winning won who won

2.

with about fifteen tim es its weight in air does gasoline allow the carburetor to run smoothly. (A) (B) (C) (D)

It is mixed To mix it When mixed Only when mixed

C h o o se th e lette r o f th e u n d e r lin e d w ord o r g r o u p o f w ord s th a t is n o t c o r r e c t.

3.

The Colorado River reaches their maximum height during April and May. A B C D

4.

Plant proteins tend to have few amino acids than proteins from animal sources A B C ----------¡5----------

5.

The Viking spacecraft has landed on Mars in July of 1976. A B C "D

6.

Admiral Byrd commanded airplane expeditions over both the Arctic or the A B ~ o' Antarctic.

— 7.

The advertising campaign will be based on the recent completed study. A B C D

_ 8.

Coronary occlusion results from a disease in which fatty substances with a large A B amount of cholesterol is deposited in the arteries. C D

- 9.

Her money gave back as soon as she threatened to take the matter to court. A B C D

-10.

Other sites o f fossil discoveries throughout Wyoming, ranging from the fiery A B Tyrannosaurus rex to the milder Triceratops, have proven equally e x cite. c D

STRUCTURE AND WRITTEN EXPRESSION

MORE PROBLEMS W IT H A D JEC TIVES ------------------------------------T h e p reviou s se c tio n d e a lt w ith variou s p r o b le m s rela ted to b o th a d jectives a n d adverbs. T h is se c tio n d e a ls w ith a few p r o b le m s th a t are r ela ted o n ly to adjectives: (1 ) -ly a d jectiv es, (2) p r e d ica te ad jectives, an d (3 ) -« ¿ a n d - in g adjectives.

S kill

R EC O G N IZE -LY ADJECTIVES

49:

G en erally w h en a w ord e n d s in -ly in E n g lish , it is an adverb. H ow ever, th e r e are a few w ords e n d in g in -ly th at are ad jectives, a n d th e se -ly ad jectives c a n cau se c o n fu s io n in th e W ritten E x p r e ssio n se c tio n o f th e T O E F L test. i * T he manager turned in his weekly report. ADJ.

NOU N

T h is e x a m p le is co rr ec t, b u t it ap p ears to b e in correct; it ap p ears that th e r e is an -ly adverb in fro n t o f th e n o u n report. H ow ever, weekly is an adjective th at d e sc r ib es th e n o u n report. T h e fo llo w in g ch a rt lists c o m m o n -ly ad jectives th at can a p p ea r in E nglish :

-LY ADJECTIVES

costly early friendly kindly

likely lively lonely manly

daily hourly monthly nightly

quarterly northerly weekly easterly^ yearly southerly lo v e ly ___________ westerly

E X ER C ISE 49: E ach o f th e fo llo w in g s e n te n c e s c o n ta in s at least o n e ad jective o f adverb e n d in g in -ly. C ircle th e -ly w ords, a n d la b e l th e m as e ith e r ad jectives o r adverbs. D raw ar­ rows to th e w ord s they d e sc r ib e. T h e n in d ic a te if th e s e n te n c e s are c o r r e c t (C ) o r in c o r ­ rect ( I ) . C

i.

i y Federal taxes are (yearly) taxes which must be paid every April.

ADV.

3.

Do you want to go to the early movie or the lately movie?

4.

She offered m e som e friendly advice about how to deal with the terribly problem .

5

Xhe quarterly reports need to be turned in at the next weekly m eeting.

6.

He did not have a manly reaction to the negatively com m ents.

7.

T he likely outcom e o f the purchase o f the costly car is that he will not be able to pay his m onthly bills.

WRITTEN EXPRESSION

--------8.

The days she spent at the beach house were lonely and solitarily.

--------

9.

--------

10.

S k ill

50:

She takes her daily m edication on a regularly schedule. T he kindly neighbor paid hourly visits to her unhealthily friend.

USE PREDICATE ADJECTIVES CORRECTLY

C ertain a d jectiv es a p p e a r o n ly in th e p r e d ic a te o f th e s e n te n c e ; th a t is, th e y a p p e a r after a lin k in g verb su c h as be, a n d th ey c a n n o t a p p ea r d ir ec tly in fr o n t o f th e n o u n s th at th e y d e ­ scrib e. T h e snake on the rock was alive. T he alive* snake was lying on the rock. In th e first e x a m p le , th e p r e d ic a te a d jectiv e alive is u sed c o rr ec tly a fter th e lin k in g verb was to d e sc r ib e th e su b je c t snake. In th e s e c o n d e x a m p le , th e p r e d ic a te a d je c tiv e a live is u se d in c o r r e c tly in fr o n t o f th e n o u n snake. In th is p o s itio n , th e a d jc c tiv e live s h o u ld be u sed . T h e fo llo w in g c h a r t lists s o m e c o m m o n p r e d ic a te a d jectiv es a n d th e c o r r e s p o n d in g fo r m s th a t can b e u se d in fr o n t o f th e n o u n :

PREDICATE ADJECTIVES PREDICATE ADJECTIVES

FORMS USED IN FRONT OF NOUN

alike alive alone afraid asleep

(ike, similar live, living lone frightened sleeping

A predicate adjective appears after a linking verb such as be. It cannot appear directly in front of the noun that it describes.

E X E R C ISE 50: E a ch o f th e fo llo w in g s e n te n c e s c o n ta in s a p r e d ic a te a d je c tiv e o r its re­ la ted fo r m . C ircle th e p r e d ic a te a d jectiv es o r r e la te d fo r m s. T h e n in d ic a te i f th e s e n ­ te n c e s are c o r r e c t (C ) o r in c o r r e c t (I). — 0—

1. T he two brothers do n ot look at all (alike)

— !—

2.

My friend brought the (alive) lobster to my house and exp ected m e to cook it.

--------

3. Are you g o in g to be lo n e in the house

--------

4. T he afraid child cried for his mother.

--------

5. Everyone else was asleep by the tim e I

tonight?

arrived hom e.

237

STRUCTURE AND WRITTEN EXPRESSION

6.

We com pleted our two projects in a like manner.

7.

All o f the crash victims were alive when they were found.

8.

She tried to walk quietly by the asleep dogs without waking them.

9.

Were you feelin g afraid w hen you heard the noise?

10. A ccording to the report, the president was shot by an alone gunm an.

S k i l l 5 1:

USE -ED A N D -ING ADJECTIVES CORRECTLY

Verb fo r m s e n d in g in -ed a n d -in g c a n b e u sed as ad jectives. For e x a m p le , th e verb al ad jec­ tives cleaned a n d cleaning c o m e fr o m th e verb to clean.

T he woman cleans the car. VERB

1 * u car. The cfeonmgwoman worked on the ADJECTIVE

T he woman put the cleaned car back in the garage. ADJECTIVE

In th e first e x a m p le , cleans is th e verb o f th e se n te n c e . In th e s e c o n d e x a m p le , cleaning is a verbal ad jective d e sc r ib in g w om an. In th e third e x a m p le , cleaned is a verb al ad jective d e ­ scrib in g car. V erbal ad jectives e n d in g in -ed a n d -in g c a n b e c o n fu s e d in th e W ritten E x p r e ssio n se c ­ tio n o f th e T O E F L test. T he cleaning* car . . . T he cleaned* woman . . . T h e d iffe r e n c e b e tw e e n an -ed a n d an -in g a d jective is sim ilar to th e d iffe r e n c e b e tw e e n th e active a n d th e passive (se e Skills 3 7 an d 3 8 ). A n - in g a d jective (lik e th e a ctiv e) m e a n s th at th e n o u n it d e sc r ib e s is doing-the a c tio n . T h e ab ove e x a m p le a b o u t th e cleaning ca r is n o t c o r r e c t b e c a u se a car c a n n o t d o th e a c tio n o f c le a n in g : you c a n n o t say th a t a car cleans itself. A n -ed a d jective (lik e th e p assive) m e a n s th a t th e n o u n it d e sc r ib e s is receiving th e ac­ tio n fr o m th e verb. T h e ab ove e x a m p le a b o u t the cleaned w om an is n o t c o r r e c t b e c a u se in th is e x a m p le a w om an c a n n o t r e c e iv e th e a c tio n o f th e verb clean: th is s e n t e n c e d o e s n o t m e a n th a t someone cleaned the w om an.

WRITTEN EXPRESSION

T h e f o llo w in g c h a r t o u tlin e s t h e k e y i n f o r m a t io n th a t y o u s h o u ld r e m e m b e r a b o u t -ed a n d -ing a d je c tiv e s :

-ED AND -ING ADJECTIVES TYPE

MEANING

-. -iNC ,—■.



-£D

USE

active

... -

passive





EXAMPLE

It does the action of the verb. ■ ■■■■; ' r a ■:

. . . the happily playing children . . . (The children ploy.)

It receives the action of the verb.

.. . the frequently played record . . . (Someone plays the record.)

E X E R C I S E 51: E a c h o f t h e f o llo w in g se n te n c e s c o n ta in s e it h e r a n -ed o r a n -ing v e r b a l a d ­ je c tiv e . C ir c l e t h e v e r b a l a d je c tiv e s . D r a w a rro w s to th e w o rd s th e y d e s c rib e . T h e n in d i­ c a te i f th e s e n te n c e s a re c o r r e c t ( C ) o r i n c o r r e c t ( I ) .

I --------

1.

I ^ The teacher gave a quiz o n the ju st (com pleting) lesson.

C —x_

2.

I T here is a (f'asci n atin g)m ovie at the theater tonight.

--------

3.

They thought that it had been a very satisfied dinner.

--------

4.

The em pty b otd es are to the left, and the filling bottles are to the right.

-----

5.

F o r lu n c h at the restau ran t she o rd ere d a m ixed salad.

--------

6.

T h e students thought that it was an interesting assignm ent

--------

7.

T he shoppers were im pressed by the reducing prices.

--------

8.

He can ’t afford to take lon g vacations to exotic places because he is a worked man.

--------

9.

I recently received several annoying phone calls from the insurance agent.

--------

10.

Today the bookkeeper is working on the unpaying bills.

E X E R C I S E ( S k ills 4 9 - 5 1 ): C ir c l e th e a d je c tiv e s in e a c h o f th e f o llo w in g s e n te n c e s . D r a w a rro w s to th e n o u n s o r p r o n o u n s th e y d e s c rib e . T h e n in d ic a t e i f t h e s e n te n c e s a r e c o r r e c t (C ) o r in c o rre c t ( I ) .

--------

1.

H er kindly words o f thanks made m e feel appreciating.

--------

2.

After the earthquake, assistance was sent to the dam aging areas.

--------

3.

Your view has som e validity; however, we do not have alike opin ion s o n the matter.

--------

4.

It is likely that the early seminar will not be the m ost interested.

--------

5.

I prefer a live theater show to a movie.

STRUCTURE AND WRITTEN EXPRESSION

_____

6.

The thesis o f your essay was not very well developed.

_____

7.

T he asleep children were wakened by the loud sound o f the crashing thunder.

_____

8.

During the nightly news show there was a lively and fascinating debate.

_____

9.

H is car was struck by an uninsured motorist.

_____

10.

The girl was all alone and feeling lonely in the darkened, frightened house.

T O E F L E X E R C ISE (S k ills 4 9 -5 1 ): C h o o se th e le tte r o f th e u n d e r lin e d w o rd o r g r o u p o f w o rd s th a t is n o t c o rr ec t. _____ 1

As the only major American river that flowed in a west direction, the Ohio was the ---- A---B C preferred route for settlers. D

___ 2.

During the annually salmon migration from the sea to fresh water, Alaska s McNeil A B River becomes a gathering place for brown bears waiting eagerly to catch their fill. ------C-----D

_____ 3

Edelman stresses the mounting evidence showing that greatly variation on a ; A B C microscopic scale is likely. D

_____ 4.

Perhaps the most welcoming and friendly of the park's wild places is the live oak A B C forest that surrounds the district’s alone visitors’ center in Gulf Breeze. D

_____ 5.

____ 6

____ 7.

Hailey’s com et, viewing through a telescope, was quite impressive. A B C D The state of deep asleep is characterized by rapid eye movement, or REM, sleep. ~A BC D Among the disputing sections of the Monteverdi opera are the sinfonía, the A B C D prologue, and the role of Ottone.

_____ 8.

Most probably because of the likable rapport between anchors, the night newscast A B C on the local ABC affiliate has recently moved well beyond its competitors in the D ratings battle.

WRITTEN EXPRESSION

_ 9.

Signing at the outset of a business deal, a contract offers the participants a certain A B C degree of legal protection from costly mistakes. D

_10. The story presented by Fischer is a headlong tale told so effectively that  B its momentum carries the reader right through the live endnotes. C

T T

T O E F L R EV IEW E X E R C ISE (S k ills 1 -5 1 ): C h o o se th e le tte r o f th e w o r d o r g r o u p o f w ord s th a t b e st c o m p le te s th e s e n te n c e . During the early nineteenth century, the S p a n i s h m i s s i o n s i n Alta, C a l i f o r n i a

to be an integral part of the economy and productive capacity of the region. (A) proved (B) they proved (C) they proved it (D) proved it

3. The daughters of Joseph LaFlesche were bom into the generation of Omaha forced to abandon tribal traditions, on the reservation, and to adapt to the white man's ways. (A) they matured (B) to mature (C) maturing (D) to maturity

2 . Still o t h e r h u r d l e s r e m a i n b e f o r e

suitable for private cars.

4. Among the most revealing aspects of mining towns their paucity o f public open space.

(A) fuel cells (B) become (C) fuel cells become (D) that fuel cells become

(A) was (B) were (C) it was (D) so

C h o o se th e lette r o f th e u n d e r lin e d w ord o r g r o u p o f w ord s th a t is n o t c o r r e c t.

5.

Factor analysis is used to discover how many abilities are involve in intelligence test A B C D performance.

6.

.

One of the early orders of marine mammals, manatees have evolved more than fifty B million years ago from land animals. C

7.

Dolphins and chimps are like in that they have been shown to have language skills. T 1 C D

8.

In the appendix at the end o f the chapter are the instructions to be used for the “

completion correct of the form. D

TT

C

STRUCTURE AND WRITTEN EXPRESSION

_____ 9.

Used sound that varies not only in time but in space, whales at close range may A “ IT C communicate with sonarlike "pictures.” D

_____ 10. The 1898 Trans-Mississippi International Exposition has the distinction of being the ~A~ B last major fair which held during the Victorian period. C D

PROBLEMS W IT H ARTICLES________________________________ A rticles are very d iffic u lt to lea r n b e c a u se th e r e are m any ru les, m a n y e x c e p tio n s , and m a n y sp ecia l cases. It is p o s sib le , how ever, to learn a few ru les that w ill h e lp y o u to u se ar­ ticles c o rrectly m u c h o f th e tim e. N o u n s in E n g lish can b e e ith e r c o u n ta b le or u n c o u n ta b le . I f a n o u n is c o u n ta b le , it m u st b e e ith e r sin g u la r o r p lu ral. In a d d itio n to th e s e g e n e ra l types o f n o u n s , th e r e are two types o f articles: d e fin ite (sp e c ific ) and in d e fin ite (g e n e r a l).

ARTICLES INDEFINITE

(General) DEFINITE

(Specific)

S k i l l 52:

COUNTABLE SINGULAR NOUNS

COUNTABLE PLURAL NOUNS

o dollar on apple

~

the dollar the apple

the dollars the apples

UNCOUNTABLE NOUNS mnnpy

dollars

apf>leS

............... ,U'Ce . —■ ■■ the money the juice

USE ARTICLES W IT H S IN G U LA R N O U N S

You can s e e fro m th e ch a rt th a t i f a n o u n is e ith e r c o u n ta b le plu ral o r u n c o u n ta b le , it is p o ssib le to h ave e it h e r th e d e fin ite a rticle the o r n o a rticle ( in d e fin ite ). W ith all c o u n t­ a b le sin g u la r n o u n s , h ow ever, y o u m u s t have an a r tic le (u n le ss y o u have a n o th e r d e te r ­ m in e r su ch as my o r each). I have money. I have books. I have a book.

(uncountable — n o article needed) (countable plural — no article needed) (countable singular — article needed)

T h e fo llo w in g c h a r t o u tlin e s th e key in fo r m a tio n th a t y o u sh o u ld r e m e m b e r a b o u t ar­ ticles w ith sin g u la r n o u n s: ARTICLES WITH SINGULAR NOUNS A singular noun m utt have an article (o, an, the) or some other determiner si'ch as my or each. (A plural noun or an uncountable noun may or m ay not have an article.)

WRITTEN EXPRESSION

E X E R C I S E 52: T h e fo llo w in g s e n te n c e s c o n ta in d iffe r e n t types o f n o u n s . C ir cle o n ly th e

c o u n ta b le sin g u la r n o u n s . M ark w h e r e a rticles (or d e te r m in e r s ) h a v e b e e n o m itte d . T h e n in d ic a te i f th e s e n t e n c e s are c o r r e c t (C ) o r in c o r r e c t (I). __ L _

1. She is taking^ trip ) with friends.

C

2. In my (yard) there are flowers, trees, and grass.

_____

3. T he m anager sent m em o to his em ployees.

_____

4. T here is car in front o f the building.

_____

5. T he child and his friends are having milk and cookies.

_____

6. She is studying to be an actress in films.

_____

7.

_____

8. We have m achinery that prints ten pages each minute.

_____

9. Teacher has many students during a semester.

_____

My neighbor was arrested for throwing rocks through windows.

10. Can you heat water for tea?

S k i l l 53:

D IS T IN G U IS H A A N D AN

T h e b asic d iffe r e n c e b e tw e e n a a n d a n is th a t a is u s e d in fr o n t o f c o n s o n a n ts a n d a n is u sed in fr o n t o f vo w els (a, e, i, o, u ) : a ¿»ook a man a /»age

an orange an illness an autom obile

In reality, th e r u le is th a t a is u s e d in fr o n t o f a w ord th a t b e g in s w ith a c o n s o n a n t so u n d a n d th a t a n is u s e d in fr o n t o f a w o rd th a t b e g in s w ith a vow el sound. P r o n o u n c e th e f o l­ lo w in g e x a m p les: a university an unhappy man

a hand an hour

a one-way street an om en

a euphem ism an event

a xerox m achine an x-ray m achine

T h ese e x a m p les sh ow th at certain b e g in n in g letters can have e ith er a c o n so n a n t o r a vow el sou n d . A w ord that b e g in s w ith u can b e g in with a c o n so n a n t y so u n d as in university or w ith a vowel so u n d as in unhappy. A w ord that b e g in s with h can b e g in w ith a c o n so n a n t h so u n d as in hand o r with a vow el s o u n d as in hour. A w ord that b eg in s with o can b e g in w ith a c o n so n a n t w so u n d as in one o r w ith a vow el so u n d as in omen. A w ord that b e g in s w ith e can b e g in w ith ei­ ther a c o n so n a n t y so u n d as in euphemism or with a vowel sou n d as in event. A w ord that b e g in s with x can b e g in with e ith e r a c o n so n a n t z so u n d as in xerox or w ith a vow el so u n d as in x-ray. T h e fo llo w in g c h a r t o u d in e s th e k ey in fo r m a tio n a b o u t th e u se o f a a n d an: .

..

...

A A N D AN A

A is used in front of a singular noun with a consonant sound.

AN

An is used in front of a singular noun with a vowel sound.

Be careful of words beginning with letters such as u. o, e, x, or h. Th-.y may begin with either a vowel or a consonant sound.

STRUCTURE AND WRITTEN EXPRESSION

E X E R C ISE 53: E ach o f th e fo llo w in g s e n te n c e s c o n ta in s a o r an. C ircle e a c h a o r an. U n ­ d e r lin e th e b e g in n in g o f th e w ord th at directly follow s. P r o n o u n c e th e w ord . T h e n in d i­ c a te if th e s e n te n c e s are c o r r e c t (C ) o r in c o r r e c t (I). I

1.

The dishwasher quit his jo b because he was making only four dollars © hour.

C

2.

It was (an) unexpected disappointm ent to receive (a) rejection letter from the university.

3.

A signature is required wherever you see a X on the form.

4.

He bought a half gallon o f milk and a box o f a hundred envelopes.

5.

An objection was raised because it was such a unacceptable idea.

6.

There are two trees in the yard, an elm tree and a eucalyptus tree.

7.

The police officer was n ot wearing an uniform when she arrested the suspect.

8.

If you do not give m e a hand, finishing the project on time will be an impossibility.

9.

She was upset when a hon est mistake was made.

10. She opened a account at a local departm ent store during a one-day sale.

S k ill

54:

MAKE ARTICLES AGREE W IT H N O U N S

T h e d e fin ite article (the) is u se d fo r b o th sin gu lar a n d p lu ral n o u n s , so a g r e e m e n t is n o t a p r o b le m w ith th e d e fin ite article. H ow ever, b e c a u se th e u se o f th e in d e fin ite a r tic le is dif­ f e r e n t for sin g u la r a n d p lu ral n o u n s , y o u m u st b e carefu l o f a g r e e m e n t b e tw e e n th e in ­ d e fin ite a rticle a n d th e n o u n . O n e very c o m m o n a g r e e m e n t e rr o r is to u se th e sin g u la r in d e fin ite article (a or an) w ith a plu ral n o u n . H e saw a* new movies. They traveled to a* nearby mountains. D o you have another* books? In th e se e x a m p le s , you sh o u ld n o t have a o r a n b e c a u se th e n o u n s are p lu ra l. T h e fo llo w ­ in g s e n te n c e s are p o ssib le c o r r e c tio n s o f th e s e n te n c e s above. H e saw a new m ovie. H e saw new movies.

(singular) (plural)

They traveled to a nearby mountain. They traveled to nearby m ountains.

(singular) (plural)

D o you have another book? D o you have other books?

(singular) (plural)

T h e fo llo w in g ch art states th e key p o in t fo r y o u to r e m e m b e r a b o u t th e a g r e e m e n t o f a rticles w ith n o u n s: ' • .. ■■’ ’

' .

AGREEMENT OF ARTICLESWITH NOUNS You should never use a or an with a plural noun.

' - •i

WRITTEN EXPRESSION

E X E R C ISE 54: E a ch o f th e fo llo w in g s e n t e n c e s c o n ta in s a o r an. C ir c le e a c h a o r a n. Draw an arrow to th e n o u n it d e sc r ib e s. T h e n in d ic a te if th e s e n t e n c e s a r e c o r r e c t (C ) o r in c o r r e c t (I).

r

X

t

— __

1. She w ent to school in (a) local community.

— !__

2. T h e doctor used (an oth er pills.

--------

3. It is necessary to have a farm or land o f your own.

--------

4. H e must contact a m em bers o f the club.

--------

5. You will n e e d a pen or a pencil.

--------

6. H e is responsible for bringing a number o f items.

--------

7. You must write a report on a subjects o f your choice.

--------

8. They crossed through several forests and a stream.

--------

9. T here will be another im portant lessons tomorrow.

--------

10. H e could n ot give m e a good reasons for what he did.

S k ill 55:

D IS T IN G U IS H SPECIFIC A N D G ENERAL IDEAS

W ith c o u n ta b le sin g u la r n o u n s it is p o ssib le to u se e ith e r th e d e fin ite o r th e in d e fin ite ar­ ticle, b u t th ey have d iffe r e n t m e a n in g s. T h e d e fin ite article is u s e d to r e fe r to o n e sp e c ific noun. Tom will bring the book tomorrow. (There is on e specific book that Tom will bring tomorrow.) H e will arrive on ¿A« first Tuesday in July. (There is only o n e first Tuesday in July.) H e sailed on the Pacific Ocean. (There is only o n e Pacific Ocean.) T h e d e fin ite a r tic le is u s e d w h e n th e n o u n c o u ld b e o n e o f several d iffe r e n t n o u n s . Tom will bring a book tomorrow. (Tom will bring any o n e book.) H e will arrive o n a Tuesday in July. (H e will arrive o n on e o f four Tuesdays in July.) H e sailed on an ocean. (H e sailed on any o n e o f the w orld’s oceans.)

STRUCTURE AND WRITTEN EXPRESSION

T h e fo llo w in g c h a r t o u tlin e s th e key in fo r m a tio n th a t y o u sh o u ld u n d e r sta n d a b o u t s p e c ific a n d g e n e r a l ideas:

SPECIFICAND GENERAL IDEAS a r t ic l e

A o r AN



-,

general idea

Use when there are many, and you do not know which one it is. Use when there »re many, and you do not care which one it is.

specific idea

Use when it is the only one. Use when there are many, and you know which one it is.

' -

THE

.

USES

MEANING

E X E R C I S E 55: E ach o f th e fo llo w in g s e n te n c e s c o n ta in s o n e o r m o r e a rticles. C ircle th e

articles. D raw arrow s to th e n o u n s th ey d e sc r ib e. T h e n in d ic a te i f th e s e n te n c e s are c o r­ r e c t (C ) o r in c o r r e c t ( I ) . ^

1.

H e took ¿ ”tnp on (&) Snake lLver.

C

2.

I’ll m eet you at (the) library later.

_____

3.

T he ball hit a child on a head.

_____

4.

H e had a best grade in the class on the exam.

_____

5.

T h e peop le who cam e here yesterday were here again today.

_____

6.

She was a m ost beautiful girl in the room.

_____

7.

T h e trip that I took last year to the Bahamas was the only vacation I had all year.

_____

8.

I n eed a piece o f paper so that I can finish the report that I am working on.

_____

9.

A basketball player threw the ball to a center o f the court.

_____

10.

T h e sixth-grade class went on a field trip to visit a Lincoln Memorial.

E X E R C I S E ( S k ills 5 2 - 5 5 ): C ircle th e a rticles in th e fo llo w in g s e n te n c e s . T h e n in d ic a te if

th e s e n te n c e s are c o r r e c t (C) o r in c o r r e c t (I). _____ 1.

a H e took a m oney from his wallet to pay for sweater.

_____

2.

T h e notebook that he left had an im portant assignm ent in it.

_____

3.

Because o f previous disagreem ents, they are trying to arrive at an understanding.

_____

4.

T h e appearance o f room could be improved by adding a green plants.

_____

5.

T he Senate passed law banning sm oking in public workplaces.

_____

6.

Each chem istry student should bring laboratory m anual to a n ext class.

_____

7.

She adm itted that she made mistake but said that she had m ade a h on est effort.

8.

H is absence from the board m eeting was a strong indications o f his desire to leave the company.

WRITTEN EXPRESSION

9.

T he car n eed ed gas, so the driver stopped at a service station.

10. Anyone taking group tour to the Hawaiian Islands must pay fee before a first o f the m onth.

T O E F L E X E R C ISE (S k ills 5 2 - 5 5 ) : C h o o se th e lette r o f th e u n d e r lin e d w o rd o r g r o u p o f w ord s th a t is n o t c o rr ec t. _____ 1.

On a trip down to the bottom of the Grand Canyon, the equipment will in all ~A~ B C probability be carried by a burros. D

_____ 2.

Ford designed the first large-scale assembly line at plant in ' A B ~T~ Highland Park, Michigan. D

_____ 3.

In the human body, blood flows from a heart through the arteries, and it returns A B C through the veins.

_____ 4.

The scholarship that Wilson received to study history at Cambridge presented an A B C ~D" unique opportunity.

_____ 5.

Observations from Earth indicate that at the solar surface, the outward magnetic A B field is a strongest at the polar regions.

Z _____ 6.

------- B-------

A radar images of Venus add details about a planet dominated by A B C volcanoes and lava.

--------- 5--------_____ 7.

In 1863 and 1864, the U.S. Congress passed the National Bank Acts , which set up a A B system of privately owned banks chartered by a federal government. C D

_____ 8.

An human ear responds to a wide range o f frequencies . T ~B~ C D

_____ 9.

Bacteria that live in soil and water play a vital role in recycling carbon, nitrogen, A B sulfur, and another chemical elements used by living things. C D

_____ 10.

During the U.S. Civil War, an American balloonist organized a balloon corps in Army. A B U D

247

248

STRUCTURE AN D WRITTEN EXPRESSION

T O E F L R EV IEW E X E R C ISE (Sk ills 1 -5 5 ): w ord s th at b e s t c o m p le te s th e s e n te n c e .

C h o o se th e le tte r o f th e w ord o r g r o u p o f

1. In econom ics, "diminishing returns" d escribes_____ resource inputs and production. (A) (B) (C) (D)

3. R arely. _remove the entire root of a dandelion because of its length and sturdiness.

among when it is among them the relationship between

(A) (B) (C) (D)

can the casual gardener the casual gardener the casual gardener will does the casual gardener’s

2. When lava reaches the surface, its temperature can be ten tim es_____ boiling water. (A) (B) (C) (D)

the temperature that of it is more

C h o o se th e le tte r o f th e u n d e r lin e d w ord o r g r o u p o f w ord s th a t is n o t c o rr ec t.

4.

5.

Operascan be broadly classified as either com edies or they are tragedies. a b r

D

Tungsten has the highest melting point of all metals, and for this reason it is often A use in equipment that must withstand high temperatures. TT C D

6. Whereas thereare forty-three ant species in GreatBritain, the same amount of ant A "B~

C

species can be found in a single tree in Peru. D 7.

People voice theirs opinions first in small groups or among friends and acquaintances. A B C ~D

8.

Inside the Lincoln Memorial is a large statue of Lincoln make from white marble. A B ~C ~ D

9.

Detailed photometric data of the area just north o f Triton s equatorial region indicate A — B the existence o f a thin, transparent layers of frost. D C

_10.

U.S. census figures indicate that people with only an elementary education can earn A B~ just half as much as college graduations. £ D

WRITTEN EXPRESSION

PROBLEMS W IT H P R E P O S IT IO N S __________________________ P r e p o s itio n s can b e u s e d in tw o ways: in a literal way a n d in an id io m a tic way. In th e literal u s e , th e p r e p o sitio n m e a n s e x a ctly w h at y o u e x p e c t. T he boy ran up the hill. She w ent in the house. In th e first e x a m p le , th e p r e p o s itio n u p m e a n s th a t th e b o y w e n t in th e d ir e c tio n u p rath er th an down. In th e s e c o n d e x a m p le , th e p r e p o sitio n in m e a n s th a t s h e w e n t into rath er th an out o f th e h o u se . In th e id io m a tic u se, w h ic h is w h a t a p p ea r s m o st o fte n o n th e T O E F L test, th e p r e p o ­ sitio n a p p ea r s in a n id io m a tic e x p r e ssio n ; th a t is, its m e a n in g in th is e x p r e s s io n h as n o th ­ in g to d o w ith th e lite ra l m e a n in g . I call up my friend. He su cceed ed in passing the course. In th e first e x a m p le , th e w o r d up h a s n o th in g to d o w ith th e d ir e c tio n up. To call u p some­ one m e a n s to telephone s o m e o n e . In th e s e c o n d e x a m p le , th e w ord in h a s n o th in g to d o w ith th e m e a n in g o f into o r inside; it is sim p ly id io m a tic th a t th e w o r d in is u s e d a fter th e verb succeed. It is im p o ss ib le to list all p o te n tia l id io m a tic e x p r e s sio n s w ith th e ir p r e p o s itio n s b e ­ cau se th e r e are so m a n y e x p r e s s io n s th a t c o u ld a p p ea r o n th e T O E F L test. H ow ever, in th is se c tio n y o u ca n p r a c tic e r e c o g n iz in g p r o b le m s w ith p r e p o s itio n s in T O E FL -type q u e stio n s. T h e n , w h e n y o u are w o r k in g in th e W ritten E x p r e ssio n s e c tio n o f th e TO EFL test, y o u sh o u ld b e aw are th a t id io m a tic e rro rs w ith p r e p o sitio n s are c o m m o n in th a t se c ­ tio n . T h e r e are tw o c o m m o n types o f p r o b le m s w ith p r e p o sitio n s th a t y o u s h o u ld e x p e c t: (1) in c o r r e c t p r e p o s itio n s a n d (2) o m itte d p r e p o sitio n s.

S k ill

56:

R E C O G N IZ E IN C O R R E C T PREPOSITIONS

S o m e tim e s an in c o r r e c t p r e p o s itio n is g iv en in a s e n te n c e in th e W ritten E x p r e ssio n se c ­ tio n o f th e T O E F L test. T he gam e was called on* because o f rain. I knew I could count in* you to do a good job. T h e first e x a m p le s h o u ld say th a t th e g a m e w as called o ff b e c a u se o f rain . T h e e x p r e s sio n called o ff m e a n s canceled, a n d th a t is th e m e a n in g th a t m ak es se n s e in th is s e n t e n c e . To call on someone is to v is it someone, a n d th is m e a n in g d o e s n o t m ak e se n s e in th is e x a m p le . In th e se c o n d e x a m p le , it is n o t c o r r e c t in E n g lish to count in someone. T h e c o r r e c t e x p r e s s io n is to count on someone.

STRUCTURE AND WRITTEN EXPRESSION

E X E R C IS E 56: E ach o f th e fo llo w in g s e n te n c e s c o n ta in s at le a s t o n e p r e p o sitio n . C ircle th e p r e p o sitio n s. T h e n in d ic a te i f th e s e n te n c e s are c o r r e c t (C ) o r in c o r r e c t (I). — 0—

1. (Aftei) school many students participate (in) sports.

— !—

2. I know I can rely (in) you to be here (on) time.

--------

3. If you need m ore light to read, turn on the lamp n ext to you.

--------

4. Parents always try to bring at their children to be thoughtful.

--------

5. I’ll have to consult to my attorney before making a decision.

_____

6. Walt has lost his keys, so he must look for them.

--------

7.

I ju st d o n ’t approve at your cheating o n the exam.

--------

8.

Sm oking is forbidden, so you should put out your cigarette.

--------

9.

Failure to pass the test will result to the loss o f your license.

--------

10.

S k ill

57:

It is unlawful for parolees to associate with known felons.

R E C O G N IZE W H E N PR EPO SITIO NS HAVE BEEN O M IT T E D

S o m e tim e s a n e c e ssa r y p r e p o sitio n h as b e e n o m itte d fro m a s e n te n c e in th e W ritten Ex­ p r e ssio n s e c tio n o f th e T O E F L test. Can you wail* me after the game? I plan* attending the m eeting. T h e first e x a m p le is in c o r r e c t b e c a u se it is n e c essa ry to say w ail f o r me. T h e s e c o n d e x a m ­ p le is in c o r r e c t b e c a u se it is n e c e ssa r y to say p la n on attending.

EX E R C ISE 57: P r e p o s itio n s h ave b e e n o m itte d in so m e o f th e fo llo w in g se n te n c e s . Mark w h er e p r e p o s itio n s have b e e n o m itte d . T h e n in d ic a te if th e s e n te n c e s are c o r r e c t (C) or in c o r r e c t (I). — !— C

1.

If you take this job , it will be necessary to deal Vother departments.

2.

Each child took o n e cookie from the plate.

3.

In the discussion, Rob sided the rest

4.

T h e board turned his suggestion for the project because it was too cosdy.

5.

H e can always d epend his friends.

6.

W hile Mrs. Sampson w ent shopping, a baby-sitter looked the children.

7.

I know Steve believes what you told him.

8.

Children should beware strangers.

9.

It was difficult to make a decision about buying a house.

10. Tom blam ed his brother the d en t in the car.

WRITTEN EXPRESSION

E X E R C I S E (S k ills 5 6 - 5 7 ): C ir c l e th e p re p o s itio n s in th e f o llo w in g s e n te n c e s . M a r k w h e r e th e y h a v e b e e n o m itt e d . T h e n in d ic a t e i f t h e se n te n c e s a r e c o r r e c t ( C ) o r i n c o r r e c t ( I ) .

_____

1.

T he students m ust hand in their homework.

_____

2.

It will be difficult to forgive you o f breaking your promise.

_____

3.

Elizabeth excels math and science.

_____

4.

She insisted on goin g to work in spite o f her cold.

_____

5.

Bob rem inds m e to his father because he looks just like him.

_____

6.

If you are cold, you should put on your sweater.

_____

7.

Mr. Sanders is n o t here now, but he will call you when he returns.

_____

8.

I do not want to interfere your plans.

_____

9.

Alan waited Marie after school.

_____

10.

Bill laughs m e w henever he looks me.

T O E F L E X E R C I S E (S k ills 5 6 - 5 7 ): C h o o s e th e le t te r o f t h e u n d e r li n e d w o r d o r g r o u p o f w o rd s th a t is n o t c o r r e c t .

____ 1

.

Amelia Earhart, the first woman to fly solo across the Atlantic, disappeared on June A B C 1937 while attempting to fly around the world. D

_____ 2.

The occurrence edem a indicates the presence of a serious illness. A B C D

_____ 3.

Atomic nuclei are believed to be composed by protons and neutrons in_ equal “A B C D numbers for the lighter elements.

_____ 4. _____ 5.

_____ 6.

According legend, Betsy Ross designed and sewed the first American flag. ----------- A B C D The middle ear is attached for the back of the throat by the Eustachian tube. A TT TT D Plants that sprout, grow, bloom, produce seeds, and die within one year are A B C classified for annuals. TT

_____ 7.

A marionette is controlled by means strings connected to wooden bars. A B C D

_____ 8.

In July of 1861, Pat Garrett killed Billy the Kid inji house close Fort Sum ner. X

"B

C

D

STRUCTURE AND WRITTEN EXPRESSION

_____ 9.

Manv comfort heating systems using steam as a working fluid operate at the ~K~ "B C D convection principle.

_____ 10.

Mars’ two small moons are irregularly shaped and covered for craters. A B 5 "IT

T O E F L REV IEW E X E R C ISE (1 -5 7 ): C h o o se th e le tte r o f th e w ord o r g r o u p o f w ord s that b e st c o m p le te s th e se n te n c e . 1. In any matter, heat tends to flow _____ to the cooler parts. (A) (B) (C) (D)

hotter parts there are hotter parts from the hotter parts toward the hotter parts

3. _____ Army camps near Washington, D.C., in 1861, Julia Ward Howe wrote “The Battle Hymn of the Republic.” (A) (B) (C) (D)

She visited After visiting When visited When was she visiting

2. Certain authorities claim that the costumes that people wear to parties___ into their personalities. (A) (B) (C) (D)

give subtle insights they give subtle insights which give subtle insights subtle insights

C h o o se th e le tte r o f th e u n d e r lin e d w ord o r g r o u p o f w ord s th a t is n o t c o r r e c t.

_____ 4.

The body depends in food as its primary source of energy. A "B T IT

_____ 5.

Regular programming was interrupted to broadcast a special news bulletins. “ A B C D

_____ 6.

Sulfa drugs had been used to treat bacterial infection until penicillin becom es widely ~A~ B C D available.

_____ 7.

Plans for both the International Monetary Fund or the World Bank were drawn up A IT C" D at the Bretton Woods Conference.

_____ 8.

Seldom Antarctic icebergs will move far enough north to disturb South Pacific A B C shipping lanes. D

WRITTEN EXPRESSION

_ 9.

In 1958, a largest recorded wave, with a height of 500 meters, occurred in Lituya "A B C D Bay, Alaska.

_10.

Exercise in swim ming pools is particularly helpful because of the buoyant A B C effect water. D

PROBLEMS W IT H USAGE. In E n g lish cer ta in g r o u p s o f w ord s h ave fu se d in th e W ritten E x p r e ssio n s e c tio n le m s are p o s sib le o n th e T O E F L test, (1) w h en to u se m ake a n d do; (2 ) w h e n

sim ila r u ses, a n d th e se w ord s are s o m e tim e s c o n ­ o f th e T O E F L test. A lth o u g h v a rio u s u sa g e p r o b ­ th e fo llo w in g p r o b le m s are th e m o s t c o m m o n : to u se like, unlike, a n d alike; a n d (3 ) w h e n to u se

other, another, a n d others.

S k ill

58:

D IS T IN G U IS H M AKE A N D DO

M ake an d do can b e c o n fu s e d in E n g lish b e c a u se th e ir m e a n in g s are so sim ilar. S in c e th e d iffe r e n c e b e tw e e n m ake a n d do is te ste d o n the T O E F L test, y o u s h o u ld le a r n to d istin ­ g u ish th em . M ake o fte n h as th e id e a o f creating or constructing. T h e fo llo w in g e x p r e s s io n s sh o w so m e o f th e p o s sib le u ses o f make: She likes to make her own clothes. Would you like to make a cake for dessert? If you make a. mistake, you should correct it. H e was unable to make a response to the threat. Do o fte n h as th e id e a o f com pleting o r perform ing. T h e fo llo w in g e x p r e s s io n s sh o w s o m e o f th e p o ssib le u se s o f do: This m orning she did all the dishes. T he students are doing the assignments. T h e janitors did the work they were assigned. You can do your laundry at the laundromat. T h e se are o n ly s o m e o f th e u ses o f m ake a n d do. M any u s e s o f m ake a n d do are id io m a tic a n d th e r e fo r e d iffic u lt to classify.

FXF.Rr.lSF. 58: E ach o f th e fo llo w in g s e n te n c e s c o n ta in s m ake or do. C ir cle m ake o r do. D raw arrow s to th e n o u n s th a t c o m p le te th e e x p r e ssio n s. T h e n in d ic a te i f th e s e n t e n c e s are c o r r e c t (C ) o r in c o r r e c t (I). J r L2.

1.

T he biology stu dent (did) several mistakes in the lab report.

^ ♦ I hop e that you will be able to (dg) m e a favor this afternoon.

STRUCTUREAND WRITTEN EXPRESSION

3.

N o matter what jo b she has, she always makes her best.

4.

The runner did a strong effort to increase her speed in the m ile race.

5.

It is com forting to think that your work can make a difference.

6.

His grade was not very good because he had not don e his homework.

7.

In this jo b you will make m ore m oney than in your previous job.

8.

H e was unable to do dinner because no o n e had d on e the lunch dishes.

9.

It is a pleasure to work with som eone who always makes the right thing.

10. If you make a good im pression at your jo b interview, you will get the job .

S k i l l 59:

D IS T IN G U IS H LIKE, ALIKE, UNLIKE, A N D D ISLIKE

Like, alike, unlike, a n d dislike are easily c o n fu s e d b e c a u se th e y lo o k so sim ila r a n d th e y have m an y d iffe r e n t u ses. T h e r e are several stru ctu res w ith like, alike, unlike, a n d dislike th a t you sh o u ld b e fa m ilia r w ith. T h e first str u c tu r es y o u sh o u ld already be fam iliar w ith are th e ad jectives alike a n d like (se e Skill 5 0 ). S tu d y th e u se o f alike an d like in th e fo llo w in g e x a m p les. John and Tom are alike. John and Tom worked in a like manner. In b o th th e se e x a m p le s , alike a n d like are ad jectiv es th a t m e a n similar. In th e first e x a m p le , alike is a p r e d ic a te ad jective d e sc r ib in g J o h n a n d Tom. B e ca u se alike is a p r e d ic a te adjec­ tive, it c a n o n ly b e u sed a fter a lin k in g verb su ch as are. In th e s e c o n d e x a m p le , like is th e ad jective fo r m th a t is u s e d im m e d ia te ly b e fo r e th e n o u n manner. T h e n e x t str u c tu r es y o u sh o u ld b e fam iliar w ith are th e p r e p o sitio n s like an d unlike, w h ic h h ave o p p o s ite m e a n in g s. B e ca u se they are p r e p o sitio n s, th e y m u st b e fo llo w e d by ob jects. John is (likeTom). John is (unlikeTom). In th e first e x a m p le , th e p r e p o sitio n like is fo llo w e d by th e o b je c t Tom. It m e a n s th a t Tom an d J o h n are sim ilar. In th e se c o n d e x a m p le , th e p r e p o sitio n u n lik e is fo llo w e d by th e o b ­ j e c t Tom. It m e a n s th at T o m an d J o h n are n o t sim ilar. T h e p r e p o s itio n s like a n d u n lik e can also b e u s e d at th e b e g in n in g o f a se n te n c e . (LikeTom ), John is tall. (U nlikeTom), John is tall. In th e first e x a m p le , th e p r e p o sitio n like is fo llo w e d by th e o b je c t Tom. It m e a n s th a t T om is tall. In th e s e c o n d e x a m p le , th e p r e p o sitio n u n lik e is fo llo w e d by th e o b je c t Tom. It m e a n s th a t T om is n o t tall. T h e fin a l str u c tu r es th a t y o u sh o u ld b e fa m ilia r w ith are th e verb s like a n d dislike, w h ic h h a v e o p p o s ite m e a n in g s. B e c a u se th ey are verb s, th ey are u s e d w ith su bjects. John and Tom like the course. John and Tom dislike the course. In th e first e x a m p le , th e verb like fo llo w s th e su b ject John a n d Tom. It m e a n s th at b o th m en th in k th a t th e c o u r se is en jo y a b le . In th e se c o n d e x a m p le , th e v e rb dislike fo llo w s th a t sub­ j e c t J o h n a n d Tom. It m e a n s th at b o th m e n th in k th at th e c o u r se is n o t e n jo y a b le .

WRITTEN EXPRESSION

T h e fo llo w in g c h a r t o u tlin e s th e stru ctu res a n d m e a n in g s o f s e n te n c e s w ith like, alike, unlike, a n d dislike: LIKE, ALIKE, UNLIKE, AND DISLIKE GRAMMAR

MEANING

USE

adjective adjective

similar similar

As an adjective, like is used before a noun. As an adjective, alike is used after a linking verb.

unlike

preposition preposition

similar different

Both prepositions are followed by objects.They can both be used in many positions, including at the beginning of the sentence.

like dislike

verb verb

enjoy not enjoy

Both verbs follow subjects.

like alike like

E X E R C ISE 59: E ach o f th e fo llo w in g s e n te n c e s c o n ta in s like, alike, unlike, o r dislike. C ircle th e ftfce w ords. T h e n in d ic a te i f th e s e n te n c e s are c o r r e c t (C ) o r in c o r r e c t (I ). 1 C

1.

The iwo routes you have chosen for the trip are (like.)

2.

T he scien ce books this sem ester are (like) the books used last sem ester.

3.

Alike the restaurant where we usually eat, this new restaurant has early-bird specials.

4.

U nlike the traditional red fire engines, the new fire engines are yellow.

5.

T he two girls disliked the fact that they were wearing alike dresses.

6.

The new piece that the pianist is preparing is unlike any she has ever played before.

7.

Like the W ashington Zoo, the San D iego Zoo has several panda bears.

8.

The insurance package offered by that company is exactly alike the package our com pany offers.

9.

Any further work done in a like fashion will be rejected.

10. S k ill

It is unfortunate that the covers for this year’s and last year’s albums are so dislike.

60:

D IS T IN G U IS H OTHER, ANOTHER, A N D O THERS

Other, another, a n d others are very easy to c o n fu s e . T o d e c id e h o w to u se e a c h o f t h e m correcdy, y o u m u st c o n s id e r th r e e th in gs: (1 ) i f it is sin g u la r o r p lu ral, (2 ) i f it is d e fin it e (the) o r in d e fin ite (a ) , a n d (3) i f it is a n ad jective (it ap p ears w ith a n o u n ) o r i f it is a p r o n o u n (it ap p ea rs by it s e lf ) . SINGULAR INDEFINITE

DEFINITE

PLURAL

1 have another book.

1 have other books.

1 have another.

1 have others.

1 have the other book.

1 have the other books.

1 have the other.

1 have the others.

N o tic e th a t y o u u s e another o n ly to refer to an in d e fin ite , sin g u la r id ea . Others is u s e d o n ly as a p lu ral p r o n o u n ( n o t a c c o m p a n ie d by a n o u n ). In all o th e r c a ses, other is c o r r e c t.

STRUCTURE AND WRITTEN EXPRESSION

E X E R C ISE 60: E ach o f th e fo llo w in g s e n te n c e s c o n ta in s other, another, o r others. C ircle other, another, o r others. T h e n in d ic a te i f t h e s e n te n c e s are c o r r e c t (C ) o r in c o r r e c t (I). ^

1. It is essential to com plete the first program before working on the (o th ers)

__ !__

2.

The waitress will bring you (th e another) bowl o f soup if you want.

_____

3.

You should pack another pair o f shoes in case that pair gets soaked.

_____

4. It is difficult to find others workers who are w illing to work such lo n g hours.

_____

5.

Since the lamp you wanted is out o f stock, you m ust choose another.

____ _

6.

T he other desk clerk must have put that message in your mailbox.

_____

7.

If your identification card is lost or stolen, you cannot g et another.

_____

8. Because they were n ot pleased with the hotel accom m odations last year, they have decided to try a other hotel this year.

_____

9.

_____

10.

As som e students m oved into the registration area, others took their places in line. The printer will not function unless it has another cartridges.

E X E R C ISE (S k ills 5 8 - 6 0 ) : C ircle th e w o r d s in th e fo llo w in g s e n te n c e s th a t are c o m m o n ly c o n fu s e d o n th e T O E F L test. T h e n in d ic a te i f th e s e n te n c e s are c o r r e c t (C) o r in c o r r e c t (I).

_____

1.

W hen the car’s odom eter reached 100,000, she decided that it was time to buy another car.

_____

2.

Every time som eon e does an error in the program, several extra hours o f work are created.

_____

3.

Like the fashions shown in this m agazine, the fashions in the other m agazine are quite expensive.

_____

4.

Because the main highway is crowded at this hour, the driver should try to find another routes to the stadium.

_____

5.

Although the two signatures are supposed to be exactly the same, they are n ot at all like.

_____

6.

T h e decorators did the shopping for the material and m ade curtains for the windows.

_____

7.

Before the administrator reads the stack o f papers on his desk, he should sign the others that are on the file cabinet.

_____

8.

T he com m ittee is d oin g the arrangem ents for the Saturday evening banquet.

_____

9.

W hen he m ade several other big mistakes, he did his apologies to the others in the office.

_____

10.

Perhaps the designer could select others styles if these are inappropriate.

WRITTEN EXPRESSION

T O E F L E X E R C ISE (S k ills 5 8 - 6 0 ) : C h o o s e th e le tte r o f th e u n d e r lin e d w o r d o r g r o u p o f w ord s th at is n o t c o rr ec t.

____ 1.The buffalo and the bison are like except for the size and shape of the head and X B C TT shoulders.

_____ 2.

Other interesting aspect of tachistopic training in recent years has been the —S B C newfound use by professional teams. TT

_____ 3.

Only about 3 percent of oil wells actually do a profit. A B C

~D

_____ 4.

Dislike sumac with red berries, sumac with white berries is poisonous. A C D

_____ 5.

Pittsburgh has reduced its sm og by requiring more complete oxidation of fuel in X B cars, and others cities can do the same thing. C TT

____ 6.

Alike all other mammals, dolphins have lungs. "A B C~ ~T5~

_____ 7.

Up to World War II almost all important research in physics had been made in ~ A B universities, with only university funds for support. C D

_____ 8.

Because the plan that was made yesterday is no longer feasible, the manager had to A B C choose another alternatives. D~~ '

_____ 9.

Particles w ith unlike charges attract each other, while particles with alike charges A ~B~' ~~C~ repel each other.

_10.

One another surprising method of forest conservation is controlled cutting o f trees. X" B " "C D

257

258

STRUCTURE AND WRITTEN EXPRESSION

T O E F L REV IEW E X ER C ISE (S k ills 1 -6 0 ): C h o o se th e le tte r o f th e w o r d or g r o u p o f w ord s th^ t b e st c o m p le te s th e s e n te n c e . 1. Wild Bill H ickok_____ for the Union Army during the Civil War by posing as a Confederate officer. (A) (B) (C) (D)

spied spying a spy was spied

2. --------was unusable as farmland and difficult to traverse, the Badlands is an area in South Dakota. (A) (B) (C) (D)

So named because it Because of It Naming it

C h o o se th e le tte r o f th e u n d e r lin e d w ord o r g r o u p o f w ord s th at is n o t co rr ec t.

_____ 3.

Titania, photographed by Voyager 2 in 1986, has significantly fewer craters than A “B “ C another moons of Uranus.

4.

The author Francis Scott Key Fitzgerald is better know as F. Scott Fitzgerald. A B C- "D

5.

The result of the failure to plan for the future is that a child from an urban area A B must be took to the country to see nature. — D

6.

This machine can print on a single pieces of paper, but only if the level is facing the B C D front of the machine.

_ 7.

The development of permanent teeth, alike that of deciduous teeth, begins before A B~ C birth .

.8 .

A crowd o f several hundred fan watched the ceremony from behind a fence. A 13 C D

_ 9. Unlike other architects of the early modem movement, Alvar Aalto stressed A Binformality, personal expression, romantic, and regionality in his w ork. C D _10. Color blindness may exist at birth or may occur later in life as a result for disease or A “ B~ C D in ju ry .

2 9 2 » 2 * 2 » 2 * 2 » 2»2 TOEFL POST-TEST SECTION 2 STRUCTURE AND WRITTEN EXPRESSION l i m e — 25 m in u t e s ( i n c l u d i n g t h e r e a d in g o f t h e d ir e c t i o n s ) N o w s e t y o u r c lo c k f o r 25 m in u t e s .

This section is designed to measure your ability to recognize language that is appropriate for standard written English. There are two types of questions in this section, with special directions for each type.

S tru c tu re D irections: These questions are incomplete sentences. Beneath each sentence you will see four words or phrases, marked (A), (B), (C), and (D). Choose the one word or phrase that best com pletes the sentence. Then, on your answer sheet, find the number of the question and fill in the space that corresponds to the letter of the answer you have chosen. Look at the following examples. Sam p le Answer

Exam ple I The president_____ the election by a landslide (A) (B) (C) (D)

won he w on yesterday fortunately

©

The sentence should read, "The president won the election by a landslide." Therefore, you should choose answer (A). Exam ple II

S am p le Answer

W hen_____ the conference? (A) the doctor attended (B) did the doctor attend (C) the doctor will attend (D) the doctors attendance

©

The sentence should read, “When did the doctor attend the conference?" Therefore, you should choose answer (B).

I GO ON TO THE NEXT PAGE TOEFL* test directions and form at are rep rin ted by perm ission of ETS, the copyright owner. However, ali exam ples and test questions are provided by P earson E ducation, Inc.

r STRUCTURE AND WRITTEN EXPRESSION POST-TEST

259

2

*

2

*

2

*

2

. rotations during 1. The planet Mercury. every two trips around the Sun. (A) (B) (C) (D)

three complete completes three the completion of three completing three of the

2

*

3.

(A) (B) (C) (D)

(A) (B) (C) (D) 5.

(A) (B) (C) (D)

dense and solar density, solar activity density, but solar activity density and activity of the Sun is

. stone, has been valued Lapis lazu li,. for ornamental purposes for more than 6,000 years. (A) (B) (C) (D)

an opaque deep blue is an opaque deep blue it is an opaque deep blue that is an opaque deep blue

(A) (B) (C) (D)

260

is known as it is known to be known as to be known

give given are given they are given

they buffer that buffer to buffer them that they buffer

10. It is at the age of approximately eighteen m onth s_____children begin to make combinations of two or three words. (A) (B) (C) (D)

when when when when

many are many do many have many of the

11. Story o f a Bad Boy, a semiautobiographical novel by Thomas Bailey Aldrich, ranks high among books_____ have incorporated their boyhood experiences.

_climb Mount Everest 6. M ountaineers. must make reservations to do so, often up to seven years in advance. (A) (B) (C) (D)

2*2

9. North Carolina's Outer Banks are a chain of low, narrow islands_____ the mainland from the frequent Atlantic storms in the area.

flight because of the flying the the flying of the flight because the

The upper levels of the Sun's atmosphere are of very lo w _____ heats the gases there to very high temperatures.

*

8. Most slang terms are simply old words additional new meanings.

times, a large part times, there was a large part part of the time for large parts of time

The helicopter is able to hover in . powered rotors produce lift even at zero forward speed. (A) (B) (C) (D)

2

7. Created by the dissolution of limestone, the underground cave system _____ Mammoth Cave is noted for its stalactites and stalagmites.

2. In prehistoric. . of western Utah was covered by Lake Bonneville. (A) (B) (C) (D)

*

want to they want to who want wanting to

STRUCTURE AND WRITTEN EXPRESSION POST-TEST

(A) (B) (C) (D)

the American authors which are American authors in which American authors are those which American authors

2 • 2 ^ 2 ^ 2 ^ 2 • 2 ^ 2 •2 12. In the La Brea tar pits of Los A ngeles------which have been preserved from the Pleistocene period. (A) (B) (C) (D)

thousands of animals are thousands are animals the thousands of animals are thousands of animals

13. _____provided aliv in g fo rn ea rly 9 0 percent of the population of the American colonies. (A) (B) (C) (D)

Farming was what What farming Farming was What was farming

14. Not o n ly _____ more brittle than hard maples, but they are also less able to withstand high winds. (A) (B) (C) (D)

soft maples are are soft maples they are soft maples soft maples

15. _____ become blocked so that heat and moisture could not escape, death would result. (A) (B) (C) (D)

Were the skin’s pores to The pores of the skin were to The skin’s pores If the pores of the skin

STRUCTURE AND WRITTEN EXPRESSION POST-TEST

26 I

2

*

2

»

2

«

2

«

2

»

2

*

2»2

W ritten Expression Directions: In these questions, each sentence has four underlined words or phrases. The four underlined parts of the sentence are marked (A), (B), (C), and (D). Identify the one underlined word or phrase that must be changed in order for the sentence to be correct. Then, on your answer sheet, find the number of the question and fill in the space that corresponds to the letter of the answer you have chosen. Look at the following examples. Exam ple I

Sam ple Answer

® The four string on a violin are tuned ~ B ~C D~ in fifths.

© (D)

The sentence should read, "The four strings on a violin are tuned in fifths." Therefore, you should choose answer (B). Exam ple II

Sam ple Answer

® The research for the book Roots taking \ BC

(B) #

Alex Haley twelve years. D

®

The sentence should read, "The research for the book Roots took Alex Haley twelve years." Therefore, you should choose answer (C).

~

_

262

____ ______ _____

___

STRUCTURE AND \WRITTEN EXPRESSION POST-TEST

TOEFL* test directions and form at arc reprinted by perm ission o f ETS, the copyright owner. However, all exam ples and test questions arc provided by Pearson E ducation. Inc.

16. The wave lengths of ultraviolet light are short than those of visible light A B C

but longer D

than those of X-rays. 17

All thoroughbreds are descended from three Arabian stallion imported into England X B £ D between 1689 and 1724.

18.

By measuring the rate of decay of potassium isotopes in volcanic ash, scientists A can date the lavers of volcanic ash and any human remains in they. -----B----C D

19.

Hundreds of partial to complete fossil skeletons of Triceratops have been gather in A B C North America from rocks of the late Cretaceous period. D

20.

By the time of the dinosaurs, turtles have already developed the hard shell A B into which their heads and legs could be drawn. ------ C-----D

21.

A zoom lens produces an inverted real image,either on the film in a camera and on A B C D the light-sensitive tube of a television camera.

22

The leaves and young twigs of the henna plant are ground into a powder to produce a A B paste that can used as a dye. “ C— D

23.

Thirty-one pairs of spinal nerves arepresent in humans, and each pair have A B C D

two

roots 24

William Randolph Hearst built a chain of newspapers that included 25 dailies and 11 ~ B C Sunday editions at their peak in 1937. D

25

The electromagnetic spectrum consists in bands o f different wavelengths. A B C D

E

E

*

STRUCTURE AND W RITTEN EXPRESSION POST-TEST

2

»

2

»

2

»

2

»

2

*

2

*

2

»

rz

26.

Lemon trees are similar in longevity and appear to orange trees but have more upright A B C growth. D

27.

Christopher Columbus, alike many other explorers, underestimated the size of the A “T T C~ Earth and overestimated the width of Asia. D

28.

Manganese, found in trace amounts in higher animals, activates a large amount of A B the enzymes involved in metabolic processes. C D

29.

The remains of Homo erectus, an extinct species of early man, was first discovered X B "TT D on the island of Java by Dutch physician Eugene Debois.

30.

The Ford Motor Company introduced the moving assembly line in 1914 so that it will A TT be able to meet the huge demand for its Model T. ' C "D

31.

By 1830, approximately 200 steamboats had become operationally on the A B C D Mississippi River.

32.

The huge Meteor Crater was created when a 63,000-ton A B Earth near Winslow, Arizona.

33.

Daniel Boone helped to build the Wilderness Road through the Cumberland Gap, A B creating a route for settlers heading westerly. -----D----C

34.

The Appalachian Mountains extend Georgia and Alabama in the south to Canada in A B C "D the north.

35.

Howard Hughes once did more than half a billion dollars in one day in 1966 when he A B received a single bank draft for $546,549,171 for his share of TWA. C D-----

iron meteorites struck the C D

9 / 264

STRUCTURE AND WRITTEN EXPRESSION POST-TEST

2 * 2 » 2 * 2 * 2 * 2 » 2»2 36.

The city of Tam pa, Florida, is located on peninsula across Tam pa Bay from S aint A B C “ D“ Petersburg.

37.

The closer it gets to D ecem ber 21, the first day of winter, the short the davs becom e A B ~ C ~ D

38.

Only about a hundred o u t of an estim ating 3,000 know n m ineral species A have been found at least reasonably suitable for use as gems B C------~ D ~

39.

Most of the y ear San Miguel Island is shrouded in fog, and strong northw est w inds A B b a tte r relentlessly the island. C D

40.

W omen have adm itted to the United States M ilitary Academy a t West Point since 1976, and the first w om en cadets graduated in 1980. C D

This is the end of Section 2. If you finish before 25 minutes has ended, check your work on Section 2 only.

(s t o p )

(s t o

p)

(s t o

p

)

(s to p ]

(s t o

p)

0

( 0

)

W h e n y o u fin ish th e test, y o u m ay d o th e fo llo w in g : * T u rn to th e D ia g n o s tic C h a r t o n p a g e s 5 8 5 -5 8 7 , a n d c ir c le th e n u m b e r s o f th e q u e s tio n s th a t y o u m isse d . * T u rn to S c o r in g I n fo r m a tio n o n p a g e s 5 8 1 -5 8 2 , a n d d e te r m in e y o u r T O E F L sc o r e. * T u rn to th e P r o g r e s s C h a r t o n p a g e 5 9 1 , a n d ad d y o u r sc o r e to th e chart.

STRUCTURE AND WRITTEN EXPRESSION POST-TEST

26S

266

SECTION THREE

READING COMPREHENSION

268

3

a

3

3

a

3

a

a

3

a

3

a

3

a

3

DIAGNOSTIC PRE-TEST SECTION 3 READING COMPREHENSION T im e — 55 m in u t e s ( i n c l u d i n g t h e r e a d in g o f t h e d ir e c t i o n s ) N o w s e t y o u r c lo c k f o r 55 m in u t e s .

This section is designed to m easure y our ability to read and understand sh o rt passages sim ilar in topic and style to those that students are likely to encounter in N orth Am erican universities and colleges. This section contains reading passages and questions about the passages. D irectio n s: In this section you will read several passages. Each one is followed by a n u m b er of questions about it. You are to choose the o n e best answer, (A), (B), (C), o r (D), to each question. Then, on your answ er sheet, find the num ber o f the question and fill in the space that corresponds to the letter of the answ er you have chosen. Answer all questions about the inform ation in a passage on the basis o f w hat is s ta te d o r im p lie d in that passage. Read the following passage: John Quincy Adams, who served as the sixth president of the United S tates from 1825 to 1829, is today recognized for his m asterful statesm anship and diplomacy. He dedicated his life to public service, b oth in the presidency and in the various ot.'-er political offices th at he Line held. T hroughout hi ; political career he dem onstrated hie unsw ervi ig belief in freedom of (5) speech, the antislave -y cause, uik 1 the l ight of A m ericans u; be fre from E u ro p ean and Asian dom ination. E x am p le I

Sam ple Answer

To w hat did John Quincy Adams devote hi ; life?

® •(E )®

(A) Im proving his personal life (B) Serving the public (C) Increasing his fortune (D) W orking on his private business According to the passage, John Quincy Adams "dedicated his life to public service.” T herefore, you should choose answ er (B). E x am p le II

Sam ple Answer

In line 4, the w ord "unswerving” is closest in m eaning to (A) (B) (C) (D)

m oveable insignificant unchanging diplom atic

The passage states th a t John Quincy Adams dem onstrated his unsw erving belief “thro u g h o u t his career.” This im plies th at the belief did not change. Therefore, you should choose answ er (C).

______________ k GO ON TO THE NEXT PAGE TOEFL® test directions and form at are reprinted by perm ission of ETS. th e copyright owner. However, all exam ples and test questions are provided by P earson Education. Inc.

READING CO M PREH ENSIO N PRE-TEST

269

3

a

3

a

3

a

3

a

3

a

3

a

3

a

3

Q u e s t io n s 1 - 1 2

Line (5)

( 10 )

(15)

Algae is a prim itive form of life, a single-celled o r sim ple m ultiple-celled organism th at is able to conduct the process of photosynthesis. It is generally found in w ater but can also be found elsewhere, growing on such surfaces as rocks o r trees. The various types of algae are classified accoiding to their pigm entation, o r coloration. Blue-green algae, o r Cyanophyta, can grow at very high tem p eratu res and u n d e r high-intensity light. This is a m icroscopic type of algae, a n d some species consist of only one cell. Blue-gree algae is the oldest form of life w ith photosynthetic capabilities, and fossilized rem ains of this type of algae m ore th an 3.4 billion years old have been found in p a rts of Africa. Green algae, or Chlorophyta, is generally found in fresh w ater. It reproduces on the surfaces of enclosed bodies of w ater such as ponds o r lakes and has the appearance of a fuzzy green coating on the water. In large quantities, this type of algae may reproduce enough to give a green color to an entire lake. . Brown algae, or Phaeophyta, grows in shallow, tem perate water. This type of algae is the largest in size and is m ost recognizable as a type of seaweed; kelp is a type of brow n algae th at has grown to lengths of up to 200 feet. Its long stalks can be enm eshed on the ocean floor, o r it can float freely on the ocean’s surface. Red algae, or Rhodophyta, is a sm all, delicate organism found in the deep w aters of the subtropics, w here it often grows w ith coral. This type of algae has an essential role in the form ation of coral reefs: it secretes lim e from the seaw ater to foster the form ation of lim estone deposits. 1. What is the authors main purpose? (A) To show what color algae is (B) To differentiate the various classifications of algae (C) To describe where algae is found (D) To clarify the appearance of the different types of algae

5. Algae rem nants found in Africa are (A) (B) (C) (D) 6.

(A) (B) (C) (D)

All types have only one cell. It can be found out of water. It can use photosynthesis. It is not a relatively new form of life.

size shape composition color

4. The word "microscopic” in line 6 is closest in meaning to (A) (B) (C) (D)

mechanical tiny visual bacterial

on the ocean floor on top of the w ater throughout ponds and lakes surrounding enclosed bodies of w ater

7. The word "coating” in line 10 could best be replaced by (A) (B) (C) (D)

3. The word "pigmentation" in line 4 means (A) (B) (C) (D)

G reen algae is generally found (A) (B) (C) (D)

2. Which of the following is NOT true about algae?

still flourishing photogenic extrem ely old red in color

8.

clothing covering w arm th sw eater

B row n algae w ould m ost likely be found (A) (B) (C) (D)

on trees near green algae on rocks in the ocean

y 270

READING COMPREHENSION PRE-TEST

mam»

3

a

3

a

3

a

3

9. According to the passage, red algae is (A) (B) (C) (D)

sturdy huge fragile found in shallow w ater

10. It can be inferred from the passage that lim estone deposits serve as the basis of (A) (B) (C) (D)

coral reefs red algae subtropical seaw ater secret passages

a

3

a

3

a

3

a

3

11. How is the inform ation in the paragraph organized? (A) Various details supporting a theory are explored. (B) Various classifications of a specific life form are described. (C) Various stages of the chronological developm ent of a life form are presented. (D) Various elem ents th a t com pose a certain life form are outlined. 12. This passage w ould m ost probably be assigned reading in a course on (A) (B) (C) (D)

chem istry physics botany zoology

READING COMPREHENSION PRE-TEST

271

3

a

3

a

3

a

3

a

3

a

3

a

3

a

3

Q u e s t io n s 1 3 -2 1

Line (5)

( 10)

(15)

(20 )

Narcolepsy is a disease characterized by m alfunctioning sleep m echanics. It can consist of a sudden and uncontrollable bout of sleep during daylight hours and disturbed sleep during nighttim e hours. It occurs m ore often in m en th an in w om en, and it com m only m akes its appearance during adolescence o r young adulthood. At least a half m illion Am ericans are believed to be affected by N arcolepsy can take a n um ber of form s during daylight hours. One com m on sym ptom of the disease during daytim e hours is a sudden attack of REM (rapid-eye m ovem ent) sleep during norm a, waking hours. This occurs in som e people hundreds of tim es in a single day, w hile o thers only have rare occurrences. D uring a sleep attack, narcoleptics may experience autom atic behavior; even though asleep, they m ay continue autom atically perform ing the activity they w ere involved in p rio r to falling asleep. They may, for exam ple, continue walking, or driving, o r stirring a pot until the activity is interrupted by external forces. Others experience cataplexy during daytim e hours; cataplexy involves a sudden loss of muscle tone th a t m ay cause the head to droop o r the knees to wobble m m in o r attacks o r a total collapse in m ore serious attacks. Cataplexy seem s to occur m ost often in conjunction w ith intense em otion or excitem ent. D uring sleep hours, narcolepsy can also m anifest itself in a variety of ways. During the transitional phase th at precedes the onset of sleep, it is com m on for hallucinations to occur. These hallucinations, known as hypnagogic phenom ena, consist of realistic perceptions of sights and sounds during the sem i-conscious state betw een w akefulness and sleep. N arcoleptics m ay also suffer from night w akening during sleep, resulting in extremely fragm ented and restless sleep. Then, upon waking, a narcoleptic may experience sleep paralysis, the inability to move, perhaps for several m inutes, im m ediately after waking. 13. W hich of the following would be the m ost appropriate title for the passage? (A) (B) (C) (D)

A G ood N ight’s Sleep A Cure for Narcolepsy An U nusual Sleep D isturbance H allucinations during Sleep

14. The w ord "m alfunctioning" in line 1 is closest in m eaning to (A) (B) (C) (D)

im properly working regularly waking incorrectly classifying harshly inte,preting

15. At w hich of the following ages w ould a person be m ost likely to develop narcolepsy? (A) (B) (C) (D)

272

10 20 30 40

READING COMPREHENSION PRE-TEST

16. Approxim ately how m any narcoleptics are there in the United States? (A) (B) (C) (D)

Fewer than 500,000 More th an 500,000 Fewer than 1,500,000 More th an ! ,500,000

17. The word "bout” in line 2 is closest in m eaning to (A) sym ptom (B) lack (C ) illness (D) period 18. W hich of the following w ould be m ost likely to occur during daily activities? (A) (B) (C) (D)

Autom atic behavior Hallucinations Night wakening Sleep paralysis

3

a

3

a

3

a

3

a

3

a

3

a

3

a

3

19. Which of the following involves a complete 21. Where in the passage does the author collapse? describe what seems to precipitate a sudden loss of muscle tone? (A) Automatic behavior (B) Cataplexy (A) Lines 12-14 (C) Hallucinations (B) Lines 14—15 (D) REM sleep (C) Lines 16-17 (D) Lines 20-22 20. When would hypnagogic phenomena most likely occur? (A) (B) (C) (D)

Just after going to bed In the middle of the night Soon after waking After getting up

READING COMPREHENSION PRE-TEST

273

3

a

3

a

3

a

3

a

3

a

3

a

3

a

3

Questions 22-30

Line (5)

( 10)

(1 5 )

( 20 )

Whereas literature in the first half of the eighteenth century in America had been largely religious and moral in tone, by the latter half of the century the revolutionary fen/or that was coming to life in the colonies began to be reflected in the literature of the time, which in turn served to further influence the population. Although not all writers of this period supported the Revolution, the two best-known and m ost influential writers, Ben Franklin and Thomas Paine, were both strongly supportive of that cause. . . . . » u Ben Franklin first attained popular success through his writings in his brothers newspaper, the New-England Courant. In these articles he used a simple style of language and commonsense argumentation to defend the point of view of the farmer and the Leather Apron man. He continued with the same commonsense practicality and appeal to the common man with his work on Poor Richard's Almanac from 1733 until 1758. Firmly established in his popular acceptance by the people, Franklin wrote a variety of extremely effective articles and pamphlets about the colonists revolutionary cause against England. . . . . , . f Thomas Paine was an Englishman working as a magazine editor in Philadelphia at the time of the Revolution. His pamphlet Common Sense, which appeared in 1776, was a force in encouraging the colonists to declare their independence from England. Then throughout the long and desperate war years he published a series of Crisis papers (from 1776 until 1783) to encourage the colonists to continue on with the struggle. The effectiveness of his writing was probably due to his emotional yet oversimplified depiction of the cause of the colonists against England as a classic struggle of good and evil. 22. The paragraph preceding this passage most likely discusses how literature influences the population (B) religious and moral literature (C) literature supporting the cause of the American Revolution (D) what made Thomas Paine’s literature successful

(A)

The word "fervor” in line 2 is closest in meaning to (A) (B) (C) (D)

war anxiety spirit action

The word "time” in line 3 could best be replaced by (A) (B) (C) (D)

hour period appointment duration

25. It is implied in the passage that (A) som e writers in the American colonies supported England during the Revolution (B) Franklin and Paine were the only writers to influence the Revolution (C) because Thomas Paine was an Englishman, he supported England against the colonies (D) authors who supported England did not remain in the colonies during the Revolution 26. The pronoun "he" in line 8 refers to (A) (B) (C) (D)

Thomas Paine Ben Franklin Ben Franklin’s brother Poor Richard

27. According to the passage, the tone of Poor R ichard’s Almanac is (A) (B) (C) (D)

pragmatic erudite theoretical scholarly

— 274

READING COMPREHENSION PRE-TEST



3

a

3

a

3

a

3

The word "desperate" in line 16 could best be replaced by (A) (B) (C) (D)

unending hopeless strategic combative

Where in the passage does the author describe Thomas Paine’s style of writing? (A) (B) (C) (D)

a

3

a

3

a

3

a

3

30. The purpose of the passage is to (A) discuss American literature in the first half of the eighteenth century (B) give biographical data on two American writers (C) explain which authors supported the Revolution (D) describe the literary influence during revolutionary America

Lines 4-6 Lines 8-9 Lines 14-15 Lines 18-20

READING COMPREHENSION PRE-TEST

275

3

a

3

a

3

a

3

a

3

a

3

a

3

a

3

Questions 3 1-4 1

Line (5)

(10)

(15)

(20)

(25)

Federal Express is a company that specializes in rapid overnight delivery o f high-priority packages. The first company of its type, Federal Express was founded by the youthful Fred Smith in 1971, when he was only 28 years old. Smith had actually developed the idea for the rapid delivery service in a term paper for an economics class when he was a student at Yale University. The term paper reputedly received a less-than-stellar grade because of the infeasibility of the project that Smith had outlined. The model that Smith proposed had never been tried; it was a model that was efficient to operate but at the same time was very difficult to institute. Smith achieved efficiency in his model by designing a system that was separate from the passenger system and could, therefore, focus on how to deliver packages most efficiently. His strategy was to own his own planes so that he could create his own schedules and to ship all packages through the hub city of Memphis, a set-up which resembles the spokes on the wheel of a bicycle. With this combination of his own planes and hub set-up, he could get packages anywhere in the United States overnight. What made Smith’s idea difficult to institute was the fact that the entire system had to be created before the company could begin operations. He needed a fleet of aircraft to collect packages from airports every night and deliver them to Memphis, where they were immediately sorted and flown out to their new destinations; he needed a fleet of trucks to deliver packages to and from the various airports; he needed facilities and trained staff all in place to handle the operation. Smith had a $4 million inheritance from his father, and he managed to raise an additional $91 million dollars from venture capitalists to get the company operating. When Federal Express began service in 1973 in 25 cities, the company was not an immediate success, but success did come within a relatively short period of time. The company lost $29 million in the first 26 months of operations. However, the tide was to turn relatively quickly. By late 1976, Federal Express was carrying an average of 19,000 packages per night and had made a profit of $3.6 million. . 31. The most appropriate title for this passage is (A) The Problems and Frustrations of a Business Student (B) The Importance of Business Studies (C) The Capitalization of Federal Express (D) The Implementation of a Successful Business The word "developed” in line 3 could best be replaced by (A) (B) (C) (D)

come come come come

up with about across into

33. What is stated in the passage about Smith's term paper? Smith submitted it through a deliver service. (B) It was written by a student of Smith'« (C) Its grade was mediocre. (D) The professor thought it had great potential. (A)

34. What was a key idea of Smith’s? (A) That he should focus on passenger service (B) That package delivery should be separate from passenger service (C) That packages could be delivered on other companies' planes (D) That passenger service had to be efficient

GO ON TO THE NEXT PAGE

7 276

READING COMPREHENSION PRE-TEST

3a 3a 3a 3a 3a 3a 3a 3 35. A “hub city” in line 11 is (A) a large city with small cities as destinations (B) a city that is the final destination for many routes (C) a city where many bicycle routes begin (D) a centralized city with destinations emanating from it 36. It can be inferred from the passage that Smith selected Memphis as his hub city because it (A) was near the middle o f the country (B) had a large number of passenger aircraft (C) already had a large package delivery service (D) was a favorite passenger airport 37. The pronoun "they" in line 16 refers to (A) (B) (C) (D)

39. How long did it take Federal Express to become profitable? (A) (B) (C) (D)

TWo months One year Three years Six years

40. Which paragraph explains what made Sm ith’s model effective? (A) (B) (C) (D)

The The The The

first paragraph second paragraph third paragraph last paragraph

41 The tone of the passage in describing Smith’s accom plishm ents is (A) (B) (C) (D)

unflattering sincere unconvincing snobbish

aircraft packages airports destinations

38. It is NOT mentioned in the passage that, in order to set up his company, Smith needed (A) (B) (C) (D)

airplanes trucks personnel faculty

READING COMPREHENSION PRE-TEST

277

3 A3 A 3

a

3

a

3

a

3

a

3

a

3

Q u e s t io n s 4 2 - 5 0

Line (5)

(10)

(15)

(20)

(25)

Perhaps better known than the Cullman Diamond is the Hope Diamond, a valuable and rare blue gem with a background of more than 300 years as a world traveler. The 112-carat blue stone that later became the Hope Diamond was mined in India sometime before the middle of the seventeenth century and was first known to be owned by Shah Jahan, who built the Taj Mahal in memory of his beloved wife. From India, the celebrated blue stone has changed hands often, moving from location to location in distant com ers of the world. In the middle of the seventeenth century, a trader from France named Jean Baptiste Tavemier acquired the large blue diamond, which was rumored to have been illegally removed from a temple. Tavemier returned to France with the big blue gem, where the stone was purchased by the Sun King, Louis XIV. Louis XIV had it cut down from 112 to 67 carats to make its shape symmetrical and to maximize its sparkle. The newly cut diamond, still huge by any standards, was passed down through the royal family of France, until it arrived in the hands of Louis XVI and Marie Antoinette. During the French Revolution, Louis XVI and his wife met their fate on the guillotine in 1793, and the big blue diamond disappeared from public sight. The diamond somehow managed to get from France to England, where banker Henry Hope purchased it from a gem dealer early in the nineteenth century. The huge blue stone was cut into a 45.5-carat oval, and at this point it took on the name by which it is known today. The diamond stayed in the Hope family for around a century, when deep indebtedness brought on by a serious gambling habit on the part of one of Henry Hope’s heirs forced the sale of the diamond. From England, the Hope Diamond may have made its way into the hands of the Sultan of Turkey; whatever route it took to get there, it eventually went on to the United States when American Evelyn Walsh McLean purchased it in 1911. Mrs. McLean certainly enjoyed showing the diamond off; guests in her home were sometimes astounded to notice the huge stone embellishing the neck of Mrs. McLean's Great Dane as the huge pet trotted around the grounds of her Washington, D.c. home. The Hope Diamond later became the property o f jeweler Harry Winston, who presented the stunning 45.5carat piece to the Smithsonian in 1958. The Hope Diamond is now taking a well-earned rest following its rigorous travel itinerary and is on display at the Smithsonian Institution in Washington, D.C., where it has been since 1958. 42. The paragraph preceding the passage m ost likely discussed (A) why gems are considered valuable (B) how the Hope Diamond was mined (C) a diamond other than the Hope Diamond (D) methods for mining diamonds 43. The main idea of this passage is that the Hope Diamond (A) came from India (B) has moved around a lot (C) has been cut several times (D) now resides in the Smithsonian

278

READING COMPREHENSION PRE-TEST

44. The pronoun "it” in line 12 refers to (A) (B) (C) (D)

its shape the newly cut diamond the royal family the French Revolution

45. It can be inferred from the passage that the author is not certain (A) who bought the Hope Diamond in England (B) who sold the Hope Diamond in England (C) how the Hope Diamond went from France to England (D) how big the Hope Diamond was in the nineteenth century

3

a

3

a

3

a

3

46. A “dealer" in line 16 is most likely a (A) (B) (C) (D)

card player miner cutter businessman

47. It can be determined from the passage that Henry Hope most likely had how many carats cut off the Hope Diamond? (A) (B) (C) (D)

21.5 45.5 66.5 67

a

3

a

3

a

3

a

3

49. Which country is NOT m entioned in the passage as a place where the Hope Diamond spent some time? (A) (B) (C) (D)

India France England Denmark

50. Where in the passage does the author describe what happened to the royal French owners of the diamond?

48. According to the passage, Mrs. McLean

(A) (B) (C) (D)

Lines 7-8 Lines 10-11 Lines 12-14 Lines 15-16

(A) donated the Hope Diamond to the Smithsonian (B) let her dog wear the Hope Diamond (C) purchased the Hope Diamond from the French (D) had the Hope Diamond cut to its present size of 45.5 carats

This Is the end of the Reading Comprehension Pre-Test.

W h e n y o u fin ish th e test, y o u m ay d o th e fo llo w in g : • T u r n to th e D iagnostic Chart o n p a g e s 5 8 8 - 5 9 0 , a n d c ir c le th e n u m b e r s o f th e q u e s tio n s th a t y o u m issed . • T u r n to Scoring In form ation o n p a g e s 5 8 1 -5 8 2 , a n d d e te r m in e you r T O E F L sco re. • T u r n to th e P rogress Chart o n p a g e 5 9 1 , a n d ad d y o u r sc o r e to th e chart.

READING COMPREHENSION PRE-TEST

279

280

READING COMPREHENSION T h e th ird se c tio n o f th e T O E F L test is th e R ea d in g C o m p r e h e n s io n s e c tio n . T h is s e c tio n c o n sists o f fifty q u e s tio n s (so m e tests m ay b e lo n g e r ). You h ave fifty-five m in u te s to c o m ­ p le te th e fifty q u e s tio n s in th is se c tio n . In this part o f th e test y o u w ill b e giv en r e a d in g p assages, a n d y o u w ill b e a sk ed tw o types o f q u e s tio n s a b o u t th e r e a d in g passages: 1.

R e a d in g C o m p r e h e n s io n q u e s tio n s ask you to answ er q u e s tio n s a b o u t th e in fo r ­ m a tio n giv en in th e r e a d in g passages. T h e r e w ill be a variety o f q u e s tio n s a b o u t e a c h r e a d in g p a ssa g e , in c lu d in g m a in id e a q u e s tio n s, d irectly a n sw e r e d d e ta il q u e stio n s, a n d im p lie d d e ta il q u estio n s.

2.

V ocabu lary q u e s tio n s ask you to id e n tify the m e a n in g s o f v o c a b u la r y w o r d s in th e r e a d in g p a ssa g es. T o a n sw e r th e se q u estio n s, you m ay have to k n o w th e m e a n in g s o f th e w ords. You c a n also id e n tify th e m e a n in g s o f s o m e o f th e w ord s by u n d e r ­ sta n d in g th e c o n te x t s u r r o u n d in g th e w ords, by u sin g stru ctu ra l c lu e s to id e n tify th e m e a n in g s o f th e w ord s, o r by b rea k in g d o w n th e u n k n o w n w o rd s in to k n o w n w ord parts in o r d e r to id e n tify th em .

G EN ERAL STRATEGIES

1. Be fam iliar w ith th e d irectio n s. The directions on every TOEFL test are the same, so it is not necessary to spend time reading the directions carefully when you take the test. You should be completely familiar with the directions before the day of the test.

2.

D o n o t sp en d to o m uch tim e reading th e passages! You do not have time to read each reading passage in depth, and it is quite possible to answer the questions correctly without first reading the passages in depth. Some students prefer to spend a minute or two on each passage reading for the main idea before starting on the questions. Other students prefer to move directly to the questions without reading the passages first.

3.

D o n o t w orry if a reading p assage is on a top ic th a t you are unfam iliar w ith . All of the information that you need to answer the questions is Included in the passages. You do not need any background knowledge to answer the questions.

4.

N ever leave any an sw ers blank on your answ er sh ee t. Even If you are unsure of the correct response, you should answer each question.There is no penalty for guessing.

281

READING COMPREHENSION

__t h e

r e a d in g c o m p r e h e n s io n q u e s t io n s



T h e R e a d in g C o m p r e h e n sio n se c tio n o f th e T O E FL test c o n sists o f five r ea d in g passages, c a c h fo llo w e d by a n u m b e r o f r ea d in g c o m p r e h e n sio n a n d v o ca b u la ry q u estio n s. T o p ics o f th e r ea d in g p a ssa g e are varied, b u t they are o fte n in fo r m a tio n a l su b jects that m ig h t be stu d ied in an A m erica n university: A m erica n history, litera tu re, art, a rch ite ctu re , g eo lo g y , g e o g ra p h y , a n d astronom y, for ex a m p le. " T im e is d e fin ite ly a factor in th e R ea d in g C o m p r e h e n s io n se c tio n . M any stu d en ts w h o take th e T O E F L test n o te that they are u n a b le to fin ish all th e q u e stio n s in this sec­ tio n . T h e r e fo r e , y o u n e e d to m ak e th e m o st e ffic ie n t u se o f y o u r tim e in this se c tio n to g e t th e h ig h e st sc o r e. T h e fo llo w in g m e th o d is th e b e st way o f a tta ck in g a r ea d in g p assage to g et th e m o st q u e stio n s co rr ec t in a lim ited a m o u n t o f lim e .

STRATEGIES FO RT H E READING C O M PREH EN SIO N Q UESTIO N S

I. Skim th e reading passage to d e te rm in e th e m ain idea and th e overall o rgan ization o f ideas in th e passage. You do not need to understand every detail ,n each passage to answer the questions correctly. It is therefore a waste of time to read the passage with the intent of understanding every single detail before you try to answer the questions. 1. Look ahead a t th e qu estion s to d e te rm in e w h a t ty p es o f q u estio n s you m ust answer. Each type of question is answered in a different way.

3. Find th e se c tio n o f th e passage th a t deals w ith each qu estio n .T h e question-type tells you exactly where to look in the passage to find correct answers. • For m ain idea questions, look at the first line of each paragraph. • For directly and indirectly answered d eta il questions, choose a key word in the question, and skim for that key word (or a related idea) in order in the passage. • For vocabulary questions, the question will tell you where the word is located in the passage. • For overall review questions, the answers are found anywhere in the passage.

4. Read th e p art o f th e passage th a t contains th e answ er carefully. The answer will probably be in the same sentence (or one sentence before or after) the key word or idea.

5. C h o o se th e b est answ er to each qu estion from th e four answ er ch o ices listed in you r t e s t b ook . You can choose the best answer according to what is given in the appropriate section of the passage, eliminate definitely wrong answers, and mark your best guess on the answer sheet.

T h e fo llo w in g sk ills will h e lp you to im p le m e n t th e se stra teg ies in th e R ea d in g C o m p re­ h e n s io n se c tio n o f th e T O E FL test.

READING COMPREHENSION

Q U E S T IO N S A B O U T T H E IDEAS O F T H E PASSAGE__________ It is very c o m m o n for r e a d in g p a ssa g es in th e R e a d in g C o m p r e h e n s io n s e c tio n o f th e T O E F L test to have q u e s tio n s a b o u t th e overall id ea s in th e p a ssa g e. T h e m o st c o m m o n type o f q u e s tio n asks a b o u t th e m ain id e a , to p ic , title, o r su b ject. O cc a sio n a lly , th e r e w ill also b e a q u e s tio n a b o u t h ow th e in fo r m a tio n in the p a ssa g e is o r g a n iz e d .

S k il l

I:

A N S W E R M A IN ID EA Q U E S T IO N S CORRECTLY

A lm o st every r e a d in g p a ssa g e o n th e T O E F L test w ill h ave a q u e s tio n a b o u t th e m a in id e a o f a passage. S u ch a q u e s tio n m ay be w o r d e d in a variety o f ways; you m ay, for e x a m p le , b e ask ed to id e n tify th e topic, subject, title, prim ary idea, o r m ain idea. T h e s e q u e s tio n s are all really ask in g w h at p rim ary p o in t th e a u th o r is tryin g to g e t across in th e p a ssa g e. S in c e T O E F L p assages are g e n e r a lly w ritten in a tr ad ition ally o r g a n iz e d m a n n e r , it is n o t d iffi­ c u lt to fin d th e m ain id e a by stu d y in g th e to p ic s e n te n c e , w h ich is m o st p r o b a b ly fo u n d at th e b e g in n in g o f a p aragrap h . If a p assage c o n sists o f o n ly o n e p a ra g ra p h , you s h o u ld stu d y th e b e g in n in g o f th a t paragrap h to d e te r m in e th e m a in id ea.

Exam ple I The passage: In the philosophy o f John Dewey, a sharp distinction is m ade betw een “intelligence" and "reasoning.” According to Dewey, intelligence is the only absolute way to achieve a balance betw een Line realism and idealism , betw een practicality a n d w isdom of life. (5) Intelligence involves "interacting with o th er things a n d know ing them ," while reasoning is merely the act o f an observer, " . . . a m ind that beholds o r grasps objects outside the world of things. . . ." With reasoning, a level of m ental certainty can be achieved, but it is through intelligence th at control is taken of events that shape (10) one's life. The question: W hat is the topic of this passage? (A) (B) (C) (D)

The intelligence of John Dewey D istinctions m ade by John Dewey Deweys ideas on the ability to reason How intelligence differs from reasoning in Dewey's works

T h e first s e n te n c e o f th is p a ssa g e d iscu sses a d istin c tio n b e tw e en th e id e a s o f “in t e llig e n c e ” an d “r e a s o n in g ” in th e p h ilo s o p h y o f J o h n Dewey, so th is is p rob ab ly th e to p ic . A q u ick c h e c k o f th e rest o f th e s e n te n c e s in th e p assage c o n fir m s that th e to p ic is in fa c t th e d if­ fe r e n c e b e tw e en “in t e llig e n c e ” a n d “r e a s o n in g .” N o w you sh o u ld c h e c k e a c h o f th e a n ­ swers to d e te r m in e w h ic h o n e c o m e s c lo sest to th e to p ic th a t y o u h ave d e te r m in e d . A n sw er (A) m e n tio n s o n ly in te llig e n c e , so it is n o t th e top ic. A n sw er (B ) m e n tio n s d istin c ­ tion s th a t J o h n D ew ey m akes, b u t it d o e s n o t say sp ecifica lly w h at type o f d istin c tio n s. A n ­ sw er (C ) m e n tio n s o n ly r e a s o n in g , so answ er (C ) is in c o m p le te . T h e b e st an sw er is th e r e fo r e ( D ) ; th e id e a o f how intelligence differs fro m reasoning c o m e s fr o m th e first s e n te n c e o f th e p assage, w h ich m e n tio n s a sharp distinction . . . between “intelligence” a n d "reasoning. ”

284

READING COMPREHENSION

I f a p assage c o n sists o f m o r e th an o n e p aragrap h , you sh o u ld stu d y th e b e g in n in g o f e a c h paragrap h to d e te r m in e th e m ain idea.

E x am p le II T h e p assag e:

Line (5)

( 10)

Nitrogen fixation is a process by w hich nitrogen is continuously fed into biological circulation. In this process, certain algae and bacteria convert nitrogen into am m onia (NH,). This newly created am m onia is then for the m ost p art absorbed by plants. The opposite process of denitrification re tu rn s n itrogen to the air. During the process of denitrification, bacteria cause some o f the nitrates from the soil to convert into gaseous nitrogen or nitrous oxide (N20). In this gaseous form the nitrogen re tu rn s to the atm osphere. T h e q u e stio n : W hich of the following would be the best title for this passage? (A) The Process of Nitrogen Fixation (B) Two Nitrogen Processes (C) The Return of Nitrogen to the Air (D) The Effect of Nitrogen on Plant Life

In a p a ssa g e w ith m o r e th a n o n e p aragrap h , you s h o u ld b e su re to read th e first s e n te n c e o f e a c h p a ra g ra p h to d e te r m in e th e su bject, title, o r m a in id ea . In E x a m p le II, th e first s e n te n c e o f th e first paragraph in d ic a tes th at th e first paragrap h is a b o u t th e p r o c ess o f n itr o g e n fix a tio n . I f you lo o k o n ly at th e first p aragrap h , you m ig h t c h o o s e th e in c o r r e c t an sw er (A ), w h ic h w o u ld b e a g o o d title fo r th e first paragrap h only. T h e first s e n te n c e o f th e s e c o n d paragrap h in d ic a tes th a t th e p ro cess o f d e n itr ific a tio n is d isc u sse d in th e se c ­ o n d p aragrap h . A n sw er (C ) is in c o r r e c t b e c a u se the return o f nitrogen to the air is th e p r o c ess o f d e n itr ific a tio n , a n d this is d isc u sse d in th e s e c o n d p aragrap h only. A n sw er (D ) is in c o r r e c t b e c a u s e the effect o f nitrogen on p la n t life is n o t d isc u sse d in th is passage. T h e b e st a n sw er to th is q u e s tio n is answ er (B ); th e two n itr o g e n p r o c e sse s are n itr o g e n fixa­ tio n , w h ich is d isc u sse d in th e first p aragraph, a n d d e n itr ific a tio n , w h ic h is d isc u sse d in th e s e c o n d p aragrap h . T h e fo llo w in g c h a r t o u tlin e s th e key in fo r m a tio n th a t y o u sh o u ld r e m e m b e r a b o u t m a in id e a q u estio n s: MAIN IDEA QUESTIONS HOW TO IDENTIFY THE QUESTION

W h a t is the t o p ic o f th e passage? W h a t is the s u b je c t o f th e passage? W h a t is the m a in id e a o f the passage? W h a t is the a u th o r’s m a in p o i n t in the passage? W ith w ha t is the a u th o r p r im a r i ly c o n c e r n e d ? W h ich o f th e follow ing w ould be th e best t it le ?

WHERETO FIND THE ANSWER

The answer to this type of question can generally be determined by looking at the first sentence of each paragraph.

READING COMPREHENSION

HOW TO ANSWER THE QUESTION

Read the first line of each paragraph. Look for a common theme or idea in the first lines. Pass your eyes quickly over the rest of the passage to check that you really have found the topic sentence(s). Eliminate any definitely wrong answers and choose the best answer from the remaining choices.

T O E F L E X E R C ISE 1: S tu d y e a c h o f th e p assages a n d c h o o s e th e b e s t a n sw e rs to th e q u e stio n s that follow . In this e x e r c ise , e a c h p assage is fo llo w e d by se v e ra l m a in id e a , to p ic , or title q u e s tio n s so th a t th e stu d e n ts can p r a c tic e th is type o f q u e s tio n . O n th e T O E F L test, o n e p assage w o u ld p r o b a b ly n o t h a v e two su ch q u e s tio n s b e c a u se th e y a re so sim ilar.

PASSAGE ONE (Questions 1-2)

Line (5)

Fort Knox, Kentucky, is the site of a U.S. arm y post, but it is even m ore renow ned for the F o rt Knox Bullion Depository, the m assive vault that contains the bulk of the U.S. governm ent's gold deposits. Com pleted in 1936, the vault is housed in a two-story building co n stru cted of granite, steel, and concrete; the vault itself is m ade of steel and concrete and has a d oor th at w eighs m ore th an twenty tons. Naturally, the m ost up-to-date security devices available are in place a t Fort K nox, and the arm y post nearby provides fu rth er protection. 1. W hich of the following best describes the topic of the passage? (A) (B) (C) (D)

2.

The city o f F ort Knox, Kentucky The federal gold depository The U.S. arm y post at F ort Knox Gold bullion

W hich of the following w ould be the best title for this passage? t (A) The Massive C oncrete Vault (B) Fort Knox Security (C) W here the U.S. Keeps Its Gold (D) A Visit to Kentucky

PASSAGE TWO (Questions 3-4)

Line (5)

One identifying characteristic of m inerals is their relative hardness, w hich can be d eterm ined by scratching one m ineral w ith another. In this type of test, a h a rd er m ineral can scratch a softer one, b u t a softer m ineral is unable to scratch the h ard er one. The Molls' hardness scale is used to ra n k m inerals according to hardness. Ten m inerals are listed in this scale, ranging from talc w ith a hardness of 1 to diam ond with a hardness of 10. On this scale, quartz (num ber 7) is h arder than feldspar (num ber 6 ) and is therefore able to scratch it; however, feldspar is unable to make a m ark on quartz. W hich of the following best states the subject of this passage?

4.

The m ain idea of this passage is th a t (A)

(A) (B) (C) (D)

The hardness of diam onds Identifying m inerals by m eans of a scratch test Feldspar on the M ohs' scale Recognizing m inerals in th e ir n atural state

(B)

(C) (D)

the hardness of a m ineral can be determ ined by its ability to m ake a m ark on o th er m inerals diam onds, w ith a hard n ess of 10 on the M ohs’ scale, can scratch all o th er m inerals a softer m ineral . ,:' ,.. ~:~;.., :,',.. 2., When a nouf1clause Is u'sed as.a subject, do not add an~~ubjeS!.: ,':",': after the noun clause. .". ." "" .. ,.' . I•. ;".

'.:-~:

:.~~.. :

\:"

:-:.::

'A subordinate clause is a dependenl c1ame. il has both a subject and a verb and is introduced by a mbordinate connector. Complex sentences with subordinate noun, adverb, and adjective clauses are covered in great detail in Skills 7-12 of the Structure and Written Expression section on pages 14I-I,,:\.

TESTOF WRITTEN ENGLISH (TWE)

EXERCISE 7C: Underline the subjects once and the verbs twice in the main clauses. Put parentheses around the subordinate noun and adjective clauses. Then indicate if the sentences are correct (C) or incorrect (I).

-'-

1. The tickets (that I ordered) they will be delivered tomorrow. 2. How I will be able to get all this work done is unclear. 3. The excuse that you gave me was not very credible. 4. What the lecturer said it was really quite amusing. 5. The place where we agreed to meet it was quite secluded. 6. The person whose friendship I cherish most is a friend from my childhood. 7. Who is responsible for the accident it is unknown. 8. That the story is on the front page of the paper it is indisputable. 9. The contractor who painted the house he did a very carefuljob. 10. Why she was the one who got the job is a mystery to me. II.

What happened just before our arrival it is unknown.

12. The clothes that we purchased at the sale were quite a good bargain. 13. The room in which the seminar will be held is rather tiny. 14. What will happen to her next it is what concerns me the most. 15. The receptionist who regularly answers the phone is out of the office. 16. What the manager wrote in the report it washighly complimentary. 17. The classmate who presented the report he did a great job. 18. How such a thing could happen is not clear to me. 19. The situation in which I found myselfwas one in which all of the facts are not known. 20. Why he has done what I told him not to do with the money that I gave him it is not certain.

399

400

TESTOF WRITTEN ENGLISH(TWE)

EXERCISE 7 (A-C): Find and correct the sentence structure errors in the following essay. (The number in parentheses at the end of each paragraph indicates the number of errors in that paragraph.) The essay discusses the following topic. Some people prefer to take vacations in quiet, natural places. while others prefer to spend their vacation time in big cities. Discuss the advantages of each type of vacation. Then indicate which you prefer and why.

1.

What you need to do before going on a vacation it is to decide where you will go on your vacation. You may decide to go to a quiet place with a quiet and natural setting. instead you may decide "to go "to a big city with a fast-paced life. Each of these types of vacation something to offer. (3 ermr~)

2.

The reasons that it can be a good idea to go to a quiet and natural location for a vacation they are numerous. first of all, a vacation in a natural setting allowing you to relax and slow dcwn the pace of your life for a while. Instead of hurrying from place to place as you are used "to doing. You can spend your time doing nothing more than enjoying the beauty of the location. Then, after are thoroughly relaxed, what you can do it is to take part in outdoor activities such as hiking or swimming. All of this will/eave you completely relaxed and free of stress by the end of your vacat.iol". (:. error;;)

3.

It can be nice to go to a quiet and natural spot for a vacation, however it can also be quite an adventure to go to a big and fast-paced city for a vacation. The main reason that it can be a good idea to take a vacation in a big city it is to take part in so many activities that are unavailable in yC'ur hometown. On a big city vacation, numerous cultural events that might not be available in your hometown, such as theatrical performances, conc~rts, and art and museum exhibits. they are available. On a big city vacation, will a/so have access to some of the world's finest restaurants and shopping. After your big city vacation has ended. You will have a whole range of new experiences that are not part of your daily life. (5 errors)

4.

for me, the type of vacation that I decide to take it depends on my life prior "to the vacation. I work as a legal assistant in a law office, this job is often repetitious and dull but is sometimes quite frantic just prior to a major case. After a slow and boring period of work. Alii want is to head to a fast-paced vacation in a big city. However, if my job been frantic and busy prior to my vacation, then want to head to a quiet and beautiful place where I can relax. Thus, I tmjoy different types of vacations, the type of vacation depends on the pace of my life before the vacation. (6 errors)

2

TESTOFWRITTEN ENGlISH (TWE)

SKill 8: EDIT WRITTEN

EXPRESSION

8A. Inversions and Agreement Errors in inversions and agreement are covered in the Structure section of this book. You may want to review these skills.

and Written Expression

Skills 15-19: Sentences with Inverted Subjects and Verbs Skills 20-23: Problems with Subject/Verb Agreement EXERCISE SA: Find and correct the errors in the following essay. (The number in parentheses at the end of each paragraph indicates the number of errors in that paragraph.) The essay discusses the following topic. Some people prefer to work for a company.while others prefer to work for themselves. Discuss the advantages of each position. Then indicate which you prefer and why.

1.

Something very important for students to decide as they near the end of their studies are whether should they work for another company or go into business for themselves. As a university student, this decision about my future are one that I face soon myself. To me, each of these positions have clear advantages, in particular depending on the stage of your career. (4 errors)

2.

There is numerous advantages to working for another company. particularly early in your career. One of the advantages are that working in someone else's company provide a situation with the security of a regular paycheck and less responsibility than you would have you were to be the owner of the company. Also, not until you start your own business you need to come up with the finances to back the company. Thus. all of this indicate that it is better to work for other people early in your career while you are gaining the knowledge and experience you need to start your own company. (6 errors)

3.

Then. later in your career, it may be advantageous for you to go into business for yourself. The main reason for going into business for yourself are that in your own company you are able to decide on what direction do you want your company to go. However, only when you have gained enough knowledge and experience are it a good idea to go into business for yourself. This is when will you be ready to deal with the responsibility, pressure, and financial needs of owning a company. (4 errors)

4.

Nothing are more important to me than having my own company one day. However, what seems very clear to me now is that beginning my career working in someone else's company are best. In this situation, not only I can work with more security and less pressure, but I can also build up my financial resources and learn from others. Then, I should manage to gain enough experience, knowledge, and confidence and build up my financial resources, I hope eventually to open my own company, where can I determine exactly how would I like the company to operate. (6 errors)

40 I

402

TESTOFWRITTEN ENGLISH(TWE)

8B. Parallel, Comparative,

and Superlative

Structures

Errors in parallel, comparative, and superlative structures are covered in the Structure and Written Expression section of this book. You may want to review these skills. Skills 24-26: Problems with Parallel Structure Skills 27-29: Problems with Comparatives and Superlatives EXERCISE 8B: Find and correct the errors in the following essay. (The number in parentheses at the end of each paragraph indicates the number of errors in that paragraph.) The essay discusses the following topic. Do you agree or disagree with the following statement! The primary reason to gel an education is to succeed ~nandally. Support your response with reasons and examples.

I.

I am a university student. and I am studying in the university for a number of reasons. Of course, one of my reasons for going to school, studying hard. and obtain a university degree is to succeed financially; the more money I make. it will be better for me. However, financial success is not my most importantest reason for going to the university. Instead, I am going to the university for a much broad reason than that: I believe that a university education will give me a much rich and better

2.

life. not just

in a financial way. (5 errors)

One way that a university education

makes your life enjoyabler is to give you the

opportunity to have a career that you really desire and appreciative. Having a career that you like is much better than a job that just pays the bills. I. for example. am studying to be a marine biologist. I will have the better career for me; I will be rewarded not only in terms of money and also in terms of enjoyment of my career. (5 errors) 3.

Another way that a university education can enrich your life is to provide a broadest knowledge. understand, and. appreciation of the world around you than you already have. It provides you with an understanding of both the history of your own culture and to influence history on the present. It also provides you with an understanding of other cultures and shows you that other cultures are neither exactly the eame as nor they are completely different from your own culture. Finally. it provides you with an understanding of the universe around you and showing you how the universe functions. (5 errors)

4.

Thus. in getting a university education, I can say that financial success is certainly one goal that I have. However, the goal of financial success is not as important as I have another goal. My primary goal in getting a university education is the goal of achieving a more full life. certainly one with financial security but more importantly one that is rewarding both in terms of professional opportunities or in terms of awareness and understanding of life around me. The closer I get to achieving this goal, I will be happier. (4 errors)

TESTOF WRITTEN ENGLISH (TWE)

Be.

Verbs

Errors in verbs are covered in the Structure You may want to review these skills. Skills 30-32: Skills 33-36: Skills 37-3B:

and Written Expression section of this boo •..

Problems with the Form of the Verb Problems with the Use of the Verb Problems with Passive Verbs

EXERCISE Be: Find and correct the errors in the following essay. (The number in parentheses at the end of each paragraph indicates the number'of errors in that paragraph.) The essay discusses the following topic. When something unexpected happens, how do you react! Use examples to support your response.

I.

When~omething unexpected happen~, different people reacted in a variety of ways. I wi~h I could reacted calmly to unexpected ~ituation~. However, unfortunately, I u~ually react with panic. The following example ~how~ my u~ual reaction to situation~ when I have be completely unprepared for them. (3 errors)

2.

Thi~ example of the way that I react to unexpected ~ituation~ ha~ occurred in hi~tory cla~~ la~t week. The profe~~or had told u~ that we will be covering the material in Chapter~ 10 through 12 in cla~~ on Thur~day. By the time I arrived in c1a~~.I have read all of the a~~igned material, and I under~tood mo~t of what I had. ~tudy. While I wa~ relax in my chair at the beginning of cla~~. the profe~~or announce~ that there would be a pop quiz on the material in the a~~igned chapter~. I wa~ preparing on the material becau~e I have ~tudied all of it thoroughly before cla~~. (8 errors)

3.

However,I wa~ face with an unexpected ~ituation. and I do not react well to unexpected ~ituation~. In~tead of feeling relaxed at the announcement of the unexpected quiz becau~e I wa~ ~o prepared. I wa~ completely fill with anxiety by the ~ituation. A~ the profe~~or wa~ write the que~tion~ on the board. I become more and more nervou~. I wa~ unable to think clearly. and I knew that I would done a bad job on the quiz becau~e thi~ wa~ what alway~ happen~ to me when I feel panic. A~ I ~tared at the que~tion~ on the board, I had been unable to think of the correct an~wer~. It wa~ a~ if I had not prepare at all for cla~~. Then, the profe~~or collected the papers from the cla~~, including my basically blank piece of paper. Ju~t after the paper~ had been collecting, the answer~ to all the que~tion~ came to me. (9 errors)

4.

You can ~een from thi~ example that my u~ual reaction to ~omething unexpected i~ to panic. In the future. I hoped that Iwill learn to react more calmly. but up to now I had not learned to react thi~ way. On the ba~i~ of my pa~t behavior, however,it ~eem~ that I currently had a ~tronger tendency to react with panic than with calm. (4 errors)

403

404

TESTOFWRITIEN ENGLISH(TWE)

80. Nouns and Pronouns Errors in nouns and pronouns are covered in the Structure tion of this book. You may want to review these skills.

and Written Expression sec-

Skills 39-42: Problems with Nouns Skills 43-45: Problems with Pronouns EXERCISE 8D: Find and correct the errors in the following essay. (The number in parentheses at the end of each paragraph indicates the number of errors in that paragraph.) The essay discusses the following topic. What part of your high school experience was the most valuable! Use reasons and examples to support your response.

1.

I was not a very good athleticism in high school, but I wanted with all of mine heart to be on the football team. My desire to be on the team had little to do with athletics and was perhaps not for the best of reasons; the strong stimuli for I to make the team was that team members were well-known in the school and he became very popular. This desire to be on the football team in high school, and the fact that through hard worker I managed to accomplish something that I wanted so much, even if its was something petty, turned out to be the single most valuable experiences of my years in high school. (8 errors)

2.

I had to work very hard to make the football team in high school, and for some time this seemed like an impossible goals. A large amount of students in my school, more than a hundred and fifty of them, spent many of theirs afternoons trying out for a team with less than forty positions. After a lot -of hard work on my part, and after I had demonstrated to the coaches that he could count on me to keep going long after everyone was exhausted, I managed to make the team as a secondary play. Even with so many effort, I was never going to be a sports phenomena or even a member of the first team, but I did accomplish my goal of making the team. (8 errors)

3.

The valuable lesson that I learned through this experience was not the joy of competitor or the much benefits of teamwork, several lesson very commonly associated with participation in team sports. Instead, the valuable lesson that I learned was that hard work and determination could be very important in helping I accomplish each goals that I want to reach. Even if others have more talent, I can work harder than it does and still perhaps find successor where them do not. (8 errors)

TESTOFWRITTEN ENGLISH (TWE)

8E. Adjectives

and Adverbs

Errors in adjectives and adverbs are covered in the Structure tion of this book, You may want to review these skills.

and Written Expression

sec-

Skills 46-48: Problems ~ith Adjectives and Adverbs Skills 49-51: More Problems with Adjectives Skills 52-55: Proble!Il!i with Article!! EXERCISE BE: Find and correct the errors in the following essay. (The number in parentheses at the end of each paragraph indicates the number of errors in that paragraph.) The essay discusses the following topic. Some students prepare early.while other students procrastinate. Which type of student are youl Support your response with reasons and examples.

I.

I understand that it seems importantly for a students to prepare early their assignments rather than procrastinate in getting assignments done. However, although I understand this clear, I always seem to wait until the finally minute to get assigning projects done. There are two reasons why I regular procrastinate on my assignments academic in spite of the fact that this is not a best way to get my work done. (9 errors)

2.

One reason that I ttind to be a eternal procrastinator is that I work much more. efficient under pressure than I do when I am not under pressure. For example, I can accomplish so much more in a two-hour' period when I have a definittily deadline in two hours than I can during an alike period without the pressure of a deadline strict. Without a deadline. the two-hour period seems to ffy by with minimally accomplishment, but with an rapid approached deadline I seem '{uitti capably of making every mio'lU~ of the two-hour period count. (10 errors)

3.

Another' reason that I tend to procrastinatti is that if I start preparing early, it takes generally more of my time. If, for example. I have paper due in six weeks. I can start working on the paper now and w(l~k on it on a day basis. a"d th[lt p;1per will take up a lot of my time and energy during the followed six weeks. However, if I wait to begin work on the paper until week before it is due. I have to go off some place where I can be lone and spend all of my time and energy that week on !-he paper, bUJ; it will only take one week of my time valuable and not six weeks. (7 errors)

4.

In summary. it seems that I always wait until !-ne la:5J;minute to complete an assignments b~c",use I am ;;fr,:;id that I Wi!1I'.'.-;st.e t-:.o much time. by p,tarting early. It would be flood idea. however. for me to make a ,~ff(lrt:to qet wor.\-done effi-:i.mt and early so th'-lt I do not al"Vays have to feel tensely about getting work done at a last minutti. (6 errors)

405

406

TESTOFWRITIEN ENGLISH(TWE)

SF. Prepositions

and Usage

Errors in prepositions and usage are covered in the Structure tion of this book. You may want to review these skills. Skills 56-57: Skills 58-60:

and Written Expression

sec-

Problems with Prepositions Problems with Usage

EXERCISE SF: Find and correct the errors in the following essay. (The number in parentheses at the end of each paragraph indicates the number of errors in that paragraph.) The essay discusses the following topic. Some people avoid confrontations at all costs, while other people seem to seek them out. Discuss each type of person. Then indicate which type of person you are and why.

1.



A

@ @

in fifths.

The sentence should read, "The four strings on a violin are tuned in fifths." Therefore, you should choose answer (B). Example

II

The research for Ihe book Roots taking --A--B-CAlex Haley twelve years. D

Sample

Answer

@

@ @

The sentence should read, "The research for the book Roots took Alex Haley twelve years." Therefore, you should choose answer (C).

446

COMPLETE TEST "TWO

TOEFL- test directions .nd format are reprinted by prnnission of ETS. 1M copyright OWMr. However; all e:rampl~ and lest qU"lions are provided by Peanon Education. Inc.

2 16.

Mosquitoes

will accepts the malaria parasite at only one stage of the parasite's -A--B--C-

complex life cycle. --D17.

The counterpart of a negative electrons ABC

18. \ Alexander Hamilton's

is the positive proton. --D-

advocacy of a strong national government A -B-

brought

he into

C-

bitter conflict with Thomas Jefferson. -D19.

There are more than eighty-four

million specimens

A Natural History's collection C anthropology D 20.

in the National

Museum of

B of biological, geological, archeological,

and

treasures.

After George Washington

---P::-

married widow Martha Custis, the couple comes to reside -B-C-D-

at Mount Vernon.

21.

Rubberized A

asphalt can hardly be classified as cutting edge at this stage in their -BC [)

development.

22.

Rhesus monkeys exhibit patterns A -B-

of shyness similar to that in humans. --C0

23.

In space, with no gravity for muscles to work against, the body becomes weakly. A -B--C---D-

24.

Fort Jefferson,

in the Dry Tortugas off the southern

A

tip of Florida, can be reach -C-

B

only by boat or plane. -D25.

Quarter horses were developed A courses short of about a quarter C

26.

Supersonic A

in eighteenth-century

Virginia to race on -B-

of a mile in length. --D-

flight is flight that is faster the speed of sound. B -CD

COMPLETE TEST TWO

447

27.

Since the dawn of agriculture 9,000 years ago, only a few animal species had been A Be 0 domesticated.

2 28.

The Betataken House Ruins at Navajo National Monument is among the largest and

A

-B-

most elaborate cliff dwellings in the country. C --029.

The island of Kauai has much streams, some of which have worn deep canyons into -A--B-

c-

the rock.

---0

30.

It is a common observation that liquids will soak through some materials but not A --B-Cthrough other.

----0-

31.

Surrounded A

by forested mountain slopes are the town of Telluride, a former B -C-

gold-mining town 7,500 feet above sea level. --032.

The newsreels of Hearst Metronome News, which formed part of every moviegoer's

---x-

experience in the era before television, offer an unique record of the events of the B C -01930s.

33.

Probably the best known of all dinosaurs. the Tyrannosaurns A B

was larger and last of -CD

the meat-eating carnosaurs.

34.

Unlikely gas sport balloons, hot air balloons do not have nets. --A-B--C [)

35.

Born in Massachusetts ---x-

in 1852, Albert Farbanks has begun making banjos in Boston B -C-

in the late 1870s.

D 36.

448

Methane in wetlands comes from soil bacteria that consumes organic plant matter.

COMPLETE TEST TWO

-A-13

C

--0-

37.

A10is Alzheimer made the first observers of the telltale signs of the disease that today A -Bbears his name. -CD

38.

Edward McDowell remembers as the composer ABC

of such perennial

favorites as "To a --D-

Wild Rose" and "To a Water Lily."

39.

Animism is the belief that objects and natural phenomena -AB

such as rivers. rocks. and

wind are live and have feelings. C -D40.

Newtonian physics accounts A

from the observation C

--a

of the orbits of the planets and -D-

moons.

This is the end of SectIon 2. If you finish before 25 minutes has ended, check your work on Section 2 only.

8888888 At the end of 25 minutes, go on to Section 3. Use exactly 55 minutes to work on Section 3.

COMPLETETESTTWO

449

SECTION 3 READING COMPREHENSION TIme-55 minutes (including the reading of the directions) Now set your clock for 55 minutes. This section is designed to measure your ability to read and understand short passages similar in topic and style to those that students are likely to encounter in North American universities and colleges. This section contains reading passages and questions about the passages. Directions: In this section you will read several passages. Each one is followed by a number of questions about it. You are to choose the one best answer, (A), (B), (C), or (0), to each question. Then, on your answer sheet, find the number of the question and fill in the space that corresponds to the letter of the answer you have chosen. Answer all questions about the information in that passage.

in a passage on the basis of what is stated or implied

Read the following passage: John Quincy Adams, who served as the sixth president of the United States from 1825 to 1829, is today recognized for his masterful statesmanship and diplomacy. He dedicated his life to public service, both in the presidency and in the various other political offices that he Line held. Throughout his political career he demonstrated his unswerving belief in freedom of (5) speech, the antislavery cause, and the right of Americans to be free from European and Asian domination. Example

Sample Answer

I

@



To what did John Quincy Adams devote his life? (A) (B) (C) (0)

@

Improving his personal life Serving the public Increasing his fortune Working on his private business

@

According to the passage, John Quincy Adams "dedicated his life to public service." Therefore, you should choose answer (B) .. Example

Sample Answer

n

In line 4, the word "unswerving" is closest in meaning to (A) (B) (C) (0)

moveable insignificant unchanging diplomatic

@ @

• @

The passage states that John Quincy Adams demonstrated his unswerving belief "throughout his career." This implies that the belief did not change. Therefore, you should choose answer (C).

450

COMPLETE TEST TWO

TOEFL- test directions and formal are reprinted by pennission of ETS, the copyright owner. Howrver. all examples.nd test questions are provided by Pearson Education. Inc.

Questions 1-10

Line (5)

(10)

(15)

Niagara Falls, one of the most famous North American natural wonders, has long been a popular tourist destination. Tourists today flock to see the two falls that actually constitute Niagara Falls: the I 73-foot-high Horseshoe Falls on the Canadian side of the Niagara River in the Canadian province of Ontario and the I 82-foot-high American Falls on the U.S. side of the river in the state of New York. Approximately 85 percent of the water that goes over the falls actually goes over Horseshoe Falls, with the rest going over American Falls. Most visitors come between April and October. and it is quite a popular activity to take a steamer out onto the river and right up to the base of the falls for a close-up view. It is also possible to get a spectacular view of the falls from the strategic locations along the Niagara River. such as Prospect Point or Table Rock, or from one of the four observation towers which have heights up to 500 feet. Tourists have been visiting Niagara Falls in large numbers since the 1800s; annual visitation now averages above 10 million visitors per year. Because of concern that all these tourists would inadvertently destroy the natural beauty of this scenic wonder, the state of New York in 1885 created Niagara Falls Park in order to protect the land surrounding American Falls. A year later Canada created Queen Victoria Park on the Canadian side of the Niagara, around Horseshoe Falls. With the area surrounding the falls under the jurisdiction of government agencies, appropriate steps could be taken to preserve the pristine beauty of the area. 1. What is the major point that the author is making in this passage? (A)

(B)

(C) (D)

Niagara Falls can be viewed from either the American side or the Canadian side. A trip to the United States isn't complete without a visit to Niagara Falls. Niagara Falls has had an interesting history. It has been necessary to protect Niagara Falls from the many tourists who go there.

2. The word "flock" in line 2 could best be replaced by (A) come (B) come (C) come (D) come see

by plane in large numbers out of boredom without knowing what they will

3. According to the passage. which of the following best describes Niagara Falls? (A)

(B) (C)

(D)

Niagara Falls consists of two rivers, one Canadian and the other American. American Falls is considerably higher than Horseshoe Falls. The Niagara River has two falls, one in Canada and one in the United States. Although the Niagara River flows through the United States and Canada, the falls are only in the United States.

4. A "steamer" (A) (B) (C) (D)

a a a a

in line 8 is probably

bus boat walkway park

COMPLETE TEST lWO

451

3 6 3 6 3 6 3 6 3 6 3 6.36 3 s.

The expression "right up" in line 8 could best be replaced by (A)

tum to the right

(B) follow correct procedures (C) travel upstream (O) all the way up 6. The passage implies that tourists prefer to (A) visit Niagara Falls during wanner weather (B) see the falls from a great distance (C) take a ride over the falls (0) come to Niagara Falls for a winter vacation 7. According to the passage, why was Niagara Park created? To encourage tourists to visit Niagara Falls (B) To show off the natural beauty of Niagara Falls (C) To protect the area around Niagara Falls (0) To force Canada to open Queen Victoria Park

(A)

8. The word "jurisdiction"

in line 17 is closest

in meaning to (A) view (B) assistance (C) taxation (O) control 9. The word "pristine" in line 18 is closest in meaning to (A)

pure and natural

(B) highly developed (C) well-regulated (0) overused 10. The paragraph following the passage most probably discusses (A)

additional

ways to observe the falls

(B) steps take by government agencies to protect the falls a detailed description of the falls between States and Canada (0) further problems that the area around the

(C)

of the division the United are destroying falls

GO Ot-; TO THE NEXT PAGE

endtime in England before she died there at a young age. In the spring of 1613, a young Pocahontas was captured by the English and taken to Jamestown. There she was treated with courtesy as the daughter of chief Powhatan. While Pocahontas was at Jamestown, English gentleman John Rolfe fell in love with her and asked her to marry. Both the governor of the Jamestown colony and Pocahontas's father Powhatan approved the marriage as a means of securing peace between Powhatan's tribe and the English at Jamestown. In 1616, Pocahontas accompanied her new husband to England, where she was royally received. Shortly before her planned return to Virginia in 1617, she contracted an illness and died rather suddenly. A major part of the folktale of Pocahontas that is unverified concerns her love for English Captain John Smith in the period of time before her capture by the British and her rescue of him from almost certain death. Captain John Smith was indeed at the colony of Jamestown and was acquainted with Powhatan and his daughters; he even described meeting them in a 1612 journal. However, the story of his rescue by the young maiden did not appear in his writings until 1624, well after Pocahontas had aroused widespread interest in England by her marriage to an English gentleman and her visit to England. It is this discrepancy in dates that has caused some historians to doubt the veracity of the tale. However, other historians do argue quite persuasively that this incident did truly take place. 41.

The main idea of the passage is that

43.

(A) folktales are often not very factual (B) Pocahontas did not really exist (C) anyone of Powhatan's daughters could have been the Pocahontas of legend (0) Pocahontas fell in love with John Smith and saved his life 42.

The expression "arose from" in line 4 is closest in meaning to (A) (B) (C) (0)

developed from went up with was told during climbed to

What is true about the name Pocahontas, according to the passage? (A) (B) (C) (0)

44.

It was the real name of a girl named Matoaka. It meant that someone was playful. Only one girl was known to have used this name. Powhatan was one of several people to be given this nickname.

How was Pocahontas held at Jamestown? (A)

(B) (C) (0)

With With With With

treated when she was

respect disdain surprise harshness

COMPlETETESTTHREE

485

45.

It can be inferred from the passage that Pocahontas

49.

(A) never intended to return to Virginia (B) had a long marriage (C) suffered from a long illness (0) did not mean to remain in England 46.

47.

(B)

The pronoun

(C) (0)

therefore in fact unexpectedly in contrast

50.

"he" in line 22 refers to

Captain John Smith probably never knew Pocahontas. Captain John Smith was never actually in Jamestown. His rescue purportedly happened while Pocahontas was in England. His account of the rescue did not appear until well after the event supposedly happened.

The word "veracity" in line 26 is closest in meaning to (A) (B) (C) (0)

(A) the governor (B) Pocahontas (C) John Smith (0) Powhatan 48.

(A)

The word "indeed" in line 21 is closest in meaning to (A) (B) (C) (D)

Why are some historians doubtful about the portion of the Pocahontas folktale dealing with John Smith?

timing location understanding accuracy

When did John Smith most likely meet Pocahontas? (A) (B) (C) (0)

In In In In

1612 1613 1616 1624

This is the end of Section 3.

If you finish in less than 55 minutes, check your work on Section 3 only. Do NOT read or work on any other section of the test.

When you finish the test, you may do the following: • Turn to the Diagnostic Charts on pages !'ilt\-:)~)().and circle the numbers of the questions that you missed . • Turn to Scoring Information on pages :;1'l-:;H~, and determine your TOEFL score . • Turn to the Progress Chart on page :)~}I, and add your score to the chart.

486

COMPLETE TEST THREE

TEST OF WRITTEN ENGLISH: TWE ESSAY TOPIC TIme-30 minutes Do you agree or disagree with the following statement? You can get a better education

{rom experience than you can in a classroom.

Use specific details and examples to support your opinion.

COMPLETETESTTHREE

487

488

1o 1o 1o 1o 1o 1o 1o 1 COMPLETE TEST FOUR SECTION 1 LISTENING COMPREHENSION Trme-approximately 3S minutes (including the reading of the directions for each part) In this section of the test, you will have an opportunity to demonstrate your ability to understand conversations and talks in English. There are three parts to this section, with special directions for each part. Answer all the questions on the basis of what is stated or implied by the speakers you hear. Do not take notes or write in your test book at any time. Do not tum the pages until you are told to do so.

Part A Directions: In Part A you will hear short conversations between two people. After each conversation, you will hear a question about the conversation. The conversations and questions will net be repeated. After you hear a question, read the four possible answers in your test book and choose the best answer. Then, on your answer sheet, find the number of the question and fill in the space that corresponds to the letter of the answer you have chosen. Listen to an example. On the recording, (man) (woman) (narrator)

Sample

@ @ @

you wilI hear:



That exam was just awful. Oh, it could have been worse. What does the woman mean?

In your test book, you will read:

(A) (B) (C) (D)

Answer

The exam was really awful. It was the worst exam she had ever seen. It couldn't have been more difficult. It wasn't that hard.

You learn from the conversation that the man thought the exam was very difficult and that the woman disagreed with the man. The best answer to the question, "What does the woman mean?" is (D), "It wasn't that hard: Therefore, the correct choice is (D).

TOEFL- test direclions and fonnal an: n:printed by pennission of ETS. the copyright O'W~r. Hown'~r. aU examples and tesl quesliom an: providal by Pearson EJucation. Inc.

8 COMPLETETESTFOUR

489

1 o 1 D 1 D 1 D 1 o 1D 1 o 1 1. (A) Watching a movie. (B) Hunting. (C) Buying film. (D) Taking photos. 2. (A) (B) (C) (D)

It's a good idea to be thrifty. He's feeling a little dirty. He'd like something to drink. Stopping for thirty minutes is a good idea.

3. (A) The flight is departing in the ncar future. (B) The plane is taking off early. (C) The man needs to make plans soon. (D) The plane is taking up space. 4. (A) (B) (C) (D)

He has never gone to any games. It is rare for the football team to win. He doesn't go to games often. It is rare for the university team to have a game.

. 5. (A) They should neighbors. (B) They should (C) They should (D) They should neighbors.

call out to their visit their neighbors. phone their neighbor's. look over their

(A)

He's going to say something in the theater. (B) What the woman said was magnified out of proportion. (C) The size of the theater was magnificent. (D) He shares the woman's opinion.

10. (A) They are unconfirmed. (B) They are dependent on futur'e research. (C) They are most probably correct. (D) They are independent of the researchers' ideas. II.

(A) She was less Ihan delighted. (B) She was quite pleased. (C) She was unable to accept it. (D) She wished she could have been more delighted.

12. (A) The lawyer delivered the letter' this morning. (B) The courier has already made the delivery. (C) The letter 10 Ihe courier has already been received. (D) The lawyer's office does not have the letter.

6. (A) (B) (C)

13. (A) (B) (C) (D)

7. (A)

14. (A) She must work lonight. (B) The shift in her plans is unlucky. (C) Her roommate is persuasive. (D) Her room male will work in her place.

It is not done yet. It was not done carelessly. It does not seem to have been done by the accountant. (D) It contains a lot of erTors. He will be far from the conference tonight. (B) He's not quite sure who the speaker will be. (C) He knows Dr. Burton well. (D) He knows that Dr. Burton will be speaking.

8. (A) Take a short nap. (B) Go out now. (C) Enjoy the reSI of the evening. (D) Have a little snack before going out.

490

9.

COMPLETE TEST FOUR

The phone is off the hook. The man will head the committee. The man is no longer responsible. The committee meeting has been put off.

15. (A) Put it away. (B) Put it off. (C) Put it out. (D) Put il down.

1 D 1D 1D 1D 1D 1D 1D 1 16. (A) That the man far. (B) That the man wants. (C) That the man graduate. (D) That the man

should not reach out so can accomplish

24.

(A) He cannot work on the assignment because of a headache. (B) He thinks the assignment will take about two hours. (C) It would be better to prepare two assignments than one. (D) He prefers not to work on it by himself.

25.

(A) The project that the woman wants is impossible. (B) Two hours is not long enough to complete the project. (C) The woman's request can be accomplished. (D) The woman should not ask for such a thing.

26.

(A) (B) (C) (D)

27.

(A) She spent her normal amount of time on it. (B) It is rare for her to finish an assignment. (C) It is rare for her to put any effort into an assignment. (D) She spent more time than usual on it.

28.

(A) (B) (C) (D)

29.

(A) That she couldn't get into the lab. (B) That she wouldn't do the assignment. (e) That her lab assignment was already done. (D) That she would stal.t working in a couple of hours.

30.

(A) (B) (C) (D)

what he

will be unable to cannot score a goal.

17. (A) She couldn't afford a new computer. (B) The computers were not on sale. (C) She was unable to get a new computer. (D) She bought a new computer. 18. (A) Visiting a doctor. (B) Attending a reception. (C) Applying to medical school. (D) IntelViewing for a job. 19. (A) It met her expectations. (B) It was rather mediocre. (C) It was what she had hoped to see. (D) It was the last performance. 20.

(A) Going home on the bus. (B) Sleeping on the bus. (C) Taking a quick walk. (D) Getting some sleep before going home.

21.

(A) The course (B) The course semester. (C) The course (D) She thinks too low. That That That That

he he he he

is free. costs $100 more this was cheaper last semester. the cost of the course is

22.

(A) (B) (C) (D)

would be at work. knew a lot about architecture. did not get the job. would not be at home.

23.

(A) The rider took the road to the hospital. (B) An ambulance took the rider to the hospital. (C) The ambulance left the hospital with the rider. (D) The motorcyclist followed the ambulance to the hospital.

He is not very impressed with it. He thinks it is fantastic. He does not want more pressure on it. It is less impressive than expected.

It was not cold enough. The snowball struck him forcefully. The snow stayed around too long. It was too cold.

She took the stairs out of necessity. She didn't want to take the elevator. It was only a few flights of stairs. She preferred to climb the stairs.

COMPLETE TEST FOUR

491

1o 1 o 1o 1o 1o 1o 1o 1 Part B Directions: In this part of the test, you will hear longer conversations. After each conversation, will hear several questions. The conversations and questions will not be repeated.

you

After you hear a question, read the four possible answers in your test book and choose the best answer. Then, on your answer sheet, find the number of the question and fill in the space that corresponds to the I~tter of the answer you have chosen. Remember, you are not allowed to take notes or write in your test book.

31.

(A) The price of textbooks. (B) History 101. (C) The university bookstore. (0) Ways to sell used books.

32.

(A) (B)

(C) (0)

35. (A) That babies sleep thirteen hours a day. (B) That the woman was taking a psychology class. (C) That more mature people required so much sleep. (0) That the need for sleep decreases with age.

He desperately needs the money. Reading doesn't interest him. He's finished using them. He'd rather have cheaper books.

36. (A) (B)

33.

34.

(A) The bookstore doesn't want to buy them. (B) He wouldn't get enough money. (C) He doesn't like the bookstore's advertisements. (0) It's too late to sell them to the bookstore. (A)

Post some advertisements. Take History 101. (C) Give the books to the boobtore nothing. (0) Keep the books.

(B)

492

COMPLETE TEST FOUR

for

(C) (0)

In psychology class. In a discussion with the man. From an article that she read. From the class textbook.

37. (A) Teens. (B) Twenties. (C) Thirties. (0) Fifties. 38. (A) Thirteen hours. (B) Nine hours. (C) Eight hours. (0) Six hours.

TOEFL-lest dirrction!; and formal are reprlnlnl by pennission of ETS. the copyright own~r.lIo\W'Yer,all rumples and lest Question!>are provided by Pearson Education. Inc.

1o 1o 1 o 1o 1o 1D 1D 1 Part C Directions: In this part of the test. you will hear several talks. After each talk, you will hear some questions. The talks and questions will not be repeated. After you hear a question, you will read the four possible answers in your test book and choose the best answer. Then, on your answer sheet, find the number of the question and fill in the space that corresponds to the letter of the answer you have chosen. Here iJi an example. On the recording. (narrator) (man)

you will hear:

Listen to an instmctor talk to his class about painting. Artist Gram Wood was a guiding force in the school of painting known as American regionalist, a style reflecting the distinctive characteristics of art from rural areas of the United States. Wood began drawing animals on the family fann at the age of three, and when he was thirty-eight one of his paintings received a remarkable amount of public notice and acclaim. This painting, called "American Gothic," is a starkly simple depiction of a serious couple staring directly out at the viewer. Sample

Now listen to a sample question. (narrator)

What style of painting

In your test book. you will read:

is known as American (A) Art from (B) Art from United (C) Art from United (0) Art from

@

regionalist?

@ @

America's inner cities. the central region of the States. various urban areas in the States. rural sections of America.

The best answer to the question, "What style of painting is known as American regionalist?" "Art from rural sections of America: Therefore, the correct choice is (0). Sample

Now listen to another sample question. (narrator)

What is the name of Wood's most successful

In your test book. you will read:

(A) (B) (C) (0)

painting?

"American Regionalist: "The Family Farm in Iowa: "American Gothic: "A Serious Couple:

Answer

is (0),

Answer

@ @ @

The best answer to the question, "What is the name of Wood's most successful painting?" is (C), "American Gothic: Therefore. the correct choice is (C). Remember, you are Dot allowed to take notes or write in your test book.

TOEFL- test directions and formal are reprinted by prnnission of ETS. the copyright owner. However. all eumples and Int questions are provided by Peanon Education, Inc.

8 COMPlETE TEST FOUR

493

1 o 1 o 1 o 1o 1 o 1o 1 o 1 39.

(A) (B) (C) (D)

How to get a professor's signature. The procedure for dropping courses. When to come and see the advisor. The effect of officially dropping a course.

40.

(A)

Any time, if the professor is willing to sign. (B) Only on the day of the talk. (C) During the first three weeks of the semester. (D) Up to three weeks before the end of the semester.

41.

(A) None. (B) One. (C) Two. (D) Three.

42.

(A) The student (B) The course student's (C) The student signature. (D) The student

fails the course. is removed from the schedule. needs to get the advisor's

45.

(A) Increased prosperity. (B) A large share of gold. (C) A healthier lumber business. (D) Little or nothing.

46.

(A) To show what a terrible life John Sutter had led. (B) To show the folly of trying to develop a business. (C) To show the effect that the discovery of gold has on individuals. (D) To show that the development of the West happened partly by chance.

47.

(A) Becoming a university student. (B) Managing time. (C) Majoring in management. (D) Spending a week in a management training program.

48.

(A) Relaxation techniques. (B) Homework assignments. (C) A personal time-management study. (D) Keeping an appointment calendar.

49.

(A) Ninety-six days. (B) Twenty-four days. (C) Seven days. (D) Fifteen minutes.

50.

(A) (B) (C)

receives a warning.

43.

(A) A woodcarving business. (B) A lumber business. (C) A construction business. (D) A jewelry business.

44.

(A) During the construction of a sawmill. (B) After prospectors had arrived. (C) Sometime after Sutter's death. (D) Before Sutter had the rights to the land.

Make an appointment. Begin the time study. Write down how they spend their time. (D) Attend another seminar.

This is the end of Section 1. Stop work on Section 1. Turn off the recording.

8888888 Read the directions for Section 2 and begin work. Do NOT read or work on any other section of the test during the next 25 minutes.

494

COMPLETETEST FOUR

2 SECTION 2 STRUCTURE AND WRITTEN EXPRESSION lime-25 minutes (including the reading of the directions) Now set your clock for 25 minutes. This section is designed to measure your ability to recognize language that is appropriate for standard written English. There are two types of questions in this section, with special directions each type.

for

Structure Directions: These questions are incomplete sentences. Beneath each sentence you will see four words or phrases, marked (A), (B), (C), and (D). Choose the one word or phrase that best completes the sentence. Then, on your answer sheet, find the number of the question and fill in the space that corresponds to the It~ller of the answer you have chosen. Look at the following examples. Example

I

Sample

The president __ (A) (B) (C) (D)

the election by a landslide.

Answer



@ @ @

won hewon yesterday fortunately

The sentence should read, "The president won the election by a landslide." Therefore, you should choose answer (A). Example

II

When __ (A) (B) (C) (D)

the did the the

Sample the conference? doctor attended the doctor attend doctor will allend doctor's attendance

The sentence should read, "When did the doctor allend the conference?" choose answer (B).

TOEfL.l~t dirl"Ctinn.s and format arc .-eptinted by ~nnissiuu of ETS. the copyright own •.'I'". However. all examples and (~I questions arr pmvided by Pearson Educaliun, Inc.

Answer

@



@

@

Therefore, you should

COMPLETE TEST FOUR

495

2 I.

Indiana's Lost River __ for a distance of 22 miles.

underground

6.

(A) (B)

travels traveling (C) to travel (0) it travels

(A) it (B) of it (C) what (0) of what

the first 2. The 1980 explosion of __ volcanic emption in the continental United States in over 60 years. (A)

Mount SI. Helens was Mount SI. Helens (C) it was Mount SI. Helens (0) Mount SI. Helens was

(B)

one cloud to 3. Static e1eco'icity __ another or between clouds and the ground creates lightning. (A) (B)

7. The light from an electrical lamp includes many different wavelengths, __ in a laser is concentrated on only one wavelength. (A) all the energy (B) it is all the energy (e) while all the energy (0) while all the energy is 8. In the Antarctic Ocean __ crustacean forms of life.

flows from the flow from (C) flowing from (0) is flowing from

4. The Model T car, introduced $850.

in 1908, __

the price was a price of (C) to be priced at (0) was priced at

reacts with a chlorine atom, an 5. __ electron is transferred from the outer shell of the sodium atom to the outer shell of the chlorine atom. A sodium atom When a sodium atom (C) For a sodium atom (D) It is a sodium atom

(B)

496

COMPLETE TEST FOUR

plankton and

(A) an abundance of (B) is an abundance of (C) it is abundant (0) an abundance is

(A) (B)

(A)

In 1858, the site \__ was to become the city of Denver was settled as a way station for outfitting gold prospectors.

9.

Flintlock muskets __ sharp bayonets were standard weapons during the American Revolution. (A) tip with (B) tipped with (C) the tips of (0) were tipped with

10. Benjamin Franklin believed that the turkey rather than the eagle __ of the United States. should become the symbol the symbol becomes (C) should symbolize becoming (D) becoming the symbol

(A) (B)

2

2 II.

2

__ to occur in the Earth's crust. pushpull and shake waves would be generated simultaneously. (A) Were a break (B) If a break (C) A break was (0) If broken

12. Fossil fuels like coal. oil, and gas produce carbon dioxide when __ ' (A) (B) (C) (0)

are burned they burned burned are they burned

14. The temperatures __ take place vary widely for different materials. (A) (B) (C) (0)

which melting and freezing at which melting and freezing which they melt and freeze at which they melt and freeze

15. In general, the cells of large animals and plants are only slightly larger than __ plants and animals. (A) (B) (C) (0)

smaller are smaller those smaller are those of smaller

13. Not until Nellie Tayloe Ross was elected governor of Wyoming in 1924 __ as go'vernor of a U.S. state. (A) (B) (C) (0)

a woman served a woman serving to serve a woman did a woman serve

COMPLETETESTFOUR

497

2 Written Directions: In these questions, underlined parts of the sentence or phrase that must be changed find the number of the question have chosen.

Expression

each sentence has four underlined words or phrases. The four are marked (A), (B), (C), and (0). Identify the one underlined word in order for the sentence to be correct. Then, on your answer sheet, and fill in the space that corresponds to the letter of the answer you

Look at the following examples. Example

I

Sample

Answer

o



The four string on a violin are tuned A -B-

c-o-

@ @

in fifths.

The sentence should read, 'The four strings on a violin are tuned in fifths." Therefore, you should choose answer (B). Example

II

Sample

Answer

The research for the book Roots taking --A--B-C-

o @

Alex Haley twelve years.

@



o

The sentence should read, "The research for the book Roots took Alex Haley twelve years." Therefore, you should choose answer (C). .

498

COMPLETE TEST FOUR

TOEFL- tesl directions and formal are nprinted by penni5sion of ETS. the copyright owner. However, all examples and lest aT'e' provided by Peaoon Education. Inc.

qUC'SlioM

2 16.

The music on a compact disk (CD) is record by lasers. A B -c-l)

17.

Alaska has more active glaciers as the rest of the inhabited -';-BC

18.

Aristotle believed that everything --A-

world combined. D

in the universe were composed

---a--

of four basic -C-

elements: earth. water, air. and fire.

D 19.

In the cold climate of the far north, mosquito eggs may remains dormant A B --C---Dautumn

20.

from

until late June.

Passengers A

have ridden the first Ferris wheel at the Columbian B -C-

Exposition

in

Chicago in 1893.

15 21.

One type of Australian

frog lays up to 25 eggs at a time and then swallows they for A --BC-

protection. D 22.

The Cro-Magnons absorbed

23.

entered the area that is today Europe and quickly eliminated -A-B-

theirs Neanderthal -C-

The Spanish introduced A

or

predecessors. 0

not only horses and also cattle to the North American B -C-

continent.

o 24.

The best. known members of the cabbage vegetable group includes head cabbage. A -B---C-ocauliflower,

25.

broccoli, kale, collard, and brussels sprouts.

White blood cells are the largest of red blood cells and are more varied in size and in ABC D shape.

COMPLETE TEST FOUR

499

2 26.

An hiccup is a spasmodic contraction

A

of the diaphragm,

B

which leads to a massive

-C-

intake of air. -027.

To make a lithograph, --A-

an artist used a flat stone of a kind that will soak up oil and -C--0-

-a

water.

28.

Alike a bar magnet, ---x-B-

the Earth has two magnetic poles. C 1)

29.

Not until Harvard College was founded in 1636 was there any colleges in America. ---,;:--BC 0

30.

Antelopes are gregarious animals that travel in herds, ranging in amount from a few A --B-

-----c-

to several thousand. -031.

A supersonic

airplane can fly faster than a speed of sound.

A

32.

If

C

i5

In 1821, Emma Willard opened officially the doors of the first school in the United

A

B

States to offer college-level courses for women. -C0 33.

The first gummed postage stamps issued in New York City in 1842. --A-B- -CIT

34.

Typical long bone such as the femur consists of a long shaft with swellings at each -A-BC 0 end.

35.

The common octopus lives Jone in a den just big enough for its body.

36.

The vacuum tube did an important

---r

ABC

BCD

contribution

to the early growth of radio and

-0-

television.

37.

St. Augustine, Florida, founded in 1565 by Pedro Menendez, was razing 21 years --AB -Clater by Francis Drake. I)

500

COMPLETETESTFOUR

38.

A bimetallic thermometer A

relies the different rates of expansion of two types of

--a

-c

metal, usually brass and copper. -039.

An ice crystal is the nuclei on which a hailstone is built. -A-BC J)

40.

Tremendous

flooding during the summer of 1993 left 8 million acres of nine --AB

midwestern

states inundated

and proved both expensively and deadly. C -0-

This Is the end of Section 2. If you finish before 25 minutes has ended, check your work on Section 2 only.

8888888 At the end of 25 minutes, go on to Section 3. Use exactly 55 minutes to work on Section 3.

COMPLETETEST FOUR

50 I

SECTION 3 READING COMPREHENSION Time-55 minutes (including the reading of the directions) Now set your clock for 55 minutes. This section is designed to measure your ability to read and understand short passages similar in topic and style to those that students are likely to encounter in North American universities and colleges. This section contains reading passages and questions about the passages. Directions: In this section you will read several passages. Each one is followed by a number of questions about it. You are to choose the one best answer. (A). (B), (C), or (D), to each question. Then. on your answer sheet, find the number of the question and fill in the space that corresponds to the letter of the answer you have chosen. Answer all questions about the information in that passage.

in a passage on the basis of what is stated or implied

Read the following passage: John Quincy Adams, who served as the sixth president of the United States from 1825 to 1829. is today recognized for his masterful statesmanship and diplomacy. He dedicated his life to public service, both in the presidency and in the various other political offices that he Line held. Throughout his political career he demonstrated his unswerving belief in freedom of (5) speech. the antislavery cause. and the right of Americans to be free from European and Asian domination. Example

I

Sample Answer

@



To what did John Quincy Adams devote his life? (A) (B) (C) (D)

@

Improving his personal life Serving the public Increasing his fortune Working on his private business

@

According to the passage, John Quincy Adams "dedicated his life to public service." Therefore, you should choose answer (B). Example

II

Sample Answer

In line 4. the word "unswerving" is closest in meaning to (A) (B) (C) (D)

moveable insignificant unchanging diplomatic

@ @

• @

The passage states that John Quincy Adams demonstrated his unswerving belief "throughout his career." This implies that the belief did not change. Therefore, you should choose answer (C).

502

COMPLETE

TEST FOUR

TOEFL- tnt dlrectiom and format are ~nlrd by permission of ETS. the copyright owntl". Howevrr. all examples and test questions are provided by Peanon Education. loc.

Questions 1-10

Line (5)

(/0)

(/5)

A rather surprising geographical feature of Antarctica is that a huge freshwater lake, one of the world's largest and deepest, lies hidden there under four kilometers of ice. Now known as Lake Vostok, this huge body of water is located under the ice block that comprises Antarctica. The lake is able to exist in its unfrozen state beneath this block of ice because its waters are warmed by geothermal heat from the earth's core. The thick glacier above Lake Vostok actually insulates it from the frigid temperatures (the lowest ever recorded on Earth) on the surface. The lake was first discovered in the 1970s while a research team was conducting an aerial survey of the area. Radio waves from the survey equipment penetrated the ice and revealed a body of water of indeterminate size. It was not until much more recently that data collected by satellite made scientists aware of the tremendous size of the lake; the satellite-borne radar detected an extremely flat region where the ice remains level because it is floating on the water of the lake. The discovery of such a huge freshwater lake trapped under Antarctica is of interest tq the scientific community because of the potential that the lake contains ancient microbes that have survived for thousands upon thousands of years, unaffected by factors such as nuclear fallout and elevated ultraviolet light that have affected organisms in more exposed areas. The downside of the discovery, however, lies in the difficulty.of conducting research on the lake in such a harsh climate and in the problems associated with obtaining uncontaminated samples from the lake without actually exposing the lake to contamination. Scientists are looking for possible ways to accomplish this. I. The purpose of the passage is to explain how Lake Vostok was discovered (B) provide satellite data concerning Antarctica (C) discuss future plans for Lake Vostok (0) present an unexpected aspect of Antarctica's geography

4.

Which of the following is closest in meaning to "frigid" in line 6?

(A)

(A) Extremely cold (B) Never changing (C) Quite harsh (0) Rarely recorded 5. All of the following are true about the 1970 survey of Antarctica EXCEPT that it

2. The word "lies" in line 2 could best be replaced by

(A) was conducted by air (B) made use of radio waves (C) did not measure the exact size of the lake (0) was controlled by a satellite

(A) sleeps (B) sits (C) tells falsehoods (0) inclines What is true of Lake Vostok?

It can be inferred from the passage that the ice would not be flat if

(A) It is completely frozen. (B) It is not a saltwater lake. (C) It is beneath a thick slab of ice. (0) It is heated by the sun.

(A) there were no lake (B) the lake were not so big (C) Antarctica were not so cold (0) radio waves were not used

6. 3.

COMPlETE

TEST FOUR

503

7. The word "microbes" in line 13 could best be replaced by which of the following? (A) Pieces of dust (B) Trapped bubbles (C) Tiny organisms (D) Rays of light 8. The passage mentions which of the following as a reason for the importance Lake Vostok to scientists?

9. The word "downside" in line 15 is closest in meaning to (A) bottom level (B) negative aspect (C) underside (D) buried section

of

10. The paragraph following the passage most probably discusses (A)

(A) (B) (C) (D)

It can be studied using radio waves. It may contain uncontaminated microbes. It may have elevated levels of ultraviolet light. It has already been contaminated.

further discoveries on the surface of Antarctica (B) problems with satellite-borne radar equipment (C) ways to study Lake Vostok without contaminating it (D) the harsh climate of Antarctica

~:

504

COMPLETE TEST FOUR

.

Questions 11-21

Line (5)

(10)

(J 5)

(20)

In the American colonies there was little money. England did not supply the colonies with coins and it did not allow the colonies to make their own coins, except for the Massachusetts Bay Colony, which received pennission for a short period in 1652 to make several kinds of silver coins. England wanted to keep money out of America as a means of controlling trade: America was forced to trade only with England if it did not have the money to buy products from other countries. The result during this prerevolutionary period was that the colonists used various goods in place of money: beaver pelts, Indian wampum, and tobacco leaves were all commonly used substitutes for money. The colonists also made use of any foreign coins they could obtain. Dutch, Spanish, French, and English coins were all in use in the American colonies. During the Revolutionary War, funds were needed to finance the war, so each of the individual states and the Continental Congress issued paper money. So much of this paper money was printed that, by the end of the war, almost no one would accept it. As a result trade in goods and the use of foreign coins still flourished during this period. By the time the Revolutionary War had been won by the American colonists, the monetary system was in a state of total disarray. To remedy this situation, the new Constitution of the United States, approved in 1789, allowed Congress to issue money. The individual states could no longer have their own money supply. A few years later, the Coinage Act of 1792 made the dollar the official currency of the United States and put the country on a bimetallic standard. In this bimetallic system, both gold and silver were legal money, and the rate of exchange of silver to gold was fixed by the government at sixteen to one. 11. The passage mainly discusses (A) American money from past to present (B) the English monetary policies in colonial America (C) the effect of the Revolution on American money (D) the American monetary system of the seventeenth and eighteenth centuries 12. The passage indicates that during the colonial period, money was (A) (B) (C) (D)

supplied by England coined by the colonists scarce ' used extensively for trade

13. The Massachusetts Bay Colony was allowed to make coins (A) continuously from the inception of the colony (B) throughout the seventeenth century (C) from 1652 until the Revolutionary War (D) for a short time during one year

14. The expression "a means of" in line 4 could best be replaced by (A) (B) (C) (D)

an example of a method for a result of a punishment for

15. Which of the following is NOT mentioned in the passage as a substitute for money during the colonial period? (A) (B) (C) (D)

Wampum Cotton Tobacco Beaver furs

16. The pronoun "it" in line 12 refers to which of the following? (A) (B) (C) (D)

The Continental Paper money Thewar Trade in goods

Congress

COMPLETETESTFOUR

50S

17. It is implied in the passage that at the end of the Revolutionary War, a paper dollar was worth (A) (B) (C) (D)

exactly one dollar just under one dollar just over one dollar almost nothing

18. The word "remedy" in line 15 is closest in meaning to (A) (B) (C) (D)

resolve understand renew medicate

19. How was the monetary system arranged in the Constitution? (A) Only the United States Congress could issue money. (B) The United States officially went on a bimetallic monetary system. (C) Various state governments, including Massachusells. could issue money. (D) The dollar was made the official currency of the United States.

506

COMPLETETESTFOUR

20. According to the passage, which of the following is NOT true about the bimetallic monetary system? (A) Either gold or silver could be used as official money. (B) Gold could be exchanged for silver at a rate of sixteen to one. (C) The monetary system was based on two metals. (D) It was established in 1792. 21. The word "fixed" in line 19 is closest in meaning to (A) (B) (C) (D)

discovered repaired valued set

Questions 22-30

Line (5)

(10)

(IS)

The human brain. with an average weight of 1.4 kilograms. is the control center of the body. It receives information from the senses, processes the information, and rapidly sends out responses; it also stores the information that is the source of human thoughts and feelings. Each of the three main parts of the brain-the cerebrum. the cerebellum, and the brain stem-has its own role in carrying out these functions. The cerebrum is by far the largest of the three parts. taking up 85 percent of the brain by weight. The outside layer of the cerebrum, the cerebral cortex. is a grooved and bumpy surface covering the nerve cells beneath. The various sections of the cerebrum are the sensory cortex, which is responsible for receiving and decoding sensory messages from throughout the body; the motor cortex. which sends action instructions to the skeletal muscles; and the association cortex, which receives, monitors, and processes information. It is in the association cortex that the processes that allow humans to think take place. The cerebellum, located below the cerebrum in the back part of the skull. is made of masses of bunched up nerve cells. It is the cerebellum that controls human balance, coordination, and posture. The brain stem, which connects the cerebrum and the spinal cord, controls various body processes such as breathing and heartbeat. It is the major motor and sensory pathway connecting the body and the cerebrum. 22.

What is the author's main purpose?

25.

To describe the functions of the parts of the brain (B) To explain how the brain processes information (C) To demonstrate the physical composition of the brain (0) To give examples of human body functions (A)

(A)

a layer of the brain beneath the cerebrum (B) a layer of nerve cells in the brain (C) a part of the brain that makes up 85 perc ~nt of the brain (0) a rid';ed layer covering the cerebrum in the brain 26.

23. The word "stores" in line 3 is closest in meaning to (A) shops (B) processes (C) releases (0) stockpiles

24. The passage states that the most massive part of the brain is the (A)

(B) (C) (0)

The "cerebral cortex" in line 7 is

The sensory cortex senses that messages should be sent out to the Muscles (B) provides a slU'face covering for nerve cells (C) is where the human process of thinking occurs (0) receives and processes information from the senses (A)

cerebrum cerebellum cerebral cortex brain stem

COMPLETETESTFOUR

507

27. The word "monitors" in line 11 is closest in meaning to which of the following? (A) (B) (C) (D)

Keeps track of Keeps hold of Gets away with Gets rid of

28. Which of the following is true about the cerebellum? (A) (B) (C) (D)

508

It It It It

is located above the cerebrum. controls breathing. is responsible for balance. is the outside layer of the cerebrum.

COMPLETETESTFOUR

29. What shape does the brain stem most likely have? (A) (B) (C) (D)

Small and round Long and thin Large and formless Short and flat

30. Which of the following could best be used in place of .pathway. in line 16? (A) (B) (C) (D)

Driveway Roadway Route Street

Questions 31-41

Line (5)

(10)

(15)

(20)

Though Edmund Halley was most famous because of his achievements as an astronomer, he was a scientist of diverse interests and great skill. In addition to studyi!1g the skies. Halley was also deeply interested in exploring the unknown depths of the oceans. One of his lesser-known accomplishments that was quite remarkable was his design for a diving bell that facilitated exploration of the watery depths. The diving bell that Halley designed had a major advantage over the diving bells that were in use prior to his. Earlier diving bells could only make use of the air contained within the bell itself, so divers had to surface when the air inside the bell ran low. Halley's bell was an improvement in that its design allowed for an additional supply of fresh air that enabled a crew of divers to remain underwater for several hours. The diving contraption that Halley designed was in the shape of a bell that measured three feet across the top and five feet across the bottom and could hold several divers comfortably; it was open at the bottom so that divers could swim in and out at will. The bell was built of wood, which was first heavily tarred to make it water repellent and was then covered with a half-ton sheet of lead to make the bell heavy enough to sink in water. The bell shape held air inside for the divers to breathe as the bell sank to the bottom. The air inside the bell was not the only source of air for the divers to breathe, and it was this improvement that made Halley's bell superior to its predecessors. In addition to the air already in the bell. air was also supplied to the divers from a lead barrel that was lowered to the ocean floor close to the bell itself. Air flowed through a leather pipe from the lead barrel on the ocean floor to the bell. The diver could breath the air from a position inside the bell, or he could move around outside the bell wearing a diving suit that consisted of a lead bell-shaped helmet with a glass viewing window and a leather body suit. with a leather pipe carrying fresh air from the diving bell to the helmet. 31.

The subject of the preceding most likely Halley's

passage was

34.

(A) childhood (B) work as an astronomer (C) many different interests (0) invention of the diving bell 32.

Which of the following best expresses subject of this passage? (A) Halley's Halley's (C) Halley's diving (0) Halley's (B)

work as an astronomer many different interests invention of a contraption experiences

"ran low " in line 8 is closest

(A) moved slowly (B) had been replenished (C) sank to the bottom (0) was almost exhausted the

35.

How long could divers stay underwater Halley's bell?

in

(A) Just a few seconds (B) Only a few minutes (C) For hours at a time (0) For days on end

for

as a diver 36.

33.

The expression in meaning to

Halley's bell was better than its predecessors because it (A) was bigger (B) provided more air (C) weighed less (0) could rise more quickly

It is NOT stated in the passage that Halley's bell (A) was wider at the top than at the bottom (B) was made of tarred wood (C) was completely enclosed (0) could hold more than one diver

COMPLETE TEST FOUR

509

37.

The expression "at will" in line 13 could best be replaced by (A) (B) (C) (0)

38.

It can be inferred from the passage that, were Halley's bell not covered with lead, it would (A) (B) (C) (0)

39.

float get wet trap the divers suffocate the divers

Where in the passage does the author indicate how air traveled from the barrel to the bell? (A) (B) (C) (0)

5 I0

in the future as they wanted with great speed upside down

Lines 8-10 Lines 11-13 Lines 17-18 Line 20

COMPLETETESTFOUR

40.

In which paragraph does the author describe the diving bells that preceded Halley's? (A) (B) (C) (0)

In In In In

the the the the

first paragraph second paragraph third paragraph last paragraph

41. This passage would most likely be assigned reading in a course on (A) (B) (C) (0)

astronomy recreation oceanography physiology

Questions 42-50

Line (5)

(10)

(IS)

(20)

(25)

(30)

Paul Bunyan is perhaps America's best-known folk hero. A fictional logger of incredible strength, he was most likely based on an actual nineteenth-century logger from the northern United States or Canada. As a folk hero, he struck a chord with Americans on some level. perhaps because he was incredibly strong but also because he was hard-working and capable, ingenious in solving problems, and fun-loving. Though there is evidence that Paul Bunyan tales were part of oral tradition in the nineteenth century, Paul Bunyan stories did not appear in written form until the early twentieth century. Journalist James McGillivray included descriptions of Bunyan in a series of essays entitled "The Round River Drive:' which appeared in a number of Midwestern newspapers between 1906 and t 91 O. However, it was through an extensive advertising campaign that Paul Bunyan moved solidly into print. Recognizing the appeal of Paul Bunyan as a figure for his company's advertising, William Laughead, an advertising executive for the Red River Lumber Company, initiated a campaign that consisted of a series of publications featuring Paul Bunyan. For several decades, the company distributed these publications free of charge and made no attempt to obtain a copyright on them. In fact, the company vigorously encouraged other writers to make use of Paul Bunyan because it felt that the use of this character enhanced the name recognition of the Red River Lumber Company inasmuch as the name of the folk hero and the name of the company had become interwoven. The Bunyan stories published by Red River and further circulated by others were tall tales of gigantic proportions. In these tales, Bunyan is depicted as a man of superhuman proportions, who is strong, hard-working, entrepreneurial. and innovative. In one story, for example, Paul is credited with digging the Great Lakes in order to create a watering hole for his giant ox, Babe. In another of these tales, Paul caused an entire winter of blue snow to fall by swearing a blue streak after he injured himself by smashing his thumb with a large hammer. A third story in the series describes Paul's role in establishing the Mississippi River. Fascination with Paul Bunyan has continued to grow, and today he is a standard of American folklore. The prevalence of Bunyan as a figure of folklore today is evidenced by references to him in countless stories, cartoons, poems, and songs as well as the numerous community festivals and logging competitions featuring Paul Bunyan that can be found throughout the sections of the country where logging has a strong tradition. 42. The purpose of this passage is to (A) (B) (C) (D)

43.

present the actual feats of a real-life logger discuss a "larger than life" folk hero describe logging in North America provide an overview of American folktales

44.

The passage states that Paul Bunyan tales first appeared (A) (B) (C) (D)

in in in in

oral stories a series of essays newspapers advertising

It is NOT stated in the passage that Paul Bunyan is known for his (A) unusual strength (B) dedication to work (C) ingenuity in difficult situations (D) serious nature

COMPLETE TEST FOUR

5/ I

45.

Which of the following CANNOT be inferred about the Red River Lumber Company's advertising campaign featuring Paul Bunyan? (A) It endured for quite a time. (B) The company did not protect its ownership of the stories. (C) The campaign did little to enhance the company's profitability. (0) The company wanted the name Paul Bunyan to be known as widely as possible.

48.

(A) Lines 8-9 (B) Lines 14-15 (C) Lines 19-20 (0) Lines 22-24 49.

47.

50.

The word "interwoven" in line 18 could best be replaced by

The word .countless. in line 28 could best be replaced by the expression (A) a large number of (B) a specified number of (C) an insubstantial number of (0) an overestimated number of

46. The pronoun "them" in line 15 refers to (A) series (B) decades (C) publications (0) writers

Where in the passage does the author discuss a weather phenomenon that Paul Bunyan supposedly caused?

Which paragraph describes the plots of some of the tales of Paul Bunyan? (A) (B) (C) (0)

The The The The

second paragraph third paragraph fourth paragraph fifth paragraph

(A) unfashionable (B) mixed together (C) not compatible (0) too separate

This Is the end of Section 3.

8888888 If you finish in less than 05 minutes, check your work on Section 3 only. Do NOT read or work on any other section of the test.

When you finish the test, you may do the following: • Turn to the Diagnostic Charts on pages 5H~-590, and circle the numbers of the questions that you missed . • Turn to Scoring Information on pages !'iHl-!'iH~, and determine your TOEFL score . • Turn to the Progress Chart on page !'i91,and add your score to the chart.

5 12

COMPLETETESTFOUR

TEST OF WRITTEN ENGLISH: TWE ESSAY TOPIC Tune-30 minutes In difficult situations, some people react calmly, while others react with panic. How do you react in difficult situations? Use specific details and examples to support your answer.

3

COMPlETETESTFOUR

5I3

514

1 D 1 D 1 D 1 D 1D 1 D 1D 1 COMPLETE TEST FIVE SECTION 1 LISTENING COMPREHENSION Trme-approximately 35 minutes (including the reading of the directions for each part) In this section of the test, you will have an opportunity to demonstrate your ability to understand conversations and talks in English. There are three parts to this section. with special directions for each part. Answer all the questions on the basis of what is stated or Implied by the speakers you hear. Do not take notes or write in your test book at any time. Do not turn the pages until you are told to do so.

PartA Directions: In Part A you will hear short conversations between two people. After each conversation, you will hear a question about the conversation. The conversations and questions will not be repeated. After you hear a question. read the four possible answers in your test book and choose the best answer. Then, on your answer sheet. find the number of the question and fill in the space that corresponds to the letter of the answer you have chosen. Usten

to an example.

On the recording,

Sample

Answer

CAl

you will hear:

@ (man) (woman) (narrator)

@



That exam was iust awful. Oh, it could have been worse. What does the woman mean?

In your test book, you will read:

(A) The exam was realIy awful. (B) It was the worst exam she had ever seen. (C) It couldn't have been more difficult. (D) It wasn't that hard.

You learn from the conversation that the man thought the exam was very difficult and that the woman disagreed with the man. The best answer to the question, "What does the woman mean?" is (D), "It wasn't that hard." Therefore, the correct choice is (D).

TOEFL- tnt dlrrctlons and lonnal are reprintrd by pennlssion of ETS. the- copyright owne-r. Howf:W'l'".all eumples and lnt questions are pro••. ided by Pearson Education. Inc.

8 COMPLETETEST FIVE

5 15

1D 1D 1 1. (A) (B) (C) (0)

2. (A) (B) (C) (0)

3. (A) (B) (C) (0)

4.

D

1 D 1 D 1D 1D 1

She has rules about how to play. Her goal is to pay for school. She is praying not to have a low score. Shell be acting in a school project.

10. (A) (B) (C) (0)

He'll make a charitable contribution. He couldn't get into the classroom. He didn't have very much to say. He was not given the chance to speak.

She'd She'd She'd She's

11. (A) (B) (C)

He thought it was extremely fruitful. He's happy he didn't attend it. A lot of people missed it. It was perturbing.

like like like a bit

something to drink. to have thirty. a bite to eat. thrifty.

She's moving in the opposite direction. She's wide awake. The rest of the people are tired. She needs to take a nap.

He'll continue to stand in line for texts. (B) He has enough to pay for the texts. (C) He agrees with the woman about the texts. (0) He thinks the woman's in the wrong line to get the texts.

(0)

12. (A) He'd like the woman to repeat herself. (B) The woman should talk to physician. (C) He shares the woman's position. (0) What the woman said was unimportant.

a

(A)

5. (A) (B) (C) (0)

6. (A) (B) (C) (0)

He was given the wrong key. The key was on top of the clock. It was lucky that he got the key. The key was at his feet. He He He He

went to the conference. saw his friends at the conference. was in his place at the conference. sent a representative.

13. (A) To see a dentist. (B) To see a cardiologist. (C) To see a podiatrist. (0) To see an ophthalmologist. 14. (A) (B) (C) (0)

15. (A) (B) (C) (0)

16. (A) 7. (A) (B) (C) (0)

8. (A) (B) (C) (0)

9. (A) (8)

(C) (0)

5I6

She will see the lawyer tomorrow. She needs to phone the lawyer. The lawyer will call her tomorrow. The lawyer has called off their meeting. There's a lot of difficult homework in it. There are not very many exams in it. There is little homework. There is no homework. Returning to it later. Coming back home. Finishing the math book. Leaving for class.

COMPLETETEST FIVE

(B) (C) (0)

17. (A) (B)

She She Her She

is too scared to try it. would like another opportunity. time is very scarce. has gone skiing for the last time.

He really enjoyed the conference. He'll be able to go to the conference. He couldn't attend the conference. He heard everything at the conference. She doesn't need a jacket for the game. She was very uncomfortable last time. She will take a jacket with her this time. Her jacket does not feel very comfortable.

He parked the car to buy the tickets. He left the car where he shouldn't have. (C) He got a speeding ticket. (0) He didn't park the car.

1 D 1 o 1 D 10 1 D 1 o 1 o 1 18. (A) She prepared' him for what he was going to do. (B) She was unprepared for what she had to do. (C) She probably didn't spend much time on her presentation. (D) She was really ready for her presentation. 19. (A) (B) (C) (D)

He He He He

20.

(A) (B) (C) (D)

That That That That

21.

(A) She listened allentively during class. (B) She must make the list five pages long. (C) She did not allend all of the class. (D) She was inallentive during some of the class.

22.

25.

(A) He didn't believe the course was hard. (B) He heard that the course was closed. (C) It was hard for him to get to the class. (D) He registered for the course.

28.

(A) (B)

29.

(A) That he would be working all weekend. (B) That no one ever worked on weekends. (C) That he would not be in the office this weekend. (D) That the office would be open this weekend.

30.

(A) They should not let what happened bother them. (B) They should keep on trying to talk to Mary. (C) They should try to flatter Mary. (D) Their project is already as good as it's going to get.

now.

(C)

(A) The tuition increase was unexpected. (B) She was prepared for the tuition increase. (C) She doesn't believe that fees were increased. (D) She believes that tuition will not go up.

24.

(A) She answered his question a minute ago. (B) She just bit her tongue. (C) It's hard for her to put the answer into words. (D) The tip of he: tongue is quite sore.

laid down their

27.

(A) He's not quite sure when the projects should be finished. (B) He's doing his project for music class

23.

are choosing part-time

(A) (B) (C) (D)

man had been in class. man didn't have the notes. didn't need the notes. lecture had been canceled.

Music class meets for the first time in December. (D) He believes the music will be available on December I.

are lying down. will lose their

26. has never gone sailing. doesn't like sailing. hasn't had much time for sailing. doesn't have any time to go sailing. the the she the

(A) Some of them (B) Some of them positions. (C) Some of them jobs. (D) Some of them newspapers.

She's unhappy about the score. She hasn't seen her score yet. She's really pleased with her score. She hasn't taken the exam yet.

He didn't go because he was sleeping. He didn't miss the commillee meeting. (C) He never returned from class. (D) He was unable to fall asleep.

COMPLETE TEST FIVE

517

1 D 1 o 1 D 1 o 1o 1 D 1o 1 Part B Directions: In this part of the test, you will hear longer conversations. After each conversation, will hear several questions. The conversations and questions will not be repeated.

you

After you heal" a question. read the four possible answers in your test book and choose the best answer. Then, on your answer sheet. find the number of the question and fill in the space that corresponds to the letter of the answer you have chosen. Remember, you are not allowed to take notes or write in your test book.

31.

(A) (B) (C) (D)

To To To To

32.

(A) (B) (C) (D)

They're both regular members. He likes to go there occasionally. She wants him to try it out. She hates to exercise alone.

33.

34.

a doctor's appointment. an exercise club. a swimming pool. a school.

(A) A limited number. (B) Racquetball courts and a swimming pool. (C) Exercise machines. but not classes. (D) Just about anything. (A) (B) (C) (D)

Visit the club once. Take out a membership. Try the club unless he hurts himself. See if he has time to go.

(A)

35.

A presentation for political science class. (B) How quickly time passes. (C) The differences between the various types of courts. (D) A schedule for preparing for a political science exam.

36.

(A) (B) (C) (D)

Three levels of courts. Only the municipal courts. The state but not the federal courts. Only the state and federal courts.

37.

(A) (B) (C) (D)

On Thursday. On Monday. In a week. Before Monday.

38.

(A) (B) (C) (D)

Plenty of time. Until Monday. About one week. Until a week from Monday.

TOEFLlest directiofU or ETS. the copyright

518

COMPLETE TEST FIVE

questions.~

and fonnat

a.-r reprinted

by permission and test

O~T. H~r. aU examples provided by Pearson Education. Inc.

1 D 1 D 1 D 1 D 1 D 1 D 1D 1 Part C Directions: In this part of the test. you will hear several talks. After each talk. you will hear some questions. The talks and questions will not be repeated. After you hear a question. you will read the four possible answers in your test book and choose the best answer. Then, on your answer sheet. find the number of the question and fill in the space that corresponds to the letter of the answer you have chosen. Here is an example. On the recording, (narrator) (man)

you will hear:

Listen to an instructor talk to his class about painting. Artist Grant Wood was a guiding force in the school of painting known as American regionalist. a style reflecting the distinctive characteristics o{art {rom rural areas of the United States. Wood began drawing animals on the family farm at the age of three, and when he was thirty-eight one of his paintings received a remarkable amount of public notice and acclaim. This painting, called "American Gothic," is a starkly simple depiction of a serious couple staring directly out at the viewer.

Now listen to a sample question. (narrator)

What style of painting

Sample is known as American

In your test book, you will read:

(A) Art from (B) Art from United (C) Art from United (0) Art from

regionalist?

@ @



America's inner cities. the central region of the States. various urban areas in the States. rural sections of America.

The best answer to the question, "What style of painting is known as American regionalist?" "Art from rural sections of America." Therefore. the correct choice is (0). Now listen to another sample question. (narrator)

Sample

What is the name of Wood's most successful

In your test book, you will read:

(A) (B) (C) (0)

Answer

@

is (0).

Answer

@ @

painting?



"American Regionalist." "The Family Farm in Iowa." "American Gothic." "A Serious Couple."

@

The best answer to the question. "What is the name of Wood's most successful painting?" "American Gothic." Therefore, the correct choice is (C).

is (C),

Remember, you are not allowed to take notes or write in your test book .

TOEFL-1M' dir«llons and formal are ~printnJ by ~nnission the copynght owner. However. all examples and lest

of ETS. questions

arr providt"d

by Prarson

F.ducalion,

Inc.

.8

COMPLETE F;ST FIVE

5 I9

1o 1o 1o 1 o 1o 1o 1o 1 39.

(A) A university administrator. (B) A student. (C) A librarian. (0) A registrar.

40.

(A) How to use the library. (B) The university registration procedure. (C) Services offered by the Student Center. (0) Important locations on campus.

41.

42.

43.

44.

(A) To provide students with assistance and amusement. (B) To assist students in the registration process. (C) To allow students to watch movies. (0) To provide textbooks for university courses. (A) In administrators' offices. (B) In the Student Center. (C) In an auditorium. (0) In the Student Records Office. (A) Natural soaps. (B) Synthetic detergents. (C) Biodegradable detergents. (0) Phosphates. (A) Synthetic detergents. (B) A major cause of water pollution. (C) Substances that break down into simpler fomls. (0) The reason for the foaming water supply.

45.

(A) They broke down into simpler forms. (B) They caused the water to become foamy. (C) They released phosphates into the water. (0) They damaged only the underground water supply.

46.

(A) Water pollution in the 1950s. (B) Nonbiodegradable synthetic detergents. (C) The foamy water supply. (0) Problems caused by the phosphates.

47.

(A) The static atmosphere. (B) The cause of changes in the atmosphere. (C) The evolution of plant life. (0) The process of photosynthesis.

48.

(A) TWo hundred million years ago. (B) TWenty million years ago. (C) TWo hundred thousand years ago. (0) TWenty thousand years ago.

49.

(A) The evolution of plants and photosynthesis. (B) The variety of gases in the atmosphere. (C) The high percentage of nitrogen. (0) The ammonia and methane in the original atmosphere.

50.

(A) Read about the composition of the atmosphere. (B) Study the notes of today's lecture. (C) Prepare for a quiz. (0) Read the following chapter.

This is the end of Section 1. Stop work on Section 1. Tum off the recording.

Read the directions for Section 2 and begin work. Do NOT read or work on any other section of the test during the next 25 minutes.

520

COMPLETETESTFIVE

2 SECTION 2 STRUCTURE AND WRITTEN EXPRESSION TIme-25 minutes (including the reading of the directions) Now set your clock for 25 minutes. This section is designed to measure your ability to recognize language that is appropriate for standard written English. There are two types of questions in this section, with special directions each type.

for

Structure Directions: These questions are incomplete sentences. Beneath each sentence you will see four words or phrases, marked (A), (B), (C), and (0). Choose the one word or phrase that best completes the sentence. Then, on your answer sheet, find the number of the question and fill in the space that corresponds to the letter of the answer you have chosen. Look at the following examples. Example

I

Sample

The president __ (A) (B) (C) (0)

Answer



the election by a landslide.

@ @ @

won he won yesterday fortunately

The sentence should read, "The president won the election by a landslide." Therefore, you should choose answer (A). Example

II

When __ (A) (B) (C) (0)

the did the the

Sample the conference?

Answer

o



doctor attended the doctor attend doctor will attend doctor's attendance

@ @

The sentence should read, "When did the doctor attend the conference?" Therefore, you should choose answer (B).

TOEFL- test dirtttions and (onnat are ~prinled by pennission or ETS. the copyright owner. How~r. all eumpln and lese quntions are prn••. idrd by Pc-anon Education. Inc.

COMPLETE

TEST FIVE

521

2 at the same time 1. Different hormones __ on a particular target issue. (A) (B)

(C) (D) 2.

(B)

(C) (D) 3.

the the the the

Sun Sun Sun Sun

(B)

(C) (D)

rocks, rocks, rocks, rocks,

(A) (B)

(C) (D)

(C) (D)

522

(C) (D) 7.

(B)

(C) (D) 8.

(B)

(C) (D)

River, River River River,

the is the is one of the one of the

COMPLETETESTFIVE

accompany is accompanied accompanied it is accompanied

Charles Darwin's first scientific book. published in 1842, __ a since substantiated theory on the origin of coral reefs and atolls. (A) (B)

(C) (D) 10.

clouds clouds that clouds that are clouds that they

Measles is a highly contagious viral disease __ by a characteristic skin rash. (A)

9.

were sea levels sea had levels having sea levels sea levels were

Hail forms within large, dense cumulonimbus __ develop on hot, humid summer days. (A)

An unknown Afterwards, an unknown When an unknown During an unknown

longest river in the 5. The Missouri __ United States, flows through seven states from its source in Montana to its confluence with the Mississippi. (A) (B)

(B)

nearly all but nearly all nearly all are which nearly all are

__ radioisotope is encountered, the first step in its identification is the determination of its half-life.

Coral islands such as the Maldives are the tips of reefs built during periods of warm climate, when __ higher. (A)

is they are it is are

Most radioactive elements oceur in igneous and metamorphic __ fossils occur in sedimentary rocks. (A)

4.

usually act usually acting they usual act the usual action

The tidal forces on the Earth due to __ only 0.46 of those due to the Moon. (A)

6.

to present presented presenting it presents

Phytoplanktons thrive where __ phosphorus into the upper layers of a body of water. (A) (B)

(C) (D)

upwelling currents circulate the circulation of upwelling currents are upwelling currents circulates upwelling currents

II.

By the end of 1609, Galileo had a 20-power telescope that enabled him to see __ planets revolving around Jupiter. (A) the call (B) he called (C) to call him (D) what he called

12. On every continent except Antarctica -_ more than 30,000 species of spiders. (A) (8) (C) (D)

some are some of the are some of the is some

14. Hurricanes move with the large-scale wind currents __ are imbedded. (A) (B) (C) (D)

that they which they in that they in which they

IS. __ the Earth's ice to melt, the Earth's oceans would rise by about two hundred feet. (A) If all (B) Were all (C) If all were (D) All was

13. Many bugs possess defensive scent glands and emit disagreeable odors when __ . (Al disturbed (B) are disturbed (C) they disturbed (D) are they disturbed

COMPlETE TESTFIVE

523

2 Written

Expression

Directions: In these questions. each sentence has four underlined words or phrases. The four underlined parts of the sentence are marked (A). (B). (C). and (0). Identify the one underlined word or phrase that must be changed in order for the sentence to be correct. Then. on your answer sheet. find the number of the question and fill in the space that corresponds to the letter of the answer you have chosen. Look at the following examples. Example

Sample Answer

I

@ The four string on a violin are tuned A -B-

c-o-

• nml) (man) (nnn-ntor)

4. (woman)

lOp

Il,p, 71

I can't imagine what he was thinking aoollt when he bought that car. Neither can I! WHAT DOES TIlE WOMAN MFAN?

(man) (nanalor)

5. (fllan) (woman) (nOfTalor)

6. (woman) (man) (nanalor)

7. (man) (woman) (narrator)

8. (man) (woman) (narrator)

12, p, 74

Arc we suppo,ento read all len chapters before the exam? A., far as I can tell, we arc. WHAT DOES THE MAN MFAN? You're oUl of apple pie! Sorry. Wh)' not try the cherr)' \\'HAT DOES THE WOMAN SUGGEST?

pie?

Do you know when the mo\;e starts? It starts at 8:00, doesn't it? WHAT DOES THE MAN MEA.'
Longman preparation course for the toefl test -the paper test-ITP

Related documents

629 Pages • 292,664 Words • PDF • 86.6 MB

740 Pages • PDF • 27.2 MB

394 Pages • 96,745 Words • PDF • 3.4 MB

222 Pages • 41,036 Words • PDF • 7.4 MB

1 Pages • 185 Words • PDF • 84.3 KB

112 Pages • 87,495 Words • PDF • 969.3 KB

3 Pages • 1,172 Words • PDF • 105.9 KB

386 Pages • 75,305 Words • PDF • 2.2 MB

10 Pages • 6,970 Words • PDF • 244.3 KB